Download SOHO`s Frequently Asked Questions

Document related concepts

Equation of time wikipedia , lookup

X-ray astronomy satellite wikipedia , lookup

Observational astronomy wikipedia , lookup

History of astronomy wikipedia , lookup

Aquarius (constellation) wikipedia , lookup

Rare Earth hypothesis wikipedia , lookup

Outer space wikipedia , lookup

International Ultraviolet Explorer wikipedia , lookup

Geocentric model wikipedia , lookup

Astrobiology wikipedia , lookup

Dialogue Concerning the Two Chief World Systems wikipedia , lookup

Planetary habitability wikipedia , lookup

Space weather wikipedia , lookup

Corona wikipedia , lookup

Advanced Composition Explorer wikipedia , lookup

Energetic neutral atom wikipedia , lookup

History of Solar System formation and evolution hypotheses wikipedia , lookup

Geomagnetic storm wikipedia , lookup

Extraterrestrial life wikipedia , lookup

Comparative planetary science wikipedia , lookup

Tropical year wikipedia , lookup

Orrery wikipedia , lookup

Astronomical unit wikipedia , lookup

Solar System wikipedia , lookup

Formation and evolution of the Solar System wikipedia , lookup

Hebrew astronomy wikipedia , lookup

Standard solar model wikipedia , lookup

Timeline of astronomy wikipedia , lookup

Transcript
SOHO’s Frequently Asked Questions
In the following pages, you will find Dr. SOHO's
responses to the public's questions: questions about
everything from sunspots and neutrinos to software and
science careers. These are taken directly from the
SOHO web site. The internal links are functional – links
that extend to other parts of the web will not work
without a browser.
The FAQs are organized into 12 major sections
which are linked for you as bookmarks to the left of the
page. Just click on the bookmark to go to the section
you want to see.
Click here to return to the Home page
Dr. SOHO's FAQ
WELCOME to Dr. SOHO's Frequently Asked Questions.
We offered, you asked, and Dr. SOHO has answered!
Here, you will find Dr. SOHO's responses to the public's questions: questions about everything from
sunspots and neutrinos to software and science careers.
These answers have been written by scientists and engineers at work on the SOHO mission. Their
original recipients ranged from elementary students to fellow scientists, but we hope they all make sense
to a wider audience. Select a category from the list below, and you are guaranteed to learn a few things.
In answering your questions, we certainly did!
If your question is not yet answered by this FAQ, please ask Dr. SOHO.
Frequently Asked Questions about:
● Basic Solar Things (How big? How hot? How far away?)
●
The Sun (Further questions about solar things)
●
Coronal Mass Ejections (CMEs) (A million tons of gas at a million miles per hour)
●
Sunspots and the Solar Cycle (Just what IS a sunspot anyway?)
●
Solar Flares (All about these powerful solar explosions)
●
The Earth (Radio interference, aurorae, and other solar-terrestrial effects)
●
The SOHO Spacecraft (The hardware)
●
SOHO's Instruments (Questions about our data and what we can see)
●
Space Missions (Other space missions and NASA topics)
●
Comets (From Comet Hyakutake to the Kreutz sungrazers)
●
Astronomy (Questions about stars, planets, and galaxies)
●
Miscellaneous Subjects (Everything else)
Go back to the "SOHO" main page.
http://sohowww.nascom.nasa.gov/explore/faq.html (1 of 2) [2000/01/11 11:56:58 AM]
Dr. SOHO's FAQ
SEND US YOUR COMMENTS
Go To
Other SOHO Web Pages
Author: Shane Stezelberger
Co-Author/Curator: Therese A. Kucera
[email protected]
Responsible Official: Art Poland([email protected])
Last modification: Wednesday, 10-Nov-1999 11:09:05 EST
http://sohowww.nascom.nasa.gov/explore/faq.html (2 of 2) [2000/01/11 11:56:58 AM]
Dr. SOHO !!!
Take advantage of Dr. SOHO...
So you still have questions about the Sun? about SOHO?
We have established an e-mail account so you can ask the SOHO scientist teams any question
about SOHO or the Sun and a SOHO scientist will respond to your inquiry !!!
To use this unique service, just send a mail message with your question(s) to:
[email protected]
If you are in school, it will help if you can tell us your age and grade level.
Before sending a question, please check the following sources of information:
An encyclopedia, a basic astronomy text book, and our list of Previously Asked Questions.
We are here to provide answers to questions about the Sun and SOHO that are not immediately available
in other places!
Also, please don't ask us to do your homework for you!
If you don't have access to e-mail, you can write us at Dr. SOHO
Code 682.3, NASA/GSFC
Greenbelt, MD 20771, USA
Go To
Other SOHO Web Pages
SEND US YOUR COMMENTS
Therese A. Kucera
[email protected]
Author/Curator: G. Dimitoglou, SM&A ESA/NASA PS
http://sohowww.nascom.nasa.gov/explore/drsoho.html (1 of 2) [2000/01/11 11:57:01 AM]
Dr. SOHO !!!
Responsible Official: Art Poland([email protected])
Last modification: 22 March 1999
http://sohowww.nascom.nasa.gov/explore/drsoho.html (2 of 2) [2000/01/11 11:57:01 AM]
Dr. SOHO's FAQ: The Basics
Go back to "Dr SOHO's FAQ"
Go back to the "SOHO" main page.
●
●
How big, how far, how bright, etc... is the Sun?
●
What do you think is the MOST important thing for a 5th grader to know about the Sun?
●
How hot is the Sun, and what are its different layers?
●
Why does the Sun shine?
●
Why is the Sun hot?
●
When will the Sun become a red giant?
●
What color is the Sun?
●
How far away from the Earth is the Sun? How far away is SOHO?
●
What is the "solar cycle?" What is a "quiet sun?"
How big, how far, how bright, etc... is the Sun?
Basic information and statistics about the Sun are available at:
http://nssdc.gsfc.nasa.gov/planetary/factsheet/sunfact.html
(Sun Fact Sheet)
http://www. seds.org/nineplanets/tnp/sol.html
(The Nine Planets: The Sun)
Go back to "Basics" questions and answers.
Hi, I'm a 5th grader who is doing a report in Science about the Sun. I needed some information
about the Sun. What do you think is the MOST important thing for a 5th grader to know about the
Sun? Thank you very much for your time.
●
Hmmm, let's see. THE most important thing about the Sun? Can I list a few things?
You probably already have the basic facts about the Sun, but here are some things that strike me as
especially important and interesting for a 5th grader:
http://sohowww.nascom.nasa.gov/explore/faq/basics.html (1 of 5) [2000/01/11 11:57:02 AM]
Dr. SOHO's FAQ: The Basics
The Sun is a star, it is just a lot closer than other stars, and the Earth and the other planets of our
solar system are in orbit around it. The Sun is the ultimate source of almost all the energy on Earth.
The Sun is Big. It is 1.3 million km (0.86 million miles) in diameter. 110 Earths strung together
would be as long as the diameter of the Sun. It is 150 million km (93 million miles) away, about
400 times the distance from the Earth to the Moon. (SOHO sits out at about 1% of that Earth-Sun
distance, by the way.)
The Sun is powered by nuclear reactions that happen in its core. In a nuclear reaction, matter is
changed directly to energy. The pressure at the Suns center is so high that hydrogen atoms fuse
together to become helium atoms + energy. This is related to Einstein's famous equation, E=mc2.
The Sun is not just a bright ball. It has a complicated and changing magnetic field, which causes
things like sunspots (dark patches on the Sun), various kinds of explosions, and other solar
activity.
The Sun emits what is called the "Solar Wind." The solar wind is a wind of particles which flows
out of the Sun's atmosphere and into the solar system. The Earth is protected from this wind by its
own magnetic field. However, sometimes the solar wind and the Earth's magnetic field interact so
that we get the aurora (the "Northern and Southern Lights") and occasionally things like power
outages and problems with satellites.
Those are the first things I can think of. Is that the sort of thing you were looking for? There is a
list of some basic facts about the Sun at:
http://www. seds.org/nineplanets/tnp/sol.html
(The Sun)
Go back to "Basics" questions and answers.
●
How hot is the Sun, and what are its different layers?
Let me answer both of those questions from the inside out. The core of the Sun is where the
important nuclear reactions are taking place. It occupies the innermost quarter or so of the Sun's
radius. Here, the temperature is 15 million Kelvin (or 27 million degrees Fahrenheit) and the
density is 13 times greater than lead. Energy radiates from the core to about 80% of the way to the
surface. Here, where the temperature is down to 4 million degrees F, the Sun's plasma becomes
"convectively unstable." Energy rises above this point via convection, as in a boiling pot of water.
The Sun's visible surface (do NOT look at it!) is "only" about 5800 K (10,000 degrees F). Above
the surface is a blanket of gases called the corona. The corona is up to 2 million deg. F, much
hotter than the surface beneath. Why these gases get so hot has been a mystery for decades;
SOHO's magnetic observations may help solve this mystery. So the short answer is: the Sun is
http://sohowww.nascom.nasa.gov/explore/faq/basics.html (2 of 5) [2000/01/11 11:57:03 AM]
Dr. SOHO's FAQ: The Basics
27,000,000 deg. F in the center and 10,000 deg. F on the surface.
Go back to "Basics" questions and answers.
●
Why does the Sun shine?
Wow, I could write a book to answer this question (and some people already have)! But, I guess
you don't want an entire book-length response, so I'll try to answer it succinctly.
The Sun is basically a great big ball of gas, mostly made up of hydrogen, the lightest element in
the known universe. Near the center of the Sun, the weight of the overlying gas increases the
pressure so much that nuclear "burning," or fusion, of hydrogen takes place, giving off a
tremendous amount of energy. The temperature at the center of the Sun is about 15 million degrees
Celsius!
The energy released in this way makes its way outwards towards the surface in the form of
radiation (light) and gas motions (just as when you boil a pot of water on your stove, the hot water
starts to bubble upwards). By the time the energy gets out to the surface of the Sun, it is spread out
over a very large area, so the temperature of any particular location is much lower than it is at the
center. In fact, the surface of the Sun is at a temperature of "only" about 5,800 degrees Kelvin.
An object at this temperature glows with a golden yellow color. Think of holding a metal bar in a
hot fire: it will start to glow red as it heats up, and gradually it will turn yellow, then white as it
gets hotter. The Sun's surface is just at the correct temperature to appear yellow in color. So this is
why the Sun shines as it does.
I've condensed the results of many years of research and understanding into the above paragraphs.
It is really amazing that we can understand in great detail what goes on inside a star almost 100
million miles away, not to mention much more distant stars and galaxies. The results now coming
in from SOHO are helping us to understand even better how our Sun works, and how its output
affects us here on Earth and sustains our life.
Go back to "Basics" questions and answers.
●
How does the Sun make herself hot?
The Sun is heated by a process called nuclear fusion. Deep inside the Sun, particles combine to
produce heavier particles and release energy. This process makes a LOT of energy. It happens
because the Sun is huge and there is a lot of pressure at the center. Fusion does not normally
happen on the Earth. See this website:
http://w ww.astro.uva.nl/~michielb/sun/home.htm
(The Virtual Sun: Its Core)
Go back to "Basics" questions and answers.
●
When will the Sun become a red giant?
Current astrophysical theory predicts that the Sun will become a red giant in about five billion
http://sohowww.nascom.nasa.gov/explore/faq/basics.html (3 of 5) [2000/01/11 11:57:03 AM]
Dr. SOHO's FAQ: The Basics
(5,000,000,000) years.
We think that stars like the Sun shine for nine to ten billion years. How old is the Sun? It is about
4.5 billion years old, judging by the age of the Moon rocks. The red giant phase will occupy most
of the last few hundred million years of the Sun's lifetime.
Go back to "Basics" questions and answers.
What color is the Sun? Is the predominant radiation from that body in fact "green," as I see on the
SOHO pages?
●
Good question. Even scientists can debate the answer to this one. Two leading answers are
"yellowish-white" and "green." Our eyes perceive sunlight as being yellow or white -- but DO
NOT LOOK AT the Sun to confirm this!
It is also correct to say it is "green," too, because the Sun's peak wavelength (500 nanometers, or
5000 Angstroms) is in the green part of the visible spectrum. Watch this site for further resources
regarding this debate. Further discussion can be found at the Stanford SOLAR Center:
http://solar-center.stanford.edu/FAQ/Qsuncolor.html
(Ask a Solar Physicist: Why does the Sun appear orange?)
Two types of images are displayed on our web pages (and throughout the SOHO community) in
"green." The C1 FeXIV coronagraph images are taken through a FeXIV filter in the neighborhood
of 5302 Angstroms (530.2 nanometers, or well within the visible range). Because it is in the
"green" part of the spectrum, the FeXIV solar emission line is traditionally known as the "green
line." Hence, we show it in a green color table.
The 195 Angstrom EIT images are one of four wavelengths which EIT sees. The "E" in "EIT"
means extreme ultraviolet. The human eye cannot see any radiation below about 4000 Angstroms,
so we humans cannot see these EIT wavelengths. The EIT team assigned a unique color table to
each of these wavelengths early in the program, but there is no intended correlation to actual
"colors." (I asked that very question myself when I signed on to this project.)
Go back to "Basics" questions and answers.
●
How far away from the Earth is the Sun? How far away is SOHO?
The Earth is about 93 million miles, or 150 million kilometers, from the Sun. This distance varies
slightly throughout the year, because the Earth's orbit is an ellipse, not a perfect circle. (The Earth's
seasons are caused by the tilt of the planet, not the varying distance to the Sun. Many people do
not realize this yet.)
SOHO is between the Earth and the Sun, 92 million miles from the latter and only about 1 million
miles from the former. SOHO isn't that much closer to the Sun, so the spacecraft is in no danger of
"burning up!"
Go back to "Basics" questions and answers.
●
What is the "solar cycle?" What is a "quiet sun?"
http://sohowww.nascom.nasa.gov/explore/faq/basics.html (4 of 5) [2000/01/11 11:57:03 AM]
Dr. SOHO's FAQ: The Basics
The Sun, our star, is not simply a quiet ball of nuclear fire out in space. It bubbles and seethes with
electromagnetic activity. Its surface gets speckled with sunspots and active regions. It occasionally
spits out clouds of plasma and energetic particles.
All of this activity follows an 11-year cycle. Every 11 years, the number of sunspots, flares, and
solar storms increases to a peak. Then, after a few years of high activity, the Sun will ramp down
to a few years of low activity. This pattern is called the "sunspot cycle," the "solar cycle," or the
"activity cycle." We have known about the cycle since the 19th century, because it was seen in
sunspot records.
The Sun spends a few years at the peak of its cycle. This is known as "solar maximum." The few
years during the less-active part of the cycle are called "solar minimum," or "quiet sun." Big solar
flares and CMEs are more likely during solar maximum, but they can occur at any time.
The phrase "quiet sun" can also refer to a magnetically "quiet" part of the Sun's surface. SOHO's
spectrographic teams often talk about observing "quiet sun" if their targets are free from filaments
or active regions.
Go back to "Basics" questions and answers.
Go back to "Dr. SOHO's FAQ."
SEND US YOUR COMMENTS
Go To
Other SOHO Web Pages
Author: Shane Stezelberger
Co-Author/Curator: Therese A. Kucera
[email protected]
Responsible Official: Art Poland([email protected])
Last modification: Tuesday, 13-Apr-1999 17:33:09 EDT
http://sohowww.nascom.nasa.gov/explore/faq/basics.html (5 of 5) [2000/01/11 11:57:03 AM]
Dr. SOHO's FAQ: Questions about the Sun
Go back to "Dr SOHO's FAQ"
Go back to the "SOHO" main page.
●
Is the Sun the brightest star?
●
What is the chemical composition of the Sun?
●
What is the surface of the Sun like?
●
How do you figure out the temperature of the Sun?
●
What is the coldest part of the Sun?
●
Does the Sun weigh as much as the Earth?
●
What is the Sun's variation like, over 1 calendar year?
●
What kind of particles is the solar wind made of?
●
Is it possible to mimic the solar wind experimentally?
●
Could the solar wind's power be harnessed and used someday?
●
I heard on the news that the Sun has a recurring event every six minutes. Can you explain?
●
How close can you get to the Sun without burning up?
●
Why is the Sun darker now than it was 20 years ago?
●
Where can I find information on visible-spectrum solar variations?
●
With all these CMEs happening, do we know how much mass the Sun is losing?
●
Will SOHO help to answer the missing-neutrino mystery?
●
Please describe all of the forces at work inside the Sun.
●
How do scientists know that the Sun has a core?
●
How do we know the density of the Sun's core?
●
I heard there are observations suggesting the Sun has no core. Does SOHO have any evidence of
this?
●
Is the Sun solid?
http://sohowww.nascom.nasa.gov/explore/faq/sun.html (1 of 34) [2000/01/11 11:57:11 AM]
Dr. SOHO's FAQ: Questions about the Sun
●
Where can I find daily sunspot numbers?
●
Is there anything unusual or ominous about the rising level of solar activity?
●
How fast is the Sun's corona expanding?
●
How can hydrogen produce Lyman-Alpha emissions at such high temperatures?
●
What are the causes of the influence of the Sun's magnetic waves?
●
Is the solar wind continuous between the Sun and the Earth?
●
Is the Sun expanding or contracting, and at what rate?
●
Where can I find solar data pertinent to Amateur Radio interference?
●
How many years until the Sun explodes or disappears?
●
How does pollution and ozone affect the Sun?
●
How do the Sun's magnetic and rotational axes relate to the Earth's rotation axis and the ecliptic?
Do the Sun's magnetic poles ever shift?
●
What was the "August 1972 solar event?"
●
What is the difference in temperature between the Sun's core and surface?
●
What practical applications does the Sun's rotation have?
●
How did scientists calculate the size of the Sun?
●
What type of star is the Sun?
●
How do you measure the amount of hydrogen in the Sun?
●
What shape is the solar magnetic field?
●
What advice do you have for displaying solar activity in a public demonstration?
●
When is the next solar eclipse?
●
Do the corona and solar wind have a net charge?
●
Where can I find information about the solar "magnetic carpet?"
●
What is the difference between a filament and a prominence?
●
Will the coming solar cycle be catastrophically destructive?
●
Has there been a small increase in the solar constant?
●
How does the Sun's UV output vary with time?
●
How does energy escape from the Sun?
●
How fast can a signal get from the Sun's core to the surface?
●
Where can I find information about the density of the Sun's outer layers?
●
What holds the Sun up? How does it just sit there in space?
●
Is the Sun moving in space?
●
How can there be a "solar wind" in space?
http://sohowww.nascom.nasa.gov/explore/faq/sun.html (2 of 34) [2000/01/11 11:57:11 AM]
Dr. SOHO's FAQ: Questions about the Sun
●
●
●
Where can I view photos of these tornadoes on the Sun? And do you have charts of long-term solar
activity?
How do you measure the wind speed of those solar tornadoes?
Is the Sun the brightest star?
The answer to your question is: yes and no. The Sun is only an average star. Some stars in this
galaxy are brighter, and some are dimmer than the Sun. Some stars are smaller than the Sun, others
are much larger.
However, the Sun is by far the closest star to the Earth. Therefore, it looks bigger and brighter in
our sky than any other star.
Still, everything is relative. The next closest star to the Earth is Proxima Centauri, about 4.5
lightyears away. It has a companion, Alpha Centauri, which is one of the brightest stars in the
southern sky. (You cannot see it from most of North America.) I read once that if you were sixty
light years away from our Sun, it would be too dim to see with the naked eye!
Go back to "Sun" questions and answers.
●
What is the chemical composition of the Sun? (if you can, list the elements and percentages, etc.)
The Sun is mostly made up of hydrogen (about 90%), with about 9% of helium and only about 1%
of heavier elements (mainly carbon, nitrogen, oxygen, neon, magnesium, silicon and iron). The
abundances of the different elements vary quite a bit among the different structures in the solar
atmosphere, but these numbers are a good, overall average.
The Sun is constantly fusing hydrogen into helium in its hot, dense core. The heavier elements,
like iron, were produced billions of years ago in some huge, long-dead star. These heavy atoms -including every atom of metal on the planet Earth -- are "left over," we think, from the nebula
which originally formed the Sun and solar system.
Go back to "Sun" questions and answers.
What is the surface of the Sun like? Is it solid, soupy, or just gas that (assuming you didn't burn up)
you would pass right through?
●
The surface of the Sun is a very tenuous gas, mostly made up of hydrogen. The temperature at the
surface is about 6,000 degrees Celsius, and the density is about 1017 atoms per cubic centimeter
(1017 is scientific notation for a 1 followed by 17 zeros).
This is about one ten-thousandth of the atmospheric density at the Earth's surface. Writing the
density another way, it's about 2 x 10-7 grams per cubic centimeter (or 0.0000002 grams per cubic
centimeter), which is over a million times less dense than water.
So, as you suggest, you would pass right through the surface, but would be likely to burn up before
you got there!
http://sohowww.nascom.nasa.gov/explore/faq/sun.html (3 of 34) [2000/01/11 11:57:11 AM]
Dr. SOHO's FAQ: Questions about the Sun
Go back to "Sun" questions and answers.
My son and a few of his 7 year old friends were wondering about the temperature of the Sun.
Specifically, how did you figure out the temperature of the Sun? How do you measure the temperature
of the Sun, or is it just an estimate? Thank you for your help.
●
The way we tell the Sun's temperature on the surface (the "photosphere") is by looking at the light
it emits. Hot things emit light. Burners on an electric stove will turn red when they get hot. Molten
steel (which is even hotter) can get white hot. The hotter something is, the "blue-er" it is. (In fact
very hot things start glowing in the ultraviolet and x-rays. People glow in the infrared - colors
"redder" than red. I don't mean the colors we reflect, like green if we are wearing a green shirt. I
mean the "light" we are actually producing ourselves because we are warm. This might be beyond
a 7 year old, though.)
Basically we can tell the temperature of the Sun's surface (about 5,600 Celsius) because of its
yellow-white color. Red stars are cooler, blue stars are hotter. We can measure it pretty accurately.
The Sun is actually different temperatures in different places (its outer atmosphere is hotter than its
photosphere and glows in ultraviolet and x-rays). Furthermore, we have some idea of how hot the
star is inside from physical models we make of the Sun, but that is more complicated.
There is a lesson about this called "How Hot is that Star?" at:
http://world.smv.mus.va.us/~jims/unit.htm
(Stellar Temperatures: How Hot Is That Star?)
It is for grades 7-12, but maybe you and your son would be interested.
Also, SOHO's EIT telescope can combine images in two different wavelengths to produce a
"temperature ratio" picture:
http://umbra.nascom.nasa.gov/eit/#RATIOS
(EIT maps of the temperature-sensitive ratio)
The darkest areas are just under one million degrees Celsius, and the brightest regions are about
1.5 million degrees.
Go back to "Sun" questions and answers.
●
Where is the coldest part of the Sun?
The coldest part of the Sun is the Sun's visible surface (the part we see with our eyes) which is
"only" about 5800 K (10,000 F). Below the surface, the Sun gets hotter and hotter until the core where it is about 15 million K. Above the visible surface is a blanket of hot gases called the
Corona, which can be over 2 million K. On the visible surface, there are small regions called
sunspots which are up to a 2000 K cooler than the surrounding surface. Therefore, they are really
the "coldest" part of the Sun.
http://sohowww.nascom.nasa.gov/explore/faq/sun.html (4 of 34) [2000/01/11 11:57:11 AM]
Dr. SOHO's FAQ: Questions about the Sun
Go back to "Sun" questions and answers.
●
Does the Sun weigh as much as the Earth?
The Sun "weighs" as much as 332,950 Earths!!!!!
("Weight" varies with the local gravity, and the Sun isn't sitting near anybody's gravitational field.
"Mass" is constant, however. It's correct to say that the Sun has 332,950 times more material than
the Earth.)
Go back to "Sun" questions and answers.
What is the typical variation of the Sun's output during the calendar year? Is this variation
noticeable on Earth? How about in space? How drastic are changes in the Sun's corona?
●
The Sun's output changes very little in one Earth year. Seasonal changes on Earth over the course
of a year are due to the tilt of the Earth's axis, which exposes different parts of the globe to the Sun
for different amounts of time each day. (Many people do not realize this yet.)
The Sun's output DOES change during the course of the solar cycle, which takes about 11 years.
Over the solar cycle, the total energy output of the Sun changes about 0.1 percent. This variation
was first measured by a NASA satellite called "Solar Max."
High-energy radiation from the Sun (ultraviolet and X-rays) varies much more. Astronomers have
been studying this radiation using spacecraft since the 1960s. The Sun's corona is very hot, so it
gives off ultraviolet and X-ray light. It changes greatly with the 11-year cycle. There is a nice
image showing this at:
http://www.lmsal.com/YPOP/Spotlight/Tour/tour07.html
(Surfing for Sunbeams: The Solar Cycle)
It shows different images of the Sun taken in X-rays by the "Yohkoh" satellite from 1991 (Solar
maximum) to 1995 (Solar minimum).
We cannot observe the Sun's ultraviolet light and X-rays from Earth's surface, because our
atmosphere blocks them. That's one reason we send up spacecraft like SOHO and Yohkoh.
Light from the Sun certainly interacts with Earth's atmosphere and surface. There are some
indications that changes in solar radiation may affect climate on the Earth. However, it is not yet
known exactly how or why. If you want a more detailed, technical discussion, there is an article
about this in Reviews of Geophysics at:
http://earth.agu.org/revgeophys/reid00/reid00.html
(The Sun-Climate Question: Is There a Real Connection?)
Go back to "Sun" questions and answers.
●
What kind of particles is the solar wind made of?
http://sohowww.nascom.nasa.gov/explore/faq/sun.html (5 of 34) [2000/01/11 11:57:12 AM]
Dr. SOHO's FAQ: Questions about the Sun
The solar wind is a stream of 1 million degree (Celsius) charged particles, mainly ionized
hydrogen (electrons and protons) which flows out of the Sun at hundreds of kilometers per second
or about 900,000 miles per hour! A good place to do some quick on-line reading about the solar
wind is:
http://umbra.nascom.nasa.gov/spartan/the_solar_wind.html
(SPARTAN 201-3: The Solar Wind)
which also has many links, including a Goddard Education page:
http://lepmp.gsfc.nasa.gov/EDucation/wsolarwind.html
(NASA Goddard Lab for Extraterrestrial Physics: Solar Wind Pages)
Go back to "Sun" questions and answers.
●
Is it possible to mimic solar wind experimentally?
In terms of mimicking the solar wind experimentally, you would need a fairly good vacuum and a
very hot source of plasma. These conditions exist in particle accelerators, like Fermilab and
CERN.
Still, such accelerators are not used to deliberately simulate the solar wind. And if you look at the
data from SOHO's sister ship, ACE:
http://www.gsfc.nasa.gov/ace/ace.html
(Explorer 71: The Advanced Composition Explorer)
you will see that the solar wind is a specific kind of plasma. There are various metallic ions
entrained in it, for example. (That's because there were metal atoms, like iron and magnesium, in
the giant gas cloud which originally formed the Sun 4.5 billion years ago. The Sun is too small to
produce iron on its own, and in any case you do not get metals out of a stellar core until it
explodes. The ACE science team would then have little left to study... besides, Old Sol is too small
to "go nova.")
Go back to "Sun" questions and answers.
●
Could the power of the solar wind be harnessed and used somehow in the future?
Hmmm. I do not believe I have ever seen any studies or technical proposals for exploiting the
power of the solar wind directly. Space-based power systems have been studied fairly heavily;
read O'Neill's The High Frontier or do a web search on the keyword phrases "solar power satellite"
and "lunar power."
I'm not sure we can "harness the power" of the solar wind (it is merely a tenuous plasma), but it
does have a strong effect on the Earth. In particular, if you look at these web pages you will see
that the solar wind causes the aurora, induces ground currents which can knock out power utilities,
and adversely affects satellites.
http://sohowww.nascom.nasa.gov/explore/faq/sun.html (6 of 34) [2000/01/11 11:57:12 AM]
Dr. SOHO's FAQ: Questions about the Sun
These symptoms are all a result of the solar wind's interaction with the Earth's magnetic field. If
Earth lacked such a field, we probably wouldn't see any aurorae -- and we would have many other
problems, too! The next question is, can we harness the power of the Earth's magnetic field? Yes,
quite easily! But it can only be done well from orbit. Do you remember NASA's Tethered Satellite
experiment? That system generated a voltage. Every time you move a metal rod through a
magnetic field, you will induce an electrical current. (It's not "free energy," because energy can
neither be created nor destroyed. It is subtracted from the strength of Earth's magnetic field and -- I
think -- the speed of its rotation.)
Go back to "Sun" questions and answers.
I saw on the news that the Sun has a recurring event every 6 minutes but we don't know what the
event is. Can you elaborate?
●
I didn't see the newscast you mentioned, so I am not sure which of the many phenomena on the
Sun they were referring to by a "recurring event?" If you can remember any more details, I can try
to answer your question. I can present a few candidates:
The Sun's surface is covered by continent-sized light and dark areas, called "granules." They are
the tops of convection cells; they dissipate and re-form every 5 minutes or so. However, these
granules have become fairly well-understood in the 30-odd years since their discovery.
The Sun's surface also bears many types of magnetic activity, among them the newly discovered
"magnetic carpet." Loops of magnetic force cover the surface, completely recirculating every 40
hours. This may be the long-mysterious energy source which heats the solar corona:
http://soi.stanford.edu/press/ssu11-97/
(Magnetic Carpet Press Release)
Also, some SOHO scientists have possibly discovered waves in a part of the solar atmosphere.
These waves have a period of 11 minutes:
http://mdiwww.nascom.nasa.gov/~zowie/papers/Quasi-periodic
(Quasi-Periodic Waves)
Go back to "Sun" questions and answers.
●
How close can you get to the Sun without burning up?
You can, of course, burn up right here on Earth if you stay outside in the wrong place for too long
without adequate protection: try sun-bathing in the Grand Canyon for a day!
The key point here is that you must have adequate protection. The Earth's atmosphere protects us
from most of the damaging ultraviolet or UV radiation from the Sun. Outside the atmosphere, the
UV dose would be lethal if you did not wear a protective spacesuit and face-mask which blocked
out the UV. This is one of the reasons that astronauts wear those bulky suits (another reason is that
they need pressure and a supply of oxygen to breathe). So really, you don't have to go more than a
http://sohowww.nascom.nasa.gov/explore/faq/sun.html (7 of 34) [2000/01/11 11:57:12 AM]
Dr. SOHO's FAQ: Questions about the Sun
few hundred miles above the Earth's surface before you would "burn up" without this sort of
protection. Remember that, in the vacuum of space, things cannot "burn" in the usual sense.
One way to answer the question is to look at a presently proposed NASA mission called Solar
Probe. Here is an excerpt from the mission description:
"The Solar Probe, a unique high-technology dual spacecraft mission to be implemented in
cooperation with the Russians, will venture deep into the solar corona, the Sun's outer
atmosphere -- far closer to the Sun than any other spacecraft has previously ventured. It will
demonstrate technologies for imaging, and taking in-situ and remote measurements at 3
solar radii above solar surface, where radiation temperatures exceed 2000 K."
See also:
http://umbra.nascom.nasa.gov/spd/solar_probe.html
(Solar Probe Page at SDAC)
http://www.jpl.nasa.gov/ice_fire/sprobe.htm
(Solar Probe Overview at JPL)
(In case you are wondering, the current record-holder is the US/German space probe Helios B,
launched on 15 January 1976. This second Helios probe passed within 45 million kilometers
(about 28 million miles) of the Sun. Helios spun once per second to distribute the incoming solar
radiation, and the spacecraft's mirror-coated skin still reached a temperature of 370 degrees
Celsius. Source: The Observer's Spaceflight Directory, Reginald Turnill, F. Warne Ltd, London,
1978.)
So you see, we are ready to build a spacecraft capable of going to within 3 Solar radii (or about 1.2
million miles) -- or we soon will be. But this is for an unmanned mission. The question you ask is
for a manned mission. The main difference here would be the high energy radiation that goes
through the surface of the ship. I would presume, one could construct the ship to allow a person to
survive, given of course a sufficient budget!
Actually, the outer layer of the Sun's atmosphere, where the Solar Probe will be traveling, has a
plasma temperature of millions of degrees. However, the material there is so rarefied (like a
vacuum) that the temperatures don't have the same effect as the denser material we are used to.
The surface of the Sun you see with your eye is called the photosphere, and it has a temperature
over 5,000 C. To get that close you would need to construct your ship out of a material that does
not melt at these temperatures, something like a carbon fiber/titanium composite. Even then, this
would be nearly impossible with today's technology.
Go back to "Sun" questions and answers.
I was wondering, why is the Sun darker than it was 20 years ago? What else can we expect from the
Sun in the near future?
●
The Sun's overall brightness and size don't change significantly in 20 years. The Sun is not
brighter or darker (in visible light) than it was 20 years ago.
http://sohowww.nascom.nasa.gov/explore/faq/sun.html (8 of 34) [2000/01/11 11:57:12 AM]
Dr. SOHO's FAQ: Questions about the Sun
We cannot say that the Sun's output doesn't change at all -- there is the sunspot/activity cycle, of
course, described below. We know from the "Solar Max" satellite that the Sun's brightness changes
by only 0.1% between the minimum and maximum of a cycle. We do not have enough data to see
any longer-term changes, from cycle to cycle. But the Sun hasn't changed drastically in recent
decades.
Perhaps you are looking at plots of the number of solar flares or sunspots? The number of flares
(bright explosions which occur in the Sun's atmosphere) goes up and down in an 11 year cycle of
changes in the Sun's magnetic field. We do not know exactly why, but we think it has to do with
motions of the ionized gas (or "plasma") beneath the Sun's surface. The number of flares at the
peak of each cycle changes from cycle to cycle, and we don't know why. Right now (late 1996),
we are near the minimum part of the cycle, and there are very few flares. In about 5 years we will
be at the peak of the solar activity cycle and we can expect more sunspots, more flares, and more
coronal mass ejections (events in which the Sun shoots out huge bubbles of magnetized gas).
If you would like a reasonably solid, pre-SOHO book about the controversies of solar variability,
try John Gribbin's Blinded By the Light: The Secret Life of the Sun (Harmony Books, 1991). Be
warned, however, that this book's speculations on dark matter are probably out of date -- to say
nothing of helioseismology.
Go back to "Sun" questions and answers.
Is information available detailing solar intensity variations in the visible spectrum on various time
scales? If so, over what dynamic range, to what accuracy, and is a standard time tick available for
correlation purposes?
●
The Sun can be described as a variable star. (Many astronomers may quibble with this statement.
Our Sun does not dramatically change its brightness, but it does dramatically change its magnetic
activity.)
The study of solar intensity/irradiance variability is actually a very active and ongoing area of
research. There is a whole field of science called "space weather." This concerns the effects of the
Sun on the Earth, the Solar-Terrestrial Connection. This is even the focus of a whole program of
NASA spacecraft, as we still do not understand all the reasons for the variability nor its real effect
on the Earth.
The total solar irradiance varies less than 3% over the 11 year solar activity cycle. The irradiance is
dominated by the visible spectrum, however other parts of the spectrum such as the ultraviolet can
vary by much larger factors. The accuracy of the measurements depends upon the exact
wavelength range, in the visible it is typically better than 0.1%. In one form or another, the Sun's
variability has been measured for hundreds of years, ranging from sunspot records to modern
satellite observations. Today, measurements are being made every day (or even every few
minutes) from a number of ground- based and space-based instruments.
A good place to start looking into this area is:
http://sohowww.nascom.nasa.gov/explore/faq/sun.html (9 of 34) [2000/01/11 11:57:12 AM]
Dr. SOHO's FAQ: Questions about the Sun
http://espsun.space.swri.edu/spacephysics/www.space.html
(WWW Space Physics Resources)
This is a nicely organized pointer to a wide variety of other web pages concerning this topic. Pages
range from databases to experiments to literature to software and much, much more.
An interesting book is "The Solar Influences on Global Change," a National Research Council
report from 1994 (ISBN 0-309-05148-7). If you are interested in a more scientific analysis you
could look up: Reviews of Geophysics 29, 4 pg 505. (November 1991) by Judith Lean, which is an
excellent article.
Go back to "Sun" questions and answers.
Given that the Sun is tossing out matter in eruption events and converting mass to energy in fusion,
is there any data on the net change in mass of the Sun?
●
I know of no data that directly measure the change in mass of the Sun as a result of eruptions and
conversion to energy. Those changes are necessarily slight: even the dense coronal mass ejections
qualify as pretty darned good vacuum, with something like 100 billion particles (1011) per cubic
centimeter. (At 1 atmosphere, you have something 3 x 1019 air molecules per cubic centimeter
around you; so the CME last week was 1/100,000,000th as dense as your environment.) The
LASCO scientists do have methods for estimating the mass of a given CME.
Ultimately, all the CMEs throughout the entire lifetime of the Sun will not subtract much mass at
all. And a star like the Sun cannot fuse all of its hydrogen into helium, so fusion will not consume
the Sun, either. (Fusion can only occur in a dense, hot stellar core. When the core runs out of
usable hydrogen, it fuses the helium, and so forth, until it can fuse no heavier elements. That is
when the fun begins. However, there may be discrepancies in our solar models, which ties into the
very real "missing solar neutrino" enigma.)
Go back to "Sun" questions and answers.
Have there been any data collected by SOHO which help explain the discrepancy in solar neutrino
flux?
●
For those of you just tuning in, a "neutrino" is a by-product of basic nuclear reactions. A big fusion
reactor like the Sun should be emitting vast amounts of neutrinos all the time. Unfortunately,
neutrinos are difficult to detect. Even more unfortunately, the first neutrino detectors (in the 1960s)
reported only about 30% of the expected neutrino flux. That result has been duplicated many times
with different experiments in the decades since.
Concerning this solar neutrino problem, yes and no: data have been collected by SOHO (notably
by the helioseismology instruments: MDI, GOLF, and VIRGO) that could help explain the
discrepancy between predicted and observed neutrino fluxes. The main difficulty now is that the
existing data (before the launch of SOHO in 1995) are not sufficient to distinguish between
problems with the "standard stellar model" (which is used to predict the type of fusion that's
occuring) and problems with our "standard particle model" (which predicts how elementary
http://sohowww.nascom.nasa.gov/explore/faq/sun.html (10 of 34) [2000/01/11 11:57:12 AM]
Dr. SOHO's FAQ: Questions about the Sun
particles should interact).
The problem is that predicted count rates for real-life neutrino detectors here at Earth depend not
just on the amount of energy released by fusion -- and hence the total number of neutrinos
produced in the Sun's core -- but also on the exact process by which the fusion comes about, which
affects the type of neutrinos that are produced and the amount of kinetic energy carried by each
neutrino.
GOLF and VIRGO and, to a lesser extent, MDI are designed to detect the natural resonant modes
of the Sun with exquisite accuracy, so that we can use slight variations in those resonances to
determine the inner structure of the Sun. This is like figuring out the composition and shape of a
bell or a wind chime by carefully studying the set of musical tones that come out of it when it is
struck. The helioseismology data from those three instruments will allow us to make a three
dimensional "map" of the density, temperature, and motion down to the Sun's inner core, where the
fusion is presumably taking place. That, in turn, will allow us to constrain more tightly the existing
pictures of fusion in the center of a star. We hope that we'll be able either to pin down an error in
the standard solar model or to point to an inconsistency in elementary particle theory.
Those data, which consist of a set of extremely precisely measured frequencies of oscillation, are
neither quick to collect nor easy to analyze: it will be another year or more (mid-1998) before we
see definitive answers about the solar core from any of the helioseismology experiments.
(Meanwhile, very interesting results are coming out about the outer layers of the Sun -- but this is
enough for one response!)
Go back to "Sun" questions and answers.
I am curious about the known forces acting within the sun. I am aware of gravity, which is a
function of its mass. I know a little about the energy released with nuclear fusion, which is where the
energy comes from in the Sun (I believe). Of course, there are electromagnetic forces which are
evident in the aurora borealis; I do not know where this plays in the overall picture of the Sun. Are
these the forces acting in the Sun? Do we know about any other forces which have some effect on the
Sun? Gravity is the one that interests me because it is both the weakest and strongest of forces as I
understand it. I would very much appreciate your comments on this topic.
●
You ask an interesting and complicated question. Let us consider the question of forces from the
inside out. Inside most of the Sun, there are only two important forces: pressure and gravity.
You're right that gravity is the weakest of all known forces. It does, however, have a couple of
things going for it. First of all, it falls off with distance much slower than the nuclear forces.
Within an atomic nucleus, the nuclear forces dominate, but outside the nucleus the forces which
have a 1/R2 dependence, gravity and electromagnetism, dominate. Gravity has an additional
advantage over electromagnetism in that it is always additive -- there's no such thing as negative
mass. The Sun as a whole is electrically neutral, so that electrical forces cancel out.
There are two kinds of pressures which is important within the Sun -- gas pressure and radiation
pressure. All the heat and light in the Sun is generated in the core from nuclear fusion, and then
slowly leaks out towards the surface. The gas pressure comes from the heat, just like on Earth.
http://sohowww.nascom.nasa.gov/explore/faq/sun.html (11 of 34) [2000/01/11 11:57:12 AM]
Dr. SOHO's FAQ: Questions about the Sun
However, on the Sun, there is so much light that it also generates a pressure. As the temperature
changes from the core to the surface of the Sun, the degree of transparency of the solar material
changes so that radiation pressure is more important in some layers than others. In layers where
radiation pressure is greatest, the Sun tends to bubble like a boiling kettle, in a process known as
convection.
In the cores of some stars, densities reach so high that nuclear forces start to become important.
This force generates another kind of pressure, known as degeneracy pressure, which is related to
the Pauli exclusion principle. This force is only important in very old stars -- much older than the
Sun. White dwarf stars are held up almost entirely by degeneracy pressure.
Above the surface of the Sun, the density falls off dramatically, and pressure becomes much less
important. In this region, known as the solar atmosphere, the Sun's magnetic field not only
becomes important, but even dominates. All the structure in the solar atmosphere (prominences,
streamers, and the like) are created by the interactions between the magnetic field and the
ever-present gravitational field. Solar flares and events like the recent coronal mass ejection (07
April 1997) are created by the release of energy embedded within the magnetic field.
Go back to "Sun" questions and answers.
●
How do scientists know that the Sun has a core?
Good question! Obviously, nobody can see the center of the Sun. We've never sent a space probe
into the Sun, either. But we think we know what it's like in there.
Basically, the only thing that provides enough energy to heat a star for billions of years is nuclear
fusion. We know the pressures and temperatures needed for this to take place, so we can figure out
from that what the center of the Sun (the "core") must be like.
This sounds simple, but it wasn't until 1938 that somebody connected nuclear fusion with stellar
evolution. That person was Hans Bethe, and he won a Nobel Prize for his efforts. And there are
still some gaps in our understanding of stellar fusion; we still don't know where all the neutrinos
are, for example.
Go back to "Sun" questions and answers.
Dear Doc, Do we know that the Sun has a dense core (13 to 14 times the density of lead), and if so,
how do we know?
●
By careful measurement of the orbital motions of the Sun and all its planets, along with an
independently measured value for Newton's universal gravitational constant, we can fairly
straightforwardly determine the mass of the Sun, and it turns out to be about 2.0 × 1033 grams (that
is: 2 times ten to the thirty-third power!) But you asked about the density at the core, and that takes
a bit more calculation....
Actually, the average density of the Sun is not very high. If you measure the radius (R=700,000
kilometers), and from that compute the volume, using V = 4/3 × pi × R3 (four-thirds pi R-cubed)
http://sohowww.nascom.nasa.gov/explore/faq/sun.html (12 of 34) [2000/01/11 11:57:12 AM]
Dr. SOHO's FAQ: Questions about the Sun
you find the volume is also huge, about 1.4 × 1033. Then the density is just the mass divided by the
volume, or about 1.43 grams per cubic centimeter -- just 43 percent higher than the density of
water! -- on AVERAGE. We know how much gas (mostly hydrogen) is there, and we can measure
its diameter fairly accurately.
But the Sun, of course, is a gaseous blob, just held together by its own self-gravitation. Every part
of it that is not at the very center is pushing down on every other part, and all of it squeezing down
that central bit. The result is that the density in the outer parts is quite low, but near the center is
extremely high.
We cannot (so far) measure the central density directly; we have to model it, understanding that
the gravitational compression is to some degree mitigated by the outward pressure of heat energy
generated in the nuclear fusion reactions burning always at the core. Such computational models
predict, for example, that about 10% of the Sun's mass is contained within a sphere having just one
tenth of the Sun's radius -- which means one one-thousandth of the Sun's volume. Thus, the density
in this central region would be about 0.1/0.001, or one hundred times the average, or 140 grams
per cubic centimeter. The density of lead is 11.35 g/cc, so there you have it! Best regards!
Go back to "Sun" questions and answers.
Sirs: I have heard that observational evidence is questioning the accepted theory that the Sun has a
core. Is this in evidence through SOHO?
●
I am not aware of any observational evidence suggesting that the Sun does not have a core, and I
would be VERY surprised if any such evidence were found.
We have a pretty good understanding of the interior structure of the Sun (and, by inference, other
stars of similar type), gained using techniques of helioseismology. These techniques are similar to
those used in seismology of the Earth, in which small motions of the surface are measured and
used to deduce the interior structure. The solar observations are much more difficult, because we
cannot place an experiment on the surface of the Sun, so all measurements have to be done
remotely.
One experiment of this type on SOHO, the Michelson Doppler Imager (MDI), has recently made
some new discoveries about the Sun's internal structure, and I wonder if this is what you have
heard about. For a long time, solar physicists have believed that the Sun's magnetic field is
generated by a 'dynamo' process, an interplay between convection of hot gases in the interior and
rotation of the Sun, which can cause an initially small magnetic field to build up to significant
strengths. However, we have never been able to tell where in the interior this dynamo action takes
place.
Recent MDI results place this action in a relatively narrow layer 38,000 miles thick and centered at
a depth of about 135,000 miles below the visible surface of the Sun. For more information on this
result (and others from MDI), look at their web page:
http://soi.stanford.edu/
(The Solar Oscillations Investigation -- and select "SOI/MDI Results")
http://sohowww.nascom.nasa.gov/explore/faq/sun.html (13 of 34) [2000/01/11 11:57:12 AM]
Dr. SOHO's FAQ: Questions about the Sun
Go back to "Sun" questions and answers.
●
Is the Sun solid?
The Sun is not a solid. It is actually a type of matter you may not have heard about before: a
plasma. Plasmas are so hot that atoms come apart into charged particles, mostly electrons and
protons in the Sun's case. This plasma is tenuous and gaseous near the surface, but it gets denser
down towards the Sun's fusion core. (Stars need high densities and pressures to ignite nuclear
fusion. Fusion in the core releases the energy which makes a star shine.)
Go back to "Sun" questions and answers.
●
Please, how can I find daily sunspot index (Wolf numbers) at SOHO www-pages?
Under our "Sunspot" FAQ heading, we provide this link:
http://www.nwra-az.com/spawx/ssne24.html
(Space Weather: 24hr Effective Sunspot Number)
Other than that, we do not have such an index at the SOHO site. Please direct such requests to Mr.
Sunspot, at:
http://www.sunspot.noao.edu/PR/mr-sunspot.html
(Sacramento Peak: Mr. Sunspot)
Go back to "Sun" questions and answers.
Lately, the folks into conspiracies and paranoid, paranormal subjects have been talking about the
Sun's huge emissions of energy. I realize that if the Earth were hit by a massive particle ejection, as in
April 1997, there might be widespread damage to satellites and power systems. I am curious, though:
is our Sun behaving more erratically than usual? Was there a particle ejection in early November
1997 more severe than the April 1997 event? Is this above-mentioned activity something that we
endure every decade or so?
●
There is nothing unusual about what is happening on the Sun these days. We're fairly certain such
eruptions have happened for most of the lifetime of the Sun, and astronomers have known about
them for more than a century.
What IS new is:
1. We can observe these phenomena better than ever before, thanks to new space observatories and
spacecraft such as SOHO, ACE, and TRACE.
2. We have more satellites, larger power grids, smaller cellphones, greater reliance upon GPS and
such, so there is a greater chance of Sun-related problems with these technological things. Check
our links for "space weather" resources to learn more about such effects.
http://sohowww.nascom.nasa.gov/explore/faq/sun.html (14 of 34) [2000/01/11 11:57:12 AM]
Dr. SOHO's FAQ: Questions about the Sun
The Sun is currently moving towards a maximum of solar activity which we expect to occur
around the year 2000. This is part of its regular 11-year solar activity cycle. Again, this cycle is
nothing new.
There are a few different things that happen as a result of solar activity. One of these is called a
"Coronal Mass Ejection" (CME). These are huge bubbles of gas that blow out of the Sun.
Sometimes they interact with the Earth's magnetic field, and this can lead to problems with
satellites and power utility grids. This vulnerability did not appear overnight, either; a large
blackout in Quebec in 1989 was triggered by a large CME. Since then, electric utilities have paid
close attention to space weather.
Lists of most of the CMEs observed by SOHO so far can be found at:
http://lasco-www.nrl.navy.mil/cmelist.html
(LASCO CME Lists -- Based on Preliminary Data)
In addition, sometimes disturbances at the Sun send out sudden, large amounts of high-energy
particles. Such particles could be dangerous to astronauts, especially ones in interplanetary space.
There was a CME and solar flare on November 6, 1997 which put out a lot of these high energy
particles. This may be what you heard of. Again, the unusual thing about this is that we could
observe it so well.
Go back to "Sun" questions and answers.
Is there any measurement of the Sun's corona expansion rate? Let's say, how many kilometers or
miles per year the Sun grows.
●
In a way, you can say that the solar wind is the Sun's corona expanding into space. The solar wind
flows out of the Sun at 100's of kilometers per second. However, the Sun as we usually think of it
is not measurably expanding. We have never reliably measured any change in radius of the Sun's
visible surface.
You can see the speed of the solar wind flowing past the WIND and ACE spacecraft at:
http://space.rice.edu/ISTP/dials.html
(Current Solar Wind Conditions)
http://sec.noaa.gov/ace/ACErtsw_home.html
(ACE Real Time Solar Wind)
Generally, fast speeds (above the average 300-600 km/s) in the solar wind are associated with
CMEs, flares, or coronal holes on the surface of the Sun. At a brisk 1,000 km/s, such disturbances
can still take 2 to 3 days to cover the 150 million km void of space between the Sun and the Earth.
This is how SOHO can (and does) contribute to space weather studies.
Go back to "Sun" questions and answers.
Hi, Dr. SOHO. I have a question about Lyman Alpha emissions. From what I know, Lyman Alpha
has something to do with recombination of electrons in hydrogen atom (from n=2 to n=1). This means
●
http://sohowww.nascom.nasa.gov/explore/faq/sun.html (15 of 34) [2000/01/11 11:57:12 AM]
Dr. SOHO's FAQ: Questions about the Sun
that the hydrogen atom should still have the electrons orbiting around the nucleus. But the
chromospheric temperatures in the Sun are above 10,000 degrees Celsius. At this temperature, I
thought that electrons would be stripped away from the hydrogen atom. Can you tell why I am wrong?
What is the source of Lyman Alpha in the Sun???
You are correct that Lyman Alpha is the n=2 to n=1 transition of the neutral hydrogen atom. One
formation mechanism for Lyman Alpha is ionization and recombination. At the temperatures of
the solar chromosphere, a substantial fraction of the hydrogen atoms have their electrons stripped
away. Some of these electrons then recombine with hydrogen ions, and start to cascade down
through the levels until they reach the ground level.
However, even at 20,000 degrees, not all of the hydrogen atoms are ionized. This is because the
density is so low that collisions are not frequent enough to knock off the electrons from all the
atoms faster than they can recombine. Thus, there is always a population of neutral hydrogen,
which also contributes to the Lyman Alpha line.
Go back to "Sun" questions and answers.
●
What are the causes of the influence of the Sun's magnetic waves?
The `waves' that you see on the Sun in some of the beautiful SOHO movies are just a surface
response to an explosion that ejects large quantities of gas and magnetic field upwards and out
from the Sun. Think of an underwater explosion: you will get water forced upwards as a great
spray, but will also get a wave on the surface of the water that travels outwards from the explosion
site. The waves on the Sun are similar.
If the explosion (or `coronal mass ejection,' to use the solar physicists' term) happens to be directed
towards Earth, then we can get geomagnetic storms and spectacular auroral effects when it arrives
here, usually a few days later.
Go back to "Sun" questions and answers.
Is the solar wind continuous from here to the Sun? Is the solar wind always a plasma? How does
the ion concentration decay with the distance from the Sun? As R2?
●
In theory, the solar wind is expected to be continuous. Measurements have been made only at
isolated locations (for example, by SOHO and Ulysses). In practice, we expect discontinuities due
to waves and inhomogeneities. Yes, it is an ionized gas made of protons and electrons.
In theory (for constant mass flux), the ion density is expected to decay as 1/r2 -- if the velocity
remains constant. The velocity actually increases in the corona as the wind is heated and
accelerated, so the density may decrease faster than 1/r2 for a while. The velocity, which averages
400-700 kilometers per second in inter- planetary space, is probably not constant close to the
Earth, as the wind impacts the Earth's magnetic field. The density actually increases at these
boundaries.
Go back to "Sun" questions and answers.
http://sohowww.nascom.nasa.gov/explore/faq/sun.html (16 of 34) [2000/01/11 11:57:13 AM]
Dr. SOHO's FAQ: Questions about the Sun
●
Is the Sun expanding or contracting, and at what rate?
(For an overview of the controversy over solar diameter variations, I recommend John Gribbin's
pre-SOHO book, Blinded By the Light: The Secret Life of the Sun, published in 1991)
The Sun is not currently expanding or contracting to any measurable extent. It may in fact be
shrinking or swelling, but we have no hard data with which to conclude that.
It is vibrating, however:
http://helios.tuc.noao.edu/helio.html
(Global Oscillation Network Group)
http://soi.stanford.edu
(The Solar Oscillations Investigation)
In addition, we expect that the Sun will someday expand into a red giant phase, but that won't be
for another 4 billion to 5 billion years or so.
Go back to "Sun" questions and answers.
Holding an Amateur Radio License, I am interested in sunspots, but have difficulty in determining
which images are or are not sunspots that affect radio communications. Can you enlighten me?
●
Sunspots are most visible in white light images. The instrument aboard SOHO which takes such
images is MDI. For example, on the daily summary database page:
http://sohowww.nascom.nasa.gov/summary/
(SOHO Summary Database)
For a particular date, look at the MDI "Intensitygram". We also have available white light images
from ground-based observatories at:
http://sohowww.nascom.nasa.gov/synoptic/
(SOHO Synoptic Database)
For example, look at the Mees white light image on the same date.
It's not really the sunspots which affect radio communications, but rather the active regions with
which the sunspots are associated. Sunspots are the most visible manifestation from the ground of
active regions, but not all active regions have sunspots. From space, we can see the active regions
much more clearly as bright areas in ultraviolet light. Look at the X-ray images from the Japanese
Yohkoh satellite on SOHO's synoptic page, or any of the SOHO EIT images (particularly the
284A line) on the summary page.
One more site, particularly for Southern Hemisphere operations, is Australia's Ionospheric
Prediction Service at:
http://www.ips.gov.au
http://sohowww.nascom.nasa.gov/explore/faq/sun.html (17 of 34) [2000/01/11 11:57:13 AM]
Dr. SOHO's FAQ: Questions about the Sun
(Ionospheric Prediction Service)
Go back to "Sun" questions and answers.
●
How many years will it take before the Sun explodes or disappears?
Here is the short answer:
Four to five billion years (4-5,000,000,000 years)
The Long Answer:
The Sun will not explode; it is too small to "go nova." It WILL expand as it fuses the last of its
core hydrogen. The outer layers of gas will swallow some inner planets (possibly even the Earth).
Then the inner parts of the Sun will stop fusing, contract, and become a white dwarf. It will remain
a small, hot, slowly- cooling dwarf for many tens of billions of years after that. (It will have most
of the heat of a full-sized star, but only a tiny surface area with which to radiate it. Therefore, it
will take a very long time to cool down. This will not help us; it will be too cold to support life on
Earth by then, even if the Earth could survive the red giant phase.)
Scientists based this picture of the Sun's future on the basic nuclear reactions in a star's core. The
rates of these reactions depend upon the mass of the star. (smaller stars burn more slowly, and thus
longer, than more massive ones, but I could be oversimplifying things.) Scientists use planetary
orbits and the laws of orbital dynamics to estimate the mass of the Sun. So, with the mass of the
Sun and the rate at which this mass is consumed by fusion, we can guess that the Sun's lifetime
will be 9 billion or 10 billion years, total.
We know (from Earth's geology and Moon rocks, for example) that the age of the solar system is
4.5 or 4.6 billion years. We think the planets formed at roughly the same time as the Sun.
Therefore, based on current scientific theory, the Sun will expand in four or five billion years.
Go back to "Sun" questions and answers.
With pollution/ozone levels as they are today (and I am assuming they will only get worse), how
does this affect the Sun? My mother has always said, "According to the Bible we are all going to go by
a destructive fire." Today she says, "I know what is going to happen. . .the Sun is going to blow up."
●
You'll have to excuse my mother, she gets a little excited. But at the same time, the question still
lingers... What are the chances that the Sun will blow up? And what kind of destruction would
be involved?
This question is often asked. Thankfully, the answer is no. In about five billion years, the Sun will
swell up, cease its nuclear fusion, and collapse into a white dwarf. You can read more about it
elsewhere in our FAQ:
http://sohowww.nascom.nasa.gov/explore/faq/sun.html#HOW_LONG
("How long until the Sun explodes or disappears?")
http://sohowww.nascom.nasa.gov/explore/faq/sun.html (18 of 34) [2000/01/11 11:57:13 AM]
Dr. SOHO's FAQ: Questions about the Sun
Pollution on Earth is a serious problem, but it doesn't affect the Sun. For one thing, the Sun is
insulated from the Earth by about 93 million miles (150 million km) of vacuum. For another thing,
the Sun is so big compared to the Earth that if we could gather up all the pollution in the world and
dump it in the Sun, it would have no effect.
Go back to "Sun" questions and answers.
I'd like to know how the magnetic axis, the rotational axis, and the ecliptic relate to each other.
Also, are the Sun's axes stable and well- understood? Do they drift or wobble? Do the magnetic poles
ever shift? If there is an offset between the Sun's axes and the ecliptic, does this manifest in aspects of
the solar wind?
●
The Sun's magnetic field is much more complicated than the simple dipole field of the Earth, and
so it does not really have a well-defined "magnetic axis". It does have a general bipolar nature,
with magnetic fields looping out of one pole and back in at the other. The axis of this background
field is closely aligned with the rotation axis. However, there are also a lot of small-scale
concentrations of magnetic field distributed all over the surface, even at times of minimum
activity.
The average field strength is comparable to that of the Earth's magnetic field (a few Gauss), but
high- resolution telescopes reveal very fine bundles of magnetic field with strengths of a few
thousand Gauss, with virtually no field in between these bundles. At times of high solar activity,
the number of sunspots increases, and the magnetic field is dominated by strong fields connecting
spots of opposite magnetic polarity, forming intense active regions that give rise to solar flares and
other forms of activity.
The number of sunspots, and the level of activity, varies with a period of about 11 years. The most
recent minimum in this "solar cycle" occurred towards the end of 1996, and we are now seeing an
increase in the number of active regions. Around the time of minimum activity, the polarity of the
Sun's magnetic field reverses, so that the full magnetic cycle is really 22 years, or twice the length
of the sunspot cycle.
Now for the relationship of the rotational axis of the Sun to the ecliptic plane. (The "ecliptic" is the
plane of the Earth's orbit. Most of the other planets' orbits are close to this plane.) The Sun's
rotational axis is tilted at an angle of just over 7 degrees to the ecliptic. The Earth's rotational axis
is tilted by about 23 degrees to the ecliptic. During the year, an observer on Earth has a continually
changing view of the Sun because of the interplay between these angles. One effect is that the
poles of the Sun are periodically tilted towards or away from Earth, with a maximum tilt of about 7
degrees occurring around the time of the Spring and Fall equinoxes (in spring, the Sun's north pole
is tilted away from Earth, and in fall it is tilted towards Earth). At the summer and winter solstices,
the Earth's equator lies exactly in the plane of the Sun's rotation, and the tilt of the solar poles
appears to be zero.
This changing tilt of the solar poles can sometimes be detected in the solar wind as seen near
Earth. There are basically two types of solar wind: fast wind that emanates from "open" magnetic
field regions near the poles, and slow wind from "closed" magnetic regions near the equator. If an
open region (called a coronal hole) gets tilted in line with the Earth, then we can see changes in
http://sohowww.nascom.nasa.gov/explore/faq/sun.html (19 of 34) [2000/01/11 11:57:13 AM]
Dr. SOHO's FAQ: Questions about the Sun
wind speed and density as it sweeps across the imaginary line connecting the Sun and Earth.
In fact, major geomagnetic events seem to happen more often in the equinox months. See, for
example, this list at the excellent Australian IPS site:
http://www.ips.gov.au/papers/richard/large_geomag.html
(Historical Large Geomagnetic Disturbances)
Go back to "Sun" questions and answers.
I was looking at a paper on the Earth's radiation environment, and it referred to the August 1972
solar event. What event was that?
●
The August 1972 event to which you refer was a major solar flare, one of the best-observed flares
in history. Very intense flares of this type occur typically once in every solar cycle (a similar event
occurred in 1989). It was almost certainly accompanied by a coronal mass ejection similar to those
now frequently observed by SOHO. However, coronal mass ejections had not yet been discovered
in 1972, mainly because the equipment necessary to observe them (space-based coronagraphs) had
not been sufficiently well-developed at the time.
(Well, HAO's Mark III coronagraph can see CMEs from the Mauna Loa observatory in Hawaii,
but orbiting spacecraft saw them first.)
This event was of more than usual importance because it was in a suitable location on the Sun to
cause severe geomagnetic effects -- as was the 1989 event. Interestingly, it occurred between
manned flights to the Moon in April and December 1972 (Apollo 16 and Apollo 17); had it
occurred during either of these flights, the dose of energetic particles and radiation might have
been lethal for the astronauts. Today, NASA takes solar weather forecasts into account when
planning manned space missions.
For a list of some of the greatest geomagnetic storms in recorded history, please check the IPS
page:
http://www.ips.gov.au/papers/richard/large_geomag.html
(Historical Large Geomagnetic Disturbances)
and remember that "recorded history" only goes back to the early 20th century!
Go back to "Sun" questions and answers.
For my Advanced Physics class in high school, we need to find out if the Sun has different
temperatures at the surface vs. the center of the Sun. Which is cooler or hotter, if there is any
difference? Also, is there a difference in density between the surface and the center?
●
The answer to your question is, yes, there is a huge difference in temperature and density between
the surface and the center of the Sun. The temperature at the surface is about 6,000 degrees Kelvin,
the density is about 1017 atoms per cubic centimeter (1017 is scientific notation for a 1 followed by
http://sohowww.nascom.nasa.gov/explore/faq/sun.html (20 of 34) [2000/01/11 11:57:13 AM]
Dr. SOHO's FAQ: Questions about the Sun
17 zeros). This is about one ten-thousandth of the atmospheric density at the Earth's surface.
Writing the density another way, it's about 2 x 10-7 grams per cubic centimeter (or 0.0000002
g/cc).
(Don't take our word for it, however; please verify these figures in an astronomy textbook.)
Most of our knowledge about conditions at the center of the Sun comes from theoretical
calculations, using models of what we think is happening inside the Sun. These calculations show
that the temperature at the center is about 15 million degrees Kelvin and the density is about 150
grams per cubic centimeter. This is hot enough and dense enough for nuclear fusion reactions to
occur (where two atomic nuclei stick together or "fuse" to form a different nucleus and also release
energy in the form of heat). The energy produced in this way is what powers the Sun and produces
all the heat and light that we receive on the Earth. (Actually, a small fraction of Earth's heat budget
is generated internally, through tidal heating and radioactive decay.)
By the way, an interesting fact is that the average density of the Sun (averaged over all depths
from the surface to the center) is about 1 gram per cubic centimeter, the same as that of water.
Go back to "Sun" questions and answers.
What are some practical applications for the rotation period of the Sun? So far, we have only been
able to come up with an flimsy idea concerning the amount of sunlight a plant receives, in respect to
the rotation period of the Sun. Secondly, are there any useful sites where we could gather additional
information?
●
For all the changes in activity that the Sun goes through, the amount of light actually seen at the
ground is remarkably constant, almost never changing by more than 0.2%. When a large sunspot
group passes across the face of the Sun, the solar brightness can go down some. However, in
general, more sunspots actually results in a brighter Sun. This is because of small patches of
brighter Sun, called faculae, which occur both around sunspots, and also in quieter areas of the
sun. During periods of maximum solar activity, there are both more sunspots and more faculae, but
the faculae actually win out.
The most important time-scale, in terms of terrestrial effects, is not the few days that it takes a
sunspot group to move across the Sun, but the 11 year cycle from maximum solar activity to
minimum, and then back to maximum again. This may have an effect on the Earth's climate. There
have been some recent studies which show a correlation between solar activity and climate
changes.
The 11 year solar cycle has a profound effect on the upper atmosphere. During maximum solar
activity, the upper atmosphere can heat up by as much as a factor of three, causing the atmosphere
to expand. This results in more drag on satellites in low Earth orbit, causing them to slowly spiral
inward, and eventually re-enter the atmosphere.
Getting back to the once-per-month time-scale of the rotation of the Sun, measuring the actual
rotation is important for several reasons. It is the Sun's rotation, and more precisely the different
rates of rotation at different latitudes, which drives the solar activity. The magnetic field at the
surface of the sun gets carried along by the ionized solar atmosphere. As different parts of the
http://sohowww.nascom.nasa.gov/explore/faq/sun.html (21 of 34) [2000/01/11 11:57:13 AM]
Dr. SOHO's FAQ: Questions about the Sun
atmosphere rotate at different rates, the magnetic field lines get stretched and sheared. The
complex magnetic fields which result from all this motion give rise to solar features such as
prominences, coronal mass ejections, and even flares.
The rotation of the Sun, when combined with measurements of solar oscillations, contains clues
about its internal structure. This field of study is called helioseismology, and is similar to
seismology on the Earth, which uses the sound waves created by earthquakes to probe the Earth's
interior. I suggest looking at the MDI/SOI homepage at:
http://soi.stanford.edu
(The Solar Oscillations Investigation)
for more information about helioseismology. In particular, follow the link about Plasma Rivers. It
also gives links to a number of other helioseismology sites, such as the GONG project.
The solar rotation rate is also related to the original formation of the Sun and the planets, and is
one of the indices used to relate the Sun to other stars.
P.S. One thing I forgot to mention was an important practical aspect of understanding solar
rotation to SOHO and other solar telescopes. Some of the instruments on SOHO don't see the
entire Sun in a single observation. Instead, they select targets and point to them. If we see an active
region on Tuesday that looks interesting, and want to look at it on Wednesday, we have to know
where it's going to appear 24 hours later. For that reason, we have a model of the solar rotation
built into our planning software, so that we can simply point to a feature and let the computer
figure out where that feature will be at the actual time of observation. If we didn't do this, we'd
probably miss it completely! Go back to "Sun" questions and answers.
Could you please advise me on how scientists were able to calculate how big the Sun is, or where I
can obtain this information?
●
In order to determine the size of the Sun, we have to figure out its distance. Then we can use
geometry to calculate the Sun's size, which is directly proportional to its distance. There is a web
page showing this:
http://scruffy.phast.umass.edu/a114/lectures/lec02/node7.html
(Examples: size of Moon, distance to Sun)
Understanding how this works takes some basic geometry.
Information on determining the distance to the Sun should be in any good introductory astronomy
text book designed for people knowing geometry. My copy of Exploration of the Universe by
Abell, Morrison, and Wolff has a section about it, for instance.
Currently we can measure the distance to the Sun by bouncing radio waves off of it (radar). We
know the speed of light and so we can tell how far the signal has gone by measuring how long it
takes to go there and back: distance = time × speed of light/2
Historically, people have usually used variations on triangulation techniques. The basic idea is
related to "parallax." Hold up your finger and look at it with each of your eyes separately. You will
http://sohowww.nascom.nasa.gov/explore/faq/sun.html (22 of 34) [2000/01/11 11:57:13 AM]
Dr. SOHO's FAQ: Questions about the Sun
notice that it appears to move with respect to things across the room. It is a bit hard to explain
without drawings, so it would be best for you to look there or in a book. The actual measurements
can get a bit complicated.
Go back to "Sun" questions and answers.
●
What kind of star is the Sun?
The Sun is basically a very ordinary star, about mid-way through its "life cycle." It is a dwarf star
(astronomers call stars either "giants" or "dwarfs" - the normal stars are "dwarfs") with a surface
temperature of about 6,000 degrees Kelvin. This makes it appear yellow in color. Hotter stars are
more blue in color, and cooler stars are redder.
Technically, the Sun is classified as a G2V star. Stars are classified according to their surface
temperature and luminosity (brightness). The Sun appears very bright because it is much closer to
us than other stars, but in fact it is not unusually luminous. If we put the Sun at the same distance
as other "nearby" stars, it would look about the same as many other stars. The main temperature
classes are denoted (in decreasing temperature) by the letters O, B, A, F, G, K and M.
Each class is then divided into ten subclasses using numbers from 0 to 9, so a G2 star is roughly
20% of the way between a G0 and a K0 star. There are also five luminosity classes, denoted by the
Roman numerals I, II, III, IV, V and VI. The Sun belongs in luminosity class V, and temperature
(or spectral) class G2, hence the designation G2V.
The reasons for this messy system of classification are historical, and no doubt we could come up
with a better system. However, astronomers have gotten used to this system, and with some
experience you can use these classifications to tell an awful lot about a star. For example, knowing
the surface temperature and luminosity class allows you to compute a star's size (a big star at a
certain temperature is brighter than a small one at the same temperature).
Go back to "Sun" questions and answers.
How is the amount of hydrogen in the Sun measured? And how do we know so much about it? It
came up in science class where we are studying astronomy, and I'm interested in knowing about it.
Thanks for your help.
●
You ask a very good question. There are several ways we can measure the relative amount of
hydrogen in the Sun.
One way is to measure the particles directly in the solar wind. The solar wind particles are ionized.
That means that they have been heated so much that one or more of the electrons orbiting the
atomic nucleus have been kicked off. The absence of the electrons gives the atom a positive
charge, and makes it sensitive to the electric and magnetic fields in the solar atmosphere. These
ions (and electrons) then get accelerated into the solar wind. It is possible to measure the mass,
charge, and energy of these particles by observing their reaction to electric and magnetic fields,
and thus deduce their composition. SOHO is one of a number of satellites which measure the
properties of particles in the solar wind.
http://sohowww.nascom.nasa.gov/explore/faq/sun.html (23 of 34) [2000/01/11 11:57:13 AM]
Dr. SOHO's FAQ: Questions about the Sun
Another, and much older, way to determine the composition of the Sun is through spectral
analysis. Every element will have characteristic wavelengths (colors) associated with it. This is a
result of quantum mechanics. The electrons surrounding the atomic nuclei can only be found in
specific orbits. For example, an electron can't be halfway between one allowed orbit and another.
If a photon of light has exactly the right amount of energy, which is directly given by its
wavelength, then it will be able to kick an electron from one orbit into a higher one. The photon
will be absorbed by the atom, and we'll see a reduction of light at that wavelength. We call these
diminished areas in the spectrum absorption lines, because they appear as dark lines in a slit
spectrum. They're also known as Fraunhofer lines, after the person who discovered them. It's
possible, by measuring the depth and width of these lines, to determine the total number of atoms
of the element which produced the absorption line.
Of course, these measurements only tell us about the outer layers of the Sun. We can't measure the
inner parts of the Sun directly, but we can infer things about it through theoretical models, and by
observing stars like the Sun at various ages in their development, some of which have shed their
outer atmospheres. Also, the relatively new science of helioseismology allows us to infer some of
the properties of the solar interior.
Go back to "Sun" questions and answers.
Dear Sir: I would like to find out about: 1. the shape of the solar magnetic field 2. its observed
components - e.g., dipole and other Could you please advise where I can find these. Thank you.
●
Hi. The Sun's magnetic field is constantly changing and cannot be easily characterized by dipoles
or quadrapoles. In fact, in some areas it is clearly non-potential.
The magnetic field exists on many scales. You can see it in the overall shape of the corona, and
also in small magnetic field loops all over the surface.
The large scale structure changes substantially over the solar cycle. See:
http://www.windows.umich.edu/sun/images/eclipsecomparison.jpg
(Comparison of the Solar Corona at Solar Maximum and Minimum)
At solar minimum it does tend to look somewhat dipolar, but results from the Ulysses space
mission suggest that it is not. See:
http://ulysses.jpl.nasa.gov/ULSHOME/ScienceResults.html
(Ulysses Science Results Page)
I expect some people do attempt to model it using a summation of different moments, but most
modern models are more complex.
Go back to "Sun" questions and answers.
I perform at the Arizona Renaissance Festival. My character is a scientist and is tutor to the Prince.
I use this role to teach science to children at the festival.
●
(I am an engineer who loves science and math. To me they are fun as well as interesting. Most kids
http://sohowww.nascom.nasa.gov/explore/faq/sun.html (24 of 34) [2000/01/11 11:57:13 AM]
Dr. SOHO's FAQ: Questions about the Sun
today are taught science and math as a required topic. They are told these are difficult and required. I
tell them science is fun and exciting.)
Typically, I perform 8 times a day to audiences ranging from 4 to 30 kids. I am very hands-on and
allow the kids to dictate the direction I go. I currently have "experiments" that demonstrate
sublimation, magnetism, combustion and two different exothermal chemical reactions.
I would very much like to add a solar telescope. I believe I can really turn kids on to astronomy if they
can observe sunspots and/or solar flares. Living in Arizona, in spite of this year's el Nino storms, I
have a great opportunity to demonstrate a solar telescope.
Can you provide guidelines for constructing a (relatively) inexpensive solar telescope? Any help you
can give will be appreciated.
Hello. What you are doing sounds great! I am not sure exactly what you would consider
inexpensive. Do you have a pair of binoculars or a small telescope? If so, you can set up a
projection system.
This is described at:
http://www.cnde.iastate.edu/staff/jtroeger/sun.html
(The Sun)
This should let you see sunspots.
The astronomy literature strongly discourages the use of those solar filters which come with most
backyard telescopes. Even with such a filter, looking directly at the Sun is too dangerous! Indirect
projection is preferred, as these websites describe.
You can construct a pinhole projector out of very simple materials (basically a piece of cardboard).
There is some information on this, as well as more discussion of using a telescope, at:
http://solar-center.stanford.edu/observe/observe.html
(Safely Observing the Sun)
I find pinhole projections are good for eclipses, but I can't see sunspots with them. Keep in mind
that sunspot numbers will increase as we approach solar maximum in 2000-2001. During the next
few years, you will thus have more things to look for.
Go back to "Sun" questions and answers.
●
When will the next solar eclipse be?
A NASA eclipse support page, with useful links, is at:
http://umbra.nascom.nasa.gov/eclipse.html
(SDAC Eclipse Information)
The NEXT total solar eclipse (after 1998 February 26) will be on 1999 August 11. It will be
visible throughout central Europe, the Middle East, and western parts of India.
Go back to "Sun" questions and answers.
http://sohowww.nascom.nasa.gov/explore/faq/sun.html (25 of 34) [2000/01/11 11:57:13 AM]
Dr. SOHO's FAQ: Questions about the Sun
Is the solar corona/wind electrically neutral? If there was a charge imbalance, could it be detected
by your instruments if the concentration of charged particles is about 12 parts per million by weight of
the solar wind?
●
We expect the net charge of the solar wind to be neutral. I think it may be possible for there to be
some charge separation over very small distances, but in general the attracting forces between
charged particles restore neutrality.
SOHO has some instruments which make in situ measurement of the solar wind near the Earth's
orbit. I don't know if they can do what you need, but you could look into it. These instruments are:
CELIAS (The Charge, Element, and Isotope Analysis System)
COSTEP (Comprehensive Suprathermal and Energetic Particle Analyzer)
ERNE (Energetic and Relativistic Nuclei and Electron experiment)
You can find links to their home pages at:
http://sohowww.nascom.nasa.gov/instruments.html
(SOHO Scientific Payload)
Go back to "Sun" questions and answers.
I would be thankful if you could give me information on the recent published discovery of a
magnetic carpet on the surface of the Sun. I would like to know, technically, how the magnetic carpet
works and how it was discovered by SOHO. (measurements and such) If you know of any other
references I could contact, it would be very helpful. Thank you and I hope to hear from you soon.
●
It sounds like you have already read some basic materials on the discovery, but just in case, here
are some web pages describing it:
http://soi.stanford.edu/press/ssu11-97/
(Solar Mystery Nears solution with Data from SOHO Spacecraft)
http://www.lmsal.com/PR/magnetic_carpet.html
(Magnetic Carpet)
With SOHO, we now have better and more regular magnetograms than before. Magnetograms are
measurements of the strength of the Sun's magnetic field at the photosphere (the surface seen in
visible light). They enabled us to actually measure how quickly the field is renewing itself - every
40 hours or so, as it turns out. People can then use the magnetograms as a basis for models of the
Sun's magnetic field, how it changes, and how much energy it gives off. There are various
assumptions that have to be made in those models, and I don't know details about the ones used for
this study.
Go back to "Sun" questions and answers.
How do you interpret the elongated, S-curved flow of plasma[?] which is imaged at approx. 02:00
(northwest) on the EIT image in He II 304 Angstrom, taken at 13:19 UT on 31 May 1997? Is this a
●
http://sohowww.nascom.nasa.gov/explore/faq/sun.html (26 of 34) [2000/01/11 11:57:13 AM]
Dr. SOHO's FAQ: Questions about the Sun
prominence which we view half as a filament and half as prominence at 60,000 K via the EIT image?
Once again I would like to thank the SOHO scientific team for the opportunity to follow this
fascinating experiment.
I'm glad to see you are keeping up with us! The image in question is indeed spectacular. I do want
to point out a bit of nomenclature: filaments and prominences are the same thing. It is called a
filament when seen against the disk of the Sun and a prominence when seen off the limb. These
structures contain cool plasma that is seen as a dark feature against the disk and a bright feature
when off the limb.
The "S-curved" feature you see is called an "eruptive prominence." These are
filaments/prominences that, due to some mechanism in the magnetic field (which is why we're
looking to study the detailed causes), causes the loops to rapidly expand outward and typically
"erupting" ejecting plasma outward. This may cause what is known as a magnetic cloud (a plasma
cloud confined by a magnetic field), and ones that are directed to the Earth can have effects here.
There is an ongoing international effort (ISTP) to investigate this and other solar effects on the
Earth. You may want to look at their web page:
http://www-istp.gsfc.nasa.gov/
(International Solar-Terrestrial Physics)
Go back to "Sun" questions and answers.
I have been hearing talk lately of an increasing likelihood of major solar events to begin occuring
in the next few months and culminating with a disastrous finale in 1999. Is it true that solar activity is
on the increase? Is this Solar Cycle 23 that I keep hearing about as menacing as some have predicted?
They say that we will experience X-ray and gamma ray radiation here on Earth the likes of which we
have never before encountered in recorded history. Some have gone as far as to predict the end of life
on the planet as a result of a cataclysmic event before the year 2000. I am a bit concerned over the
rumors and thought I would inquire with people that follow this activity as their life's work.
●
It is true that we are in the beginning of Solar Cycle 23, and that solar activity is increasing and
will continue to do so for about five years -- that is, until maybe A.D. 2003. However, there is
absolutely no danger to anybody here on Earth. The Sun has been doing its thing for about 5
billion years now, and as far as we know, it is just about halfway through its life as a normal star.
Each "cycle" of solar activity lasts about 11 years, from minimum activity through maximum and
back to minimum again. The most recent minimum occurred in about August 1996. We have been
through 22 of these cycles since humans started taking regular observations of the Sun over 200
years ago, and there were many more cycles before that. For most of the Sun's life, humans were
not even aware of its activity. The only difference in modern times is that we have the capability to
observe and monitor the Sun in great detail, often from space. This is why "solar storms" have
gotten so much publicity lately.
Cycle 23 shows no signs of being drastically different from the last few cycles in terms of its
strength or duration. We hope to have lots of exciting observations during this cycle, but nothing
http://sohowww.nascom.nasa.gov/explore/faq/sun.html (27 of 34) [2000/01/11 11:57:13 AM]
Dr. SOHO's FAQ: Questions about the Sun
that should worry people going about their daily lives!
Go back to "Sun" questions and answers.
I'm trying to track down a very brief report that I saw in the paper some time ago and,
unfortunately, failed to clip. It said that the Sun appeared to be warming up very slightly -- i.e., there
appeared to have been a small increase in the solar constant. Could anybody there help me track this
down?
●
There was a story about a possible increase in the solar constant in the September 26, 1997 issue of
Science. Actually, there were two articles: a general news summary article, "Did Satellites Spot a
Brightening Sun?" by Richard Kerr on p. 1923, and a technical article by Richard Willson on p.
1963.
These are a little before the time period you specify, but perhaps the story was picked up by other
news sources later. It is still, as of December 1997, a hot topic at scientific meetings. Because
long-term solar variability may have subtle effects on the Earth's climate, many people in and out
of the solar science community are very interested in it.
Here are some web sources concerning that story:
http://www.xs4all.nl/~carlkop/solar.html
(Solar Activity and Climate)
http://www.phillynews.com/inquirer/97/Sep/26/national/SUN26.htm
(Sun is Brightening, Data Indicate)
I guess it all indicates that the solar "constant" isn't all that constant after all.
Go back to "Sun" questions and answers.
I am interested in Sun photometry as a means of measuring haze. In working through the basic
calculations that are usually performed, I have encountered two difficulties that, as far as I can tell,
are simply ignored by those working in the field It is my hope that NASA may have data that will allow
these calculations to be put on firmer ground. My questions are:
●
1. The assumption that is usually made in the field of Sun photometry is that the output of the Sun is
constant within the narrow wavelength being measured. Of course, the output is almost certainly not
exactly constant, so the question is really how much variation one can expect over various time
intervals. I suspect the variation is small, but the accuracy of an instrument can't be determined
without this vital information. There is a lot of data (for example, from NASA) regarding total output
and for wavelengths below 400 nm. However, I need data for 525 nm and other wavelengths above
400 nm. Does NASA have such data? If not, is it planning to gather such data?
2. This question is in regard to calculating the Sun-Earth distance. I know how to make the
calculation but have been unable to determine error bounds for the calculation. I am familiar with
DE200/LE200 and have looked at Chapter 5 of the "Explanatory Supplement to the Astronomical
Almanac" which gives a great deal of information about the calculation. However, that mathematical
model was adjusted to various observations. Although the maximum error for some of these
http://sohowww.nascom.nasa.gov/explore/faq/sun.html (28 of 34) [2000/01/11 11:57:13 AM]
Dr. SOHO's FAQ: Questions about the Sun
observations is given, I do not understand how one goes from the observation error bounds to the
error bounds for the Earth/Sun distance. Where can I obtain the Earth/Sun distance error bounds?
The visible-light solar irradiance is extremely well known. It does fluctuate by a small fraction of a
percent over the 11-year solar cycle, largely because sunspots are darker than the rest of the solar
surface (so that when the Sun is covered with spots, it emits slightly less light). It also varies very
slightly on about a five minute period, as a result of solar oscillations that brighten and dim the
surface (and which are being studied by three SOHO instruments: GOLF (which detects solar
irradiance and surface Doppler shift with a very stable photometric cell); VIRGO (which looks at
brightness variations in large portions of the Sun in several wavelengths); and MDI (which images
the Sun in Doppler signal).
You asked about visible light irradiance measurements; I did a quick abstract search at the
Astrophysics Database Service:
http://adswww.harvard.edu
(Astrophysics Database Service)
and found a reference to "The Visible Solar Spectral Irradiance from 350 to 850 nm as Measured
by the Solspec Spectrometer During the Atlas I Mission", which is an article by Thuiller, et al. in
the journal Solar Physics, volume 177, issue 1/p, pp. 41-61. Solar Physics is published by Kluwer
Academic Publishers in Dordrecht, The Netherlands. You can probably find it at your nearest
university library.
Thuiller's article is just one in a long, grand tradition: the irradiance of the Sun has been
extensively measured both from the ground and from space. That article should get you started it'll have references back through the Literature.
Regarding Sun-Earth distance - A pretty universal way to determine error bounds in a complex
calculation is to calculate the result several different ways for different input parameters, all within
the input error bounds. If you are working by hand, you should be able to figure out which types of
input error will maximize the error in your calculation in each direction (+ or -). Then you can
recalculate a couple of times. If you're working on a computer and it's not too expensive to
calculate the result, you can do the calculation hundreds of times with a statistical distribution of
input "measurements" and measure the statistics of the output parameters.
Go back to "Sun" questions and answers.
●
Could you please explain to me how energy escapes the Sun? Thank you.
Wow, you've managed to ask a very complicated question in just one line! The process by which
energy escapes from the Sun is very complex. Since we can't see inside the Sun, most of what
astronomers know about this subject comes from combining theoretical models of the Sun's
interior with observational facts such as the Sun's mass, surface temperature, and luminosity (total
amount of energy output from the surface).
All of the energy that we detect as light and heat originates in nuclear reactions deep inside the
Sun's high-temperature "core." This core extends about one quarter of the way from the center of
http://sohowww.nascom.nasa.gov/explore/faq/sun.html (29 of 34) [2000/01/11 11:57:13 AM]
Dr. SOHO's FAQ: Questions about the Sun
the Sun to its surface (the Sun's radius is a little over 1 million miles). Above this core, we can
think of the Sun's interior as being like two nested spherical shells that surround the core. In the
innermost shell, right above the core, energy is carried outwards by radiation. This "radiative
zone" extends about three quarters of the way to the surface. The radiation does not travel directly
outwards - in this part of the Sun's interior, the density of hot gas is very high, and the radiation
gets bounced around countless numbers of times, following a zig-zag path outwards. It takes about
170 thousand years for radiation to make its way from the core to the top of the radiative zone! So
if the Sun were to "turn off" its energy supply today, we would not know about it for a very long
time!
In the outermost of the two shells, the gas in the Sun's interior is too cool and opaque to allow
radiation to pass. Instead, huge convection currents form and large bubbles of hot gas move up
towards the surface (similar to a boiling pot of water that is heated at the bottom by a stove).
Compared to the amount of time it takes to get through the radiative zone, energy is transported
very quickly through this outer "convective zone."
When the energy finally reaches the surface, it is radiated away into space as the light and heat that
we detect here on Earth. It takes about eight minutes for light to reach Earth from the Sun.
Go back to "Sun" questions and answers.
I am wondering how long it would take for the fastest possible signal to transit from the core of the
Sun out to the surface of the Sun. The issue is the density is far greater than with normal dielectrics,
so the velocity of propagation ought to be far slower. But with the density gradient from core to
surface, it gets complicated. So, how long would it take for an EM signal to transit the radius of the
Sun, assuming it took a direct path (i.e., ignoring scattering).
●
I'm a bit puzzled by your question about EM transit time. You mention the fact that the density of
the Sun greatly increases its internal dielectric constant; and then you ask me to disregard
scattering in calculating electromagnetic transit time through the Sun.
But, as you no doubt know, the dielectric constant (and index of refraction) is determined by the
scattering characteristics of the medium under consideration!
I'll just throw some information your way and hope this addresses your confusion.
The optical scattering length inside the Sun is short (just how short depends on depth and on
wavelength) compared to the radius of the Sun. As a result, no coherent optical signals can
propagate through the Sun -- it's not transparent.
Incoherent signals (such as information about the core temperature) take millions of years to
propagate optically from the Sun's center to its surface.
Relativity has nothing to do with these effects; they are due to classical electromagnetic scattering
by the ions and electrons that make up the Sun's bulk. There are much faster ways to get
information from the center of the Sun to the edge. Neutrinos, for example, can travel through the
Sun at (essentially) the speed of light in vacuo. They take only a few seconds to get from the
center of the Sun to its surface. Sound waves propagate out in a matter of one to several hours.
http://sohowww.nascom.nasa.gov/explore/faq/sun.html (30 of 34) [2000/01/11 11:57:13 AM]
Dr. SOHO's FAQ: Questions about the Sun
Go back to "Sun" questions and answers.
I am trying to find detailed data for densities/temperatures of the solar atmosphere out to many
millions of kilometers if not out to 1+ AU. I figured SOHO might have been used to collect such data
or that you might know where to look. Has SOHO been used to study the solar atmosphere in detail?
Where (else) should I look?! I have found such data out to 400km, but no further. I have spent days in
university and college libraries to no avail. (FYI: I have a B.S. in math/physics from the '60s and a
general background in science.)
●
Yes, SOHO studies the Sun's atmosphere extensively. The temperature and density of the Sun's
atmosphere vary from place to place, and often our determinations of general structure are
dependent on models. In general the density and temperature at the Sun's "surface" the
photosphere, are about 10e-7 grams per cubic centimeter and 6000 K respectively. From there the
temperature increases and the density decreases as you go up. At a height of about 3000 km above
the surface the temperature is about 1 million degrees and the density is about 1e-15 g/cc. The
density continues to decrease from there. I think the temperature eventually goes down again
(slowly) as you continue outwards, but I am having trouble finding a source on that at the moment.
You can read about this on the net at:
http://solar.physics.montana.edu/YPOP/Spotlight/SunInfo/Structure.html
(Solar Structure)
(See "chromosphere," "transition region," and "corona" in particular)
We measure the properties of the solar wind at the SOHO orbit (which is just a bit inside of 1 AU).
You can find that at:
http://umtof.umd.edu/pm/index.html
(the Latest 48 Hours of Solar Wind Data)
Some books which might be useful to you would be:
Sun, Earth and Sky by Kenneth Lang (Springer) (see page 106 in particular)
Guide to the Sun by Kenneth Phillips (Cambridge University Press) (see p.133)
Astrophysics of the Sun by Harold Zirin (Cambridge University Press)
Go back to "Sun" questions and answers.
Hi, I am 7 years old and I was wondering... how does the Sun just sit there? What is holding it in
place and keeping it from falling or moving around?
●
Thank you for asking Dr. SOHO! The Sun actually does move through space.
Look at a picture of a spiral galaxy with its "pinwheel" arms. Our Sun orbits the center of the
Milky Way galaxy, just like the Earth orbits the Sun itself. The Earth takes one year to orbit the
Sun, and the Sun takes 240 million years to orbit the galaxy.
The Sun also gets pulled around by the gravity of nearby stars. Our solar system is being pulled
http://sohowww.nascom.nasa.gov/explore/faq/sun.html (31 of 34) [2000/01/11 11:57:13 AM]
Dr. SOHO's FAQ: Questions about the Sun
very slowly toward the constellation Lyra.
The key things to remember are:
❍ The Sun's motion through space is very slow, compared to a human lifetime. (After millions
of years, however, we would notice changes in the positions of stars. The dinosaurs lived
under different constellations 65 million years ago, for example.)
❍ As the Sun moves, its gravity pulls Earth and all the other planets with it.
I hope this helps. The Sun doesn't just sit there -- it moves around the center of our galaxy, and all
the planets are moving right along with it. Please ask us at Dr. SOHO if you have more questions
in the future. Good luck!
P.S. If you go to a very dark place, you can see the Milky Way in the night sky. It looks like a vast,
ghostly backbone stretching across the whole sky, and it is brightest toward the constellation
Sagittarius. That is where the center of the galaxy is.
Go back to "Sun" questions and answers.
Is the Sun moving in space? At what speed? Where is it going to? Where does it come from? Does it
encounter other celestial bodies going through its path, and how do we know of such encounters?
Many thanks for your time and commitment.
●
The Sun orbits around the center of the Milky Way galaxy, in the same way as the Earth orbits the
Sun. It takes about 250 million years to complete one orbit traveling at about 200-300
kilometers/sec. In an orbit, you follow an elliptical path, so you are not really going toward or
away from something in particular.
Even in our galaxy, with over 100 billion stars, space is fairly empty. The Sun does encounter
comets, meteors and dust (as does the Earth) but these are inconsequential to the Sun. The chances
of the Sun (or any of the Solar system) encountering a "solid" object such as another star are
negligible. The Sun and Solar System do encounter dust clouds, but these effects are very slight
and hard to document. Scientists have tried examining the fossil record, but there is only the
slightest hint (still debated) that there have been periods of heavier interstellar dust fall on the
Earth.
In terms of the space around the Sun, here is an answer from a colleague, Dr. Sten Odenwald, from
the "Ask the Space Scientist" website:
"We are located inside what appears to be a roughly spherical zone about 600 light years
across, where most of the interstellar medium has been swept out. This 'Local Bubble' may
have been created 100,000 years ago when a supernova went off.
"The Sun and the Local Bubble are located on the inner edge of what is called the 'Orion
Arm' or 'Orion Spur' in the great spiral pattern of the Milky Way. As we look in the
direction of Cygnus, we are looking down this Arm and see all of the star clusters and
interstellar dust clouds that are part of the arm which extends from Cygnus, across Perseus,
and off towards Carina. If we look off towards Perseus, careful studies of the reddening of
starlight show that we encounter this arm about 300 - 400 parsecs from the sun. If we look
http://sohowww.nascom.nasa.gov/explore/faq/sun.html (32 of 34) [2000/01/11 11:57:13 AM]
Dr. SOHO's FAQ: Questions about the Sun
towards Sagittarius, we do not encounter the spiral arm interior to the Sun's location for
about 1500 parsecs or so. Towards Cygnus, a sharp increase in obscuration occurs at a
distance of 700 parsecs causing the 'Great Rift' in Cygnus."
Go back to "Sun" questions and answers.
I was curious about how I can view those photographs of tornadoes on the Sun. Also do you have
any charts on your site about sunspot and electromagnetic activity over a long period of time?
●
For images and video of the recent tornado on the Sun, I would direct you to this address for CNN.
We do not yet have anything on this on our main web site:
http://www.cnn.com/TECH/space/9804/28/tornadoes.sun.reut/index.html
(CNN - Space craft finds solar tornadoes as wide as the Earth - April 28, 1998)
Keep an eye on the CDS home page, too. Click on "TORNADOES:"
http://orpheus.nascom.nasa.gov/cds/home/
(UK SOHO CDS SITE at RAL)
Also, information on recent sunspot activity can be found at a site very near where you live at
http://www.nwra-az.com/spawx/comp.html
(Comparison of Sunspot Number Indices)
Though this is short-term, there may be links to longer-term sites. There is a lot of information on
the cycle at:
http://wwwssl.msfc.nasa.gov/ssl/pad/solar/sunspots.htm
(The Sunspot Cycle)
Go back to "Sun" questions and answers.
I heard on the news about the tornadoes on the Sun the size of Africa and the wind speed of over
300 mph. I was just wondering, how do they measure the wind speed on the Sun?
●
The principle involved in measuring "windspeeds" on the Sun is the same as that used by police
radars to catch speeding motorists. Police radars bounce a radio signal off of an approaching
vehicle, and watch the change in frequency of the reflected signal. This change depends on the
speed of the vehicle. The principle involved in this measurement is called the Doppler effect. In
the case of SOHO observations of the Sun, the observed wavelength (or frequency) of light that is
emitted from hot gas in the Sun's atmosphere changes according to whether the gas is moving
towards or away from the detector. By measuring these wavelength changes, scientists can
determine the speed at which the hot gas is moving.
Go back to "Sun" questions and answers.
●
My question for you is: I don't understand how there can be solar wind. I thought there was no
http://sohowww.nascom.nasa.gov/explore/faq/sun.html (33 of 34) [2000/01/11 11:57:13 AM]
Dr. SOHO's FAQ: Questions about the Sun
friction in space. I was under the impression that there has to be some kind of resistance that causes
friction which in turn causes wind. If you could just explain to me how solar wind is created, I would
be grateful.
Thanks for asking Dr. SOHO! People often think of space as a vacuum, but it is not a perfect
vacuum. The Sun is constantly shining, sending photons in all directions. Photons are "particles"
that make up light and other radiation.
The Sun is also sending out the "solar wind." The solar wind is simply a stream of charged
particles - mostly protons and electrons - which push off of the Sun's surface into space. This
"wind" is very, very thin. Even with the solar wind flowing through it, interplanetary space is still
almost a vacuum.
As you can see, the solar wind is not like the winds in Earth's atmosphere. Its cause and
acceleration are still largely unknown; that's part of why we launched SOHO.
The solar wind does cause some drag (and some electromagnetic effects) upon objects in space,
including satellites. However, the drag caused by sunlight itself (remember those photons?) is
thousands of times stronger. Photons can put pressure on objects they strike. This pressure is what
makes "solar sails" possible. But that is another story. I hope this helps!
Go back to "Sun" questions and answers.
Go back to "Dr. SOHO's FAQ."
SEND US YOUR COMMENTS
Go To
Other SOHO Web Pages
Author: Shane Stezelberger
Co-Author/Curator: Therese A. Kucera
[email protected]
Responsible Official: Art Poland([email protected])
Last modification: Wednesday, 22-Dec-1999 17:20:38 EST
http://sohowww.nascom.nasa.gov/explore/faq/sun.html (34 of 34) [2000/01/11 11:57:14 AM]
Dr. SOHO's FAQ: Coronal Mass Ejections
Go back to "Dr SOHO's FAQ"
Go back to the "SOHO" main page.
●
What is a CME made of?
●
Did the April 1997 CME force the shuttle home early,
and can a CME affect Mir?
●
What happened on the Sun in January 1997?
●
Where can the public get warnings of CMEs?
●
What is the white circle in LASCO images?
●
Where can I get visuals for a CME presentation?
●
Do CMEs really travel faster than light, or are my numbers wrong?
●
Why don't CMEs travel at the speed of light?
●
Can a CME affect the shape of the Earth's magnetic field?
●
Can CMEs damage satellite TV systems?
●
Was the April 1997 CME dangerous? Was it larger than most?
●
Our facility recently had some power outages, and they coincided with a CME. Is there a
connection?
●
When was the largest solar eruption ever recorded?
●
Did the April 1997 CME create a shockwave?
●
How frequent are CMEs? Can they hit other planets?
●
Can CMEs start a fire on the Moon?
●
Can a CME ignite a planet's core?
●
Why aren't CMEs thought to originate in the solar
core? Could they be related to dark matter?
●
Is there a solar storm coming?
●
Will there be a CME just before Easter this year?
http://sohowww.nascom.nasa.gov/explore/faq/cme.html (1 of 16) [2000/01/11 11:57:19 AM]
Dr. SOHO's FAQ: Coronal Mass Ejections
●
Are you certain that the recent CME was unrelated to the twin sungrazing comets?
I'm not clear on what exactly the mass being ejected consists of. Is it like a large ball of ignited
hydrogen flying/flaming through space? Is it some form of radiation? If it is radiation, why is it
traveling so slow compared to light speed, i.e. days instead of minutes to reach Earth?
●
The coronal mass ejection (CME for short) consists of hot gas at a temperature of about 1 million
degrees C that is ejected from the Sun's outer atmosphere, the corona. It is mainly made up of
hydrogen, with a few percent of helium and very small amounts of heavier elements. The gas is so
hot that the atoms are almost all ionized, so the electrons that are normally bound within the atoms
are free to "float around" within the gas (or plasma, to use the technical term for a hot, ionized
gas). Note that nothing is "burning" or combusting in the conventional sense.
The LASCO images that you see on the web record sunlight that is reflected off the electrons in
this plasma cloud as it moves outwards from the Sun. The typical speed of a CME is about 400
kilometers per second, so it takes a few days to cover the distance from the Sun to the Earth (about
150 million kilometers).
There are also several types of "radiation" associated with the CME. For example, as the CME
moves outwards from the Sun, it generates a shock wave (like the shock ahead of a supersonic
aircraft) that can accelerate particles in interplanetary space to high energies. There are also high
energy particles and X-rays from the flare that accompanied the CME back at the Sun (I won't go
into the details of how solar flares and CMEs are related; that is a tricky, ongoing topic of solar
research). However, none of this radiation is energetic enough to penetrate the Earth's protective
atmosphere and magnetic field, so it poses no threat to anybody on Earth. Also, I should point out
that in no sense is the CME cloud radioactive.
Go back to "CME" questions and answers.
Was the April 1997 CME the "real" reason the space shuttle (STS-83) came home early? What do
the poor guys on Mir have to look forward to when this radiation hits them without the benefit of an
atmosphere for protection?
●
Before I answer your questions about the aborted Shuttle mission and potential dangers for Mir
cosmonauts, I should reiterate that a coronal mass ejection (CME) is not radioactive in any way. It
is merely a plasma (ionized gas) made up of protons and electrons.
The early return of the Shuttle in April 1997 was not connected in any way with activity on the
Sun. The reason for the early return was a mechanical failure in one of the fuel cells on board
shuttle Columbia. A shuttle carries 3 fuel cells,and "The Rules" say all 3 must be working properly
for a mission to stay in orbit.
This failure occurred over the weekend of 5-6 April 1997. The decision to bring the Shuttle home
early was taken around noon on Sunday, a full day before the CME occurred on the Sun. We
would love to be able to predict these events that far in advance, but we do not have that capability
yet!
http://sohowww.nascom.nasa.gov/explore/faq/cme.html (2 of 16) [2000/01/11 11:57:19 AM]
Dr. SOHO's FAQ: Coronal Mass Ejections
The question about the dangers for cosmonauts on board Mir is a very good one. I am not an
expert in the biological effects of space radiation, but I will make the following comments. Mir is
in an orbit that keeps it well inside the Earth's magnetosphere (the region where the Earth's
magnetic field shields it from interplanetary phenomena). Even without an atmosphere around it,
Mir is well-protected by the magnetosphere.
The only time astronauts and cosmonauts on Mir might be in any danger would be if they pass
through the so-called "Van Allen belts," where energetic particles from space leak into the
magnetosphere over the poles and get trapped in the Earth's magnetic field. This would be a
serious concern in the aftermath of a very large solar flare, that might produce particles with
extremely high energies and enormous fluxes of X-rays. The flare on Monday (7 April 1997),
while it was the largest we'd seen for some time, did not come anywhere near this category. So
there was no immediate danger to any of the Mir astronauts from this event. However, this is one
of the concerns that needs to be kept in mind when considering long-term stays in space in the
future, for example on the International Space Station.
Johnson Space Center in Houston, which runs NASA's astronaut program, maintains a Space
Radiation Analysis Group. This group tracks solar activity with the aid of observatories all over
the world. (Solar research is highly cooperative, you will find.) Johnson can take precautions if a
major solar event occurs during a shuttle mission. A Russian counterpart body does the same for
Mir crews.
Go back to "CME" questions and answers.
I am amazed about this project. Although it's not my field, I'd like to know: what is a "solar
tsunami," and what happened at the beginning of this year with our Sun?
●
On 6 January 1997, the coronagraphs on SOHO observed a "halo" event erupting from the Sun. A
coronagraph is a telescope with a built-in eclipse; the bright Sun is blocked by a disk so you can
view the faint corona outside the Sun.
Anyhow, a halo event looks like a circle around the Sun, because it is moving along the direction
you are looking. Therefore, if it is coming right at the Earth, and you are looking straight at it, it
looks like a big, expanding circle coming at you.
The LASCO coronagraph team announced that this halo was a CME (Coronal Mass Ejection)
heading towards earth, and that it was moving at slightly more than a million miles per hour, or 1.6
million kilometers per hour. It reached earth on the 10th of January. Because this thing had a lot of
magnetic field, it was able to interact very powerfully with the Earth's magnetic field, causing
various geomagnetic disturbances.
Energetic particles and current systems were detected at observing stations near the north and
south poles, and satellites recorded large jumps in energetic particles. We heard reports of radio
communications interference at Antarctic research outposts. Finally, the "magnetic cloud"
resulting from this CME literally compressed the Earth's magnetic field somewhat. (Do not worry;
the field returned to its normal shape afterward.)
http://sohowww.nascom.nasa.gov/explore/faq/cme.html (3 of 16) [2000/01/11 11:57:19 AM]
Dr. SOHO's FAQ: Coronal Mass Ejections
Interestingly enough, after this had all calmed down, the Sun did it again. The same region came
around one month later (the Sun fully rotates every 28 days) and let out another blast which had
nearly the same effect. That was on February 7. March 7 passed by without incident....
Go back to "CME" questions and answers.
How and from whom will members of the public receive an early warning of an impending
destructive, or potentially destructive, coronal mass ejection (CME)?
●
I don't think that there's currently a system in place that warns the general public of CMEs. (Keep
in mind that these events are difficult to detect without a coronagraph in space; LASCO's
immediate predecessor re-entered back in 1989.)
There is a "space weather bureau" associated with the NOAA. It is called the "Space Environment
Center," and you can visit their web page at:
http://www.sec.noaa.gov
(NOAA Space Environment Center)
for more information about them. The SEC's mission is to provide real-time monitoring and
forecasting of space weather events, just as NOAA's mission (as a whole) includes monitoring and
forecasting of weather on Earth. They generate forecasts and storm warnings that are distributed to
an electronic mailing list; you may be able to get on the list, if you want.
The catch is that space weather really doesn't directly affect public safety in the same way that
ground weather does. There is no direct hazard to human life on the surface of the Earth from
CME's, because of the shielding afforded by the Earth's atmosphere and magnetic field.
CMEs (and the geomagnetic storms that they can cause) do affect our technological tools. So
airlines, power utilities, and telecommunications companies are very interested in CME reporting,
while most "civilians" shouldn't need to worry (so long as your phones still work!).
Go back to "CME" questions and answers.
Greetings! I downloaded the LASCO C3 Christmas 1996 comet and Jan. 15 [1997] coronal mass
ejection.... Kudos to your efforts in putting that material on the Web. BTW, what is the inner circle
inside the occulting black disk? Is this the true size of the Sun?
●
Those coronal mass ejections, to give you a sense of scale, mass more than every living human
being on Earth combined -- and they travel at about a million miles an hour. You're right, the white
disk inside the LASCO C3 occulter (on most of their images) is the size of the Sun -- they put it in
there for scale.
For another sense of scale, the LASCO C3 coronagraph's field of view is the same linear size
(almost exactly) as the constellation Orion. (If you look closely, you'll recognize the constellation
Sagittarius in the Christmas 1996 movie.)
I'm not sure how dark the night skies are where you are -- but if you get out just after astronomical
dusk on a REALLY DARK night, you can see (with your naked eye) some of the same material
http://sohowww.nascom.nasa.gov/explore/faq/cme.html (4 of 16) [2000/01/11 11:57:19 AM]
Dr. SOHO's FAQ: Coronal Mass Ejections
that LASCO sees. It appears in the night sky as "zodiacal light" (stress on the "i", which is
pronounced "eye"), which is an extension of the Sun's corona.
The zodiacal light is several degrees broad (N/S) and extends in an enormous band (E/W) all the
way around the Zodiac -- though it fades below visibility somewhere around 90 degrees from the
Sun.
The zodiacal light is presumed to be reflected by small dust particles in orbit around the Sun;
however, I use "orbit" loosely as, for very small particles in close proximity to the Sun, light
pressure becomes important in holding them up.
Most of the pictures on the SOHO Gallery pages have been specially processed to remove the
zodiacal light from the raw downlinked images -- the dust particles aren't nearly as interesting to
solar physicists as is the "electron corona", which is made of plasma rather than flecks of solid
material.
Go back to "CME" questions and answers.
I am a student looking for as much information on CMEs, specifically their effects on the Earth's
upper atmosphere, for an informative presentation I am giving. I was wondering if you could point me
in the right direction (charts and graphs for overhead projection, etc.) with some links, or perhaps an
informative package (I know of none that exist).
●
While no one single package exists that I know of, there are many resources available on CMEs.
For one thing, there is a collection of about 50 overheads that you can get at the lower part of our
Gallery page. From our home page at:
http://sohowww.nascom.nasa.gov
(SOHO: The Solar and Heliospheric Observatory)
click on the "Picture Gallery" button, and scroll down to "SOHO Portfolio" and "SOHO Slide
Presentation" in PDF format. They have a number of images relating to CMEs. Also the EIT page
and, probably better, the LASCO page (accessible from the Gallery) have video and stills on
CMEs.
There are lists of most SOHO/LASCO CMEs at this address:
http://lasco-www.nrl.navy.mil/cmelist.html
(LASCO Coronal Mass Ejection Lists -- Based on Preliminary Data)
This location at Lockheed has some CME information and images:
http://www.lmsal.com/YPOP/Program/ToC.html
(Yohkoh Public Outreach Program: Table of Contents)
For printed info, look up the two articles on SOHO in the August and September 1996 issues of
Sky and Telescope by Kenneth Lang. These are well presented. Also, the link from the SOHO to
the ISTP page provides a wealth of additional info and links. In fact, this is more focused on
Sun-Earth connections and includes the magnetosphere, more appropriate for your interests:
http://sohowww.nascom.nasa.gov/explore/faq/cme.html (5 of 16) [2000/01/11 11:57:19 AM]
Dr. SOHO's FAQ: Coronal Mass Ejections
http://www-spof.gsfc.nasa.gov/istp/
(International Solar-Terrestrial Physics)
Go back to "CME" questions and answers.
The news reported that the solar ejection was traveling at nearly 2 million miles per hour. If the
speed of light (186,000 mph) cannot be exceeded, how is this possible?
●
I'm afraid there is a slight confusion here. The speed of light is actually 186,000 miles every
second. If we expressed it in miles per hour, the speed of light would come out to be about
671,000,000 mph. Thus, 2 million miles per hour is only about 0.3% of the speed of light.
Astronomers tend to express measurements of velocity in terms of seconds rather than hours. They
also use metric units rather than English units. Hence, an astronomer would express the speed as
900 kilometers per second (kps) rather than 2 million mph. And the speed of light would be
300,000 kps, which is equivalent to 186,000 mps.
Go back to "CME" questions and answers.
If it takes approximately 8 minutes for light to reach the Earth from the Sun, why does it take so
long for this coronal ejection to reach us? Wouldn't a plasma wave travel at the speed of light?
Thanks.
●
You are correct in that light -- and other radiation, too -- takes about 8.4 minutes to reach the
Earth.
However, coronal mass ejections (CMEs) are plasma, not radiation. These clouds of ionized gas
(particles, protons, and electrons, with a magnetic field) do not travel at light speed.
The range of speeds for coronal mass ejections is 75 kilometers per second to over 1,500 km/sec.
Roughly once or twice per solar cycle, we may detect a very fast CME traveling at up to 2,000
km/sec. The solar wind itself flows constantly at 400 to 800 kilometers per second (in our part of
the solar system, that is. Faster solar wind flows from the Sun's polar regions.).
Go back to "CME" questions and answers.
Is it possible to have an explosion on the Sun which is large enough to send an electro-magnetic
field shifted so that it could disrupt the Earth's magnetic field in any measurable way? By measurable,
I mean such as disruption of the axis or magnetic north pole of the Earth. I realize this would require
an immense amount of energy, but I have no idea as to the plausibility of such an idea. I am a novice
in such areas of physics, but I do understand the basics of magnetism and the potential since electric
fields are vastly more powerful than gravitational ones. I will appreciate any insight you could give me
as to this possibly very silly question.
●
No, your question isn't silly at all! In fact, that's exactly what's been in the news from SOHO for
the last couple of days (in early April 1997). Such explosions (depending on the way that they're
http://sohowww.nascom.nasa.gov/explore/faq/cme.html (6 of 16) [2000/01/11 11:57:19 AM]
Dr. SOHO's FAQ: Coronal Mass Ejections
observed) are often called "flares;" if they throw off a significant amount of material from the Sun,
the ejection is called a "coronal mass ejection" (or "CME"). These things occur relatively
frequently -- small ones happen perhaps every day; larger ones (such as the one that happened a
few days ago) occur maybe once a month. Their occurrence varies slightly with the overall solar
cycle.
But it's relatively rare (several times per year, or so we think) that a big one will be pointed
directly at the Earth. To understand the effect of CMEs on the Earth, you have to understand the
relationship between coronal material and the magnetic field. The solar corona is so hot that the
gases in it lose some of their electrons in the powerful collisions between atoms. So the material,
called plasma, is a mixture of positively-charged ions and negatively-charged electrons. (You can
spot a plasma here on Earth if you look at a Neon or fluorescent light; or at an electrical spark).
Plasmas behave in strange ways near magnetic fields: because they're electrically conductive, they
can steer magnetic fields; and they, in turn, are steered by them.
So a CME drags a piece of the Sun's magnetic field with it -- loops of magnetic force are stretched
and dragged into interplanetary space by the inertia of the expanding plasma.
A large part of the impact of these CMEs on the Earth is the effect of those loops of force on the
Earth's field when they arrive. Normally, the Earth's own magnetic field diverts the solar wind
around the Earth, shielding us from what we would see as a field of ionizing radiation coming
from the Sun. (The atmosphere also does a fine job of that). When one of these CMEs strikes, the
lines of force can either rebound or combine with the Earth's (depending on which way the CME's
magnetic field is pointed, compared to the Earth's).
In either case, the measurable effect is that the protected "bubble" of space around the Earth (the
'magnetosphere') changes shape -- for example, on 11 January 1997, the magnetosphere was
squashed down below the geostationary altitude satellites orbit, and many comsats (including
Telstar 401) were directly exposed to the particle stream coming from the Sun. When the
magnetosphere changes shape, the Earth's field moves slightly, and any long enough piece of wire
(including telephone lines and power grids) acts as an electrical generator. This can cause
telephone and power outages, such as the one in Quebec in March 1989.
So, yes, the Earth's magnetic field moves and the changes can be detected.
However, to the best of my knowledge the changes are not permanent -- the field rebounds to (as
near as we can measure) its original shape after the CME passes. Similarly, I don't think you'll
have trouble using your compass to point (more-or-less) north, even in a powerful "magnetic
storm." Compasses may become noticeably disturbed in polar regions, however, where magnetic
field lines get close to the surface of the Earth.
Go back to "CME" questions and answers.
Do these solar mass ejections (for example, 9 April 1997) present any danger to home T.V. satellite
systems? Can these events fry the receiver?
●
Don't fear. The CME cloud won't fry your receiver. It may pose some danger to the satellite,
http://sohowww.nascom.nasa.gov/explore/faq/cme.html (7 of 16) [2000/01/11 11:57:19 AM]
Dr. SOHO's FAQ: Coronal Mass Ejections
depending on its orbit! The Australian IPS (Ionospheric Prediction Service) has some good
background material on solar- terrestrial effects:
http://www.ips.gov.au/papers/
(IPS: Interesting Facts and Educational Material)
In particular, see:
http://www.ips.gov.au/papers/richard/sat_out.html
(Satellite Outage Caused by Sun)
Go back to "CME" questions and answers.
All the coverage of the recent solar event has definitely intrigued me. I do not recall ever being
informed via the news of the possible outcome of these types of solar emanations. My question is, have
these things really been occurring throughout my 29 years here on earth, and if so, is this one worse
than others were? Given the nature of the other comments I've heard from people who may not know
what they're talking about (i.e. that dramatic increase in earthquake activity will occur on 13 April)
and other reports like that, I am quite concerned. What, exactly, would be the "worst case scenario"
with solar activity?? What is happening to the Sun? What should I expect, if anything and finally,
what should I do to prepare? Is there anything? Can a solar emanation of major scale truly wreak
havoc on our planet? Pardon me for my ignorance, but I'd like to sleep a little better tonight, if
possible :)
●
Yes, these events have been going on for as long as we've been observing the Sun, and probably
have been for all of human history. The only difference now is that we're able to see them and
study them much better than we were ever able to do before. The April 7, 1997 event wasn't even a
particularly big one -- just one that we observed particularly well. The most significant effect that
such a solar storm could produce would be on communication and power systems. There have
been cases in the past where power outages were triggered by solar storms.
Also, Earth orbiting satellites would see an increase in the radiation they were traveling through. I
understand that the power companies and satellite operators are taking steps to minimize any
chances that this particular event will cause any problems.
Remember that this is a perfectly normal event -- the Sun is producing these all the time. In fact,
this is at least the third one to pass the Earth this year alone [as of mid-April 1997]. I think I can
safely say that it will not cause any earthquakes or tidal waves.
The cause of these coronal mass ejections is still something under study. Basically, a small part of
the Sun's atmosphere becomes magnetically unstable and bubbles out away from the Sun. The
Washington Post called it a "solar burp," and that's as good a description as any!
Go back to "CME" questions and answers.
I work at a large Water Filtration Plant, and last November (1997), we had a few power
interruptions (surges?), which caused us to lose power to equipment and left us scurrying around to
●
http://sohowww.nascom.nasa.gov/explore/faq/cme.html (8 of 16) [2000/01/11 11:57:19 AM]
Dr. SOHO's FAQ: Coronal Mass Ejections
rectify the situation. About a week later, I learned that there was a major solar flare which shot out
from the Sun and headed for our direction (Earth - North America). The solar flare occurred on a
Tuesday, and by Thursday morning we had our power problems. Is it safe to say that these solar flares
were the cause (directly or indirectly) to our power problems? Our local power supplier could not be
specific as to the cause for their (and our) power problems (I don't think they knew). I do understand
that solar flares can be a major headache for power supply companies as they tend to disrupt the
power grid. I guess that my question could be reworded as: Is it possible that these charged particles
emitted from our Sun (during solar outbursts - i.e. flares) could take up to two or more days to reach
our planet? And one more related question; Can I or other members of the public have access to the
information gathered by the SOHO or Yohkoh satellites in relation to solar flares? Would we have
enough warning giving by SOHO or Yohkoh for us to take preventative action?
The phenomena from the Sun which is most likely to cause power outages are called "cornal mass
ejections" (CME's). The Sun ejects large magnetic bubbles of gas. Some of these are directed
towards the Earth. These CMEs take a few days (say 75-90 hours) to get here. They can interact
with the Earth's magnetic field, and sometimes this causes power outages on the ground. Currents
in the Earth's magnetic field can induce currents in the ground, or in (I think) long-distance
high-voltage lines. If these currents get too large, they can burn out transformers. Given enough
warning time, power utilities can reroute power to reduce such overloads.
I can't say if that was your problem. The best people for that are at the Space Environment Center.
They have a help page at:
http://www.sec.noaa.gov/info/helpme.html
(SEC Comments Page)
Try sending a message to one of the "Induced Currents" people.
There is information about solar flares and CME's on the web. You can find some information on
SEC's Space Weather page:
http://www.sec.noaa.gov/today.html
(Today's Space Weather)
This is where you can find the best predictions as to how solar activity may affect the Earth. This
can still be quite hard to predict, although we are getting better at it.
Flares (which are sudden enhancements of emission from part of the Sun) can be detected reliably
with X-ray detectors on satellites. CMEs are harder to detect. There is a list of the ones we have
observed with SOHO at:
http://lasco-www.nrl.navy.mil/cmelist.html
(LASCO CME Lists -- Based on Preliminary Data)
We have may have missed some, though, and of the ones we see, very few affect the Earth. This
list is not updated frequently enough for prediction purposes -- i.e., roughly once per day -- but
when we see anything interesting, we tell the SEC right away.
Go back to "CME" questions and answers.
http://sohowww.nascom.nasa.gov/explore/faq/cme.html (9 of 16) [2000/01/11 11:57:19 AM]
Dr. SOHO's FAQ: Coronal Mass Ejections
When was the largest solar eruption recorded? What was its size in comparison to the April 1997
event? Are there any indicators in your observations with SOHO to actually predict solar eruptions?
●
One of the largest solar eruptions occurred on March 7, 1989 at the peak of the last solar cycle. Its
terrestrial effects included a major power outage in Canada. Compared to the April 1997 event, the
region on the Sun's surface which produced the March 1989 eruption was significantly larger in
area (about 10 times bigger) and more prolific in terms of the number and intensity of solar flares
it produced. One useful indicator of eruptions is an increase in flaring activity on the solar surface
-- as measured in X-rays.
However, like monitoring seismic activity, one cannot accurately predict the next "big one" when
it comes to solar eruptions. The interaction of solar activity and the Earth is something called
"space weather." People in the space weather field say they are as advanced as regular terrestrial
weather was in, say, the early 1950s, before weather satellites and advanced computer modeling.
Go back to "CME" questions and answers.
I'd like to do something about SOHO for the German amateur magazine "Sterne und Weltraum".
From images No. 2 and 3 of the 4 LASCO pictures of the 7 April 1997 event, I computed that the
expansion speed of the bubble (or halo, if you like) is of the order of 1500 km/s. More than a solar
diameter in less than half an hour. Is this correct? Is this a new record value? Or did I make some
mistake? 1500 km/s is well above sound speed, even at coronal temperatures (2 million K?). So, this is
not just an expanding cloud, it is a real shock wave. Many thanks in advance for a reply.
●
Thank you for your question about the LASCO CME event of April 7th, 1997. I think the speeds
you measured are a little high. The highest speeds that we found were about 800 kilometers per
second for the arcade of loops that heads off to the southwest (bottom left of the images). This is
the first part of the CME that we saw in the images. The "halo" has a speed that varies from about
650 km/s in the south to about 450 km/s in the north.
But, and this is an important point, remember that these speeds are measured in the plane of the
sky. The real speed cannot be less than this measured value, but it could be much higher. Think of
an object that comes almost directly towards you, and imagine taking a series of photographs as it
gets nearer. The object will appear to move very little from one picture to the next, even though it
could be traveling towards you at a very high speed.
By measuring the time taken for the CME to reach Earth, we can get its average speed over that
time. For this event, this average speed worked out to be about 600 km/s. This suggests that the
CME slowed down quite a bit on its journey from the Sun to the Earth.
To drive a shock wave in the solar wind, the CME disturbance must be traveling faster than the
surrounding solar wind, which typically has a speed of 300-400 km/s in interplanetary space (this
value would be smaller near the Sun, maybe about 100 km/s). In a "normal" gas, the difference in
speeds would have to be greater than the sound speed (i.e. the CME would need to be traveling
supersonically relative to the solar wind).
http://sohowww.nascom.nasa.gov/explore/faq/cme.html (10 of 16) [2000/01/11 11:57:19 AM]
Dr. SOHO's FAQ: Coronal Mass Ejections
However, the solar wind is not a "normal" gas. It is a very tenuous plasma, and is in fact a better
vacuum than anything we can make on Earth. This means that the solar wind is what we call a
"collisionless plasma," in which the individual gas particles hardly ever interact with each other.
Sound waves do not travel very well in this sort of medium!
The shocks that form in a collisionless plasma like the solar wind are very different to the shocks
in a normal gas. The important speed in this situation is the Alfven speed (this is the speed at
which magnetic waves called Alfven waves travels through the plasma). The CME needs to be
traveling at a super-Alfvenic speed relative to the solar wind in order to drive a shock. The Alfven
speed at Earth is about 60 km/s, and is probably about 500 km/s in the Sun's corona. So the event
on April 7th could probably drive a shock, though not a very strong one. This is exactly what we
observed in the solar wind!
Go back to "CME" questions and answers.
I would like to know: Have there been very many of these magnetic clouds, and have other planets
or moons been in their path?
●
Interplanetary magnetic clouds are still an area of active research. We don't have all the answers
yet. That's part of why SOHO and her sister ships exist. We do know that the Sun spits out these
coronal mass ejections (CMEs) frequently: several times per week during solar minimum and a
few times per day at maximum. We are keeping a list of the CMEs that LASCO (SOHO's
coronagraphs) sees: http://lasco-www.nrl.navy.mil/cmelist.html
(LASCO CME Lists -- Based on Preliminary Data)
You can see how often they occur, although many of these events are not very large at all.
Magnetic clouds ejected from the Sun can, I am certain, hit the Moon and other planets just as
easily as the Earth. The Sun has been spitting these ejections out for a long time, and it does not
seem to care which planets, comets, or asteroids get in the way when it spits one out.
Orientation, though, is one key factor. Like a lawn sprinkler, a narrow cloud of charged particles
only affects a certain angle of the solar system as it expands into space. For a solar eruption to
affect Mars, for example, it has to erupt from a particular longitude on the Sun's surface. Keep in
mind that the Sun rotates about once a month, like a big nuclear lawn sprinkler.
There are other factors. The Earth has a strong magnetic field; the Moon does not. I would guess
that weird interactions might happen if a magnetic cloud/storm hit while the Moon was passing
Earth's magnetic tail. Without a magnetic field above it, the Moon's surface gets bombarded
directly by the solar wind. This bombardment is why the lunar soil is so rich in the Helium-3
isotope, for instance.
When a CME's magnetic cloud reaches the Earth, huge electrical currents are induced in the
Earth's magnetic field. We still don't have all the answers about the aurora, a related phenomenon.
We do know that aurorae happen on other planets, like Jupiter and Saturn. These planets, like
Earth, possess magnetic fields and atmospheres. Check out these VERY cool pictures:
http://antwrp.gsfc.nasa.gov/apod/ap980109.html
(Astronomy Picture of the Day: Saturnian Aurora)
http://sohowww.nascom.nasa.gov/explore/faq/cme.html (11 of 16) [2000/01/11 11:57:19 AM]
Dr. SOHO's FAQ: Coronal Mass Ejections
http://antwrp.gsfc.nasa.gov/apod/ap980123.html
(Astronomy Picture of the Day: Jovian Aurora)
Go back to "CME" questions and answers.
I saw an article about fires on the Moon. Is there any truth to that? Could this be related to the
magnetic clouds ejected from the Sun?
●
No interplanetary magnetic activity could possibly ignite a fire on the Moon. The Moon has no
atmosphere, so normal combustion is simply impossible. Any volcanic activity is, as far as we
know, long extinct. Its surface has been baked dry by billions of years' exposure to the vacuum of
space.
Two exceptions are those huge ice deposits in the two lunar polar regions, of course. The scientists
who discovered these deposits think the ice came from comet impacts. In any case, they are
regions of very icy dirt -- but they still would not burn.
Go back to "CME" questions and answers.
I understand that when magnetic fields intercept a planet, they create an electrical current. Is it
possible to bring a planet to life by creating internal fires this way and boiling out water and oxygen
from within the planet's core, thereby creating an atmosphere capable of supporting life?
●
First of all, conventional fire or combustion requires gaseous oxygen. As is indicated below, that
substance isn't typically located in a planetary core. In the presence of oxygen, you can actually
burn parts of a planet's crust, which is happening in Centralia, Pennsylvania:
http://www.theequalizer.com
(Coal Mining In Centralia: A Burning Issue)
but not a planet's core.
I don't think "igniting" a planetary core would get you very much water and oxygen. When a
planet forms, it is very hot and molten for a variety of reasons (radioactive decay and gravitational
contraction, for example). Heavy elements, like iron, sink to the bottom. Light elements, like
hydrogen and oxygen, do not. This is partly why terrestrial planets, like Earth and Mercury, have
dense (but molten) iron cores.
You'll find more light elements up near the surface than down at the core. (Please don't take our
word on planetary science as gospel; consult a reliable astronomy textbook.) The result is that any
oxygen and water will find its way to the surface early on. Even if you could somehow ignite the
core, it wouldn't help make the planet or its atmosphere more livable for humans.
For suggestions on further reading in the subject of terraforming, please look up:
http://www.astrobiology.com
(The Astrobiology Web)
http://sohowww.nascom.nasa.gov/explore/faq/cme.html (12 of 16) [2000/01/11 11:57:19 AM]
Dr. SOHO's FAQ: Coronal Mass Ejections
Go back to "CME" questions and answers.
CMEs blast the matter of the corona outward at a rapid velocity. But many of them appear to be
global phenomena. Why is it thought that the events originate near the surface rather than from the
core, which would be an obvious symmetrical origin for global phenomena? Also, the density changes
globally on the 11 year solar cycle. How is this explained if not via something exiting from the core to
increase the fluidization of the matter of the star? Couldn't these be both related to and due to dark
matter?
●
CMEs are thought to originate in the corona, rather than the deep interior of the Sun, for a couple
of reasons. Most importantly, CMEs exist in a part of the Sun (the corona) where the magnetic
field dominates the structure and dynamics of the ionized gas. In most cases, one can actually see a
magnetically confined structure that somehow becomes unstable and then expands and releases its
material to become a CME. The magnetic field is what runs the show.
However, below the photosphere the magnetic field is relatively unimportant to the dynamics and
structure of the fluid flows. The solar interior is almost nothing like the tenuous gases in the outer
corona -- much more energetic fluid flows drag the magnetic field around, rather than vice versa.
The idea of global, surface originated CMEs is not so far fetched as it may appear at first glance.
Rearrangement of a single portion of the global magnetic field of the Sun can render other portions
unstable to ejection. Furthermore, high-speed compression or "blast" waves have been seen
spreading across the face of the Sun after filament disappearances and solar flares (two events that
are thought to be associated with CMEs). These blast waves cover a large portion of the Sun's
surface, and there is speculation that they could trigger other portions of the corona to eject.
In short, there are several obvious mechanisms whereby global CMEs could be triggered by
surface phenomena, but no real reason to suppose that they are directly triggered from deep within
the Sun.
You do make the rather astute observation that, since CME frequency recurs on an 11-year cycle,
they must have something to do with the solar interior. Indeed, in a broad sense one may say that
CMEs are caused by the dynamo action in the center of the Sun, but there is a big difference
between assigning causality in a general sense and assigning it to a specific event. One may say
that the Sun makes human activity possible, but one may not correctly say that sunlight caused (for
example) the Watergate scandal -- a specific set of human activity.
In response to your last question, no, CMEs have nothing to do with dark matter. Dark matter is,
by definition, matter that isn't glowing and that is therefore undetectable to astronomers. The Sun
and its corona do not qualify as dark matter.
Go back to "CME" questions and answers.
A teacher of mine says that he recently read that a big solar storm is expected to hit earth sometime
within the next two years. How true is this?
●
http://sohowww.nascom.nasa.gov/explore/faq/cme.html (13 of 16) [2000/01/11 11:57:19 AM]
Dr. SOHO's FAQ: Coronal Mass Ejections
The Sun has an 11 year cycle of activity. This activity is related to the magnetic field structure near
the surface of the Sun. One of the manifestations of this activity is "solar storms." Solar storms are
a variety of eruptions of mass and energy from the solar surface. Flares, prominences, sunspots,
and coronal mass ejections are the common harbingers of solar activity, as are plages and other
related phenomena seen at other wavelengths. They all involve sudden releases of stored magnetic
energy, which accelerates the hot gases near the surface or in the corona of the Sun. Sometimes
these particles make it all the way to the Earth and beyond by flowing along the Sun's magnetic
field into interplanetary space. When the material collides with the Earth's magnetic field and
trapped radiation belts, it can dump particles into our upper atmosphere to cause the Aurora. The
same 'charged' particles can produce their own magnetic fields which can modify the Earth's
magnetic field and affect compass readings. The changing magnetic fields can also 'induce'
electricity in long pipelines, or produce electrical surges in our power grids leading to brownouts
and blackouts.
So what will happen? The current solar cycle is expected to peak in about two years. The
predictions for this cycle can be compared to the last 300 years as being fairly high in activity but
not the highest. Actually, the activity was extremely high in the late 1950's and the previous cycle,
peak in 1989 was also higher than the upcoming one.
Go back to "CME" questions and answers.
Is there a Coronal Mass Ejection expected to take place around Easter of this year? There is a
"rumor" which I have heard regarding a huge CME which will take place specifically on Holy
Thursday (the Thursday before Easter) of this year. This "rumor" is, indeed, tied into certain religious
beliefs. However, I wish to discover if there is any scientific basis for this predicted solar event, which
is supposed to have a big impact on Earth in terms of a "bright flash of light," and magnetic upset.
Thank you.
●
For obvious reasons I do not wish to comment on religious beliefs; however, I can discuss some
facts about CME's. Presently, SOHO sees CMEs every 1 to 2 days. Therefore, the chance of a
CME occurring on Holy Thursday is fairly good. As for impacts on the Earth, this occurs when
CMEs are directed towards the Earth (CMEs can and do occur almost anywhere on the Sun, and
are therefore directed in many directions).
Such Earth-directed events are sometimes known as "Halo CMEs" because of their appearance in
coronagraphs. These occur with a frequency of 1 to 2 weeks. Many, but definitely not all, are
associated with active regions on the Sun. As the Solar rotation rate is 28 days and active regions
can last over many rotations, you can also see how there may be a 28 day period. I'm trying to
point out that there is a random CHANCE a halo CME can occur on that day. In terms of their
terrestrial effects, Halo CMEs can indeed cause geomagnetic disturbances and cause increased
aurorae.
The last topic to address is predictions. Presently, we do not have the understanding of CMEs to be
able to truly predict their occurrence. When there is a strong active region (large magnetic fields)
near the central meridian of the Sun, we expect higher CME (that is, Halo CME) activity, but can
not really predict them. We are not at the stage where we can accurately predict detailed Solar
activity one month in advance, especially not down to the single day level.
http://sohowww.nascom.nasa.gov/explore/faq/cme.html (14 of 16) [2000/01/11 11:57:19 AM]
Dr. SOHO's FAQ: Coronal Mass Ejections
To see a list of the CME's observed with SOHO, please look at:
http://lasco-www.nrl.navy.mil/cmelist.html
(LASCO CME List)
Go back to "CME" questions and answers.
Is there any possible connection between the twin sungrazing comets in early June 1998 and the
nearly simultaneous coronal mass ejection?
●
The mass ejections are presumed to be quite unrelated to the fact that two comets had evaporated
in the solar atmosphere just hours earlier.
We have not done any extended analysis on this question, but there are several reasons to make
this presumption. The release of mass, and both kinetic and magnetic energy in the CMEs is
enormous, compared to the relatively small mass and kinetic energy of little comets such as these.
In addition, the CMEs appear to be launched through some mechanism (as yet not fully
understood) in which magnetic energy builds up and is stored in closed magnetic field structures
that arch very high above the solar surface. Then something goes unstable, and there is a sudden
release of energy and outrush of plasma. In this case, the CME front was closely followed by an
erupting cool filament, which is the tangled mass you may have seen following the initial CME.
From the activities of this filament as it rotated across the solar disk during the two weeks
preceding June 1st, it was evident that energy was building, and we were all expecting an eruption
of some sort. It happened just after the comets arrived and burned up in the solar atmosphere. The
comets' closest approach would have been in the northern hemisphere, probably northeast, as
viewed from SOHO, while the CME originated in the southwest.
Another SOHO team member replies:
The consensus is that there is probably no connection between sungrazing comets and CMEs.
Remember, SOHO sees coronal mass ejections almost daily, but she sees these comets far less
frequently. To confirm this, compare LASCO's CME lists with our comet list. They are all on
LASCO's web site at:
http://lasco-www.nrl.navy.mil/
(LASCO Home Page)
CMEs obviously do not need to wait for a comet in order to lift off from the Sun.
A CME is a huge cloud of gas which expands to become larger than the Sun itself in a few hours.
These sungrazing comets - even the very bright ones --are relatively small, even as comets go.
Most comet experts think the sungrazers are house-sized chunks or smaller. Bright comets, like
Halley and Hale-Bopp, often have nuclei many kilometers wide. Scientists do not think such tiny
comets could have much effect on these huge solar eruptions.
However...
Having said that, our scientists are quick to point out that nobody has thoroughly investigated this
problem yet. There may indeed be a connection which hasn't been discovered yet. Comets are just
http://sohowww.nascom.nasa.gov/explore/faq/cme.html (15 of 16) [2000/01/11 11:57:19 AM]
Dr. SOHO's FAQ: Coronal Mass Ejections
dirty snowballs (or icy mud-balls, we are not sure!) without any magnetic field. However, all those
molecules of ice and dust plowing into the lower corona could possibly destabilize existing
magnetic structures. Possibly. Again, nobody is certain.
There is so much we don't know about CMEs themselves that even speculating about connections
right now (as I just did!) will not be productive. We're not certain what causes a CME to erupt in
the first place. That's part of why we launched SOHO in the first place: to gather data on CMEs.
Basically, we think there's no connection - but we are not sure. Thanks for asking, and please write
us again if you need more information.
P.S. I point you to the following website on Kreutz sungrazing comets. Dr. Biesecker is an active
SOHO scientist who has made more comet discoveries than anyone else on our team:
http://sungrazer.nascom.nasa.gov
(Doug Biesecker's Science Pages)
In particular, this page lists papers in the scientific literature about sungrazing comets.
http://sungrazer.nascom.nasa.gov/kreutz.biblio.html
(Sungrazing Comet Bibliography)
Go back to "CME" questions and answers.
Go back to "Dr. SOHO's FAQ."
SEND US YOUR COMMENTS
Go To
Other SOHO Web Pages
Author: Shane Stezelberger
Co-Author/Curator: Therese A. Kucera
[email protected]
Responsible Official: Art Poland([email protected])
Last modification: Tuesday, 13-Apr-1999 17:33:09 EDT
http://sohowww.nascom.nasa.gov/explore/faq/cme.html (16 of 16) [2000/01/11 11:57:19 AM]
Dr. SOHO's FAQ: Sunspots and the Solar Cycle
Go back to "Dr SOHO's FAQ"
Go back to the "SOHO" main page.
For related topics, see also our pages on solar flares and coronal mass ejections.
●
●
Where can we find a daily sunspot count on the web?
●
Is a sunspot dark? Are they strictly a visible-light phenomenon?
●
What library sources would be good for sunspot and solar flare information?
●
I cannot find sunspots with my telescope. Is it my hardware, or is the Sun not cooperating?
●
Why does it seem like most sunspots are appearing at 20 degrees latitude?
●
What is the north/south orientation of SOHO's daily images?
●
We are measuring the rotation of the Sun for a project. Any advice?
●
When is the next solar maximum? Will it really be drastically weaker than previous maxima?
●
Is there a connection between the orbits of Jupiter or Hale-Bopp and the solar cycle?
●
Does Jupiter have any influence on the solar cycle? Both have periods just over 11 years.
●
Is there a relation between planetary alignments and the solar cycle?
Would you be able to point us to a URL source for a daily sunspot count?
I believe that this site's plot will fit the bill for your needs quite nicely:
http://www.nwra-az.com/spawx/comp.html
(Space Weather: Comparison of SSN Indices ant Northwest Research Associates)
You might also try the Sunspot Index Data Center, at the Royal Observatory of Belgium:
http://www.oma.be/KSB-ORB/SIDC/index.html
(Sunspot Index Data Center)
Go back to "Sunspot" questions and answers.
http://sohowww.nascom.nasa.gov/explore/faq/sunspots.html (1 of 7) [2000/01/11 11:57:22 AM]
Dr. SOHO's FAQ: Sunspots and the Solar Cycle
My question is: is a sunspot dark, as far as UL>visible light goes? Is energy output reduced in this
area? Does UV decrease as well as all other parts of the electromagnetic spectrum, or is this more of a
visible light phenomenon?
●
Sunspots appear dark in visible light because they are cooler than the surrounding solar
atmosphere. A typical sunspot temperature is about 3500 degrees Celsius, instead of the 6000
degrees C of the adjacent regions of the photosphere (the visible surface of the Sun). However,
sunspots still radiate a lot of light; if you could somehow place a sunspot against the black
background of space, it would appear about 10 times brighter than the full Moon.
As with many solar phenomena, the reason for the dark sunspots can be found in solar magnetism.
The spots are the regions where the Sun's magnetic field breaks through the photosphere from
below. This strong magnetic field in spots inhibits the convection flows that usually carry heat up
from the interior, so the sunspot is cooler than the surrounding areas.
With respect to the second part of your question:
Spots appear in pairs, with opposite magnetic "polarity" for each spot. In one spot, the magnetic
field is directed upwards (out of the Sun) and in the other, it goes back down into the Sun. These
opposite poles are joined by magnetic loops. The region around a sunspot group where the strong
magnetism dominates the motion of gases in the solar atmosphere is usually referred to as an
active region.
Active region loops connecting spots of opposite magnetic polarity can be seen above the limb of
the Sun in photographs taken in the red light emitted by hydrogen atoms trapped within the loops.
These loops can also constrain very hot electrified gases in the Sun's corona at temperatures
around one million degrees, where they glow in ultraviolet and X-rays. Thus the areas above
sunspots appear bright at these wavelengths. The parts of the corona outside of these "active
regions" cannot constrain these hot gases so effectively, so they appear darker in X-ray
photographs.
Go back to "Sunspot" questions and answers.
As part of my course, I have to do a 20 minute presentation and an essay on a subject related to
physics. I have chosen the title "SUNSPOTS AND SOLAR FLARES," and I am currently trying to
find out as much information as possible so that I can process it all and use the best of it to give me
sufficient knowledge for a good presentation. Can you recommend any sites or other references I may
have available to me in the university library?
●
There's a great deal of literature out there on these subjects, but here's a few references that you
might find in your library:
1. Astrophysics of the Sun, by Harold Zirin, Cambridge University Press (a new edition of a fairly
old book, published in 1988)
2. Solar Astrophysics, by Peter Foukal, Wiley (more mathematical than Zirin, and also more
recent).
3. The Sun as a Star, by Roger Tayler (recently published).
http://sohowww.nascom.nasa.gov/explore/faq/sunspots.html (2 of 7) [2000/01/11 11:57:22 AM]
Dr. SOHO's FAQ: Sunspots and the Solar Cycle
Books of a more general nature are:
1. Sun Earth and Sky, by Kenneth Lang.
2. Guide to the Sun, by Kenneth Phillips (New Scientist level).
For a good on-line discussion of solar flare theory, have a look at:
http://hesperia.gsfc.nasa.gov/sftheory/
(The Solar Flare Theory Page)
For reviews of recent research, the Annual Reviews of Astronomy and Astrophysics is always a
good place to look. Other journals include Solar Physics and the Astrophysical Journal
Go back to "Sunspot" questions and answers.
First, Thank You for taking the time to help us (the general public) understand what's going on
around our little place in the universe. I have a 4.5 inch telescope. I also have a mylar filter for solar
observing. I have been looking at the Sun now for about two months using 91x and 36x powers, and
have yet to see anything but a round disk. My question is this, is the system I'm using not the right
setup to see sunspots, or is the Sun in a "mellow" period of its cycle?
●
(This question was received in April 1997.)
I don't know much about the telescope set-up you are using, but my guess is that you haven't seen
anything because there hasn't been much to see. The Sun is indeed in a "mellow" period of its
cycle [Ed. note: as of April 1997]; we are still close to sunspot minimum. You can check the
images seen by other optical telescopes. Go to:
http://sohowww.nascom.nasa.gov/synoptic
(SOHO Synoptic Database)
Click on a date, and then click on images which are "White Light." There are some sunspots, but
they are few and not very big, so maybe you just can't see them -- yet! As sunspot maximum
approaches, these sunspots will of course increase in size and number.
If you are still wondering about your telescope, write back, and I will find someone who knows
more about that.
Go back to "Sunspot" questions and answers.
Why does it seem (just by looking, mind you) that most sunspots occur around 20 degrees North or
South Latitude?
●
It is indeed true that sunspots appear at certain latitudes. Those latitudes change with the sunspot
cycle: The first spots in a solar cycle (what we are seeing now, in early 1997) appear around 30
degrees North or South of the equator. As the solar cycle continues, spots appear closer to the
equator. This is due to over-all changes in the Sun's magnetic field, but we don't really know why
they occur.
http://sohowww.nascom.nasa.gov/explore/faq/sunspots.html (3 of 7) [2000/01/11 11:57:22 AM]
Dr. SOHO's FAQ: Sunspots and the Solar Cycle
There is a plot of the change of sunspot latitude with solar cycle at:
http://www.hao.ucar.edu/public/slides/slide18.html
(The Butterfly Diagram)
Go back to "Sunspot" questions and answers.
What is the north/south orientation of SOHO's daily images? I want to determine the sidereal
rotational period of the Sun by observing sunspots.
●
Almost all of the daily images that you see on the SOHO web sites are oriented with solar north at
the top. By "solar north," we mean the north rotational axis. The east limb is at the left of the
images and the west limb is at the right (i.e. the opposite to a terrestrial map, because we look "up"
at the Sun, not "down" on it like a map). The exception is that every once in a while we rotate the
spacecraft and in that case the images are rotated too
With the normal orientation, however, new features first appear on the left of the images as they
rotate around the east limb, and then make their way across the disk to the west.
You should try measuring the rotation at different latitudes and see if you can find "differential
rotation". You might find that sunspots near the equator rotate faster than those at higher latitudes.
If you try to do this, you should remember that the Sun's rotation axis can be tilted by as much as 7
degrees towards or away from Earth, depending on the time of year (this is known as the "solar B
angle"). The effect of this on the images is that the equator moves up and down depending on the
time of year, so the apparent center of the disk is usually not at zero latitude.
Go back to "Sunspot" questions and answers.
My partner and I are doing a science project on the rotation of the Sun. We plan to do this by
measuring sunspots. We are unsure where to go from here and would appreciate it if you could send
us information on the Sun and how to accurately measure the rotation of the Sun.
●
That sounds like a very interesting project. I hope that the Sun cooperates and gives you some
good sunspots to observe. We're heading into a period of higher solar activity, and that does mean
more sunspots.
Measuring the rotation of any object, including the Sun, is in some sense a simple thing to do. All
you have to do is to track some object, like a sunspot, and watch it move. However, there are some
things that need to be kept in mind.
First of all, where is the rotation axis of the Sun? It unfortunately doesn't come with latitude and
longitude markings painted on it, so it isn't obvious when you look at the Sun. Of course, the way
that we've established where the rotation axis is, is by looking at the rotation of features like
sunspots. The Sun's rotation axis is aligned at an angle of about 26 degrees to the Earth's.
Depending on the time of year, the axis could be canted to either the left or the right. The precise
angle, for any given date, can be looked up in the Astronomical Almanac, which should be
available at your local library. It's listed as the "Position Angle of Axis P", and is known as the
http://sohowww.nascom.nasa.gov/explore/faq/sunspots.html (4 of 7) [2000/01/11 11:57:22 AM]
Dr. SOHO's FAQ: Sunspots and the Solar Cycle
P-angle. Positive values of P mean that the rotation axis is canted to the left.
If you're using a telescope with an equatorial mount, then the telescope is already oriented with
respect to the Earth's rotational axis. You can then make your measurements relative to this axis,
and apply the corrections for the P-angle directly. If, on the other hand, you're using a telescope
with an azimuthal mount, then the situation is a little more complicated. The orientation of the
Sun's axis to the sky changes as the Sun moves across the sky. At sunrise, the north pole will be on
the right side of the Sun, and at sunset, the north pole will be on the left. The best thing to do in
that case is to make your measurements at local noon. Then, the rotation axis of the Earth will
appear to be straight up from the Sun, and you can apply the P-angle correction from there. Note
that local noon is not the same thing as noon on your clock. If you like, I can tell you how to
calculate local noon at your site.
Another complication is that we're not looking at the Sun from its equator. Instead, the Sun's
rotation axis is canted at an angle of about 7 degrees to the plane of the Earth's orbit. This means
that sometimes the north pole of the Sun is tilted toward us, and sometimes it's tilted away from us.
This angle is listed in the Astronomical Almanac as "Heliographic Latitude B0", and is known as
the B-angle. Positive values of B mean that the north pole of the Sun is tilted towards us.
The Sun does not rotate as a solid body, like the Earth does. The rotation rate is the fastest at the
equator -- about once every 25 days. However, this slows as one approaches the poles. For
example, at a latitude of 45 degrees, the Sun rotates more like once every 28 days. Also, the
rotation rate is slightly different depending on what kind of feature one is looking at. This is
because the magnetic fields associated with different features are anchored at different depths in
the solar atmosphere, and the rotation rate also depends on the depth.
What you'll need to do, once you've taken your measurements, is to convert your observations into
latitude and longitude on the Sun. I suggest looking at a textbook on spherical geometry. Don't
forget to take the P and B angles into account. Once that's done, the rotation rate for any given
sunspot is the change in latitude with time.
If you have more questions, feel free to ask.
Go back to "Sunspot" questions and answers.
I heard part of a news report some time ago that the coming sunspot maximum was not going to
occur, or would not be as extensive as normal (with the usual references to the Little Ice Age). They
also mentioned unusual magnetic field patterns. Is this real, or was it the normal media garbling of a
science story? When is the next solar maximum expected to peak?
●
I don't know where you heard this report, but it is probably, as you say, another example of the
media getting it wrong. There are no known indications that the coming solar cycle will be
drastically weaker than previous cycles. Having said that, I must remind you that modeling and
predicting solar-cycle behavior still has plenty of unknowns.
In fact, the present solar cycle (Cycle 23, which started late in 1996) is expected to be of similar
strength to the last couple of cycles. There are some indications that it might be somewhat larger
than the last cycle, which peaked between 1989-1991.
http://sohowww.nascom.nasa.gov/explore/faq/sunspots.html (5 of 7) [2000/01/11 11:57:22 AM]
Dr. SOHO's FAQ: Sunspots and the Solar Cycle
You can read a report on this topic at the following web site:
http://www.sec.noaa.gov/info/Cycle23.html
(Summary of Panel Findings at the Space Environment Center)
Recent solar cycles have lasted for about 11 years. As you can see, the consensus points to a new
maximum in the 1999-2001 window.
Go back to "Sunspot" questions and answers.
Is the sunspot activity, which is at its low occurrence, related to Jupiter's orbit, which is traveling at
its fastest speed? We noticed that the perihelion date of Hale-Bopp was in the middle of the recent
sunspot activity. Is there any relation to that?
●
The short answers are: no, and no.
The solar cycle varies with an 11-year average period, although it can vary by a few years.
Juptier's "year" is 11.86 Earth years. They are close, but scientists do not believe there is a causal
relation between the two. The sunspot cycle is driven by magnetic field activity in the Sun's
interior -- but scientists aren't quite sure about the specifics of the process.
Jupiter has a large, strong magnetic field. If its magnetic field were a visible object, it would
appear in Earth's skies to be larger than the full Moon. But the Sun's magnetic field is much larger
and stronger still.
Comets, on the other hand, have no detectable magnetic field of their own. (We drew this
conclusion from spacecraft observations of Comet Halley back in 1986, which failed to find any
magnetic field as close as 4,000 km from the comet's nucleus. There was a colloquium here at
Goddard on the subject of comets just yesterday (25 April 1997), and a Goddard scientist
reiterated this point.)
I say this because sunspots are known to be magnetic phenomena. They are, in fact, the places on
the photosphere where large magnetic field lines have protruded from the Sun's surface. Their
cause lies within the Sun, not without. It is unlikely that magnetically-neutral comets have much
effect on sunspots.
(When I say, "magnetically neutral," I may not be precisely correct. Comets and their tails can and
do become electrically charged by the solar wind, but you'd need to consult a comet expert for
further details.) Thanks for your inquiry!
(Postscript: Jupiter orbital parameter from "TRW Space Data," 4th edition,
TRW Science and Technology Group, 1992.)
Go back to "Sunspots and the Solar Cycle" questions.
Does Jupiter have any influence on the solar cycle? I ask, because both Jupiter's orbit and the
sunspot cycle have approximately an 11-year period. Even if Jupiter does not cause the solar cycle,
could it control the cycle's intensity? Could Jupiter (or other planets) raise tides on the Sun in the
●
http://sohowww.nascom.nasa.gov/explore/faq/sunspots.html (6 of 7) [2000/01/11 11:57:22 AM]
Dr. SOHO's FAQ: Sunspots and the Solar Cycle
same way the Moon causes tides on the Earth?
This is an oft-raised question; scientists who have looked at it have found no causal relation
between the approximately 11-year solar cycle and Jupiter's 11.86-year orbital period. Those are
"Earth" years, of course.
Tides are a gravitational phenomenon. Sunspots are believed to be magnetic. Although Jupiter has
a huge and powerful magnetic field (if it were visible, it would look bigger than the full Moon in
our sky), the Sun's magnetic field is bigger and stronger still.
Jupiter's gravity does make the Sun "wobble" back and forth as the big planet revolves around it.
Remember that Jupiter has 99% of the non-solar mass in our solar system, so none of the other
planets produce much of an effect. This type of wobbling is what allows astronomers to detect
possible large planets circling nearby stars.
The following page at Dr. Sten Odenwald's ASK THE ASTRONOMER site contains many
references to journal articles on this and similar subjects. Nature and Science should be widely
available in local libraries, but Solar Physics may be harder to find -- try a large university library.
Good luck, and keep us in mind for future questions!
http://image.gsfc.nasa.gov/poetry/astro/q923.html
(Do the planets affect the sunspot cycle?)
Go back to "Sunspots and the Solar Cycle" questions.
Using the site at: http://www.fourmilab.ch/cgi-bin/uncgi/solar I found that the planets seem to be
most aligned at the periods of maximum solar activity. Has your team found this to be true?
●
The SOHO team has not been studying the relationship of the position of the planets to solar
activity. The solar cycle is believed to be driven by a magnetic dynamo process in the interior of
the Sun. There has been no scientific demonstration that the positions of planets can drive or be
physically related to the 11-year solar cycle.
Go back to "Sunspots and the Solar Cycle" questions.
Go back to "Dr. SOHO's FAQ."
SEND US YOUR COMMENTS
Go To
Other SOHO Web Pages
Author: Shane Stezelberger
Co-Author/Curator: Therese A. Kucera
[email protected]
Responsible Official: Art Poland([email protected])
Last modification: Wednesday, 22-Dec-1999 17:20:38 EST
http://sohowww.nascom.nasa.gov/explore/faq/sunspots.html (7 of 7) [2000/01/11 11:57:22 AM]
Dr. SOHO's FAQ: Solar Flares
Go back to "Dr SOHO's FAQ"
Go back to the "SOHO" main page.
●
●
Why do solar flares happen?
●
Can you predict solar flares? If so, when will the next one be?
●
Can you predict changes caused by the Sun's magnetic storms?
●
Who issues solar flare warnings?
●
Can SOHO act as an early warning system for other satellites?
●
How do solar flares affect Mir's cosmonauts?
●
I heard a CME is expected. Will it cause any damage?
●
Is there a classification system for flare stars?
●
Where can I find detailed data about solar flares?
●
Why don't solar flares cause spikes in soft X-rays?
●
What are these "meter-wave radio bursts" I keep hearing about?
●
Do EIT scientists use a flare loop model?
●
Does a sun flare start a heat wave?
Why do solar flares happen?
On the internet, the best place to start learning about solar flares is the following NASA web page:
http://hesperia.gsfc.nasa.gov/sftheory/
(The Solar Flare Theory Page)
This has a number of descriptions, questions and answers, and links to other interesting pages
including information on how Solar flares may affect YOU! Below are a couple of sections taken
from this page. Please feel free to ask about any details, and thanks for your interest.
A flare is defined as a sudden, rapid, and intense variation in brightness. A solar flare occurs when
magnetic energy that has built up in the solar atmosphere is suddenly released. Radiation is
emitted across virtually the entire electromagnetic spectrum, from radio waves at the long
http://sohowww.nascom.nasa.gov/explore/faq/flare.html (1 of 12) [2000/01/11 11:57:27 AM]
Dr. SOHO's FAQ: Solar Flares
wavelength end, through optical emission to x-rays and gamma rays at the short wavelength end.
The amount of energy released is the equivalent of millions of 100-megaton hydrogen bombs
exploding at the same time!
The first solar flare recorded in astronomical literature was on September 1, 1859. Two scientists,
Richard C. Carrington and Richard Hodgson, were independently observing sunspots at the time,
when they viewed a large flare in white (visible) light.
Solar flares are thought to result from the build up and explosive release of magnetic energy in the
solar atmosphere. The outer layer of the Sun is convective, meaning that the gas rolls up and down
like in a pot of boiling water. This ionized gas (plasma) drags the Sun's magnetic field with it,
twisting it and strengthening it. In some regions the magnetic field becomes particularly strong and
breaks out into the solar atmosphere as discrete, loop-like structures. In active regions where flares
occur, these structures either interact or become internally unstable, giving a flare. The signs of a
flare are gas rapidly heated to high temperatures, electrons and ions accelerated to high energies,
and bulk mass motions. The energy in the magnetic field is thought to be converted into these
things through a process called magnetic reconnection, in which oppositely directed magnetic field
lines "break" and connect to each other and part of their energy is transferred to the gas in the solar
atmosphere. This is the basic picture. Some aspects of it may not be entirely correct and many of
the details are not yet understood.
Go back to "Solar Flare" questions and answers.
●
Can you predict the solar flares? If so, when will the next solar flare occur?
It is impossible to know for sure when the next solar flare will occur, for the same reason it's
impossible to know when the next lightning strike will occur -- they are random, chaotic events.
We can only predict when one is likely to occur, up to a few days in advance. Solar flares happen
on or near "solar active regions", which are places on the surface of the Sun where the magnetic
field is strong.
Sometimes, big parts of the magnetic field cancel each other out and there are bright, sudden
releases of energy (like a humongous spark!) These huge sparks could almost be described as
"lightning on the Sun;" they are solar flares.
Big active regions have things called "sunspots" under them -- and the really big sunspots can be
seen with your naked eye, IF you look at the Sun at sunrise or sunset through mist or fog. (NEVER
look at the Sun during the day or you'll damage your eyes!!!). Individual sunspots come and go
over periods of a few weeks; and the number of them grows and shrinks on an 11-year cycle.
We're just exiting "solar minimum" now, in mid-1997 -- there have been very few active regions,
few flares, and almost no sunspots for a while. But over the next four or five years, the Sun will be
getting gradually more active, until big flares happen almost every day. Then it'll start fading back
again.
Go back to "Solar Flare" questions and answers.
http://sohowww.nascom.nasa.gov/explore/faq/flare.html (2 of 12) [2000/01/11 11:57:27 AM]
Dr. SOHO's FAQ: Solar Flares
Is it possible to measure and predict the changes that will take place because of the Sun's magnetic
storms, in the far or near future?
●
The wording of your question is a little confusing. The short answer is:
1. We can make some short-term predictions of solar activity, although they are not always very
accurate (about like weather predictions).
2. If we are looking at the Sun at the right time in the right way with the right instrument (see
example below), we can sometimes give warning that a particular solar event or set of events is
likely to affect the Earth, such as by causing a geomagnetic storm.
3. Long-term changes in the level of solar activity, especially in the total ultraviolet radiation from
the Sun, would probably lead to major changes in the Earth's average temperature.
4. But we do not now have the ability to make long-term predictions of the Sun's behavior (that is,
on timescales like a human life span or a few generations). Most of the current effort in this area is
to try to project what the next (approximately) 11-year solar cycle will be like -- e.g., how big,
how long, when it will reach its peak.
Now, a more detailed answer to your first question:
(a) Groups such as the National Oceanic and Atmospheric Administration's (NOAA) Space
Environment Center at Boulder, Colorado:
http://www.sec.noaa.gov
(Space Environment Center)
spend a lot of time and energy on making more accurate, short-term predictions of solar activity,
using space-borne and ground-based measurements of features on the surface of the Sun; NOAA
has experts on geomagnetic disturbances and the effects resulting from them.
(b) We could improve our early warning system for geomagnetic disturbances if we had several
monitors in space which could image the solar corona. In space, the monitors would never be
"clouded out." To do the job right, we would need some of the monitors able to see the back side
of the Sun, since the Earth can be affected by events on the side not facing us.
Another option would be placing solar monitors in space with different view angles, so we could
triangulate approaching CMEs. NASA is studying this approach for the STEREO mission:
http://umbra.nascom.nasa.gov/stereo_facts.html/
(The STEREO Mission Fact Sheet)
For an example of looking in the right way at the right time, go directly to an International Solar
Terrestrial Physics program (ISTP) page:
http://www-istp.gsfc.nasa.gov/istp/cloud_jan97/event.html
(ISTP Sun-Earth Connections Event: January 6-11, 1997)
A brief synopsis of this example follows:
On 06 January 1997, a coronagraph on SOHO imaged a mass ejection from the solar corona
headed directly towards SOHO (and hence, directly towards the Earth). Since light travels faster
http://sohowww.nascom.nasa.gov/explore/faq/flare.html (3 of 12) [2000/01/11 11:57:27 AM]
Dr. SOHO's FAQ: Solar Flares
than most of the particles from the event, SOHO scientists were able to warn that particles
(electrons and protons from the corona) would arrive at Earth about 80 hours later. (It turned out
that this solar event probably caused an AT&T satellite to short out and stop transmitting
numerous TV channels. The satellite was unrecoverable.)
But one instrument such as the SOHO coronagraph cannot do the whole monitoring job by itself,
since it has other observations to make. Sometimes it cannot observe, such as during the spacecraft
maneuvers which keep SOHO in a stable orbit. Also, sometimes the data from the observations
have to be stored on board for a few hours until we can contact the spacecraft. Staying in
continuous contact with the spacecraft would help, but NASA's Deep Space Network must divide
its time among many other space missions besides SOHO.
(c) Studying the effect of long-term solar activity on the Earth climate is still a fairly new science,
and there is still disagreement over how much the Sun drives the Earth's climate system. The Sun
provides almost all of the energy to the Earth (a little bit is generated internally), and most
scientists believe that long periods of solar inactivity (such as the Maunder minimum, around AD
1645-1715) probably caused mini-ice ages, while long periods of enhanced activity could shrink
the polar caps and raise the level of the oceans.
But, at least since the Industrial Revolution, human activity has been competing with the solar and
geodynamic (e.g., volcanos) influences on the Earth's atmosphere. A lot more work needs to be
done to have reliable predictions of the response of the Earth to the Sun's changes. In the
meantime, measurements are being made of how the Earth's environment is changing (by NASA's
Earth Science program and others), and scientists in different fields are trying to understand which
factors are responsible for the changes measured.
(d) The biggest unknown is probably predicting long-term changes in solar activity. We hope to
make some progress with SOHO in understanding more about what really causes solar activity in
the first place, by combining different kinds of measurements from different instruments. The
helioseismology instruments measure what is going on inside the Sun (using as probes the sound
waves that the Sun itself generates); that information will be correlated with activity at the visible
surface of the Sun and in the outer layers of the solar atmosphere. Already there is new
information on the dynamo that generates the solar magnetic field. See:
http://soi.stanford.edu/results/agu96.html
(Solar Dynamo Position)
We hope that SOHO will continue operating for several more years, so we can see what happens to
this dynamo as the solar activity increases from its current (1997) minimum level. Go back to
"Solar Flare" questions and answers.
What safety concerns do solar flares and prominences cause? How do we detect them? Who sends
out warnings?
●
That's a good question. As you know, a solar flare is a sudden, intense brightening from part of the
Sun's surface. They erupt very quickly, in just a few minutes, and they emit radiation from
virtually the entire electromagnetic spectrum.
http://sohowww.nascom.nasa.gov/explore/faq/flare.html (4 of 12) [2000/01/11 11:57:27 AM]
Dr. SOHO's FAQ: Solar Flares
This radiation (gamma rays, x-rays, visible light, etc.) reaches the Earth in about 8.3 minutes.
Earth's atmosphere protects us from the radiation, and the Earth's magnetic field shields us from
most charged particles. Basically, if you're on the ground, you are safe from flares!
The polar regions can absorb slightly higher radiation doses during solar events - charged particles
stream down to the poles along the magnetic field. That's why we see "northern and southern
lights." As a result, airline passengers flying near the Arctic may get higher-than-normal radiation
doses after a solar flare. I am told that transatlantic airliners carry radiation monitors for this
reason. I know that the space shuttle carries such monitors. The shuttle flies high above the
atmosphere, so it is less protected from flares. In theory, a large-enough flare could force NASA to
cancel a shuttle launch or end a mission early -- but that hasn't happened yet. Orbiting shuttles are
also well within the Earth's magnetic field, which gives the astronauts some protection.
The Sun follows an 11-year activity cycle. Flares occur more often at the peak of the cycle (next
peak: 2000-2001). It is still impossible to predict an individual flare, just as it is impossible to
predict exactly when a tornado will happen. We know what solar conditions can accompany flares,
though. One group that monitors these conditions is NOAA's Space Environment Center. Their
excellent web page is at:
http://www.sec.noaa.gov/
(NOAA Space Environment Center) In particular, they post daily forecasts with real-time solar
data at this page:
http://www.sec.noaa.gov/today.html
(Today's Space Weather)
On this page, you will find an orange graph of X-ray levels. When this level "spikes," that means a
flare has happened. Below this is a bar graph with green, yellow, and red bars. The length of these
bars indicates the "danger level" for satellites in Earth orbit. (You asked how we detect solar flares.
Well, besides solar telescopes and spacecraft like SOHO, there are special solar flare detectors on
most of America's weather satellites. That's where NOAA gets most of its "space weather" data.)
This "space weather" that people are now talking about can affect machines as well as people. A
solar flare can fry a satellite's computers. When the magnetic field gets charged up by a solar
storm, it can induce power blackouts on the Earth itself -- this happened in Canada in 1989.
You also mentioned prominences. Prominences are simply big "ropes" of gas hovering just above
the Sun's surface. Sometimes these ropes will break loose and fly off into space. This eruption can
take place during a CME or "coronal mass ejection." A CME can take 2 to 4 days to reach the
Earth. When it hits, it can charge up Earth's magnetic field and cause even more of those nasty
space weather effects. Luckily, the LASCO instrument on SOHO has a pretty good track record
for detecting dangerous CMEs. We at SOHO warn NOAA whenever we see one.
Basically, you're safe from flares and CMEs on the ground. However, your phone company and
electric company may not be so lucky. They are the ones that watch space weather carefully.
Go back to "Solar Flare" questions and answers.
http://sohowww.nascom.nasa.gov/explore/faq/flare.html (5 of 12) [2000/01/11 11:57:27 AM]
Dr. SOHO's FAQ: Solar Flares
Is it possible SOHO could mitigate the destructive effects of solar flares on satellite electronics by
providing an early warning system? How plausible and how effective would this be? Also, how much
of a threat are solar flares to orbiting satellites? Is there any published evidence which discusses solar
flares and their effects?
●
You are right, knowledge of how to predict solar flares would indeed be beneficial in many ways.
Unfortunately, we are not yet very good at such predictions! But let me take a step back, before
going further. When the media speak of solar flares and their damaging effects, they may
sometimes actually be speaking more generally about solar emissions and the damage of several
sorts related to these emissions.
Now, in addition to its usual, very steady output of heat and light, the Sun emits short, explosive
bursts of energy in two forms, as:
(1) radiant energy (heat, light, radio waves, x-rays, gamma rays, and ultraviolet), and
(2) energetic particles.
Explosive releases of radiant energy are called "flares." Sometimes flares eject very high-energy
particles that can be damaging, as can the gamma rays, etc. But large (enormous!) explosive
releases of solar plasma (i.e. very hot, ionized gases) can occur without any flare. These are called
"coronal mass ejections" (CMEs). Sometimes flares and CMEs occur together, but either can occur
without the other, and frequently do.
As to predictions, we have worked very hard for decades to learn the solar conditions that will give
rise to a large flare, but it is still pretty much like predicting hot summers, or earthquakes -- you
can do a moderately good job on long term prediction, but the short term predictions needed for
operational damage control lack adequate precision. We still have a lot of work to do.
CMEs are no easier to predict, but there is a difference here. When a flare is observed, its
dangerous radiations arrive within minutes. But when a CME is observed leaving the Sun (which
we can do with SOHO!), the plasma cloud, with all of its entrained electrical and magnetic energy,
cruises for about two and a half or three days across the 93-million mile chasm of interplanetary
space before impacting the Earth -- and all the sensitive satellite and ground-system technologies
that mankind has deployed. So, this kind of prediction is "almost there," and we continue to work
very hard on making it reliable.
The U.S. government agency responsible for "space weather" predictions is the National Oceanic
and Atmospheric Administration (NOAA). I will list below their web address, so that you may see
for yourself what kind of activities they are up to. And to answer your question about a flare
reference, the second address is for a web site here at NASA/Goddard that has a nice tutorial on
the subject.
NOAA Space Environment Center, Boulder, Colorado: http://www.sec.noaa.gov/
(Space Environment Center, choose"Space Weather") NASA Goddard Solar Flares, Greenbelt,
Maryland: http://hesperia.gsfc.nasa.gov/sftheory/
(The Solar Flare Theory Page)
Go back to "Solar Flare" questions and answers.
http://sohowww.nascom.nasa.gov/explore/faq/flare.html (6 of 12) [2000/01/11 11:57:27 AM]
Dr. SOHO's FAQ: Solar Flares
How would the radiation from the solar flares affect Mir and its cosmonauts? Do they have special
protection against it?
●
The radiation from a big solar flare affects Mir and its cosmonauts in two ways (Note: this also
applies to other spacecraft in low-Earth orbit, including a space shuttle).
One way is that the ultraviolet radiation from the flare heats up the upper atmosphere, causing it to
expand outwards from its normal location. This results in an increased "drag" force on the
spacecraft, causing it to lose altitude. It can be boosted back to its correct orbit by firing small
rocket engines. In Mir's case, the unmanned Progress craft can help reboost the space station
periodically.
The second effect -- and this one is more dramatic -- is the potential health hazard to the astronauts
from energetic particles that arrive in great numbers shortly after the flare. The Earth's magnetic
field provides some protection from these particles, and the metallic body of the spacecraft would
also act as a shield to some extent. So the risk for Mir astronauts is not as great as it would be on,
say, a flight to the Moon, which goes outside the protective magnetic field of the Earth.
However, astronauts in low Earth orbit could still be exposed to dangerous radiation doses
following a large flare. The exposure is greatest for spacecraft in so-called "polar" orbits that go
close to the Earth's north or south poles, because particles can more easily penetrate the
magnetosphere in these locations. (Some unmanned satellites fly in polar orbits, at or near 90
degrees of inclination. Neither the shuttle nor Mir do, although the latter comes close in its 51.6
degree inclination orbit.) Perhaps you have heard of the "Van Allen Radiation Belts?" Almost all
of those particles in the Van Allen belts originally came from the Sun.
Large or chronic doses of ionizing radiation can cause, among other health effects, cancer.
We at SOHO are not experts in the biological effects of space radiation, and I can't really say how
big a dose would be needed to be of serious concern. I do know that Johnson Space Center, which
runs Mission Control and the entire American manned space program, maintains a Space
Radiation Analysis Group (SRAG). This SRAG keeps track of solar activity, and they decide
when the astronauts are in danger. Their counterparts in Russia do the same for Mir's cosmonauts.
NOAA and various solar observatories in the western USA keep Johnson's SRAG informed of
emerging solar events.
Again, we are not part of Johnson's chain of command, so I am uncertain about the protocols for
radiation danger. Should a large event occur during a space mission, Mission Control could
probably rearrange the astronauts' schedules, cancel or postpone spacewalks (a spacesuit gives
someone less shielding than the shuttle's mid-deck), or even plan an abort. (Unmanned spacecraft
like SOHO have nowhere to abort to, but SOHO survived her first solar particle event in
November 1997 just fine.)
Remember, I am just speculating about Houston's contingencies. If you want further information
on this, you might try looking at the Johnson Space Center web site. Their address is:
http://www.jsc.nasa.gov
(JSC Home Page)
http://sohowww.nascom.nasa.gov/explore/faq/flare.html (7 of 12) [2000/01/11 11:57:27 AM]
Dr. SOHO's FAQ: Solar Flares
Solar flare radiation hazards are a concern for Mars mission planners. Manned Mars missions
might be scheduled for around AD 2007 or 2018 simply because the rate of major solar flares is so
much lower during these solar minimum periods.
Go back to "Solar Flare" questions and answers.
On a radio talk show, a guest told us a CME is expected which will create much damage. Does this
site have anything to say about that?
●
Hello! I need to clear up a couple of points first: CMEs (coronal mass ejections, which are
magnetic-gas clouds ejected from the Sun) and solar flares (which are sudden bursts of activity on
the Sun's surface) are two different things. However, they sometimes occur together.
Also, SOHO is a scientific research mission, not an operational forecasting satellite. The official
site for "space weather" forecasts in the USA is:
http://www.sec.noaa.gov/
(The NOAA Space Environment Center)
and, in particular:
http://www.sec.noaa.gov/today.html
(NOAA's report of Today's Space Weather)
Having linked you all over the map, I'll try to answer your original question! Space weather is the
technical term for the Sun's various effects on the Earth's environment -- mainly Earth's magnetic
environment (global warming and long-term climate are a different area entirely).
These effects can take many forms. Hard radiation from a solar flare will hit the Earth in minutes,
at (I think) the speed of light. Our atmosphere and magnetic field protect us from this, although
you would be in danger if you were standing on the Moon. Charged particles in an Earth-directed
CME take 70 to 95 hours to reach the Earth. When they hit, they flow around the planet's magnetic
field. When this happens, the induced currents can cause problems for communications satellites,
long-range radio transmissions, utility grids, and even oil pipelines. (Geomagnetic currents can
increase pipeline corrosion rates.)
The exact timing and severity of such effects is still hard to forecast. The worst problems people
have reported in years past include shorted-out satellites and major power blackouts. In March
1989, there was an extensive blackout in eastern Canada caused by a solar event. GPS signals can
also be disrupted; I read in Aviation Week magazine that some Allied soldiers in the 1991 Persian
Gulf War had difficulty locking their GPS signals. At the time, it was thought to be Iraqi jamming.
In reality, it was simply a high level of solar activity.
1989-1991 was the last solar cycle peak, you see. The Sun follows an 11-year cycle. We are just
coming out of a sunspot minimum, and the next peak will be around 2000-2001. At sunspot
maximum, solar flares and CMEs will probably happen much more frequently.
I hope this helps. Basically, solar eruptions won't cause the sky to fall or society to collapse, but it
is unwise for certain parts of our technological civilization to ignore them. Hence, we have the
NOAA Space Environment Center, and SOHO, among other things.
http://sohowww.nascom.nasa.gov/explore/faq/flare.html (8 of 12) [2000/01/11 11:57:27 AM]
Dr. SOHO's FAQ: Solar Flares
A Southern Hemisphere perspective on space weather is provided by the Ionospheric Prediction
Service:
http://www.ips.gov.au
(IPS Home Page)
Go back to "Solar Flare" questions and answers.
I understand that some stars produce more solar flares than others, yet I was wondering if there is a
classification system for stars concerning their productivity of solar flares. If you know or can send it
to me, I would be much obliged.
●
A colleague at SOHO gave me this answer:
"No, there really is no categorization of stars by flare activity. It used to be that some of the dMe
stars were the only known flare stars. Now we know the RS CVn stars have even bigger flares, but
in fact flares have been found on many types of stars now. This is no surprise, since the Sun flares.
But the amount of flaring is not reflected in any categorization, i.e. there is no Type X RS CVn
star or Type Y dMe star where X and Y would reflect some flare parameter." ("dMe" is a form of
M-class star).
A solar physicist, who works at the Solar Data Analysis Center out here at NASA Goddard,
pointed out to me that, while the Sun qualifies as a "flare star", it isn't especially active compared
to many of the flare stars that have been discovered. This sort of makes sense, in that until recently
it was rather difficult to detect any but the very largest flares on remote stars.
To generate flares, a star must have a corona and a strong magnetic field; no-one has yet been able
to fully explain why the Sun has a strong magnetic field, or why it behaves the way it does -- the
emergence of active regions, and the nature of the 11-year solar sunspot cycle (which involves the
Sun's magnetic field reversing itself every 11 years!) are still deep mysteries.
Work is being done on probing the solar interior with SOHO's MDI instrument, which is engaging
in the "Solar Oscillations Investigation". You can spot more stuff (including solar magnetograms
and cutaway views of interior flows in the Sun) at:
http://soi.stanford.edu
(SOI Web Home Page)
Go back to "Solar Flare" questions and answers.
Where can I find and keep track of solar proton events, fluence and spectra, maybe flux? Solar
flares are fascinating, but I have a practical interest, I am a solar array designer, and I would like to
keep my flare models based in reality.
●
There are a number of sites which keep track of solar flares. An excellent place to start, which
contains many links, is:
http://espsun.space.swri.edu/spacephysics/resrc/solar.flares.htm
(SWRI, Space Plasma Physics: WWW Resources)
http://sohowww.nascom.nasa.gov/explore/faq/flare.html (9 of 12) [2000/01/11 11:57:28 AM]
Dr. SOHO's FAQ: Solar Flares
I'm not sure how detailed your flare modeling effort is, (I assume you are interested in the high
energy flux from UNTITLEDUNTITLED flares) but the above link has pointers to the
NASA/Goddard Solar Flare Theory Page which contains a nice overview:
http://hesperia.gsfc.nasa.gov/sftheory
(The Solar Flare Theory Page)
Another good place to go to is the Space Environment Center, which is part of NOAA:
http://www.sec.noaa.gov
(NOAA Space Environment Center)
This is really the main "Space Weather" page, and it probably has the most direct information for
you in terms of terrestrial effects from the Sun.
Other useful flare sites include:
http://osse-www.nrl.navy.mil/solarflare/flarelib.htm
(OSSE Solar Flare Observations from the Gamma Ray Observatory)
http://umbra.nascom.nasa.gov/SEP/seps.html
(Solar Proton Events)
http://umbra.nascom.nasa.gov/batse/batse_years.html
(BATSE Solar Flare Server from the Gamma Ray Observatory)
I think this is plenty for you to get started. Please feel free to contact us about questions you may
have concerning any of the above sites and links.
Go back to "Solar Flare" questions and answers.
Please explain why impulsive spikes in the flare emission are often seen in microwaves, extreme
ultraviolet (EUV), and hard X-rays, but not in soft X-rays.
●
Microwaves and hard X-rays are produced directly by the bremsstrahlung interactions between
fast accelerated electrons and protons in the coronal plasma. Accordingly, their time profiles
correlate well with the electron injection profile, which can have variations on timescales of
milliseconds.
EUV radiation is formed in the transition region and upper chromosphere, where accelerated
electrons are eventually absorbed. The absorbed electrons act to heat the plasma. EUV emission is
produced when the heated plasma cools rapidly by radiation. Because of this causal relationship,
the EUV time profile is correlated with that of the electrons.
Soft X-rays are produced when some of the plasma that is heated by electrons cannot cool
sufficiently by radiation (because it is not dense enough, or the electron heating rate is faster than
the cooling rate). In this case, the heated plasma expands upwards in a process called evaporation.
The timescale for the build-up of soft X-ray plasma is determined by how fast evaporation occurs
(typically about twice the sound speed) and the size of the flaring coronal loop. For typical loop
lengths of 109 cm, this timescale is about 30-60 seconds, i.e, much longer than EUV and HXR
spikes.
http://sohowww.nascom.nasa.gov/explore/faq/flare.html (10 of 12) [2000/01/11 11:57:28 AM]
Dr. SOHO's FAQ: Solar Flares
Go back to "Solar Flare" questions and answers.
Please explain the mechanisms of the different kinds of meter wave burst, and relate their
occurrence to a flare's time evolution.
●
Solar radio bursts have been classified into 5 different classes, when they were first discovered in
the 1950s/60s, mainly by Australian radio astronomers at metric wavelengths:
(a) Type I bursts are produced by plasma turbulence that occurs most of the time (about in 60%)
even if there are no solar flares.
(b) Type II bursts have very low frequency-drifts, indicating a low velocity that corresponds to
shock waves in the solar corona. They propagate out into interplanetary space after large flares.
(c) Type III bursts are most common and originate from propagating electron beams with
relativistic velocities of v/c=0.1-0.3, producing plasma waves on their way.
(d) Type IV bursts are believed to originate by some coherent emission mechanisms from trapped
electrons in flare loops.
(e) Type V bursts are a mixture of Type III bursts with some extended emission in time after an
electron beam passes.
Type III and V bursts occur during the main impulsive phase of a flare, while type IV bursts
endure much longer, often for hours after the flare.
Go back to "Solar Flare" questions and answers.
Do the project people at EIT use a flare model loop (similar to the x-ray model loop described at the
Yohkoh website) in your analyses of the extreme ultraviolet (EUV) activities?
●
No and yes. There are a few loop models that are widely used. The basic structure of the loop is
similar in all the models, while some of their details may vary. I would say, yes, modeling loops
seen by EIT should be very similar to that seen by Yohkoh. Notice that I use the word "should,"
which is key here in terms of our modeling.
EIT is (as of late 1997) just beginning to model loops! Therefore, we haven't really progressed to
the point of using any specific model. Up until this point we have concentrated on other important
areas, such as day-to-day operations of the instrument, calibration, global morphology, etc. We are
really just beginning to scratch the surface of all the EIT data (Yohkoh has the advantage of being
well into their mission).
We are seeing some discrepancies with previous loop models, such as the temperature variations
with height (EIT is more isothermal than some of the models). We are still investigating this,
however.
Have you seen the Goddard Solar Flare Theory website?
http://hesperia.gsfc.nasa.gov/sftheory
(The Solar Flare Theory Page)
http://sohowww.nascom.nasa.gov/explore/faq/flare.html (11 of 12) [2000/01/11 11:57:28 AM]
Dr. SOHO's FAQ: Solar Flares
Go back to "Solar Flare" questions and answers.
●
Does a sun flare start a heat wave?
A solar flare doesn't cause a heat wave here on Earth, but it certainly heats things up on the Sun!
Seriously, however, there may be some relationship between solar activity and the Earth's climate.
If so, we haven't figured out the details yet:
http://hesperia.gsfc.nasa.gov/sftheory/questions.htm#weather
(Do solar flares have an effect on the weather?)
http://gcrio.ciesin.org/CONSEQUENCES/winter96/sunclimate.html
(The Sun and Climate)
Go back to "Solar Flare" questions and answers.
Go back to "Dr. SOHO's FAQ."
SEND US YOUR COMMENTS
Go To
Other SOHO Web Pages
Author: Shane Stezelberger
Co-Author/Curator: Therese A. Kucera
[email protected]
Responsible Official: Art Poland([email protected])
Last modification: Monday, 08-Nov-1999 10:05:02 EST
http://sohowww.nascom.nasa.gov/explore/faq/flare.html (12 of 12) [2000/01/11 11:57:28 AM]
Dr. SOHO's FAQ: The Earth
Go back to "Dr SOHO's FAQ"
Go back to the "SOHO" main page.
●
What is a solar storm?
●
Is this latest solar eruption dangerous, for us or our computers?
●
Can CMEs cause people harm, and if so, where can the public receive warning of them?
●
Will the aurora be seen further south after this latest eruption?
●
At what latitude will I be able to see the aurora?
●
When in the future will be a good time to see the aurora?
●
SOHO has made many predictions of aurorae recently, all of which were wrong. Why?
●
Where can we get advance warning of future aurorae?
●
Where can I find satellite pictures of the aurora borealis?
●
Does solar activity affect Earth weather and/or "el Nino?"
●
Are there any studies linking Earth weather to the sunspot cycle?
●
Are there any studies linking earthquakes to the solar cycle?
●
Where can I find out about the current status of the magnetosphere?
●
Could solar storms contribute to the Earth's magnetic field reversals?
●
Why does the Earth spin?
●
How fast does the Earth spin?
●
Where can I find solar-cycle data to compare with forest-growth records?
●
Where can I find out about upcoming eclipses?
●
Do other planets' magnetic fields affect Earth?
●
What is the distance between the Earth and the Sun?
●
How can I calculate the position of the Sun any time of year?
●
I think I saw halo rings around the Sun. What were they?
http://sohowww.nascom.nasa.gov/explore/faq/earth.html (1 of 19) [2000/01/11 11:57:35 AM]
Dr. SOHO's FAQ: The Earth
●
●
Where can I find pictures of the Earth?
●
Can nuclear explosions disrupt the Earth's magnetic field?
●
Where can I find the latest A and K radio propagation indices?
Would you mind answering these questions:
What is a solar storm? How do they start up? How do they affect the globe? When and where will one
come again?
A geomagnetic storm (also called a "solar storm") occurs when the magnetic field of the solar
wind couples with the magnetic field of Earth. The solar wind is composed of high-speed charged
particles embedded in a magnetic field.
Storms tend to occur either when the Sun emits a vast magnetic bubble called a "coronal mass
ejection" (CME) or when certain kinds of structures in the Sun's magnetic field sweep by the Earth
as the Sun rotates.
When the fields of the Earth and solar wind meet, they can create instabilities in the
magnetosphere which result in increased currents in the Earth's ionosphere and/or magnetosphere.
These currents can cause power-utility blackouts at high latitudes as these currents induce other
currents in power lines on the ground.
There can also be problems with Earth satellites as charged particles disturb their electrical
systems. This last effect can also be a problem if the solar wind "pushes" the magnetosphere on the
sunward side of Earth down close enough to Earth that spacecraft in Earth orbit as exposed directly
to the solar wind. If this happens, the satellite may be vulnerable to electrical charging problems.
We are not yet able to predict the occurrence of a major geomagnetic storm very reliably. With the
SOHO, POLAR, and WIND spacecraft, it is possible to tell when a CME is headed for (and
starting to affect) the Earth. Most such events do cause storms, but not necessarily big ones.
The main threat these storms pose is towards technology. There was a large power outage in
Quebec in 1989 which is thought to be the result of a solar storm. A number of satellites have also
been lost during extreme storms. There is some threat of increased radiation dosage to astronauts
and even people in airplanes flying high over the poles. As you can see below, several government
entities keep track of this "space weather" phenomena. They issue alerts to power utilities, airlines,
and communications companies during geomagnetic storms.
There are a number of places you can go for more information. The Educational site of the
International Solar Terrestrial Physics project:
http://www-spof.gsfc.nasa.gov/istp/outreach/
(Mission to Geospace)
especially:
http://www-spof.gsfc.nasa.gov/istp/outreach/geospace.html
(What is Geospace?)
http://sohowww.nascom.nasa.gov/explore/faq/earth.html (2 of 19) [2000/01/11 11:57:35 AM]
Dr. SOHO's FAQ: The Earth
The Australian Ionospheric Prediction Service has some excellent essays at:
http://www.ips.gov.au/papers/
(IPS Home Page)
And the USA's Space Environment Center also has information at:
http://www.sec.noaa.gov/
(NOAA Space Environment Center)
Go back to "Earth" questions and answers.
Dear Dr. SOHO: I am concerned about the effects of this latest solar eruption -- am I or my
computer in any danger?
●
April 7, 1997 was an exciting date for scientists, but a large eruption on the Sun poses no danger to
humans on the Earth. The atmosphere and magnetic field of this planet shield us (and most
low-altitude computer systems) quite sufficiently from the particle radiation of solar flares. (If you
were a communications satellite, then I would worry. I would worry about flares and coronal
mass ejections -- which are two different things, by the way. But you are not a satellite, so I would
not worry.)
We are learning more about these events, and the excitement generated by this phenomenon is
because we are just beginning to observe them well for the first time. In fact, the Earth has endured
countless numbers of solar storms for eons -- many of them were much more severe than this one.
It is unfortunate that some of the excitement may be also resulting in some anxiety -- there is
certainly nothing to worry about! As far as your computer goes, it is at much greater risk during a
thunderstorm than it is during a geomagnetic storm. Still, there is a chance that generators can be
affected, but the statistics boil down to a very minor probability. In other words, instead of hiding
inside and preparing for the worst, I am going to be outside looking for some of the most glorious
aurora (the "Northern and Southern Lights") to be seen for many years! (Although we may not see
any as far south as here, near Washington, DC. Predicting auroral strength is still "difficult,
bordering on impossible" at present.)
You don't have to be a scientist to be excited about this -- the Earth's environment, "geospace," is
an exciting and dynamic place. Learning more about our surroundings doesn't change Nature or
make us any more prone to these events, it only increases our understanding of it. I, for one, am
thrilled to see beautiful pictures of our Sun erupting, followed by the display of aurorae above the
Earth's polar regions afterward.
Go back to "Earth" questions and answers.
Is it a possibility that these CMEs could cause people harm, such as radiation exposure? If so, why
haven't we been warned?
●
http://sohowww.nascom.nasa.gov/explore/faq/earth.html (3 of 19) [2000/01/11 11:57:35 AM]
Dr. SOHO's FAQ: The Earth
No, there's not much to fear from CMEs from a purely biological standpoint. These mass ejections
are simply ionized gas -- mostly protons and electrons. As they sweep past the planets, they can
damage electronics systems aboard spacecraft. Fortunately, as indicated below, the Earth's
magnetic field and atmosphere protect us from this type of solar activity.
Now, when a solar flare is ejected, many particles are accelerated to high velocity. They become
what we call cosmic rays, although cosmic rays can also come from other sources in deep space.
These particles can damage biological tissues -- but, again, we here on Earth protected in two
ways. First, the Earth's magnetic field tends to divert the particles around the planet, protecting
humans at low latitudes from charged cosmic rays and the solar wind.
Second, the atmosphere itself stops most incoming particles. This is most important at high
latitudes, where the magnetic field can actually direct particles downward toward the atmosphere
(and the ground!). When a strong burst of particles are stopped by our atmosphere, they make the
upper layers glow just like a neon light -- causing the aurora or "Northern and Southern Lights."
(While flying at high altitudes, high latitudes, or both, pilots and passengers are exposed to higher
radiation doses from the Sun and other cosmic sources. Such doses are not nearly large enough to
cause health effects after one flight, but cosmic radiation can be a concern for career pilots and
pregnant crew members. For this reason, most airlines, especially those with transatlantic routes,
monitor cabin dosage and solar activity. I am being vague because we at SOHO aren't experts in
low-level radiation issues. Please consult your library for reputable books on this subject.
Although it is easy for discussions of "radiation" to become burdened with political rhetoric, you
must remember that no military project or power utility invented solar flares and cosmic rays.)
Most other hazards of solar activity are due to our use of (and dependence upon) technology.
When bursts of particles strike the Earth's magnetic field, they deform it. The changing magnetic
field induces currents in long power lines and telephone wires, which can damage the equipment
attached to those wires.
When an aurora happens, the fluorescing atmosphere also gives off radio interference, which
disrupts our global communications nets and can make aircraft navigation difficult.
While people here on the planet's surface are well shielded from solar storms, astronauts in low
Earth orbit are not. Astronauts in a space shuttle and in space stations (like Mir) need to worry
about the very strongest solar events, though they are mostly within the Earth's shielded space.
Astronauts on any future interplanetary missions would be completely unshielded from cosmic
radiation, unless they brought their own radiation shielding with them.
Spacecraft that are in higher Earth orbits (in geosynchronous orbits, for example) are also at risk.
Comsats (space-nerd lingo for "communications satellites") and other types of satellite are
occasionally disabled -- often permanently -- by solar events.
Go back to "Earth" questions and answers.
●
Given the size of this (April 1997) event, would any auroral activity likely be viewed farther south
http://sohowww.nascom.nasa.gov/explore/faq/earth.html (4 of 19) [2000/01/11 11:57:35 AM]
Dr. SOHO's FAQ: The Earth
than normal? I think it was 1986 when the aurora was visible down to latitude 38 while I was visiting
friends down that way.
To put it simply, it is very difficult to predict the extent of auroral activity before it happens. (I am
typing this from somewhere below latitude 40. We failed to see auroral activity in suburban
Washington, DC last night.) There are just too many unknowns, which is part of why the
International Solar-Terrestrial Physics project exists. At this writing, (10 April, 20:27 UT), there
are unofficial reports coming in from auroral-tracking spacecraft that increased auroral activity has
begun.
However, scientists (and we have at least one expert on geomagnetohydrodynamics in this office)
can't yet predict the maximum southward extent of aurora borealis/australis just by looking at the
initial solar event.
It is very well-established, though, that strong geomagnetic storms include unusually bright auroral
activity, extending unusually far south. There were aurorae seen as far south as Florida a few times
in the last several solar cycles. It takes a bigger solar event than the April 1997 one to create that
level of activity.
Go back to "Earth" questions and answers.
I'm in Britain and am just wondering: will I be able to see the 'Northern Lights' where I am, 52
degrees North?
●
Thanks for your query. Unfortunately, I cannot provide you with any satisfying answer. Neither
can the physicists here; prediction of the southward extent of an aurora based on observation of the
formative solar event is problematic.
In other words, we just don't know how to predict that sort of thing. Yet.
Scientists can warn of stronger and more southward- extending aurorae as a complement of
geomagnetic storms. But it is difficult to put a number, or a latitude, on it. "Difficult and
approaching impossible," as one scientist told me.
I can tell you that they felt that the April 1997 storm was a moderate to
slightly-more-than-moderate storm event. When it hit the Earth, it caused bright aurora borealis
(and aurora australis, too, I believe).
How long have you lived at 52 N? Have you ever seen northern lights from there? If so, did such
times correspond to previous known solar storms? In my opinion, you might stand a chance -certainly a better one than that of us here at latitude ~40 N.
To check the state of the auroral oval at any time, try this NOAA web site:
http://www.sec.noaa.gov/pmap/index.html
(NOAA Auroral Activity)
Good luck, and query Dr. SOHO again if you think we can help you!
http://sohowww.nascom.nasa.gov/explore/faq/earth.html (5 of 19) [2000/01/11 11:57:35 AM]
Dr. SOHO's FAQ: The Earth
Go back to "Earth" questions and answers.
Dear Sirs: I am planning a trip to Alaska in 1999 to see the Aurora Borealis. I was wondering if
you could help me plan my trip by sending me information about places where I could get a good view
of the Aurora, and also the time of the year when chances to see the Aurora are at best. Is the fall of
'99 a good time? I was hoping not to have to endure Alaska's winter. (I live in Brazil, near the
Equator). I'd really appreciate if you could help me.
●
Northern Alaska and Norway are two of the best places to see the aurora borealis. Since the aurora
is a direct result of solar storms, a strong solar flare can result in an aurora at lower latitudes -- as
happened in April 1997. We cannot yet accurately predict when solar storms will occur, but your
best chances of seeing an aurora (even from a weak storm) will be during the fall-winter months,
when the skies are darker and the air is clearer.
There is a useful Web site at:
http://www.imv.uit.no/english/science/publicat/waynorth/wn1/contents.htm
(Aurora Borealis, at the Tromso Museum web site)
which contains more information that you will find interesting.
The Sun's 11-year activity cycle is currently (early 1998) ramping up toward a maximum, which is
expected early in the next decade. Solar activity will be higher next fall than it is presently. This
will bolster your chances for good aurorae. Good luck!
To check the auroral status in the future, I recommend these two web sites:
http://www.sec.noaa.gov/pmap/index.html
(NOAA Auroral Activity)
http://www-spof.gsfc.nasa.gov/istp/polar/
(The POLAR Spacecraft)
Go back to "Earth" questions and answers.
Over the last six months (since April 1997), there have been a number of predictions of major
auroral events based on the CMEs detected by SOHO. All of these have proved to be in error. Is it
known why this should be so, and what the mechanism is that causes aurora to occur?
●
SOHO was designed for research, not weather prediction -- but our record is better than you
suggest! Look, for example, at the data presented by Dr. Nancy Crooker, Boston University at the
1997 American Geophysical Union's Fall Meeting: SOHO saw 12 halo CMEs between December
1996 and June 1997 -- that is, 12 CMEs which appeared to be headed straight for Earth.
Geomagnetic events accompanied nine of these ejections. (See the newsletter EOS, 16 December
1997.) "Geomagnetic events" may not imply a bright, low-latitude aurora every single time -- but
there were disturbances in the Earth's magnetic field.
Granted, SOHO scientists probably notified the NOAA space weather team about more than 12
http://sohowww.nascom.nasa.gov/explore/faq/earth.html (6 of 19) [2000/01/11 11:57:35 AM]
Dr. SOHO's FAQ: The Earth
events in that six-month period; I do not have the full statistics right here, believe it or not. Still, I
would contest any implication that SOHO is not assisting the space weather community.
Unfortunately, scientists are like weather-people, and our predictions can often be wrong. SOHO
can observe and track CMEs from their formation on the Sun. During the 93 million mile journey
to Earth, the plasma disturbance associated with the CME can change significantly (e.g. weaken).
The aurora is produced when energetic particles (electrons) within the CME plasma and Earth's
magnetosphere are accelerated in Earth's magnetic field and radiate. The brightness of the
radiation depends upon (among other things) the angle at which the CME plasma collides with the
Earth's magnetic field, how much kinetic energy is contained by the CME, and how many particles
get accelerated. These parameters are very hard to predict.
Accordingly, solar forecasters can predict when an auroral event can occur, but not always whether
it will be a significant one.
Go back to "Earth" questions and answers.
I live in central Massachusetts, and last April [1997] I was treated to an unbelievable Aurora while
taking some shots of Comet Hale-Bopp. I was wondering: how much advance notice is available
regarding the Aurora Borealis. Do we know hours in advance? Or is it a few days? Or, might it be
even longer?
●
Also, I know that it is rather unusual to get to see it this far south (although I've been told that
because of the depletion of the ozone layer it is more common).
Could you please shed some light (no pun intended) on this subject? I really don't want to miss the
next one. Please don't tell me to go to Alaska.
(I'm not aware of any connection between aurorae and ozone depletion, but that may just be my
ignorance. I'm a solar physicist rather than an auroral expert.)
I'm glad that you saw the aurora. I still have vivid memories of watching the aurora from a spot
probably not very far from you, in Amherst, Massachusetts. I was assisting at a star party at the
time, and I and about 40 others were treated to the aurora as an added benefit.
Up until recently, we would normally have very little notice of an aurora. Aurorae are caused by
solar storms which sweep past the Earth. Recent studies have shown that coronal mass ejections
(CMEs) - bubbles of magnetic gas thrown out from the Sun at a few hundred kilometers per
second - are the source of the magnetic storms which cause the aurorae.
The procedure that has been set up is: we give our observations and reports to the NOAA Space
Environment Center (which is the official U.S. space weather site). They combine the information
we send them with other data, e.g. GOES X-ray data, and make the official forecast. Therefore, the
place to go for a heads-up on high geomagnetic activity (including the aurora) is:
http://www.sec.noaa.gov/today.html
(Today's Space Weather)
http://sohowww.nascom.nasa.gov/explore/faq/earth.html (7 of 19) [2000/01/11 11:57:35 AM]
Dr. SOHO's FAQ: The Earth
and also here:
http://www.sec.noaa.gov/pmap/index.html
(Auroral Activity from NOAA/TIROS)
With the LASCO coronagraph on SOHO, we now have the sensitivity to see CMEs that are
actually headed toward the Earth. This gives us a 2-3 day warning of the kind of activity that leads
to aurorae. Of course, that doesn't tell you whether or not it will be visible as far south as
Massachusetts. With our better understanding now of the phenomena, data from other instruments,
even from some ground-based observatories, could serve as indicators that an (enhanced) aurora
could be in the offing in the next 2-3 days.
Actually, there's always an aurora at some level near the poles, caused by the solar wind streaming
past the Earth. In Arctic locations, it is visible almost every clear and dark night as a shimmering
"curtain" of mostly green and red light. The aurorae are caused by energetic electrons that collide
with oxygen and nitrogen atoms in the upper atmosphere and cause these atoms to "fluoresce"
(light up). The red and green colors are characteristic of oxygen atoms. Magnetic storms from
CMEs intensify the phenomenon, and allows it to be seen at greater distances from the poles.
However, we still cannot predict the southward (or northward for aurora australis) extent of
auroral activity. The interacting phenomena of solar wind, magnetic field, and upper atmosphere
are quite complex.
Another SOHO scientist answers this question:
Growing up in New York and now living outside of Washington, D.C., I have never seen an
aurora, so I envy you!
The electrons that cause the aurora get energized in the Earth's magnetic field (contrary to popular
opinion, they are not energetic solar particles). The strongest particle energization (and the
brightest aurora) occurs during a "geomagnetic storm", when strong electric fields and large
currents are generated in the magnetosphere (the region of space dominated by the Earth's
magnetism). Geomagnetic storms are closely linked to explosions on the Sun called coronal mass
ejections (CMEs) that sometimes get directed towards Earth. Around the time of a strong
geomagnetic storm, the aurorae can get bright enough and the auroral oval can expand far enough
south that it can be seen from latitudes around the northern US.
The aurora that you saw last April was caused in just this way. There was a CME on the Sun on
April 7th that caused a geomagnetic storm at Earth around April 10th. Bright aurorae were visible
in the northern US for a couple of nights around this time.
The typical warning time for an event of this type is only a few days. Solar physicists are trying to
understand how to predict CMEs, but right now the best we can do is to detect their occurrence. It
takes maybe four to five days for one of these events to travel from the Sun to the Earth, and then
you may see an auroral display.
So, to see another aurora from your location, you'll need clear skies a few days after a solar
eruption that happens to be directed toward Earth and that has just the right properties to cause an
intense geomagnetic storm. With increasing solar activity in the next few years, you should have a
http://sohowww.nascom.nasa.gov/explore/faq/earth.html (8 of 19) [2000/01/11 11:57:35 AM]
Dr. SOHO's FAQ: The Earth
good chance of catching another one. But if you want to be sure of seeing an aurora, then sorry you'll have to go to Alaska!
Additional sites with related information can also be found at:
http://www.dxlc.com/solar
(Solar Terrestrial Activity Report)
http://www.pfrr.alaska.edu/~pfrr/AURORA/
(The Aurora: Information and Images)
The Alaska people seem to have stopped forecasting for the summer [1998], but they may be
useful later. NOAA is the group officially in charge of "space weather" forecasting for the United
States. The "pmap" site contains regularly updated maps of the Aurora from space. You could
check how close the auroral oval is getting to your area.
Go back to "Earth" questions and answers.
At the moment, I am doing a research project in Greenland. Could you tell me where I could find
some satellite pictures of Northern Lights?
●
You might try:
http://www-spof.gsfc.nasa.gov/istp/polar/
(The POLAR Spacecraft)
A colleague also suggested the page:
http://www.sec.noaa.gov/pmap/index.html
(Auroral Activity from NOAA)
Go back to "Earth" questions and answers.
I am a reference librarian, and just today I overheard a group of students discussing our awful
weather this last week or so. Aside from "el Nino," do solar storms in any way affect our "local"
weather? I do remember some years ago visiting in Michigan during a period of heavy solar activity
and experiencing awesome electrical storms -- no rain, just lots of lightning and thunder.
●
The question of the terrestrial effects of solar activity is one of the major scientific problems
NASA is tackling right now. Therefore, the short answer is: we are still learning. I will try to give
some brief descriptions and pointers to much more extensive Web pages.
Solar activity certainly has an effect on the Earth's environment; in particular, on satellites, power
systems, communications, navigational systems, and humans in space. See:
http://www.nas.edu/ssb/spwpt5nw.html
(Space Weather: A Research Perspective)
Typical visual responses to solar activity are the "northern" and "southern lights," which are
http://sohowww.nascom.nasa.gov/explore/faq/earth.html (9 of 19) [2000/01/11 11:57:35 AM]
Dr. SOHO's FAQ: The Earth
caused by solar energetic particles being trapped in the Earth's magnetic field.
Solar activity probably has an effect on long term climatic changes on the Earth. An example is
the Maunder Minimum (1645-1715), which was a period of very low recorded solar activity. This
coincides with what was known in Europe as the Little Ice Age, where it was cold enough to ice
skate on the Thames River in London. The exact mechanism of this relationship is still under
study. Solar activity may also have a small effect on global warming, but again, this is pushing our
current understanding. The Sun probably changes in brightness by 0.3% during the 11-year
sunspot cycle, for example -- some experiments have already demonstrated this variability (e.g.
ACRIM on the "Solar Max" satellite and ACRIM-II on UARS).
However, solar activity is NOT the cause of El Nino. The ocean-current flow patterns in the
Pacific Ocean which cause this effect are under study, but I do not believe any demonstration has
linked this to solar activity. Additionally, solar activity has not been shown to affect "local"
weather (thunderstorms, etc.)
Places of interest into the field of the solar effects on the Earth are:
http://www-istp.gsfc.nasa.gov/
(The International Solar-Terrestrial Physics homepage)
http://umbra.nascom.nasa.gov/spd/secr/
(Sun-Earth Connection Roadmap)
Follow these links down to see many of the latest developments in the field. The Solar-Terrestrial
relationship is one of growing interest and relevance, so stay tuned for more exciting
developments!
I hope this answers your questions. In short, the Sun probably strongly influences Earth's climate,
but probably not its weather. I do not mean to be elusive on the actual effects, but this is a topic of
ongoing research and is not easily summarized in a few paragraphs.
Go back to "Earth" questions and answers.
I have found that there seems to be a correlation between sunspot activity and our Earth weather
systems. Are there any studies that have been or are being done on this subject on the net?
●
Many scientists and laymen have a great interest in the effects that the Sun has on the Earth, both
short term and long term. Given how variable the weather is over the planet, and the number of
other factors which probably have an influence on weather, it is rather difficult to make clear,
detailed connections between solar activity and Earth weather.
This does not mean that there is not a connection -- only that there is a lot of work yet to do to
show reliable correlations that can be given the ultimate test: being used to make predictions. I
have not yet seen any studies relating sunspots and terrestrial weather, on the internet or elsewhere.
Given the number of data bases that are becoming more accessible through the web, I expect such
studies are coming. So I think the best answer to your first question is to stay tuned....
There are many clear, short-term "space weather" effects of solar activity:
http://sohowww.nascom.nasa.gov/explore/faq/earth.html (10 of 19) [2000/01/11 11:57:35 AM]
Dr. SOHO's FAQ: The Earth
http://www.nas.edu/ssb/what.html
(Space Weather: A Research Perspective)
such as geomagnetic storms (which give rise to, for example, the aurora and communication
blackouts). However, much of the emphasis on Sun-Earth connection studies has been on looking
for longer-term correlations of solar activity with global climate.
Even in the area of climate, however, it is hard to allow for effects such as human sources of
pollution and global warming. So a great deal of time, thought, and effort are required to do the job
right, and the field is still young. But this is clearly a very important area of research for
humankind, and NASA is now sponsoring some of the needed research. In fact, here is a quote
about a NASA research announcement which I received recently:
"NASA has just issued Research Announcement NRA 97-OSS-15, a joint solicitation of the
Office of Space Science and the Office of Mission to Planet Earth [or Earth Science] for
basic research proposals relating to the study of solar influences on global change. The
objective of this program is to stimulate research that enhances understanding of the role of
solar variability in affecting terrestrial global climate. The program will support new
research investigations with particular emphasis being given to investigations that (1) make
use of data from past or current space missions, and (2) support past, current, or possible
future space missions, including investigations involving theory, modeling, and historical
data on the connections between the behavior of the Sun and climate."
(Scientists have long suspected that there are connections between the Earth's long-term climate
and solar variations, but again, the details remain unclear. For example, in the late 1600s an early
1700s, European astronomers recorded few sunspots. During these years, a "little Ice Age"
occurred in the Northern Hemisphere, with harsh winters and mild summers. The Thames river
froze over. We call this period the Maunder Minimum, but we do not know why it happened.)
Go back to "Earth" questions and answers.
●
I am looking for collaborative studies on sunspot activity and earthquakes. Can you help?
I have not seen or heard any discussion of a physical relationship between solar activity and
earthquakes. That does not mean that there cannot be such a relationship, but it is not obvious why
there should be one. The power of a solar flare or an ejection of matter from the Sun can be
enormous near the Sun, but the resulting mechanical force at the Earth is a tiny fraction of what
is needed to shift the Earth's crustal plates, as happens in an earthquake. Earthquakes result from
motions of tectonic plates, and the mechanical forces on these plates from solar flares or solar
magnetic storms 150 million kilometers away is almost certainly negligible.
When I got a similar question a few months ago, I contacted a scientist here at Goddard Space
Flight Center who works on earthquake studies. He also was not aware of any studies of
correlations between solar activity and earthquakes, and he did not expect that solar activity could
cause earthquakes. However, he did mention that it was worth considering the possibility that
some electrical (rather than mechanical) coupling of the ionized solar material to the Earth's plates.
But I suspect that we would have heard if anyone had yet found something like an approximate
http://sohowww.nascom.nasa.gov/explore/faq/earth.html (11 of 19) [2000/01/11 11:57:35 AM]
Dr. SOHO's FAQ: The Earth
11-year cycle in earthquake frequency and magnitude, such as has been found for sunspots. If solar
activity and earthquakes are well correlated, they should show similar behavior over time. (It
would not be a good test to try to relate just a few earthquakes and sunspots on a one-to-one basis,
since there are too many other factors which might be important.)
For more information on earthquakes, you might want to check out the USGS National Earthquake
Information Center web page at:
http://wwwneic.cr.usgs.gov
(USGS National Earthquake Information Center)
They have a catalogue of earthquakes from 2100 B.C. to the present, and a variety of information
on what is known about earthquakes.
Go back to "Earth" questions and answers.
I am an amateur astronomer, and I like to watch the aurorae. Does your spacecraft provide any sort
of warning information that is displayed in a manner a layman can use to determine the status of the
magnetosphere?
●
We at SOHO do not post any formal bulletins of impending geomagnetic (and thus, auroral)
activity. The NOAA does, however, at their Space Environment Center. We do notify NOAA
when we see any suspicious events. I think you will find what you are looking for here:
http://www.sec.noaa.gov/today.html
(Today's Space Weather)
and here:
http://www.sec.noaa.gov/pmap/index.html
(NOAA Auroral Activity)
SOHO observes the early eruption of coronal plasma (called CMEs, or coronal mass ejections)
from the solar surface. These eruptions are sometimes (but not always) associated with solar flares
and can travel at over 1,000 kilometers per second. Depending on where it is emitted on the
surface and how fast it is moving, the energetic plasma associated with CMEs can affect the
Earth's aurora within 2 to 4 days. SOHO's LASCO instrument produces images of CMEs as they
erupt and travel from the Sun:
http://lasco-www.nrl.navy.mil/lasco.html
(LASCO Home Page)
To view more layman information on magnetospheric effects, there is a useful site at the
International Solar-Terrestrial Physics project:
http://www-spof.gsfc.nasa.gov/istp/outreach/
(Mission to Geospace)
And for more auroral data, check the POLAR spacecraft:
http://sohowww.nascom.nasa.gov/explore/faq/earth.html (12 of 19) [2000/01/11 11:57:36 AM]
Dr. SOHO's FAQ: The Earth
http://www-spof.gsfc.nasa.gov/istp/polar/
(The POLAR Spacecraft)
Go back to "Earth" questions and answers.
I saw Dr. Art Poland on CNN today talking about the potential impact of a solar storm, and it made
me think about periodic changes in the Earth's magnetic fields. I had read somewhere that we know
from geologic records that the Earth's magnetic field occasionally switches, and that -- in a sense -the north pole becomes the south pole. I recall this information being presented as a mystery of sorts,
and I wondered if it might be possible that the source of these occasional changes might be solar
activity, perhaps incredibly intense solar storms that disrupt things on the Earth on a mammoth scale,
sufficient to reverse the Earth's polarity.
●
As far as we know, the magnetic field reversals of the Earth's field are not related to the Sun. It is
thought that they are due to motions inside the Earth's liquid outer core. Geophysicists do complex
models of the interior and they have started to get to the point where they can simulate the field
reversals. I believe there was an article about that in Physics Today sometime in the last few years.
A fairly technical web write-up of some results is at:
http://www.igpp.lanl.gov/Geodynamo.html
(Core Convection and the Geodynamo)
Another article on the field reversals appeared in American Scientist in the November-December
1996 issue. An abstract is at:
http://www.sigmaxi.org/amsci/articles/96articles/Fuller.html
(The Reversal of the Earth's Magnetic Field)
Go back to "Earth" questions and answers.
●
Why does the Earth spin?
We think that the Sun, the Earth, and the other planets were formed out of a big cloud of gas a long
time ago. The cloud of gas was spinning. As the cloud shrank down (because of gravity) to form
the Sun and planets it started spinning more -- just as spinning ice skaters spin faster when they
pull in their arms. The spinning of the Earth is left over from this original spinning.
Go back to "Earth" questions and answers.
●
How fast is the Earth spinning, in miles per hour?
The radius of the Earth = 6,400 km = 4,000 miles, so at the equator the Earth is 2×pi×r =25,000
miles around.
The earth rotates once around on its axis every 24 hours*, so the velocity at the equator is about
25,000 miles/24 hour, just over 1000 miles/hour.
http://sohowww.nascom.nasa.gov/explore/faq/earth.html (13 of 19) [2000/01/11 11:57:36 AM]
Dr. SOHO's FAQ: The Earth
At higher latitudes the Earth's circumference perpendicular to the rotation axis is smaller, so the
velocity is lower.
*Factoid for the day: Actually the rotation period of the Earth is 23.94 hours, a bit shorter than 24
hours -- 24 hours is the time it takes for the Sun to return to the same place in the sky. This is
determined by the Earth's orbit around the Sun as well as the the Earth's rotation about its own
axis. (In the course of 24 hours, the Earth travels around the Sun just a little. That movement
changes the apparent positions of the stars relative to the Sun in the sky.)
Go back to "Earth" questions and answers.
As a forester in the UK, I am very interested in the growth, survival and seeding of broadleaves and
conifers. On a program last night in the Equinox series, the SOHO project was featured. This was
fascinating, and I wondered whether any past data on solar activity was available in the public domain
or if you could send it to me. I would be most interested to compare this against tree growth, mortality
and mast years for various tree species that my organization has kept since 1919. It would be
interesting to see whether there is any connection, or has this already been done?
●
Do you have access to the web? In the USA, at least, the best place to go for such information is
the National Geophysical Data Center. Probably the best indicator of solar activity over that time
period would be the sunspot number -- most other things (flares, radio flux, etc.) have not been
measured very regularly until the last few decades, if then.
Take a look at:
http://www.ngdc.noaa.gov
(National Geophysical Data Center)
Sunspots are discussed on:
http://www.ngdc.noaa.gov/stp/SOLAR/SSN/ssn.html
(Sunspot Number)
I don't know if there have been studies of possible relationships between solar activity and plant
growth, although Dr. Jack Eddy used tree ring data in his pioneering solar-cycle research in the
1970s. In general, there have been some studies of the effects of solar activity on climate. There
may be some relationship, but if so, we still don't really understand the details or why. See:
http://hesperia.gsfc.nasa.gov/sftheory/questions.htm#weather
(Do solar flares have an effect on the weather?)
http://gcrio.ciesin.org/CONSEQUENCES/winter96/sunclimate.html
(The Sun and Climate)
Go back to "Earth" questions and answers.
●
Please, can you tell us about the time of the 26 February 1998 solar eclipse at the towns of
http://sohowww.nascom.nasa.gov/explore/faq/earth.html (14 of 19) [2000/01/11 11:57:36 AM]
Dr. SOHO's FAQ: The Earth
Oransestad (Aruba) and Winenstad (Curacao)? Exactly what time is the first contact and the second
contact, in universal time (UT)?
Much information about future eclipses, such as the February 1998 eclipse, can be found at the
SDAC eclipse page:
http://umbra.nascom.nasa.gov/eclipse
(Eclipse Information at the Solar Data Analysis Center)
Try this site first, then ask Dr. SOHO if you have any remaining questions. Good luck!
Go back to "Earth" questions and answers.
I know the Sun's magnetic field can affect the Earth. What about the Moon's magnetic field, or
those of the other planets in our solar system?
●
You're quite correct in stating that the Sun's magnetic field affects the Earth. The Sun has a very
powerful magnetic field which is carried out by the solar wind. Sometimes the interaction of the
magnetic field and the solar high-energy particles can produce spectacular effects like the aurora
borealis.
The Moon's magnetic field is extremely low -- so low that it's not clear that it really has one!
There is magnetism in the lunar surface, but no apparent global field. It seems that the Moon once
had a magnetic field, which is why the surface is magnetized, but the interior dynamo has long
since died down.
One source which talks about this is:
http://www-ssc.igpp.ucla.edu/~ccasler/ANCIENT/
(On The Source of the Ancient Lunar Magnetic Field)
Like the Earth, some other planets in the solar systems have magnetic fields. Some of these fields,
like Jupiter's, are very strong. However, the magnetic fields do not reach as far as their
gravitational fields. Gravity decreases from a planet as the square of the distance, while the
magnetic field (more or less) decreases according to the cube of the distance. For example, at the
Earth's surface, a distance of about 4000 miles from the center, the gravitational force is 10
meters/second2 or 1g, and the magnetic field strength is about half a Gauss. Ten times further
away, at 40,000 miles, the force of gravity is 100 times smaller, but the magnetic field strength has
gone down by 1000.
Thus, in comparison to gravitational effects, the effect of the magnetic fields of other planets in the
solar system is negligible at the Earth.
Go back to "Earth" questions and answers.
I am a student living in Vancouver, B.C. and I am doing an experiment to find the wattage of the
Sun. Can you please tell me (or help me to find) the distance of the Earth from the Sun in each of the
twelve months?
●
http://sohowww.nascom.nasa.gov/explore/faq/earth.html (15 of 19) [2000/01/11 11:57:36 AM]
Dr. SOHO's FAQ: The Earth
A preliminary answer: the Earth's AVERAGE distance from the Sun is 93 million miles, or 149.6
million km. (This is from Bate, Mueller, and White, Fundamentals of Astrodynamics, 1971.)
Perfectly circular orbits don't happen in nature; Kepler's big achievement was proving that
planetary orbits are ellipses. The eccentricity of the Earth's orbit (the "e" value in equations of
orbital dynamics) is 0.017. (This number I took from the 1992 TRW "Space Data Book.")
The Earth's "perihelion" (or perigee, or simply periapsis -- whatever term you prefer, it is the
Earth's closest approach to the Sun, at the lowest point of the orbit) occurs during this planet's
northern winter. Likewise, Earth is farthest from the Sun during northern summer. These exact
dates are listed at:
http://riemann.usno.navy.mil/AA/data/docs/EarthSeasons.html
(US Naval Observatory: Times of the Earth's Perihelion and Aphelion 1992-2005)
For a system of calculating the Earth-Sun distance each month, you'll need some formulas from a
science textbook or orbital-mechanics book. The math is too extensive for this answer, and you'll
probably need help from your science teacher in finding the right book. How much math do you
have? You can calculate these orbital positions without calculus, but you will need much geometry
and algebra.
Go back to "Earth" questions and answers.
How can I calculate the position of the Sun in the sky at any given time of the year, for my location
on Earth?
●
The easiest way to obtain the position of the Sun would be the Astronomical Almanac, published
by the US government (Department of the Navy). This book, which is updated annually, should be
available in your local library.
If it is not, below is low precision formula suggested in the Almanac. The error in this formula is
about 0.01 degrees in right ascension and declination. If you need more precision, let me know and
I can send you a much more detailed calculation, e.g. which accounts for many of the planets.
I hope this is useful to you. Let me know if you have any questions concerning the calculations.
(Given jd - The Julian date of the day (and time) in question. Except where noted, all values are in
degrees.)
n = jd - 2451545.0
mean longitude = L = 280.466 + 0.9856474 × n
mean anomaly = g = 357.528 + 0.9856003 × n
Put L and g in the range of 0 to 360 by adding multiples of 360
Ecliptic longitude = Lambda = L + 1.915 × sin(g) + 0.020 × sin(2g)
Ecliptic Latitude = Beta = 0
Obliquity of ecliptic = eps = 23.440 - 0.0000004 × n
http://sohowww.nascom.nasa.gov/explore/faq/earth.html (16 of 19) [2000/01/11 11:57:36 AM]
Dr. SOHO's FAQ: The Earth
Set f = 180/PI and t = tan2(eps/ 2) (tangent squared of eps/2)
Right ascension = alpha = lambda - f × t × sin(2 × lambda) + (f/2) × t × t × sin(4×lambda)
Declination = delta = arcsin(sin(eps) × sin(lambda))
Distance of Sun from Earth in AU = R = 1.00014 - 0.01661 × cos(g) - 0.00014 × cos(2×g)
Equatorial rectangular coords of the Sun in AU =
x = R × cos(lambda)
y = R × cos(eps) × sin(lambda)
z = R × sin(eps) × sin(lambda)
Equation of time (apparent time minus mean time) = E (in minutes)
E = (L - alpha) × 4
Horizontal parallax = 0.0024
Semidiamater = 0.2666 / R
Go back to "Earth" questions and answers.
I was talking with my grandmother, and she mentioned an incident in which there were rings
coming off from the Sun. This happened around 1923 or 1924. The best guess I have been able to
come up with is that this was a rare halo phenomenon. What is your opinion?
●
I can't say from your description exactly what your grandmother saw. It is possible that it was
some kind of halo phenomenon. Rings around the Moon are pretty common. Halos around the Sun
can happen too, but usually the Sun is so bright you can't see them. You can probably find books
about halos and related things (such as rainbows and sundogs) in your library.
You can see an image of a halo around the Sun at:
http://covis.atmos.uiuc.edu/guide/optics/halos/hal.html
(Haloes -- Atmospheric Optics)
It is part of a general site about atmospheric optics at:
http://covis.atmos.uiuc.edu/guide/optics/
(Atmospheric Optics)
Another detail that I forgot: the rings, according to her, were of different colors.
The colors make it sound all the more like it might be some sort of rainbow/halo effect.
Go back to "Earth" questions and answers.
●
Where can I find pictures of the Earth?
The following site should be helpfull:
http://sohowww.nascom.nasa.gov/explore/faq/earth.html (17 of 19) [2000/01/11 11:57:36 AM]
Dr. SOHO's FAQ: The Earth
http://www.nasm.si.edu/earthtoday/
(Earth Today: A Digital View of Our Planet)
Go back to "Earth" questions and answers.
I've been studying the possible effects of Solar activity upon the Earth's magnetic field. This, as well
as planetary gravitational fields and their effect on our planet. I am concerned that we are in for
severe effects. New millennium paranoia you might ask? Maybe...
●
Here's the pessimistic scenario you may have heard. A coming planetary alignment coupled with
intense solar activity will cause measurable changes in the planet's electro-magnetic field... a pole
shift, some say...
Oh, my question:
Can nuclear detonations within and without the Earth's crust and atmosphere also disrupt or affect
this magnetic field?
I don't think you need worry too much about the Earth's magnetic field changing too soon - it only
happens about once every 100,000 years on average! Currently, we're approaching the maximum
activity portion of the Sun's 11-year activity cycle, and that causes some concern for the people
who own and operate satellites, power grids, and phone lines; but for the rest of us, there's not
much to concern us.
You can bet that intense solar activity changes our planet's magnetic field; that's what causes the
induced voltages that worry people who operate power grids, phone lines, and the like. But there's
no evidence that the dynamo deep within the Earth is affected by solar activity - so, as soon as a
solar eruption has passed by, the Earth's field generally springs back to its former state.
Your actual question, about nuclear detonations, is pretty open-ended. Nuclear detonations can
certainly disrupt or affect the Earth's magnetic field - it's just a matter of how large a detonation
you're thinking about. I don't believe that any nuclear device that has been detonated has ever
caused a measurable change in the Earth's global magnetic field. But in the absurd limit (a nuclear
explosion large enough to destroy the entire Earth) the field would certainly be disrupted too!
(Nuclear tests detonated at high altitudes in the early 1960s did inject all sorts of short-lived
radiation into the radiation belts, but I'm unaware of any associated global magnetic-field data.)
Somewhere between current nuclear technology and "planet buster" bombs, there is a size of
detonation that would disrupt the magnetic field. But it would have to be a very, very large
detonation - perhaps large enough to destroy all life on the surface!
Go back to "Earth" questions and answers.
How do I find the latest (or better yet, a curve/graph of past ones) A and K radio propagation
indices?
●
If you are interested in the effects of solar activity on radio propagation, you might want to contact
the NOAA Space Environment Center -- they specialize in such things and have a help page at:
http://sohowww.nascom.nasa.gov/explore/faq/earth.html (18 of 19) [2000/01/11 11:57:36 AM]
Dr. SOHO's FAQ: The Earth
http://www.sec.noaa.gov/info/helpme.html
(Space Environment Center Comments Page)
Go back to "Earth" questions and answers.
Go back to "Dr. SOHO's FAQ."
SEND US YOUR COMMENTS
Go To
Other SOHO Web Pages
Author: Shane Stezelberger
Co-Author/Curator: Therese A. Kucera
[email protected]
Responsible Official: Art Poland([email protected])
Last modification: Monday, 08-Nov-1999 10:05:03 EST
http://sohowww.nascom.nasa.gov/explore/faq/earth.html (19 of 19) [2000/01/11 11:57:36 AM]
Dr. SOHO's FAQ: The SOHO Spacecraft
Go back to "Dr SOHO's FAQ"
Go back to the "SOHO" main page.
●
Where, when, and how was SOHO launched?
●
Who designed SOHO?
●
How much does SOHO weigh?
●
Where can I find diagrams of SOHO and her parts?
●
What can you tell me about SOHO, as a system? How does it all work together?
●
How is SOHO propelled?
●
How much fuel does SOHO have?
●
Where can I find information about SOHO's thrusters?
SOHO's Orbit
❍ How fast is SOHO moving?
❍
Where exactly is SOHO's orbit?
❍
Where is SOHO in relation to the Earth's bow shock?
❍
Is there any possibility that SOHO will burn up?
❍
Please tell us about the orbits of the SOHO and TRACE spacecraft.
❍
How can SOHO stay at the L1 point, which is unstable?
❍
Why won't SOHO be sucked in by the Sun's gravity?
❍
How is the SOHO orbit stable? Or isn't it?
❍
What calculations are necessary to predict SOHO's orbit?
❍
Can you share with me some of the math behind those Lagrange points?
❍
Why does SOHO use a "halo" orbit?
❍
Where can I find orbital elements for SOHO's orbit?
●
So is SOHO a "spacecraft" or a "satellite?" Both? Neither?
●
What is SOHO made out of?
http://sohowww.nascom.nasa.gov/explore/faq/spacecraft.html (1 of 23) [2000/01/11 11:57:46 AM]
Dr. SOHO's FAQ: The SOHO Spacecraft
●
●
What effects do solar eruptions have on spacecraft?
●
Can solar wind particles and UV radiation affect spacecraft?
●
How hot does SOHO get?
●
How do you communicate with SOHO? How do you control SOHO?
●
Is SOHO's software written in Ada?
●
What is the expected duration of the SOHO mission?
●
Has the SOHO mission been extended beyond its 2.5 year primary period?
●
Was SOHO damaged in November of 1997?
●
How did the spacecraft get out of the atmosphere?
Where, when, and how was SOHO launched?
SOHO was launched at 08:08 UT on 2 December 1995 from Launch Complex 36B at Cape
Canaveral Air Force Station, Florida. SOHO's launch vehicle was a Lockheed Martin Atlas IIAS
rocket with a hydrogen/oxygen-propelled Centaur upper stage.
SOHO's launch was originally scheduled for August 1995, but a variety of range, spacecraft, and
launcher issues delayed it to October, late November, and finally early December.
The Atlas IIAS (soon to be replaced by the single-engine Atlas III) is an advanced version of an
early American ICBM. The Atlas has been a workhorse launcher for a few decades; John Glenn
launched on one in 1962. The Centaur upper stage has been sending probes into deep space for
many years, too. Viking, Voyager, and Cassini all used Centaurs.
You can see movies of SOHO's launch and deployment, as well as a transcript of countdown
activities, on our home page:
http://sohowww.nascom.nasa.gov/artwork/#launcher
(SOHO Spacecraft Artwork)
Go back to "Spacecraft" questions and answers.
●
Who designed SOHO?
The SOHO spacecraft was designed and built by the Matra Marconi Space company, formerly
Matra Espace:
http://www.matra-marconi-space.com/
(Matra Marconi Space Bienvenue/Welcome)
For a space mission this complicated, there wasn't one single engineer designing the whole
spacecraft. There were several design teams who assembled all of SOHO's subsystems; the space
design process gets done at the "systems" level. The 12 instruments were each designed by a
different team, each drawn from different laboratories, universities, and countries.
http://sohowww.nascom.nasa.gov/explore/faq/spacecraft.html (2 of 23) [2000/01/11 11:57:46 AM]
Dr. SOHO's FAQ: The SOHO Spacecraft
SOHO was assembled and tested in Toulouse, France and Portsmouth, England. She was flown in
2 large containers to Cape Canaveral, Florida, where final assembly was performed in late 1995.
Launch occurred on 02 December of that year.
Each of the 12 instruments on board SOHO was built at laboratories all over Europe and the USA.
Contributors included the Max Planck Institutes, Stanford University, the US Naval Research
Laboratory, UK's Rutherford Appleton Laboratory, the Finnish Meteorological Institute, the
Smithsonian Astrophysical Observatory, the University of Kiel, and the Institute d'Astrophysique
Spatial, Orsay (IAS).
The whole mission was led by our three Project Scientists: Dr. Vicente Domingo, Dr. Bernhard
Fleck, and Dr. Art Poland.
Go back to "Spacecraft" questions and answers.
●
How much does SOHO weigh?
SOHO had an initial mass of 1861 kilograms (just over 4,100 pounds). She certainly masses a little
less than that now, because we've used up some of our rocket propellant. SOHO launched with 235
kg of propellant in her tank.
(Remember, "weight" is relative to a gravity field; "mass" is not. SOHO would weigh less on the
Moon and more on Jupiter, and in deep space she weighs nothing at all. But her initial mass would
always be 1861 kilograms, no matter where.)
Go back to "Spacecraft" questions and answers.
●
Where can I find a picture of the satellite labeling the parts?
There are a number of diagrams showing the locations of instruments on the spacecraft about
halfway down the SOHO web page:
http://sohowww.nascom.nasa.gov/artwork/
(SOHO Spacecraft Artwork)
In particular, see:
http://sohowww.nascom.nasa.gov/gif/instruments-coronalA.gif
(Coronal Instruments)
http://sohowww.nascom.nasa.gov/gif/instruments-coronalB.gif
(More Coronal Instruments)
http://sohowww.nascom.nasa.gov/gif/instruments-particles.gif
(Particles Instruments)
http://sohowww.nascom.nasa.gov/gif/instruments-helioseis.gif
(Helioseismology Experiments)
for diagrams showing where the different instruments are. There are also images and movies on
the artwork page showing the solar panels. The radio dish used for communication with Earth is on
http://sohowww.nascom.nasa.gov/explore/faq/spacecraft.html (3 of 23) [2000/01/11 11:57:46 AM]
Dr. SOHO's FAQ: The SOHO Spacecraft
the same end as the solar panels -- you can see it in some of the movies.
If you can print out PostScript files, there is also link to a nice PS diagram of the spacecraft near
the top of the page.
Go back to "Spacecraft" questions and answers.
I'd like to ask: is it possible to obtain information regarding system aspects of the SOHO satellite?
(kinds of sensors, data processing, sending data to Earth, etc.) I'm not interested in details, but I'd like
to see how the whole system works in block diagrams, for example.
●
A satellite like SOHO can be broken up into two parts. First of all, there is the spacecraft section.
This part of the satellite is responsible for communicating with the ground, supplying power,
maintaining the attitude and stability of the satellite, and making orbital maneuvers. On SOHO, the
spacecraft section also has a highly accurate clock which provides time signals to the instruments.
Think of the spacecraft section as the vehicle.
The other part of the satellite is the instrument package. In some sense, the instruments are
independent of the spacecraft section. The scientists are able to send commands to the individual
instruments without affecting the spacecraft as a whole. However, the instruments are dependent
on the spacecraft: they can only communicate through it, and get all their power from it. There are
12 different instrument packages on SOHO. Each has their own optics and sensors, and each are
independent. For more information, look at the instrument pages at:
http://sohowww.nascom.nasa.gov/instruments.html
(SOHO Scientific Payload)
You asked about sensors. The spacecraft section contains two main sensor packages. The fine Sun
sensor is used to keep the spacecraft pointed at the center of the Sun. This sensor is highly
accurate, but has one drawback - it doesn't know where "up" is on the Sun. To measure the
satellite's roll angle, the spacecraft also has a star tracker which looks out to the side of the
spacecraft. Using these two sensor packages in conjunction, the spacecraft is able to measure
exactly where it is pointed, and can maintain its attitude through a set of three spinning reaction
wheels (plus a fourth kept as a spare).
The various instruments use a variety of detectors. Some (e.g. EIT, LASCO, MDI) use CCDs,
similar to those in modern camcorders. Others (e.g. SUMER, UVCS, CDS) use
microchannel-plate-based detectors which convert light into amplified electronic pulses, and then
count each individual photon. The latter type of detector is extremely sensitive. The CDS
instrument also includes a hybrid detector which combines a microchannel-plate and a CCD. The
various particle experiments (CELIAS, COSTEP, ERNE) have their own detectors, which operate
in various unique ways, plus some designed to measure the absolute intensity of the Sun in various
wavelength bands (e.g. GOLF).
Communication with SOHO is through the NASA Deep Space Network (DSN). This is a
collection of antennas at various places about the globe which is used to communicate with
satellites. Mostly, the Deep Space Network is used with satellites which are not in close Earth
orbit, such as the Mars Global Surveyor. Since SOHO is three times further away than the Moon,
http://sohowww.nascom.nasa.gov/explore/faq/spacecraft.html (4 of 23) [2000/01/11 11:57:46 AM]
Dr. SOHO's FAQ: The SOHO Spacecraft
it's definitely out in deep space.
Telemetry comes down in major frames, with one major frame every 15 seconds. These major
frames are specially encoded, using an algorithm called Reed-Solomon Encoding, so that the
frames can be reliably reconstructed even when the telemetry is somewhat noisy. This is similar to
the error correction schemes used within computers and compact disks, but probably more
sophisticated. Although it's possible for the satellite to be in continuous DSN coverage, this is not
a typical mode of operation. When the satellite is not in contact with the ground, the telemetry is
stored on board on a tape recorder. This tape recorder is then dumped to ground through a separate
high-speed channel. For example, if the spacecraft has been out of contact for 8 hours, then it will
take about two hours to dump that data from the tape recorder to the ground. After the tape
recorder has been dumped, one of the instruments, MDI, uses that same high-speed channel to
send its own observations down. Thus MDI can only observe when the spacecraft is in real-time
contact and not dumping the tape recorder. There are periods in the year when special effort is
made to keep SOHO in continuous contact so that MDI can observe all the time.
Within the major frames are a series of telemetry packets. There are packets for the spacecraft as
well as packets for each instrument. The packets for the spacecraft are sent through a special
NASA communications network to the Flight Operations Team (FOT) and the packets for the
various instruments are sent to each instrument team. Each team has their own set of computers
which receive these packets through an ethernet connection, and process these packets in various
ways. Packets from tape recorder dumps are also sent to the instrument teams, but as a simple file
transfer. The packets are also sent to a special NASA facility which stores all the packets and then
sends them out to the instrument teams as a set of compact disks, with the real-time data and the
tape recorder dumps all sorted together into a single package. These are referred to as the final
data.
Commands sent to the spacecraft and instruments are similarly bundled up into blocks and sent up,
and basically traverse the system in the opposite direction. Commands from the instrument first go
to a central system which verifies that the commands are coming from a proper computer and that
they are indeed for the correct instrument. There may also be some checking on the syntax of the
command itself - this differs from instrument to instrument. The whole process takes only a few
seconds.
I work on the CDS team, so I can best describe how the CDS packets are processed. The way other
instrument teams work is very similar. First of all, the packets are stored as disk files so that one
can go back and reprocess them at any time. We have some programs written in C on Unix
workstations which scan through the telemetry and display parameters about the current state and
health of the instrument on the screen. Another program also scans the telemetry and looks for
science data, and writes it out into files which are more easily used by the data analysis software.
We use a format called FITS (Flexible Image Transport System), which is a standard within the
astronomical community. Each instrument team processes its own telemetry, using their own
software.
Below are some "nuts-and-bolts" numbers for the mass and power budgets of some SOHO
instruments:
Coronal Diagnostic Spectrometer (CDS) Mass 100kg, avg. power 58 W, telemetry rate 11.3kbit/s,
http://sohowww.nascom.nasa.gov/explore/faq/spacecraft.html (5 of 23) [2000/01/11 11:57:46 AM]
Dr. SOHO's FAQ: The SOHO Spacecraft
length 1.7m
Energetic and Relativistic Nuclei and Electron experiment (ERNE) 382 x 185 x 163 mm, 9.305kg,
768 bits/s
Extreme-Ultraviolet Imaging Telescope (EIT) ~2kbits/s nominal, ~24kb/s high-rate (shared with
LASCO)
Large-Angle and Spectrometric Coronagraph (LASCO) 135 x 34 x 32 cm, 7.9kbp/s nominal,
26.7kbps high-rate
Michelson Doppler Imager (MDI) Mass 56.5kg, avg. power 38 W, telemetry rate 160kbit/s
high-rate, 5kbps continuous
Solar Ultraviolet Emitted Radiation (SUMER) focal length 1302.77mm,
Solar Wind Anisotropies (SWAN) total mass 13.25kg, avg. power 11W, telemetry 200 bps
Ultra-Violet Coronagraph Spectrometer (UVCS) 100kg, 2.bm length, 0.9m diameter, 2.5A/27V =
71 W avg power, 5kbits/s telemetry
Go back to "Spacecraft" questions and answers.
●
How is the satellite propelled?
It is propelled with hydrazine-fueled thrusters.
Deep in the heart of SOHO's service module, in the center of the spacecraft, is a big, spherical tank
of hydrazine rocket fuel. (Hydrazine is nasty, toxic stuff, but it is highly energetic!) This tank feeds
four sets of thrusters, located on long posts sticking up out of the service module. They're very
hard to see in pictures of the spacecraft.
These thrusters burn the monopropellant hydrazine (no oxidizer necessary) on a pelletized catalyst
bed. There is enough hydrazine fuel aboard for another 20 years (as of early 1998) of orbital
maneuvers. SOHO's mission probably won't last that long, so we do not need to worry about
running out of fuel!
Some other rocket fuels were used to launch SOHO, of course; the spacecraft didn't fly all the way
out to the L1 point under her own power. SOHO's Atlas IIAS launch vehicle burned liquid oxygen
and kerosene. The Centaur upper stage burned LOX and liquid hydrogen -- the most powerful
propellant type yet available, and the propellant used by the space shuttle.
All of SOHO's electrical energy for day-to-day operations comes from her solar cells. SOHO was
built with little or no battery capacity; batteries aren't necessary when the spacecraft is in full
sunlight 100% of the time.
(Specifically, and according to prelaunch ESA documents, SOHO had 950 watt-hours of NiCd
battery charge. I assume most of this electricity was consumed in the first few minutes of the
mission, before the solar array deployment.)
http://sohowww.nascom.nasa.gov/explore/faq/spacecraft.html (6 of 23) [2000/01/11 11:57:46 AM]
Dr. SOHO's FAQ: The SOHO Spacecraft
Go back to "Spacecraft" questions and answers.
●
How much fuel does SOHO have?
SOHO was born with 235 kilograms (about 518 pounds, but "pounds" are meaningless in
weightlessness) of hydrazine propellant. SOHO burned some of this fuel during the halo orbit
injection maneuver. We will use the rest of it a few grams or kilograms at a time, every 3 to 4
months, for orbit-maintenance maneuvers. SOHO must fire her thrusters periodically to stay in that
unstable halo orbit. We also need to "unload" the momentum wheels periodically. Between
maneuvers, SOHO "coasts" in its halo orbit.
At the current rate of consumption, SOHO's fuel supply will last about 20 years.
Go back to "Spacecraft" questions and answers.
I need the following information to analyze our CELIAS data and would appreciate the following
information: 1. What fuel is used during thruster burn? What is the atomic weight of the fuel? What is
the temperature of the fuel? 2. Does the gas from the thruster hit anything? 3. Is there a cloud
surrounding a spacecraft when thruster firing occurs? 4. Is it possible for you to determine the
pressure of the fuel measured around the spacecraft when the thruster is operating? 5. How far from
CELIAS are the thrusters? Are they pointing nearby the instrument? 6. Could you please e-mail a list
of thruster burns since launch, if available?
●
As a MMS engineer I work at Goddard on the SOHO operations and was given your questions.
1. What fuel is used during thruster burn? What is the atomic weight of the fuel? What is the
temperature of the fuel?
It's N2H4 (Hydrazine). In the Tank the temperature is about 20 Celsius. During long burn (several
minutes) the chamber temperature (inside the thruster) could increase up to 740 C.
2. Does the gas from the thruster hit anything?
●
●
●
The propulsion module is below the instruments. There are 8 thrusters with the following
configuration:
Thruster 1 and 2 are aimed at +30 degrees off +X axis, 1 on -Z and 2 on +Z. They are located at
the top of the service module on -Y and +Y.
Thruster 3 and 4 are at the bottom of the Service Module.
Thruster 5, 6, 7 and 8 are located at the top of the Service Module (each at the end of a boom),
exactly one at each corner of the SVM.
3. Is there a cloud surrounding a spacecraft when thruster firing occurs?
When a Station Keeping is performed, most of the time thrusters 1 and 2 are used (Delta-V on X
axis), therefore there might be some plume effects. Nevertheless, nothing significant has been
detected so far (checked with the SUMER team).
4. Is it possible for you to determine the pressure of the fuel measured around the spacecraft when
http://sohowww.nascom.nasa.gov/explore/faq/spacecraft.html (7 of 23) [2000/01/11 11:57:46 AM]
Dr. SOHO's FAQ: The SOHO Spacecraft
the thruster is operating?
We don't know the pressure but I can tell you how much fuel was used, for instance: 2 kg for
Station Keeping on 1997 September 4.
5. How far from CELIAS are the thrusters? Are they pointing nearby the instrument? 6. Could you
please e-mail a list of thruster burns since launch, if available?
It would be easier for me to fax it. I can also fax you drawings showing the location of the thruster.
To do that I need your fax number.
Go back to "Spacecraft" questions and answers.
●
How fast is the satellite SOHO moving?
The spacecraft is orbiting around the first Earth-Sun Lagrange point (or simply "L1"). This is the
gravitational balance point between the Earth and the Sun. (It is only about 1 percent of the
distance to the Sun; the Sun is much more massive, but it is much farther away.)
This point orbits the Sun with the Earth, so SOHO is moving around the Sun at about the same
speed as the Earth -- about 30 kilometers per second. SOHO isn't exactly at the L1 point -- it is
orbiting around it with a period of about 180 days. There is an diagram of this orbit at:
http://seal.nascom.nasa.gov/gif/halo_orbit.gif
(SOHO's Halo Orbit)
I calculate that it is moving around the L1 point at a few hundred meters per second.
Go back to "Spacecraft" questions and answers.
My officemate and I were discussing a line in the recent New York Times article (10 April 1997)
entitled: "Storm on Sun is Viewed From Spacecraft" and had a specific question with regard to a line
which reads: "...was launched on Dec. 2, 1995, and is now balanced between the gravity of Sun and
Earth, in orbit around Sun." Our question is: "Huh?" The writer for the Times must have goofed.
What are the orbital elements of SOHO's orbit? What do you think he meant in that quoted line?
●
Actually, The N.Y. Times was not wrong but the phrasing perhaps led to some confusion. SOHO
is in position about 930,000 miles sunward of the Earth at the first Lagrangian point. This is the
point between the Sun and the Earth where the gravity pulls of the two are balanced. Thus, SOHO
moves around the Sun just as the Earth does, in a relatively stationary position, although in a small
"halo" orbit. This graphic illustrates the orbit:
http://seal.nascom.nasa.gov/gif/halo_orbit.gif
(SOHO's Halo Orbit)
For more information about Lagrange Points see:
http://map.gsfc.nasa.gov/html/lagrange.html
(Lagrange Points)
http://sohowww.nascom.nasa.gov/explore/faq/spacecraft.html (8 of 23) [2000/01/11 11:57:46 AM]
Dr. SOHO's FAQ: The SOHO Spacecraft
Go back to "Spacecraft" questions and answers.
Can you tell me the usual position of SOHO in relation to the Earth's bow shock? Are there any
times when SOHO and satellites within the bow shock are in direct alignment between the Earth and
the Sun?
●
(The bow shock is where the solar wind encounters the Earth's magnetic field.)
As SOHO is in orbit around the first Lagrangian point, it is roughly 1.5 million kilometers (1
million miles) away from Earth. The Earth's bow shock usually sits at around 60,000 km altitude,
and the Van Allen belts are below that. (Note that the Earth's magnetic field does get compressed
to lower altitudes during geomagnetic storms.)
So SOHO is well outside the the Earth's bow shock.
This positioning permits SOHO's particles experiments, like CELIAS, to directly sample the solar
wind in situ.
Viewed from Earth, SOHO is always at least 6 degrees off the Sun, so it is impossible to find an
Earth-Sun alignment including SOHO and any other stationary satellite.
(Keep in mind that geosynchronous communications satellites do eclipse the Sun occasionally, and
this creates nasty radio interference for the satellite operators. SOHO stays far enough off of the
Sun's disk -- as seen from Earth -- to avoid such radio interference.)
At least one other spacecraft, the Advanced Composition Explorer (ACE), shares a SOHO-like
halo orbit around the Sun-Earth L1 point. NASA's WIND spacecraft is usually in such an orbit as
well. Both ACE and WIND obtain very precise measurements of the solar wind as it flows past
L1.
Go back to "Spacecraft" questions and answers.
●
Is there a possibility that SOHO will burn up?
No -- not anytime soon, that is. SOHO is not very close to the Sun, so the Sun won't "burn it up."
SOHO is only five times further out than the Moon. She isn't as close as NASA's planned Solar
Probe mission will get, or even as close as the US/German Helios probes got in the mid-1970s.
The usual way that a spacecraft can burn up is if it falls into Earth's atmosphere. SOHO won't do
that either, because she is orbiting the Sun, not Earth.
Eventually, after the mission is over and regular orbit maintenance ends, SOHO may drift into an
Earth-crossing solar orbit. Then, she may someday burn up in the Earth's atmosphere. But that is a
long way off.
Go back to "Spacecraft" questions and answers.
●
Dear Sirs: Please can you kindly inform us about the altitude of both SOHO and TRACE from the
http://sohowww.nascom.nasa.gov/explore/faq/spacecraft.html (9 of 23) [2000/01/11 11:57:47 AM]
Dr. SOHO's FAQ: The SOHO Spacecraft
Sun, and the period of rotation around the Sun for each. Many thanks for your help.
SOHO is in a special orbital location known as the first Lagrange point, or L1 for short. This is a
point between the Earth and the Sun where the gravitation forces balance such that SOHO always
stays between them. This point is about 1% of the distance to the Sun, about 1,500,000 kilometers,
three times farther away than the moon. Normally, an object in an orbit about the Sun that is closer
than the Earth would have a faster orbit. However, since the Earth is pulling in the other direction,
SOHO ends up orbiting the Sun once a year, just like the Earth, and never leaves the region
between the Earth and the Sun. In some sense, one could say that SOHO is in orbit simultaneously
about the Earth and about the Sun, rotating about each once a year. Since the Earth never gets
between SOHO and the Sun, SOHO can observe 24 hours a day, 365 days a year.
TRACE is in an orbit much closer to the Earth, about 600-650 kilometers above the surface. It
orbits around the Earth about once every 90 minutes (and around the Sun once a year, along with
the Earth). Rather than circle around near the equator like most satellites, it orbits over the poles.
This is how TRACE avoids the Earth's shadow. As the seasons progress, the sunlight strikes the
Earth from a different angle. If TRACE's orbit remained constant, then it would reach a point
when it did start going into the Earth's shadow. This does happen, but the Moon intervenes to
delay it. The Moon affects TRACE's orbit by causing it to precess, and this is enough to keep
TRACE from going into shadow until next November.
Once TRACE starts going into shadow, it has a day-night cycle like most other satellites. Not only
can it not observe the Sun while it's in shadow, but it won't be getting any power from its solar
panels either for a good fraction of its orbit. Because it's only getting a fraction of the power during
this "shadow season," TRACE will probably stop observing for several months until the shadow
season is over.
Go back to "Spacecraft" questions and answers.
I've read about SOHO, but I can't understand how it manages to orbit the L1 Lagrange point,
which is unstable?! Can you enlighten me?
●
You are right; the First Earth-Sun Lagrange point (L1) is unstable. If we tried to keep SOHO
stationary precisely at that point, the spacecraft would steadily drift away.
That is why the SOHO spacecraft is in a "halo" orbit around the L1 point (with about a 6 month
period). The semi-diameters of this halo orbit are:
- within ecliptic, Earth-Sun line ~ 200 000 km
- within ecliptic ~ 650 000 km
- out of ecliptic ~ 200 000 km
On each point of its orbit, the acceleration compensates the difference between Sun and Earth
attraction. But this halo orbit is not inherently stable, either. Left by itself, SOHO would steadily
drift away from the L1 region -- even from its current halo orbit.
Therefore, the spacecraft is equipped with rocket thrusters. Every 2 to 3 months, we use these
thrusters to adjust the orbit. We never let SOHO drift far enough to cause any problems. SOHO
http://sohowww.nascom.nasa.gov/explore/faq/spacecraft.html (10 of 23) [2000/01/11 11:57:47 AM]
Dr. SOHO's FAQ: The SOHO Spacecraft
arrived in this halo orbit with an estimated 20-25 year fuel supply; she can continue this
gravitational balancing act well into the next solar cycle.
(You will need to ask a NASA flight dynamicist to explain how a spacecraft can orbit an empty
point in space, even a Lagrange point, in the first place!)
Go back to "Spacecraft" questions and answers.
●
Will SOHO be sucked in by the Sun's gravity? Are there any drawings of SOHO's flight path?
We didn't have any simple drawings of the SOHO orbit, so we made one for you. It is at:
http://sohowww.nascom.nasa.gov/explore/img/orbits_graphic.gif
(The SOHO Orbit: GIF image 360x323 pixels)
This picture is NOT at the true scale. Compared to the Sun, the Earth and SOHO are actually a lot
smaller and farther away. SOHO is in orbit around the Sun, but is actually pretty close to the Earth
- SOHO is about 99% of the distance from the Sun to the Earth. Because it is in a nearly circular
orbit, SOHO will not get sucked into the Sun. The Earth itself is in no danger of being sucked in
anytime soon, right?
At first glance, this image also appears to violate Kepler's laws of orbital motion - how else could
SOHO stay near the Earth all the time while orbiting significantly closer to the Sun? However,
SOHO orbits around a special gravitational balance point between the Earth and Sun (the L1 point)
which allows it to orbit with the same period as the Earth. For more information about Lagrange
Points see:
http://map.gsfc.nasa.gov/html/lagrange.html
(Lagrange Points)
Go back to "Spacecraft" questions and answers.
How is the SOHO orbit stable at the Lagrangian Point? Even if it is at the cancelation point
between Earth-Sun gravity, would not the solar wind and electromagnetic pressure push it toward the
Earth and cause it to fall to Earth? Does it orbit around the center of Earth-Sun mass?
●
Your suspicions are correct; the SOHO orbit is not, strictly speaking, stable. The Sun-Earth L1
point itself is not stable. Any perturbing force would tend to push a spacecraft away from the L1
point. Besides solar photon pressure, there are gravitational perturbations from the Moon and
planets with which to contend. SOHO does not orbit on the orderly page of a math textbook, but
rather in a crowded solar system.
So, how does SOHO do it? She cheats. The spacecraft orbits around the L1 point without sitting
stationary at the point. Every 3 to 4 months, we close all the instrument lens covers and fire our
thrusters in what's called a station-keeping maneuver. Without these frequent adjustments, our
orbit would drift away from L1. Between adjustments, the spacecraft stays very near the Earth-Sun
line of sight.
http://sohowww.nascom.nasa.gov/explore/faq/spacecraft.html (11 of 23) [2000/01/11 11:57:47 AM]
Dr. SOHO's FAQ: The SOHO Spacecraft
Every spacecraft ever sent to L1 has used this (SOHO, ACE, WIND, and ISEE-3) station-keeping
technique. After 2.5 years, SOHO still has enough thruster propellant for 20 more years of orbit
adjustments. Whatever limits our mission life, it probably won't be the fuel supply!
(As SOHO team members have said, the term "stable" may mean different things to an astronomer
or an aerospace engineer. An orbit which is unstable on a 100,000-year time scale may be perfectly
acceptable to a 5-year space mission.)
For a look at the mathematics involved in Lagrange points and halo orbits, we recommend the
Stanford Solar Center's FAQ:
http://solar-center.stanford.edu/FAQ/QL1.html
(How is SOHO Orbit Location Calculated?)
Go back to "Spacecraft" questions and answers.
I'm a second-year engineering student. I'm very interested in the calculations involved in placing
SOHO in the L1 orbit. Is it possible for you to send me either an overview of the calculations or
detailed calculations?
●
Consider the 4 phases: - launch
- parking orbit around the Earth (at low altitude: about 200 km)
- transfer orbit with several Mid-Course Corrections:
MCC1 with 2 Delta V: 3 and 1.9 m/s
MCC2 with 3 delta-V: 16, 9.2 and 6 m/s
- halo orbit
Ax 200 000 km
Ay 670 000 km
Az 120 000 km
Halo Orbit Insertion: Delta-V of 3 m/s
In fact, I'm afraid I don't know much about all the detailed calculations. May be you'd better ask
somebody from Flight Dynamics (at GSFC, code 500). Also, check the Stanford Solar Center's
FAQ and Links:
http://solar-center.stanford.edu/FAQ/QL1.html
(How is SOHO Orbit Location Calculated?)
Go back to "Spacecraft" questions and answers.
●
How do you calculate the location of the Lagrange points? Why are they stable?
I had to go back to a textbook that I haven't used since graduate school for a precise answer of how to
calculate the Lagrange points.
http://sohowww.nascom.nasa.gov/explore/faq/spacecraft.html (12 of 23) [2000/01/11 11:57:47 AM]
Dr. SOHO's FAQ: The SOHO Spacecraft
Taking the example of the Earth going around the Sun, let M1 be the mass of the Sun, and M2 be the
mass of the Earth. The distance between them is represented by the symbol a. Define the coordinate
system to be measured from the center of mass of the Earth-Sun system, with x being measured along the
Earth-Sun line, and y being in the perpendicular direction. It turns out that the equation for the potential
energy of the system is given by the rather complicated equations:
e = x/a
n = y/a
e1 = M2 / (M1 + M2)
e2 = -M1 / (M1 + M2) = e1 - 1
V = ((M1 + M2)G/a) × ((e2 / sqrt((e-e1)2 + n2)) (e1 / sqrt((e-e2)2 + n2)) - (e2 + n2)/2)
The Lagrangian points are then defined as the points where the derivatives of V with respect to
both e and n are both zero. As you can imagine, solving these equations is a rather messy business,
but the answer for the L4 and L5 points turns out to be surprisingly simple. They always form an
equilateral triangle with the two bodies, in this case the Earth and the Sun, no matter what the
actual values of M1 and M2 are.
P.S. I forgot to reply to one point in your question, i.e. under what conditions are the L4 and L5
points stable.
In an ideal system consisting only of two massive bodies, the L4 and L5 points are always stable.
When you start adding more bodies, then things get more complicated. Thus, in a formal sense, the
L4 and L5 points of the Earth-Sun system aren't really stable, because there are other planets in the
solar system which also have to be taken into account. They cause perturbations which eventually
can knock an object out of a stable Lagrange orbit.
The question then comes down to what kind of stability are you really interested in. Is it years?
Decades? Centuries? Astronomers tend to think in terms of extremely long time scales, millions or
even billions of years. On the other hand, if one is thinking in terms of the stability of a satellite or
space station at a Lagrange point, then much shorter time scales are more important - you'd still
call it stable even if it did wander away after 1,000 years or so.
The largest planet in the solar system is Jupiter, and thus its L4 and L4 points are much more
stable than those of Earth. There are, in fact, a family of asteroids, known as the Trojan asteroids,
which lie in or near the Jovian Lagrange points.
Further discussion of Lagrange points and halo orbits is available at the Stanford Solar Center's
FAQ:
http://solar-center.stanford.edu/FAQ/QL1.html
(How is SOHO Orbit Location Calculated?)
http://sohowww.nascom.nasa.gov/explore/faq/spacecraft.html (13 of 23) [2000/01/11 11:57:47 AM]
Dr. SOHO's FAQ: The SOHO Spacecraft
Go back to "Spacecraft" questions and answers.
Why is SOHO in a "halo orbit" around L1? Why can't the spacecraft simply sit motionless at the
gravitational balance point itself?
●
SOHO is in orbit around a point in space called the first Lagrangian point, or L1. This is
approximately the location where the gravitational forces of the Sun and Earth balance (I say
approximately because one also has to consider another force, the centrifugal force, which occurs
because the Earth is in almost a circular orbit about the Sun). Similarly, an object at the L1 point
feels no force, but if it moves slightly toward either the Sun or the Earth, then the gravitational
forces take over. L1 is what physicists refer to as an "unstable equilibrium".
In principle, SOHO could just to sit at the L1 point and not orbit around it. However, this would
cause tremendous practical engineering difficulties. In order to keep it at L1, the spacecraft would
have to arrive there with exactly zero velocity - i.e. it would have to slow down to a stop at just the
right time and place. It was much easier to have it arrive at L1 with a small velocity and then use
onboard rocket motors to put it into an orbit about L1. The orbit that is used is called a "halo"
because if you were to trace out the path of SOHO as it goes around in its orbit, the path would
look like a halo.
Another consideration is that the ground-based radio antennas that communicate with SOHO
cannot point directly at the Sun without frying the sensitive electronics that they use to transmit
and receive signals. L1 is directly on a line between the Earth and the Sun, so SOHO needs to
positioned slightly off this line. The only way to do this is to put it a halo orbit about L1. Finally,
the orbit is not stable. Left alone, SOHO would gradually drift away from L1 and get swept
towards either the Earth or the Sun. Every couple of months, engineers send commands to fire the
onboard rocket motors to adjust the spacecraft motion and keep it in the desired orbit.
Go back to "Spacecraft" questions and answers.
Can you help me please to find the orbital elements of the SOHO satellite? I like to use this
elements in my planetarium program, Voyager II. How far stay the SOHO satellite from the Sun, and
what is the position of the Earth?
●
SOHO is at the first Lagrange (L1) point between the Earth and the Sun. This is approximately 1.6
million kilometers directly towards the Sun. The Sun is approximately 150 million kilometers
from the Earth. The L1 position is not stable and SOHO is actually in an orbit about this point.
You can obtain the exact orbital elements for any day of the mission by looking at:
http://sohowww.nascom.nasa.gov/data/ancillary/#orbit
(The SOHO Ancillary Data)
and following the links. Additionally, there is an online service (Satellite Situation Center) at:
http://sscweb.gsfc.nasa.gov
(The Satellite Situation Center)
which provides either tabular data or plots of many satellites, including SOHO.
http://sohowww.nascom.nasa.gov/explore/faq/spacecraft.html (14 of 23) [2000/01/11 11:57:47 AM]
Dr. SOHO's FAQ: The SOHO Spacecraft
I am not familiar with the Voyager II program, so I can't say what format you need to input the
data. If the above sites don't provide you with the information you need, please let me know what
details you require.
P.S. Here are the orbital elements, in various formats, for today, 1998 June 10:
GCI_X FLOAT 665238.
GCI_Y FLOAT 1.46106e+06
GCI_Z FLOAT 502594.
GCI_VX FLOAT -0.0828258
GCI_VY FLOAT 0.0153136
GCI_VZ FLOAT 0.0310732
GSE_X FLOAT 1.63926e+06
GSE_Y FLOAT -358109.
GSE_Z FLOAT -120065.
GSE_VX FLOAT -0.0590714
GSE_VY FLOAT -0.230147
GSE_VZ FLOAT 0.0224175
GSM_X FLOAT 1.63926e+06
GSM_Y FLOAT -313384.
GSM_Z FLOAT -210826.
GSM_VX FLOAT -0.0590714
GSM_VY FLOAT -1.38185
GSM_VZ FLOAT 1.67591
SUN_VECTOR_X FLOAT 2.90674e+07
SUN_VECTOR_Y FLOAT 1.36761e+08
SUN_VECTOR_Z FLOAT 5.92944e+07
HEC_X FLOAT -2.84021e+07
HEC_Y FLOAT -1.47521e+08
HEC_Z FLOAT -126521.
HEC_VX FLOAT 28.6581
HEC_VY FLOAT -5.78491
HEC_VZ FLOAT 0.0224364
CAR_ROT_EARTH FLOAT 1937.00
HEL_LON_EARTH FLOAT 5.78500
HEL_LAT_EARTH FLOAT 0.00700000
CAR_ROT_SOHO FLOAT 0.00000
HEL_LON_SOHO FLOAT 5.78700
HEL_LAT_SOHO FLOAT 0.00700000
Go back to "Spacecraft" questions and answers.
●
Is SOHO a "satellite" or a "spacecraft?" Both? Neither?
Because neither ESA nor NASA maintains an "Academie Francaise" to police everyone's
terminology, the jargon is a bit looser than you might expect. Both terms are applicable, but:
http://sohowww.nascom.nasa.gov/explore/faq/spacecraft.html (15 of 23) [2000/01/11 11:57:47 AM]
Dr. SOHO's FAQ: The SOHO Spacecraft
Traditionally, people call a vehicle a "satellite" if it is in Earth orbit or if it stays in Earth orbit. The
Apollo missions were in Earth orbit briefly, en route to the Moon, but they wouldn't have been
considered "satellites."
Traditionally, people call a vehicle a "spacecraft" if it ventures beyond Earth orbit. I can arguably
say that SOHO has never been in Earth orbit, so it is clearly a "spacecraft." Other examples
include NEAR, Cassini, Giotto, and Vega.
Technically, however, a "spacecraft" is any vehicle that flies in space, period (e.g. the space
shuttle). Also, the strict definition of "satellite" is anything that orbits anything else. Well, SOHO
does orbit the Sun, doesn't it? What about all our landers on Venus, the Moon, and Mars? The
Galileo spacecraft is a satellite of Jupiter. Even the Pioneers and Voyagers cruising out of our solar
system are still technically locked in orbit about the galactic core.
The short answer is: we usually call SOHO a "spacecraft!"
Go back to "Spacecraft" questions and answers.
●
What is SOHO made out of?
SOHO is made out of all sorts of things -- metals, ceramics, plastics, etc.
Specifically, SOHO's main component is various alloys of aluminum. The spacecraft's frame, and
most of its exterior, is aluminum. Thin layers of this metal make an exceptional insulation in the
vacuum of space.
The solar arrays are silicon.
Various secondary components are made from exotic materials, like titanium and composites.
SOHO has a composite high-gain antenna dish, for example. Composites are just starting to be
used for major pieces of modern spacecraft; aluminum is traditional, and thus well-understood.
ESA and NASA needed to accurately control SOHO's thermal flexing, too, and an aluminum
construction makes this reasonably straightforward.
The instruments themselves are often constructed of aluminum. LASCO's optics box, for example,
is aluminum 6082-T651 plate.
Go back to "Spacecraft" questions and answers.
Could you please explain the impact of the recent solar eruptions (April 7th, 1997) on satellites
orbiting around the globe, especially the Hubble Space Telescope, as well as the SOHO (which is
presumably even closer to the Sun)? Will the eruptions effect climate and is this the beginning of the
new 11 year active cycle on the Sun?
●
Space is not always a very kind environment. When designing a space satellite, one must always
take into account the radiation environment that it will encounter in space. For this reason,
electronics intended for space applications must always be made of special "rad-hardened"
components. Regular "off-the-shelf" computer chips, for example, will not work very well in space
-- unless they are heavily shielded, of course. Shielding is massive, however, and launch costs are
http://sohowww.nascom.nasa.gov/explore/faq/spacecraft.html (16 of 23) [2000/01/11 11:57:47 AM]
Dr. SOHO's FAQ: The SOHO Spacecraft
expensive.
The Sun is constantly emitting high-energy particles together with the solar wind. These particles
are called "cosmic rays," and are produced when the ionized particles, such as protons and
electrons, are accelerated by strong electric fields. Magnetic cloud events, such as the one in April
1997, will have a large number of such particles associated with it. Such particles can affect space
electronics by causing memory errors, false signals in detectors, and even some permanent
damage. Because of these particles, sensitive electronics will tend to degrade after years of
constant exposure.
Satellites which are in low Earth orbit, such as Hubble, see a different environment than one which
is in deep space, like SOHO. The Earth's magnetic field deflects many of these high-energy
particles away from the Earth. However, SOHO, at four times the distance of the moon, is outside
the Earth's magnetosphere, and is thus relatively unprotected.
There are further differences for low Earth orbit satellites. Electrons tend to get trapped within
regions of the magnetosphere called the Van Allen belts, and satellites which travel through these
regions will see an a greatly increased level of radiation. These levels intensify when a magnetic
cloud event from the Sun dumps a large number of electrons into the belts.
Occasionally, all these particle and radiation effects can make a satellite malfunction. At least one
Canadian "Anik" communications satellite was damaged by solar activity in 1991.
The major effect on the Earth of these magnetic cloud events is spectacular Northern lights. Some
particularly large ones have been known to cause electrical outages (because of the extra currents
produced in power lines). However, the April 1997 one was not really very big. We are seeing an
increase in solar activity as we enter a new 11 year cycle.
Go back to "Spacecraft" questions and answers.
Hi, Dr. SOHO. I need to know about dangers of solar wind to the satellites (including SOHO).
What type of particles (energy range) can affect the satellites. Can they actually penetrate the
equipment? What about UV radiation (what wavelength)? What equipment is most vulnerable? Can
you answer me ASAP?
●
High energy particles in the solar wind in the MeV range do penetrate into the interior of the
spacecraft and affect the SOHO instruments. Those instruments which use CCD detectors see
these particles as occasional bright streaks across the image, which need to be removed in the data
analysis.
Another way in which these particles affect the instruments is to change a bit in memory. Such
errors can be detected, because that piece of memory will then fail a parity check. Depending upon
where the error occurs, it may be corrected without interrupting processing, or it may cause a
reboot of one of the electronics boxes. (HINT: When building any part of a spacecraft, be sure to
give it a good "on-off" switch. Cycling the power can cure many vexing problems with laser
printers, lawn-mower engines, and satellite software!)
http://sohowww.nascom.nasa.gov/explore/faq/spacecraft.html (17 of 23) [2000/01/11 11:57:47 AM]
Dr. SOHO's FAQ: The SOHO Spacecraft
Particle radiation also affects one particular spacecraft system which, by design, must be in
sunlight all the time: the solar arrays. Radiation slowly degrades a solar array throughout a
spacecraft's mission life. Designers must provide enough collection area for the "end of life"
solar-cell potential, not the "beginning of life" potential. Therefore, SOHO was launched with
slightly larger solar panels than were absolutely necessary in December 1995.
You also ask about UV radiation. Of course, our instruments are developed specifically to observe
in that region of the spectrum. However, sometimes the UV radiation acts to keep itself from being
observed. If the optical surfaces are not absolutely clean, the UV can act to "polymerize" the
contaminants on the surface. They can turn dark to UV light.
The most important wavelengths are probably in the region of the strong Lyman alpha line at 1216
Angstroms. If this polymerization is going to happen, it will happen in the first few weeks of
exposure. The fact that it didn't happen on SOHO is a testament to the excellence of the cleanliness
controls used in building the instruments and the spacecraft.
For more information about the space environment, and how it affects spacecraft, I suggest that
you contact the National Oceanographic and Atmospheric Administration at:
http://www.sec.noaa.gov/
(NOAA Space Environment Center)
Go back to "Spacecraft" questions and answers.
I have been exploring SOHO with my Grandson, and we came up with a question that I can't
answer or find the answer to: What are the external as well as the internal temperatures of the SOHO
modules? Being that close to the Sun, one would think that a man-made object would burn up in short
order, perhaps in about 6 to 8 weeks at the max.
●
Interesting question, sir! The short answer is this: SOHO just isn't that close to the Sun. The Earth
is 93 million miles (150 million km) from the Sun. SOHO is about one million miles closer, or just
over one percent of the Earth-Sun distance. Let us call it 1.01 times closer.
In the vacuum of space, radiation is the only way most objects can heat up or cool down. The Sun's
radiation fades out according to the inverse square law: if you get twice as close, you will be 22 = 4
times hotter. Therefore, SOHO only gets about... 1.012 = 1.02 times more heat from the Sun than
the Earth does. (Well, the Earth is much bigger, but you see what I mean.)
Here's the funny thing that I learned while answering your question: even in the depths of space,
with the blackness of the universe behind her and the heat of the blazing Sun in her face, SOHO's
components are mostly at room temperature (20 to 40 degrees Celsius)! SOHO was designed and
built in room- temperature laboratories, and careful thermal design has maintained such
temperatures.
Now, there are exceptions. SOHO, like most spacecraft, has thick insulation over most of its skin. I
do not know how warm that gets. Our OSR, or "front windshield," on the Sunward side of SOHO
is only 11 C (51.8 degrees Fahrenheit). On the opposite (dark) side of SOHO, our Earth-pointing
http://sohowww.nascom.nasa.gov/explore/faq/spacecraft.html (18 of 23) [2000/01/11 11:57:47 AM]
Dr. SOHO's FAQ: The SOHO Spacecraft
radio antenna sits in permanent shade at -124 degrees C.
Temperatures for some other SOHO modules on 08 January 1998:
Solar arrays (backside): 44 deg. C
MDI front aperture: 46.8 deg. C
MDI filters: 35 deg. C (electrically heated)
MDI camera: -75 deg. C (passively cooled)
UVCS instrument 26 deg. C EIT telescope body: 23.8 deg. C
EIT camera: -66.3 deg. C
LASCO (C2 telescope): 20.0 deg. C
LASCO (C3 telescope): 19.4 deg. C
batteries: 3.8 deg. C (batteries like to be cooler)
hydrazine fuel tank: 21.2 deg. C
hydrazine thrusters: (I don't know, but they're 800 deg. when firing!
Thermally, SOHO has a much easier life than most satellites. Earth-orbiting satellites like Hubble
telescope pass in and out of sunshine once every 90 minutes. This can be like flying from a warm
oven to a cold freezer sixteen times per day. All that hot-and-cold cycling can strain a spacecraft's
components, which is why insulation is so important. So is good thermal design and thermal
testing. SOHO may be slightly closer to the hot Sun, but we are in sunlight 100% of the time. We
do not need to worry about thermal cycles and day-to-night transitions.
P.S. NASA is working on a new spacecraft called "Solar Probe." Its job is to fly VERY close to
the Sun and make measurements.
How close is close?
It will get within three times the Sun's radius, or only 1.3 million miles away. At this distance,
NASA says "the radiation temperature exceeds 2000 Kelvin." That is about 1725 degrees C. To
survive such temperatures, Solar Probe will be a tiny spacecraft hiding behind a huge heat shield.
Go back to "Spacecraft" questions and answers.
●
How do you communicate with SOHO? How do you control SOHO?
We send SOHO commands by radio, and she sends us data back the same way.
Controllers here at Goddard can type commands into computers. (In some cases, commanding a
SOHO instrument is a simple, near- real-time "point and click!") The computers send these
commands through NASA's worldwide communications network to one of the Deep Space
Network (DSN) antennas. That first leg of the journey from the controller to SOHO takes less than
a second.
The DSN is a set of big radio dishes, some of them 70 meters across. After the commands are
transmitted through the DSN antenna, they require 5 seconds to reach SOHO at the speed of light.
SOHO receives these radio signals through a small antenna dish pointed toward Earth. We send
such commands several hundred times each day; the science programs keep us and our instruments
quite busy.
http://sohowww.nascom.nasa.gov/explore/faq/spacecraft.html (19 of 23) [2000/01/11 11:57:47 AM]
Dr. SOHO's FAQ: The SOHO Spacecraft
The data which SOHO transmits back to Earth follows the same path in reverse: radio signals go
from SOHO to the Deep Space Network, to NASA's network, to the SOHO control rooms, to an
individual computer. When SOHO isn't scheduled to use a DSN antenna, she stores the data in
special onboard recorders for later transmission. For more information on the DSN see:
http://deepspace.jpl.nasa.gov/dsn/index.html
(The Deep Space Network Home Page)
SOHO is physically controlled using momentum wheels and rocket thrusters. These momentum
wheels act like gyroscopes; they spin constantly, and they force the spacecraft to point at the Sun
all the time. Every few months, we fire SOHO's thrusters to keep the orbit fixed between the Earth
and the Sun.
Go back to "Spacecraft" questions and answers.
It's my understanding that the software that controls the SOHO satellite itself was written in Ada. Is
there any information available on this?
●
Your understanding is correct. Actually, there are several software routines running on board
(without taking into account those of the many individual instruments):
1. The COBS (Central OnBoard Sw) runs in CDMU on a 1750 processor, written in ADA (some
part in ASSEMBLER)
2. The Attitude Control Unit SW, also on 1750 chip, written in ADA.
3. The Star Tracker SW (written in Assembler) and the Solid State Recorder SW (also on a 1750
processor).
4. One example from the Instrument side of the aisle: LASCO and EIT share one computer
between them. It has multiple processors. One is "a radiation-hardened, 32-bit Sandia Lab SA3000
(15 MHz clock), based on the National Semiconductor 32C016." (This is straight from the LASCO
instrument paper.) The other processors for this "electronics box" are mostly Intel 8031-based
ASICs. The LASCO Electronics Box software is mostly written in C.
5. Last but not least, we should mention our ground system. The Flight Operations Team uses
Hewlett-Packard workstations and a variety of custom-built boxes to run SOHO's command
software. The Instrument side of the aisle is a veritable Who's Who of desktop computing: Silicon
Graphics workstations, IBM workstations, Sun Microsystems (pardon the inevitable pun -- and we
need both SunOS an Solaris systems), VMS systems, a few Intel machines, and even one lone
Macintosh! Each instrument's systems run off-the-shelf operating systems, custom control
software, and commercial data-processing packages like IDL.
Go back to "Spacecraft" questions and answers.
●
I would like to know: what is the expected duration of the SOHO mission?
Wow, that question is like asking the aging movie star how old she is! Still, we will give a straight
http://sohowww.nascom.nasa.gov/explore/faq/spacecraft.html (20 of 23) [2000/01/11 11:57:47 AM]
Dr. SOHO's FAQ: The SOHO Spacecraft
answer.
Although the expected duration is much longer than the original plan. The SOHO project proposal
states that the nominal mission is 2.5 years. The first 2.5 years of operation will be finished in
May, 1998.
However, the spacecraft itself is in good shape. In fact, we have enough rocket fuel on board to
last for 20 years or more. (The spacecraft uses small amounts of rocket fuel to stay correctly
positioned in its halo orbit around L1. The orbit is slightly unstable, so if SOHO is left
uncontrolled, she will gradually drift into a more conventional orbit around either the Sun or the
Earth.)
Most of us expect that we will continue to operate SOHO at least through a complete solar cycle
(i.e. another eight to ten years, after 1996). After that, well, it's anyone's guess.
Unlike Hubble, SOHO probably isn't destined for the Air and Space Museum when it's done; the
L1 point is too far away for any retrieval. SOHO's fate will probably be similar to that of IUE, the
fabled International Ultraviolet Explorer. (I hope we're not being presumptuous by comparing our
mission to that particular spacecraft.) After nearly 18 years of steady output, IUE's controllers
burned the satellite's thrusters to depletion (knocking it out of its geosynchronous zone and
preventing any future explosive hazard), then commanded its radio transmitter "OFF" (to eliminate
future interference on that frequency). If we do that to SOHO someday, she will drift into a solar
orbit -- possibly an Earth-crossing one. Go back to "Spacecraft" questions and answers.
I am curious to know how much longer SOHO will be operated. I read that the program
(operations) was funded for 2 years. It has been 2 years since the 1995 launch. Did SOHO receive
more funding? As long as the satellite continues to operate, I don't see why more funding should not
be provided for operations.
●
[Dr. SOHO's note: This question was received in March 1998.]
Yes, fortunately we have received more funding to continue operations. You probably already
know (I see by your address that you work for Lockheed-Martin) that it makes sense to fund
operations of each spacecraft as long as possible, because of the high capital cost of building and
launching each one. In SOHO's case, a year of operations runs something like 1%-3% of the initial
cost (depending on how you count; these things are never as simple as one would like).
Meanwhile, the Sun itself is obliging us by changing modes. When SOHO was launched, the Sun
was in its minimum activity phase. With each passing month, it's getting more complex as it
prepares to flip the direction of its main magnetic field, sometime around AD 2000-2001.
SOHO funding was in jeopardy for a while, but thanks in part to the wide and positive public
response to our initial scientific results, we're (knock on wood) in good shape on that front. The
spacecraft has enough fuel to keep on station for approximately 30 more years; and, while most of
the instruments show some minor signs of wear and tear, all 12 of them are in good shape to take
data for years to come.
Go back to "Spacecraft" questions and answers.
http://sohowww.nascom.nasa.gov/explore/faq/spacecraft.html (21 of 23) [2000/01/11 11:57:47 AM]
Dr. SOHO's FAQ: The SOHO Spacecraft
●
I heard rumours (in November 1997) that the SOHO is damaged?
I assure you that the SOHO spacecraft and all of her instruments (there are 12) are fine. On
November 19, 1997, the SOHO spacecraft entered into an Attitude Survival Mode or Emergency
Sun Reacquisition (ESR). This was due to a problem with the Attitude Control Unit Side A.
During the ESR recovery process, the ACU-A was powered on, and it has been functioning
nominally ever since (as of this writing, January 1998). All of the instruments were successfully
tested and taking data within about 48 hours of the ESR.
Temporary glitches like this are fairly common in spacecraft, and we are presently NOT concerned
about any imminent failure aboard SOHO.
Could SOHO be damaged by future solar flares? Possibly. Flares can degrade solar arrays and
"fry" electronics, as is discussed elsewhere in this FAQ. SOHO has not yet experienced a truly
huge solar flare, like the 1972 or March 1989 events. There have been several solar-proton events
(SPE) so far, but none of these have caused physical damage to SOHO. SPEs can and do interfere
with some scientific observations, especially the LASCO and EIT imaging programs. The extra
particle flux causes the infamous "snowstorm" effect in out images. However, everything returns
to normal when the flux recedes.
ADDENDUM, FEBRUARY 1998:
This is an involved story, but it may be classified as a "SOHO damage" issue.
At 03:05 UT on 04 February 1998, SOHO's LASCO instrument had a software anomaly. In other
words, our coronagraph stopped taking pictures for several hours. LASCO's software is a
complicated piece of code, but such anomalies are thankfully rare. This software also runs the EIT
instrument, so EIT stopped taking images at this time, too.
When we restarted the science programs later that morning, we saw that EIT's "light leak" had
grown significantly.
You see, EIT looks at narrow wavelengths of UV light. A set of special filters in the front of the
telescope blocks out unwanted stray light. Most of EIT's four filter positions had always had some
amount of stray light leakage. Now, these leaks were bright and pronounced in all four filters. This
incident caused EIT to switch back to its "Al+1 pre-filter." Excepting calibrations, every EIT
image since 04 February 1998 has used the Al+1 filter to reduce stray light interference.
One theory holds that the two failures are unrelated. It is thought that, deep in the middle of the
night, EIT was struck in the face by a dust-grain-sized piece of space debris. The SOHO Meeting
Minutes for 04 February 1998 note that SOHO experienced an unexpected wheel torque at 03:46
UT. This is consistent with a micrometeoroid impact. However, EIT was not taking images
because of the software problem, so we cannot accurately "fix" the exact time at which the light
leak grew. We will probably never know for sure. Here are some of the morning-after images:
http://umbra.nascom.nasa.gov/eit/new_leaks.html
(SOHO EIT light leaks, 1998 February 4)
Go back to "Spacecraft" questions and answers.
http://sohowww.nascom.nasa.gov/explore/faq/spacecraft.html (22 of 23) [2000/01/11 11:57:47 AM]
Dr. SOHO's FAQ: The SOHO Spacecraft
●
How does the spacecraft get out of the atmosphere?
There's nothing really difficult to getting out of the atmosphere. It just takes a lot of power. SOHO
was launched by an Atlas-Centaur rocket on Dec 2, 1995. You can see a movie of the launch at:
http://sohowww.nascom.nasa.gov/artwork
(SOHO Spacecraft Artwork)
A rocket generates enough power to lift its own weight. It does this by burning a very powerful
fuel and throwing out hot gases at high velocity from the nozzles at the bottom. This hot gas
pushes the rocket in the opposite direction, straight up.
Besides power, there's one big difference between a rocket and an airplane. In order to burn fuel,
you have to have oxygen. An airplane, or for that matter the fire in a fireplace, gets its oxygen
from the air around it. A rocket, though, has to be able to keep burning its fuel even after it has left
the atmosphere behind it. This means that a rocket has to carry its own oxygen along with it. It
does this by cooling the oxygen to temperatures so low that the oxygen turns into a liquid, which
takes up less room than normal oxygen.
A rocket actually works better outside of the atmosphere. The atmosphere slows the rocket down
as it travels through it, because the rocket has to push the atmosphere out of the way. Once the
rocket gets outside the atmosphere, there's nothing left to slow it down.
Go back to "Spacecraft" questions and answers.
Go back to "Dr. SOHO's FAQ."
SEND US YOUR COMMENTS
Go To
Other SOHO Web Pages
Author: Shane Stezelberger
Co-Author/Curator: Therese A. Kucera
[email protected]
Responsible Official: Art Poland([email protected])
Last modification: Wednesday, 22-Dec-1999 17:20:38 EST
http://sohowww.nascom.nasa.gov/explore/faq/spacecraft.html (23 of 23) [2000/01/11 11:57:47 AM]
Dr. SOHO's FAQ: Scientific Instruments
Go back to "Dr SOHO's FAQ"
Go back to the "SOHO" main page.
●
How do you take a picture of the Sun?
●
Where can I find a summary of each SOHO instrument's goals?
●
How do your "difference movies" work?
●
How does a spectrograph work?
●
Can you explain how SUMER can see different temperatures?
●
Where can I get numerical data from SOHO?
●
Where can I find analyzed UVCS data?
●
How can I build my own coronagraph?
LASCO
❍ What is that bright object moving through LASCO's field of view?
❍
What is this object in a LASCO C3 image?
❍
Is this object (in a C3 image) one of the planets?
❍
What are the dots in the background of LASCO images?
❍
What are those sudden, flickering objects in the SOHO movies?
❍
Can you describe this tube-like object in a LASCO image?
❍
Why does the bottom half (or top half) of each LASCO image look depleted at this time?
MDI
❍
Can you put solar sounds on the web in real time?
❍
What time standard is displayed in the MDI image times?
❍
Where can I find the MDI dopplergrams?
❍
Where can I find information about the 2.65-hour solar oscillations that were once detected
by the Crimean Astrophysical Observatory?
EIT
http://sohowww.nascom.nasa.gov/explore/faq/instruments.html (1 of 22) [2000/01/11 11:57:57 AM]
Dr. SOHO's FAQ: Scientific Instruments
●
❍
Is "up" always at the top of SOHO's images?
❍
To which temperatures do the EIT 171A and 284A images correspond?
❍
Why do SOHO's CCDs "bake out" occasionally? What IS a bakeout, anyway?
❍
Why do the EIT images seem to "jump" on March 4, 1998?
❍
Why did the noise level in EIT's pictures suddenly jump in April 1998
❍
Why do some loops in EIT appear dark, and others bright?
❍
What is that dark band around the Sun in EIT?
❍
How long is a single EIT exposure?
❍
Do the different EIT wavelengths come from different cameras, different telescopes, or
different filters?
❍
Why don't sunspots appear in EIT images?
●
Can you tell me about SOHO's use of wavelet-compression techniques in its data?
●
Did a solar flare zap SOHO in April 1998?
●
It seems easier to look at sunspots in negative images. What do you think?
How do you take a picture of the Sun?
There are a number of ways to take the Sun's picture. One way is to use film, like you use to take
pictures of your family. Before the Sun's light gets to the film, you have to put it through special
filters that block out most of its light -- otherwise it will probably be over-exposed.
More recently, people have started using electronic detectors. The light causes changes in the
detector which can be recorded by computers. Then the information can be transmitted around the
world or across the solar system.
This is important for distant space probes like SOHO. None of SOHO's cameras use film; they are
all digital. SOHO transmits these digital pictures via radio signals to NASA's communications
network on Earth.
Then we process them, display them on the web, and analyze them for scientific purposes.
Go back to "Instrument" questions and answers.
I was wondering if you or someone else in the administration of the SOHO program could suggest
that each of the instrument teams be required to post a brief essay on the information that their
respective teams are seeking, and the most important and or mysterious things they are trying to figure
out. This way, each of the teams can see what the other teams are confused about and thus a cross
correlation of data may result. Who would be capable of mandating such a thing?
●
Your suggestion is highly sensible. Such information should, of course, be on the Web. The
question is, is it?
An overview of the SOHO mission, and the basic goals of the instruments, is available from the
http://sohowww.nascom.nasa.gov/explore/faq/instruments.html (2 of 22) [2000/01/11 11:57:57 AM]
Dr. SOHO's FAQ: Scientific Instruments
SOHO homepage by following the links "About:"
http://sohowww.nascom.nasa.gov/descriptions/mission/
(The SOHO Mission)
This isn't really very detailed, though.
The SOHO homepage also allows you to go to the individual instrument home pages. The question
is, do the various instruments have this information? I've looked at it instrument-by-instrument:
CDS: It's not really easy to find. If one follows the "Gallery" link, there are a few words under
"SOHO Mission Description," and "Purpose of Mission." More detailed information can be found
by following "Science" and then "Technical Information," but that requires either a PostScript or
PDF viewer:
http://orpheus.nascom.nasa.gov/cds/cds_homepage.html#TECHNICAL
(Coronal Diagnostic Spectrometer)
CELIAS: A very good description of the scientific goals of the instrument is available from their
homepage:
http://www.cx.unibe.ch/phim/soho/instrument.html#Science
(The CELIAS Instrument)
COSTEP: Hmmm, it's in German:
http://ifkki.kernphysik.uni-kiel.de/www_root/soho/ephin.html
(das Kieler Experiment auf SOHO)
EIT: Follow the link to "EIT Science Topics:"
http://umbra.nascom.nasa.gov/eit/EIT.html#SCIENCE_TOPICS
(SOHO Extreme ultraviolet Imaging Telescope (EIT))
ERNE: This is described on their home page:
http://www.srl.utu.fi/projects/erne/erne.html
(Space Research Laboratory -- ERNE)
GOLF: Some description is available on the home page:
http://www.medoc-ias.u-psud.fr/golf/
(GOLF (SOHO) Home Page)
LASCO: Follow the link "What is LASCO?:"
http://lasco-www.nrl.navy.mil/about_lasco.html
(About LASCO)
MDI/SOI: Follow the link "SOI Science Objectives:"
http://soi.stanford.edu/science/objectives.html
http://sohowww.nascom.nasa.gov/explore/faq/instruments.html (3 of 22) [2000/01/11 11:57:57 AM]
Dr. SOHO's FAQ: Scientific Instruments
(SOI Science Objectives)
SUMER: Follow the link "Description of the SUMER Investigation:"
http://sohowww.nascom.nasa.gov/descriptions/experiments/sumer/investigation.html
(The SUMER Investigation)
SWAN: Follow the link "main scientific objective:"
http://www.geo.fmi.fi/PLASMA/SWAN/swanscience.html
(FMI/GEO, SWAN -- Science)
UVCS: Follow the link "Scientific Purpose:"
http://cfa-www.harvard.edu/uvcs/
(UVCS)
VIRGO: Follow the link "Science Objectives:"
http://sohowww.nascom.nasa.gov/descriptions/experiments/virgo/
(VIRGO)
So, it seems that the most problematic instruments are CDS and COSTEP. If you read German,
then you'd find a good description of the scientific goals of COSTEP. I'll try to encourage the CDS
team to put a more readily accessible scientific description on the Web.
Hope this answers your question. If you have any others, please address them to
[email protected].
Go back to "Instrument" questions and answers.
I am interested in how you extract information from the images. I found your "difference" images
fascinating. From a mathematical point of view, how do you approach the problem? Do you know of
any references you might direct me to? Is there a carryover between the procedures which you folks
use and medical imaging?
●
We use difference subtraction as a quick way to look for coronal mass ejections (CMEs) in a series
of LASCO coronagraph images. We simply subtract the earliest image from each subsequent
image. Because they change position from image to image, CMEs become quite apparent.
The difference image is essentially the derivative of the pixel brightness values over a series of
images. In other words, anything that does not change from image to image gets subtracted out,
while the varying features of the image get enhanced.
For other space missions, such techniques do have an application in medical imaging. Techniques
developed for the Hubble Space Telescope have recently been used in a non-surgical technique for
mammography:
http://www.hq.nasa.gov/office/oss/pubs/health.htm
(HST and Women's Health: NASA Technology in Your Doctor's Office)
http://sohowww.nascom.nasa.gov/explore/faq/instruments.html (4 of 22) [2000/01/11 11:57:57 AM]
Dr. SOHO's FAQ: Scientific Instruments
Much of the commercial off-the-shelf image-processing software that we use in the SOHO offices
(for example, IDL) is used throughout the medical-imaging community.
For references, check any books or research articles including the terms "Digital Image
Processing."
Go back to "Instrument" questions and answers.
I am doing a science project and I needed to know what a spectrograph is, how it works, and how it
is used. Please simplify things because I am only a 7th grader, and I am doing a demonstration about
spectrum analysis.
●
Simply put, a spectrograph separates out light into the colors that it's made of. In other words, it
forms a rainbow out of the light that goes into it. There are two ways that one can do this. One
way, which I'm sure you're familiar with, is to use a prism. Different colors bend by different
amounts as they pass into and out of the glass or crystal of the prism, and this breaks up the light
into its spectrum.
Scientists, however, use a tiny, fine grating to form the spectrum. This has several advantages over
prisms. First of all, the spectrum from a grating is of higher quality than that from a prism. Also,
gratings can be used for parts of the spectrum that are invisible to the eye -- colors which do not
pass through a prism's material.
A single color of light is a traveling wave in space, like waves in the ocean. For each color, there is
a specific distance between one peak of the wave and the next. Scientists refer to colors by this
length, and use the term "wavelength." For example, the wavelength of red light is about twice that
of blue light.
A typical grating consists of grooves in a mirror with a regular spacing. Light waves hitting the
grooves scatter off of them. The trick of a grating is that at a certain angle, the red light scattered
from one groove will exactly line up with the light scattered from the next groove. In other words,
the waves from each groove will be going up and down at the same time. At other angles, the
waves will be doing different things -- one will be going up while the other will be going down. At
all angles but the right one, the waves will cancel out. Only at the correct angle for that wavelength
will one see that color. Different colors have different angles.
The reason why spectroscopy is important is that different materials glow at different colors. We
call these "emission lines." By looking at what lines are emitted by a hot object like the Sun, we
can tell what it's made of, and many other properties like how hot it is, how fast it is moving, and
so on. Looking at different parts of the spectrum, we see different aspects of the Sun. For example,
the light we can see with the eye comes from the part of the Sun that's at about 7000 degrees
Kelvin. However, at shorter wavelengths, in what we call the Extreme Ultraviolet, we see gas
above the visible surface of the Sun (in the solar atmosphere) at temperatures of several million
degrees. X-rays, which are even shorter wavelengths, allow us to see even hotter material.
Go back to "Instrument" questions and answers.
http://sohowww.nascom.nasa.gov/explore/faq/instruments.html (5 of 22) [2000/01/11 11:57:57 AM]
Dr. SOHO's FAQ: Scientific Instruments
I am having a lot of trouble seeing how the data that has been collected by SUMER can be used to
answer some of SOHO's purpose questions. I have seen lots of neat pictures, and I suppose these do
give information about atoms, but I can't hook up a calorimeter to my computer screen. I have read
some of Harrison and Wilhelm's articles, and the articles in the Solar Physics journal on SOHO, but I
have come to realize that these are just a little beyond me (I'm a first-year honors science undergrad).
I just need a little bit more of a foundation to understand what they're talking about to the point where
I can interpret it and explain it to others in my own words.
●
The data collected by SUMER (and UVCS, and CDS) are spectral data. Specifically, we are
looking at ultraviolet spectra from the Sun, which show lines and continuum from electron
transitions by ions in the Sun's atmosphere. There are few different ways we can tell temperatures
from this data:
1. We often know what ion forms a particular spectral line. Certain ions are formed at particular
temperatures. In general, the hotter it is, the more energy there is to strip off electrons from an
atom. Scientists have calculated and measured these things, so we know, for instance, that He II
(once ionized helium; He I would be neutral helium) is generally formed and emits radiation at
about 50,000 K (K is degrees Kelvin, which is Celsius plus 273.15). Fe XV (iron ionized 14 times)
is usually formed around 2 million K. Thus if we see a feature in a He II line, but not in a Fe XV
line, we will assume its temperature is closer to 50,000 K. If the feature is visible in a Fe XV line,
we know there it contains plasma close to two million degrees.
Examples of this are provided by the EIT instrument on SOHO. This instrument takes images in
four different wavelength ranges. You can compare the images taken in the range dominated by He
II emission to those taken at wavelengths dominated by a Fe XII line. They can show quite
different features because they show plasmas at different temperatures. See:
http://sohowww.nascom.nasa.gov/data/summary/gif/971031/seit_00284_fd_19971031_1905.gif
(EIT 284 angstrom Fe XII image)
and:
http://sohowww.nascom.nasa.gov/data/summary/gif/971031/seit_00304_fd_19971031_1918.gif
(EIT 304 Angstrom He II image)
(warning, these are big images).
Of course, some things are seen in both lines - often, that means that the feature have plasmas at a
range of temperatures, often at different locations.
2. You can also determine temperature by comparing different spectral lines, often emitted by the
same ion. The transitions of electrons from one orbital state to another are sometimes governed by
temperature and density. If we can correctly calculate how they are dependent, we can predict how
the ratio of two lines will change as a function of temperature. These calculations can be pretty
complex. See:
http://wwwsolar.nrl.navy.mil/chianti.html
(CHAINTI: A Database for Astrophysical Emission Line Spectroscopy)
http://sohowww.nascom.nasa.gov/explore/faq/instruments.html (6 of 22) [2000/01/11 11:57:57 AM]
Dr. SOHO's FAQ: Scientific Instruments
to see the sort of effort put into it. It is hard to convey this sort of thing with pretty pictures, so you
don't often see it on our web pages.
3. Another way to determine temperature is to look at the width of a spectral line. As you may
know, the Doppler shift will cause a line to shift slightly in wavelength if the source of the light is
moving towards you or away from you. The hotter a plasma, the more the ions in the plasma are
moving around in random directions. This will lead to a "broadening" of the line when you look at
the emission of all these ions together. The broadening is proportional to the temperature. This a
bit tricky, though, because there are other things which cause spectral lines to broaden (turbulence,
for instance).
Go back to "Instrument" questions and answers.
How can I get numerical data from SOHO? I need some numerical data to include in my research
(i.e sunspot numbers, etc). Can you help me with that (show me how to get these)?
●
SOHO observations are available by using the SOHO catalog at:
http://sohowww.nascom.nasa.gov/data/catalogues/main.html
(SOHO Main Catalog)
You'll only be able to access those FITS files (Flexible Image Transport format, a way NASA
handles space image data) which are in the public domain. Check each instrument home page for
details about proprietary analysis periods.
However, I suspect that this is not what you mean, since you mention sunspot numbers. Probably,
what you want is to look at the various data maintained by the National Oceanic and Atmospheric
Administration (NOAA) at:
http://www.sec.noaa.gov/
(NOAA Space Environment Center)
You can also look at their gopher server at:
gopher://gopher.sec.noaa.gov/
(NOAA Gopher Server)
where there is a wealth of information. You may also try:
http://www.ngdc.noaa.gov/stp/SOLAR/SSN/ssn.html
(Sunspot Number)
Go back to "Instrument" questions and answers.
●
What data has been collected by the UVCS? Has it been analyzed? Where can I find it?
Some UVCS data certainly has been analyzed. You can find out about some of the results on the
UVCS home page:
http://sohowww.nascom.nasa.gov/explore/faq/instruments.html (7 of 22) [2000/01/11 11:57:57 AM]
Dr. SOHO's FAQ: Scientific Instruments
http://cfa-www.harvard.edu/uvcs/
(UVCS Home Page)
This page gives some abstracts to papers on UVCS, although admittedly some of them are pretty
technical. There have also been some popular articles mentioning UVCS results - see Science, 17
Oct. 1997, p. 387, and 25 July 1997, p. 479. To look at examples of actual analyzed data, you will
have to go to the scientific journal articles, though. Please inquire further if you need more details.
Go back to "Instrument" questions and answers.
I love observing the Sun through my telescope and a solar filter. Is it possible for me to build a
coronagraph and observe the corona from low altitudes, or should I just forget it and stick to my solar
filter?
●
There's one simple way that any telescope can be turned into a coronagraph, and that is to observe
during a total solar eclipse. The Solar Data Analysis Center at Goddard keeps information about
upcoming eclipses at:
http://umbra.nascom.nasa.gov/eclipse/
(Eclipse Information at the Solar Data Analysis Center) If you've never seen a total solar eclipse,
then I highly recommend that you think about going to one. It's a truly amazing experience.
Of course, that's not really what you asked. I talked to some people here more familiar with
coronagraphs than myself, and we came to the conclusion that the amount of scattered light from
the atmosphere near sea level would overwhelm the light from the corona. That is why all solar
coronagraphs are built on top of high mountains (such as the Pic du Midi observatory in France).
During an eclipse, the scattered light from the atmosphere is suppressed, because you're looking
through air which is itself shadowed by the moon just as you are.
Building a chronograph is a very exacting task. If it is externally occulted, then the occulter has to
be very far away from the telescope objective, or one won't be able to look near enough to the Sun.
Internally occulted coronagraphs are also very difficult. The first optical element, whether it is a
lens or a mirror, has to be of extremely high quality lest internal scattering overwhelm the corona.
SOHO's triple coronagraph, LASCO, was assembled in special clean rooms. Keeping the optics
and structure very clean throughout the assembly was very important. LASCO's light baffles and
internal structure are black-anodized to reduce stray light scattering. One cannot simply paint a
coronagraph black on the inside, because paint can flake off -- thus producing debris to scatter
more stray light through the coronagraph.
Go back to "Instrument" questions and answers.
Do you know what the bright object is at the top of the LASCO C3 picture on 1-12-1998 at 21:43,
and again on 1-13-1998 at 11:56?
●
That object is the planet Venus, which was close to inferior conjunction at that time (i.e. it was
very close to the Sun as seen from Earth). Venus is so much brighter than the solar corona that it
http://sohowww.nascom.nasa.gov/explore/faq/instruments.html (8 of 22) [2000/01/11 11:57:57 AM]
Dr. SOHO's FAQ: Scientific Instruments
saturates the C3 CCD camera, so it appears as a large white disk. Venus is not much bigger than a
single C3 pixel at its closest approach.
The horizontal lines on either side of the disk are caused by this saturation. Charge from incoming
photons can fill a CCD pixel completely, and the excess charge will spill over into neighboring
pixels. Because of the way this CCD is constructed, this "bleeding" occurs preferentially along the
horizontal direction.
LASCO sees planets several times per year, especially Venus and Mercury, as their orbits take
them close to the Sun in our sky. Many such times, we will be caught unawares, and a planet's
appearance will send us scrambling for the Astronomical Almanac to identify the intruder.
Go back to "Instrument" questions and answers.
What the heck is this thing? Is it your comet? http://www.eagle- >net.org/IWP/comet2.htm
Please respond, I am losing sleep over this thing! Thanks...
●
Don't worry, you can start getting some sleep again. That object is definitely the planet Mars. I
confirmed it with the LASCO team. To someone like myself, who's used to looking at LASCO
images, it's quickly picked out as a planet, and in this case it's Mars. On LASCO images, planets
always look like that.
(Compare its location on our images, relative to the Sun, with Mars' position in the Nautical
Almanac. Johannes Kepler would not lie to us, even about Mars -- which always was Kepler's
biggest headache!)
Obviously, this image doesn't look anything like the planet Mars. That's because the size and shape
of planetary images on the detector are being driven by their brightness, not by their intrinsic size.
Planets are just too darned bright for a detector meant for observing the faint corona. First of all,
such bright objects end up looking much bigger than they really are. Even the largest planet,
Jupiter, would be just a fraction of a pixel in the LASCO C3 camera. What you're really seeing is
scattered light from the telescope around the image of the planet itself. Since the planet is saturated
in the detector, its apparent brightness is much less than its real brightness, and one sees this halo
of scattered light much more clearly.
The extensions of the image to the left and right, making what look like wings, or a disk like the
rings of Saturn, are also an artifact of the brightness of the planet, but for a different reason. The
LASCO detector is a CCD, similar to those used in camcorders and digital still cameras. CCDs
work by converting the light that falls on them into electrons. These electrons are stored within the
CCD wells until the CCD is read out. When CCD cameras are exposed to very bright light levels,
the electrons tend to start spilling over into adjacent pixels. This is often referred to as either
"blooming" or "bleeding." To minimize the effects of this, CCDs are built with channel stops so
that the bleeding is mostly restricted to one direction. In the LASCO detector, these channels stops
are arranged horizontally, so images always bleed in the horizontal direction.
LASCO's raw images do not show any structure or gradations in saturated regions, by the way.
The camera absorbs photons up to a maximum "digital number," and then it starts to bleed.
http://sohowww.nascom.nasa.gov/explore/faq/instruments.html (9 of 22) [2000/01/11 11:57:58 AM]
Dr. SOHO's FAQ: Scientific Instruments
Go back to "Instrument" questions and answers.
Would you please identify the planetary object in the lower left view of the attached LASCO C3
image of 1998 May 26?
●
The bright object in the southeast (lower left) quadrant of the SOHO/LASCO-C3 images you refer
to is, surprisingly, not a planet at all.
Rather, it is the 1st magnitude star Aldebaran, the blazing eye of Taurus, the Bull. It sure looks
like a planet, though, as its brilliance is sufficient to saturate the C3 telescope's CCD detector
slightly. This causes the little horizontal bar, a result of the CCD phenomenon know as
"blooming," analogous to a "solarized" image on photographic film. Of course, we adjust our
exposure times to be optimal for the faint solar corona, as that is of most interest to our
experiment. At that exposure, most of the objects we've seen that are bright enough to saturate the
detector have been planets, but a good bright star can do it too, as is the case here.
Taurus, like most of the ecliptic constellations, transits LASCO's field of view once each year as
SOHO and Earth both revolve around the Sun.
Go back to "Instrument" questions and answers.
I have a question concerning the LASCO C2 coronagraph QuickTime movies on your page:
http://seal.nascom.nasa.gov/gallery/current/. I noticed there are bright dots in these movies which
don't change much from frame to frame. My question is: are these just artifacts? If not, what are
they? The only other things I would guess them to be are stars, but that seems like a very long shot,
and, as I said, I'm just guessing.
●
The short answer is that your guess was right: most of the "dots" you see in these images are stars.
The C2 coronagraph has a field of view that is 3 degrees across. The C3 field is even bigger; at 16
degrees, it is big enough to include the whole of the constellation Orion. The telescopes are
designed to detect very faint coronal signals, and one of the consequences of this design is that
they detect background stars in C3's field of view down to about 10th magnitude. The faintest stars
you can see with your eye from the ground are about 6th magnitude, which is a factor of 40 times
brighter than 10th magnitude. So LASCO sees many more stars in any region of the sky than you
can see with the naked eye, even from the best observing sites.
If you watch a movie covering a few days of LASCO data, you will see the stars move slowly
toward the right on the images. This is because SOHO lies between the Earth and the Sun. Its orbit
follows the Earth around the Sun, so as the Earth moves, the stars appear to move.
Some of the dots are due to cosmic rays (energetic particles that are continuously present in
interplanetary space) striking the detector during an exposure. These particles produce a bright
spot if they pass through just one "pixel" on the CCD digital camera, and occasionally show up as
a bright streak if they pass through several pixels at a glancing angle. These streaks do not persist,
of course; they only affect single frames of image data.
Go back to "Instrument" questions and answers.
http://sohowww.nascom.nasa.gov/explore/faq/instruments.html (10 of 22) [2000/01/11 11:57:58 AM]
Dr. SOHO's FAQ: Scientific Instruments
●
What are those sudden, flickering objects in the LASCO images?
The bright streaks that you refer to are caused by energetic particles that were emitted from the
Sun several hours earlier in a large solar flare and coronal mass ejection (CME). When these
particles strike the LASCO CCD camera, they leave bright streaks, giving the appearance of a
"snowstorm".
There are always a few particle "hits" in any LASCO image, just from the general background of
cosmic rays in space, but the rate of hits increases dramatically after a big flare and can sometimes
take several days to go back to normal levels.
You can see movies of the latest LASCO and EIT data on the web, both on the SOHO web site,
and at the LASCO home page:
http://lasco-www.nrl.navy.mil/lasco.html
(LASCO Home Page)
I have a backlog of items to ask you about, and have included a frame from the C3 1/4 scale gif
movie of April 24/25 1998. It appears to show a flux tube of energy of huge proportions exiting the
Sun on a SE to NW 45 degree diagonal. This occurred about 12 hours after the X-2 X-ray event of
April 23 1998, which was preceded by the M-2 proton event of April 20 1998. The whole gif is very
dramatic.
●
The sequence you refer to, on 23rd April 1998, is indeed an interesting one, but I'm afraid I have to
tell you the particular frame at 20:44 UT is a bit of a red herring. The data is good, and valid
enough, but the very unusual streak from lower left to upper right is not of solar origin. You can be
sure of that, because it appears in only one frame, and not in accompanying data from the C2
telescope. Our interpretation is that this streak was caused by a random dust particle passing across
the C3 telescope's field of view.
In a typical C3 exposure, the shutter is open for about 19 seconds. Fast-moving particles
(micrometeorites) pass across the field from time to time; that is not unusual. They appear as a
streak rather than a point in the image because they are moving rapidly with respect to the
spacecraft. One can then calculate how far such a particle is from the telescope's objective lens by
noting how far out out of focus it is, i.e., how broad the streak. A very distant particle is in good
focus and appears as a very thin streak. Closer particles make broader tracks. This particle is
unusual only in that it is brighter than most, and very, very close to our objective aperture, a "close
encounter," so to speak. The dark swath down the center of the bright streak results from the way
our optical system handles such a source. The tapering and distortion of the bright band in the
areas near the central occulting disk result from the instrumental vignetting function (The optics
cause a variation in brightness across the field of view). The streak is somewhat broader in the
upper right than in the lower left; this indicates the particle was moving closer to (or father from )
the aperture as it crossed the field.
●
I had hoped that the "dust particle" explanation for all "anomalies" would have stayed on the
shelf for a little while longer. Several things about your reply do not jive with my experience.
First, one-frame events are very common on the C3, as the interval is usually one hour between
http://sohowww.nascom.nasa.gov/explore/faq/instruments.html (11 of 22) [2000/01/11 11:57:58 AM]
Dr. SOHO's FAQ: Scientific Instruments
shots. Second, as you should very well know the C2 image is a different spectrum than the C3 so
many things that are visible on the C3 don't even show up on the C2. I have attached the C3 1/4
resolution gif so that you might also see that there is no "streak" or "passing." The flash is also
pinched at the Sun and broadens as it moves outward from the Sun in both directions.
1. Your first point is precisely the one I was wished to make; I am sorry I was not clear: As
the field of view of the Lasco C3 is a cone of +/- 8 degrees, it is perfectly reasonable that a
particle close to the aperture will cross the field entirely during one 19-second exposure. At
the solar distance of 1 AU, the width of the C3 field is about 42 million kilometers; it is not
reasonable to propose that large clouds of solar plasma have propagated across such a
distance in an hour.
2. Both the C2 (orange) and C3 (clear) filters are designed with very broad spectral
response, and their passbands differ but slightly. They are for the most part coincident, with
the C2 being slightly more restricted at the blue end. This is to improve the focal properties
of C2. Specifically, C3's clear filter has >80% transmittance between approx. 5000-8000 A.
C2's orange passband is about 5400-6300 A. (C2's limiting magnitude is somewhat lower
than C3, so C3 does see some dimmer objects. This is a function of brightness, not color or
spectrum)
3. Early in the mission, we thought we saw increased instances of "objects" in our field of
view whenever the CDS instrument opened its doors. So, on 1996 June 28, we performed a
test. C3 took a rapid series of images as CDS opened both the GIS and NIS doors. After
each opening, subsequent C3 images showed many streaks. We interpret them to be
particles from SOHO's insulation knocked off by CDS's doors. They were narrower and
brighter than many subsequent "debris" streaks, but they do strongly suggest that nearby
physical particles would be visible to LASCO.
4. Thank you for forwarding a 1/4 resolution .gif. However, we do not work with these, but
with the full resolution data which, perhaps unfortunately, is too voluminous to put on the
Web because of the bandwidth restrictions. Nevertheless, I am glad you can see the pinching
of the streak in the areas near the occulter. This is, as I said in my earlier reply, an effect due
to the vignetting function of the optical system. The coronagraph optics can distort objects
very near the edge of the occulter; you can see this when a planet or bright star passes
through the field of view.
For further information I refer you to the SOHO Web Page and references therein; to some
excellent books now available, such as Ken Lang's Sun, Earth, and Sky, or Herb Friedman's
Between Sun and Earth. You may find the answers to more subtle instrumental questions in
Rudolf Kingslake's classic series on optical design. Specifically, coronagraphic questions
are to be found in the scientific literature. Start with the SOHO Web Page, click on LASCO,
or consult the "LASCO User's Handbook" at:
http://lasco-www.nrl.navy.mil/handbook/hndbk.html
(LASCO HANDBOOK FOR SCIENTIFIC INVESTIGATORS)
Go back to "Instrument" questions and answers.
http://sohowww.nascom.nasa.gov/explore/faq/instruments.html (12 of 22) [2000/01/11 11:57:58 AM]
Dr. SOHO's FAQ: Scientific Instruments
●
The LASCO images show very low solar wind mass in the southern pole region. Is this true, or
is this related to the imaging telescopes' present view of Sol? Thanks to all who are involved
with this experiment for making this information and beautiful images available for my
observation.
The raw LASCO images do not show very much detail; one needs to process them to bring
out the contrast, coronal streamers, etc. One "quick and dirty" way of processing them is to
subtract a background image from them. That is, we create an artificial image from a
long-term average of many actual images. When we subtract this background from any real
image, the short-term changes jump out much more obviously.
This is what we do to our pictures on the web.
Now, this technique is only an approximation. The solar corona evolves over time, so the
background image becomes obsolete after a while. We do update these backgrounds
regularly, but because of SOHO's orbital motion (and other reasons), they can never quite
stay current. One of the chief results of this lag time is that the southern part (lower half) of
our C2 and C3 images sometimes looks dim and "depleted." (Other effects may contribute
to this depletion, but the background model is the strongest contributor.)
Remember, however, that we are near the minimum of the 11-year solar cycle. Coronal
outflows during solar minimum tend to cling to the equatorial streamer belts, which are so
prominent in our images right now (mid-1996).
In summary: Most of that depletion you see in the south is due to the image processing.
Things do look much more symmetrical after further processing routines. I thank you for
your interest in our products -- and please query Dr. SOHO again if you have further
questions.
Go back to "Instrument" questions and answers.
●
Dear Ladies and Sirs, As I read in "Spektrum der Wissenschaft" (the German issue of
"Scientific American"), the Sun is (among other things) comparable to a large, oscillating bell.
I really like:
http://soi.standord.edu/results/sounds.html
(solar Sounds from MDI)
But is it possible to constantly couple the data from MDI or GOLF with a music sound (bass,
cello or anything, that effects a well-done, suitable sound) here on these webpages? For me, it's
a fantastic imagination to hear a rhythm and to know that it's the ~5 minutes old sound of the
Sun.
It is not possible to listen in real-time, or even near-real-time, to the notes coming from the
Sun. There are two main reasons for that. The first is that it takes a lot of computer
processing to separate the individual notes from one another. If you sample a single piece of
the Sun, you will `hear' literally thousands of discordant notes combined, which sounds a lot
http://sohowww.nascom.nasa.gov/explore/faq/instruments.html (13 of 22) [2000/01/11 11:57:58 AM]
Dr. SOHO's FAQ: Scientific Instruments
like noise or an uninteresting rumble.
The second reason is that you have to speed up the incoming data by a factor of about
40,000 to make it audible to humans. So each second of sound would require about 12
HOURS of solar data. (A human ear can hear frequencies from 20 Hz to 20,000 Hz. Solar
oscillations happen in the milliHertz range.)
There are some solar sounds on the web, at the Solar Oscillations Investigation site at
Stanford University -- they're just not "real time." The SOI home page is, of course:
http://soi.stanford.edu/
(The Solar Oscillations Investigation)
and the sounds are, as you have found, available at:
http://soi.stanford.edu/results/sounds.html
(Solar Sounds)
Currently, they are from about 45 days' worth of data that were collected during the spring
and summer of 1996.
Go back to "Instrument" questions and answers.
●
There is a time listed on the heading of the SOHO MDI Intensitygrams. Is that time UTC, EST,
or some special space time?
The time listed on the MDI images is TAI time. TAI stands for (in French) "Temps
Atomique Internationale," or International Atomic Time. It is very similar to UTC:
Coordinated Universal Time.
The difference is that UTC is based on a celestial quantity (the rotation speed of the Earth);
while TAI is based on the time displayed by a particular atomic clock. Cesium atomic
clocks keep time better than the Earth does: the Earth is slowing down gradually (and
measurably) due to the drag induced by lunar tides.
This gradual slowing of the Earth's rotations is, I think, the reason for the occasional "leap
seconds" that get added to a year occasionally. The two time standards, UTC and TAI,
started out synchronized. They are currently something like 16 seconds apart.
You can find out more about these times at the United States Naval Observatory's Time
Service Department:
http://tycho.usno.navy.mil
(Time Service Department)
(The Naval Observatory keeps the United States' atomic clocks.)
Go back to "Instrument" questions and answers.
http://sohowww.nascom.nasa.gov/explore/faq/instruments.html (14 of 22) [2000/01/11 11:57:58 AM]
Dr. SOHO's FAQ: Scientific Instruments
●
I have following queries regarding SOHO images: 1. Where are the dopplergrams available in
the SOHO database. I could not locate them using search. 2. At what interval of time are these
dopplergrams available.
There are a couple of places where dopplergrams can be retrieved from the SOHO database:
1. Mt. Wilson dopplergram images can be retrieved from the SOHO Daily Images web page
at:
http://sohowww.nascom.nasa.gov/data/latestimages.html
(SOHO Latest Images)
under "Non-SOHO Synoptic Data." This web page will show you the dopplergrams as GIF
images with a delay of about 12 to 24 hours. You can then access the actual FITS files (with
the same delay) by following the links "The SOHO synoptic database" and then "SOHO
Synoptic Database Search Engine". This will allow you to search for dopplergrams by date,
and the result will be links to the FITS files.
You can also access the Mt. Wilson dopplergrams directly from the source at:
http://www.astro.ucla.edu/~obs/intro.html
(Mt. Wilson 150-Foot Solar Tower)
2. MDI dopplergrams are not made available as part of the SOHO Summary Database.
However, you can access these images from the MDI/SOI Web page at:
http://soi.stanford.edu/
(The Solar Oscillations Investigation)
by following the "Quick-Look Data Image Atlas" link. For access to MDI FITS files, please
contact the MDI/SOI team.
Go back to "Instrument" questions and answers.
●
I am interested in the 2.65 hour long [solar] pulsations reported by the Crimean Astrophysical
Observatory in 1975. I would like to see this report either confirmed or refuted. I have been
unable to find anything on this on the internet or in publications.
In the 1970's both the Crimean Astrophysical Observatory and the Wilcox Observatory
measured oscillations that were barely statistically significant and that looked like g-modes.
However, SOHO/MDI has an order of magnitude less noise in its data and has not observed
these oscillations. This information is from a paper presented by P.H. Scherrer at the Spring
1998 American Geophysical Union Meeting.
Go back to "Instrument" questions and answers.
●
Should I assume that all EIT images have the Solar axis straight up? Is this axis sometimes
tilted towards the camera and sometimes away (as SOHO circles the Sun)?
http://sohowww.nascom.nasa.gov/explore/faq/instruments.html (15 of 22) [2000/01/11 11:57:59 AM]
Dr. SOHO's FAQ: Scientific Instruments
The spacecraft is usually aligned such that it points directly at Sun center with Solar North
up. This does not change as we move around the Sun. There are a few occasions (every 6
months or so) when we do special maneuvers, such as roll the spacecraft pointing around the
Sun for some special observations. At these times, our images may not have Solar North at
the top. Also, in many of the EIT images on the web after observations resumed following
the gap from June-Oct 1998 North is to the top left, again because the spacecraft has rolled.
Furthermore, the poles do tip towards and away from us at different times of the year, as you
suggest. Because SOHO is basically in the direction of Earth, you can get an idea of the
amount of this tilt by looking up the solar "B-angle" in an astronomical almanac.
Go back to "Instrument" questions and answers.
●
I've searched the site, unsuccessfully, for the temperature range of emission spectra Fe IX/X
171 Angstrom, and Fe XV 284 Angstrom utilized in the Extreme Ultraviolet Imaging Telescope.
Could you help me out? The SOHO Observatory web site is so informative and exciting for an
amateur such as myself. Anything you would like to add concerning the EIT science objectives
would be interesting.
The EIT images taken in the Fe IX/X 171 Angstrom bandpass looks at the coronal plasma at
about 1 million K, while the Fe XV 284 samples the 2 million K (degrees Kelvin) plasma.
While I'm at it, the Fe XII 195 Angstrom is about 1.5 million K and the He II 304 is
sensitive to the chromospheric plasma at about 80,000 K. More information, including EIT's
goals, is on the web at):
http://umbra.nascom.nasa.gov/eit/EIT.html
(The Extreme Ultraviolet Imaging Telescope)
If you would like more information or an explanation of any of these details, please let me
know. We're very pleased that our spectacular images (yes, even to the "professionals,"
these images are amazing) are getting the public interested in the Sun.
Go back to "Instrument" questions and answers.
●
I am curious about the CCD bakeout that you periodically experience. Is this an elevated
temperature bakeout of the CCD to anneal electron traps in the bulk caused by particle
bombardment in space? What kind of CCDs are used on SOHO?
One of EIT's scientists wrote a concise essay describing these "bakeouts" at this link:
http://umbra.nascom.nasa.gov/eit/CCD_bakeout.html
What's a CCD bakeout, anyway?
The following answer was provided by an MDI scientist. (Although MDI uses a CCD
camera, it does not require periodic bakeouts.):
There are several different types of detectors on SOHO. Several instruments use CCDs:
MDI, LASCO, and EIT, for example. EIT's CCD is the only one that regularly gets baked
http://sohowww.nascom.nasa.gov/explore/faq/instruments.html (16 of 22) [2000/01/11 11:57:59 AM]
Dr. SOHO's FAQ: Scientific Instruments
out; I'll assume that's the one you mean. It's a special "thin substrate, back-illuminated" CCD
that is sensitive to the EUV/XUV (extreme ultraviolet) light that EIT detects. The CCD
devices are normal charge traps, just like in a commercial video camera -- but their
manufacturing standards are much higher, and the 'chip' is extra thin. XUV light must
penetrate through the silicon substrate of the chip from the back side to get to the charge
traps.
The CCDs on all the instruments are kept very cool (about 200 Kelvin, or -80 degrees
Celsius) to lower their dark current and make them more sensitive. There are a number of
reasons why one would bake them out periodically.
First, because the CCD is cold, stray gas molecules inside the spacecraft tend to stick to it -so a deposit tends to build up slowly on the CCD. This is how some vacuum pumps,
"cryopumps" and "sorption pumps," work here on Earth. Briefly heating the CCD can drive
off this layer of contamination.
The second main effect, which you mentioned, is electron trapping in defects in the crystal
structure of the CCD. Defects are caused by cosmic rays and other ionizing radiation.
EIT's CCD is especially sensitive to both of those effects. The first (sorption of stuff onto
the surface) affects it because most materials strongly absorb XUV -- which is why it's still
something of a "spectral frontier:" it's very hard to work with light at wavelengths around
200 Angstroms, because most materials don't reflect it very well and all materials absorb it
relatively well. So any deposit on the EIT CCD tends to absorb the XUV photons before
they get to the detector itself, on the back side of the CCD.
Furthermore, the XUV photons are themselves energetic enough to slightly damage the
crystalline structure in the CCD -- so your EIT detector slowly degrades as you use it. (In
1996 the shutter jammed open for a few hours as a solar active region was crossing the disk;
and this left a little "burn" on EIT's field of view near the southeast edge of the Sun). This
effect is also seen on the Yohkoh spacecraft, which uses a similar CCD to image soft X-rays
(around 1 Angstrom wavelength) from the Sun.
Baking helps to fix both problems: heating the CCD makes frozen gases "boil off" and
either drift away or stick to something else; it also shakes the crystal structure enough to
repair small defects. But the mechanism is poorly understood, and different "experts" will
tell you different stories about what's going on inside the CCD during the heating process.
In point of fact, EIT is baked out periodically because it works better that way -- even if we
don't know exactly why.
Go back to "Instrument" questions and answers.
●
Why do the EIT images seem to "jump" suddenly on 1998 March 4?
The reason for the apparent "jumping" was an imaging problem. There was some software
failure on SOHO that caused the aligning mechanism to drift. It was discovered fairly
quickly and fixed the same day. Things do not always work perfectly, and this is one
http://sohowww.nascom.nasa.gov/explore/faq/instruments.html (17 of 22) [2000/01/11 11:57:59 AM]
Dr. SOHO's FAQ: Scientific Instruments
example.
Click on "LASCO/EIT Real Time Movies" from here to get a list of the latest available
movies, both in .mpeg and animated .gif format. These movies are regularly updated, so that
they show roughly the latest 48 hours of data, depending on the scheduling of contacts with
the spacecraft.
Incidentally, there was another large flare and CME on April 20 [1998], on the opposite
limb to the event of April 23. This earlier event also produced lots of particles that impacted
SOHO.
Luckily, the Earth's magnetic field largely shields us (here on the planet's surface) from
potentially harmful effects of such high energy particles.
Go back to "Instrument" questions and answers.
●
I have noticed that quite a few of the images from the EIT the past few days [April 1998] have
had a remarkable amount of noise or dots or whatever you want to call them on the images. Is
this due to some problem with the processing of the images, or is there something on the lens of
the instrument??
Actually, neither explanation. It was a true Solar phenomenon. On April 20 there was a
large Solar flare and a Coronal Mass Ejection. These events produced highly energetic
protons (one form of cosmic rays). This event was such that the particles headed toward
SOHO and the Earth (not to worry, this has no effect on people due to the Earth's magnetic
field) but when the particles impact the EIT CCD detector they show up as "points of light."
Technically, the protons interacted with the detector and freed electrons which are captured
and reported by the CCD. A movie of part of this event can be viewed from the EIT
homepage:
http://umbra.nascom.nasa.gov/eit/
(SOHO Extreme ultraviolet Imaging Telescope)
There have been other similar events. One well- documented event can be found at:
http://www.lmsal.com//eit/eit_proton_19971106.html
(EIT Proton Detection)
Here you can see the EIT movie as well as the associated GOES X-ray detections. Another
similar event happened on May 2, 1998.
Go back to "Instrument" questions and answers.
●
In the FeIX/X 171A image taken 1998 April 24, there are brightly lit loops on the East limb and
in the Northern part of the same limb are some dark loops. What makes these loops either dark
or bright? I presume that it is temperature, but what makes some of them have much higher
temperature than others?
You are correct that the brightness reflects temperature differences in the loops. Loops have
http://sohowww.nascom.nasa.gov/explore/faq/instruments.html (18 of 22) [2000/01/11 11:57:59 AM]
Dr. SOHO's FAQ: Scientific Instruments
different temperatures for a number of reasons. I will briefly point out the extreme: the
bright loops seen in the Fe IX,X typically have temperatures > 1.2 million Kelvin. The
source of the heat input is one of the major questions that SOHO was launched. Basically, it
is a magneto-hydrodynamic (MHD) phenomenon. The loops outline strong magnetic fields.
It is an debated question as to whether they are heated at the tops and the heat conducted
down or heated from below from magnetic reconnection, or twist or other mechanism. The
exact mechanism may depend upon how hot the loop is.
The "dark" loops seen in the Fe IX,X images are usually very cool loops, some about
50,000-80,000 Kelvin. These loops usually are from material in the chromosphere that has
for a variety of reasons extended into the corona. This cool material absorbs the light
(photons) from the hotter coronal material behind it (like dark smoke) and therefore appear
dark.
Go back to "Instrument" questions and answers.
●
Almost all EIT images have a band around the limb of the Sun that is either darker (304 A) or
darker (171 A). They do not look natural. What causes that?
You are correct, the ring is artificial (i.e. part of the camera). As time goes on, places on the
CCD become less sensitive to incoming light. This happens most on the limb (where the
Sun is actually brightest, at least for the iron lines). This is one of the reasons for a bakeout:
to reduce this effect.
Go back to "Instrument" questions and answers.
●
How long is a single EIT exposure?
The exposures vary depending upon the wavelength, and the type of feature we are looking
at. Typical exposures are:
171 - 7 seconds
195 - 12 seconds
284 - 152 seconds
304 - 52 seconds
Go back to "Instrument" questions and answers.
●
Are all EIT images made with the same camera with the use of different filters, or are there
separate cameras? I noticed that all images are always made at different times.
All of the images are made with the same camera. Actually, the telescope design does not
use different filters but different quadrants on the mirrors. Each quadrant of the mirror is
specially coated to reflect only the wavelength of light of interest, i.e. one of the four. All
four wavelengths are imaged on the same 1024x1024 pixel CCD.
Go back to "Instrument" questions and answers.
http://sohowww.nascom.nasa.gov/explore/faq/instruments.html (19 of 22) [2000/01/11 11:57:59 AM]
Dr. SOHO's FAQ: Scientific Instruments
●
I am reading papers in Scientific Computing magazine (issue 37, April 1998) on wavelet
transform in astronomy. I have learned that in the SOHO mission several lossless and lossy data
compression algorithms have been implemented (in particular the paper refers to LASCO and
EIT instruments). I am currently working on the IBIS imager on board the INTEGRAL ESA
satellite. We have problems with telemetry data rate, and therefore we are thinking about
implement compression algorithms. May you help me find some information or at least where I
can find information about loseless algorithms?
Some information can be obtained at:
http://lasco-www.nrl.navy.mil/handbook/hndbk_10.html
(Chapter 10: Onboard Software: Image compression)
However, this is a little out of date. The only algorithm LASCO found satisfactory from the
given list was RICE compression. It is lossless. For the RICE compression algorithm we are
typically getting about a factor of 2.5 compression. In addition to the algorithms described
on that page, LASCO has implemented a wavelet compression. This can either be lossless or
lossy. We found the lossless wavelet was slower than the RICE and is therefore not used.
Details on the lossy compression depend a lot on how structured your data is: is it 12, 14, 16
bit, etc?
Another current Solar mission, TRACE, is using some variant of JPEG compression. I
believe this also goes from lossless to lossy. You may wish to contact them as well:
http://vestige.lmsal.com/TRACE/
(< Welcome to T R A C E on-line >)
I hope this steers you in the right direction. Let me know if there are more specifics you can
not find.
Go back to "Instrument" questions and answers.
●
I read that a solar flare zapped SOHO on 21 April 1998. If this is true, what was the extent of
damage, if any, to SOHO?
Thanks for asking Dr. SOHO! And thank you for your concern for SOHO's safety.
There was a solar flare on 20 April 1998. Although flares can damage spacecraft
components, SOHO suffered no permanent physical damage from this flare.
However, the flare did affect our scientific observations for a day or two. SOHO's sensitive
spectrographs, like CDS and SUMER, try not to point at potential flare regions in the first
place. The 20 April flare also caused a "solar proton event," or SPE. This SPE increased the
"cosmic-ray flux" in SOHO's imagers, LASCO and EIT.
When you look at a LASCO movie, for example, you will see many flickering sparkles in
the background. These are energetic particles striking the camera. Usually, they cause no
damage. The SPE caused many more of these particles to hit LASCO during each exposure,
resulting in a "snowstorm" effect in our images. Check LASCO's web page for movies of
http://sohowww.nascom.nasa.gov/explore/faq/instruments.html (20 of 22) [2000/01/11 11:57:59 AM]
Dr. SOHO's FAQ: Scientific Instruments
this effect:
http://lasco-www.nrl.navy.mil/daily_mpg/1998_04/
(Index of /daily_mpg/1998_04/)
There are other effects on SOHO's observations, too. LASCO's C1 Fabry-Perot filter cannot
calibrate itself in the midst of such high particle flux. So most C1 images during these
periods cannot be analyzed spectroscopically.
However, everything aboard SOHO went back to normal after the flare's particle fluxes
receded.
What sort of damage can a spacecraft suffer during large flares? Most satellites are
solar-powered. Those broad panels solar cells slowly degrade during several years of
exposure to solar radiation. The extra radiation of a solar flare can cause a swift burst of
faster degradation. (SOHO was equipped with plenty of extra solar array area, as are most
other spacecraft.) Onboard computers can be "fried" by a flare, although I cannot describe to
you the exact process. SOHO's electronics are radiation-hardened, but this hardening does
not guarantee perfect immunity to space radiation. Solar arrays and electronics are two
spacecraft systems vulnerable to flare damage.
For more information on other possible damage to SOHO, please see our FAQ:
http://sohowww.nascom.nasa.gov/explore/faq/spacecraft.html#DAMAGE
("I heard rumors [in November 1997] that the SOHO is damaged?")
Go back to "Instrument" questions and answers.
●
Why don't sunspots show in the 'SOHO EIT, He II line, 304' images?
Actually they do, but generally as bright features rather than dark ones. Sunspots are the
dark regions seen in the Photosphere (the part of the Sun you see with your eye). These
regions are relatively dark (as compared to the surrounding Sun) because they are cool. The
cooling mechanism is complicated but basically, they are regions of strong magnetic fields.
These fields continue rising above the photosphere into the chromosphere (where you see
the He II 304) and into the Corona (where we see the EIT Fe iron lines).
In these regions, the places where the magnetic field is strong are actually hotter than the
surrounding area and they appear bright. In the corona you see the bright loops which are
anchored (the footpoints) to where the Sunspots are. The chromosphere (and transition
region) is an in-between region.
You can read some more about these regions at the SOHO education pages:
http://sohowww.nascom.nasa.gov/explore/materials.html
(SOHO Poster: New Views of the Sun)
as well as following some of the links on:
http://sohowww.nascom.nasa.gov/explore/faq/instruments.html (21 of 22) [2000/01/11 11:57:59 AM]
Dr. SOHO's FAQ: Scientific Instruments
http://sohowww.nascom.nasa.gov/explore/links.html
(SOHO Links !!!)
Go back to "Instrument" questions and answers.
●
I noticed something while playing around with an ultraviolet picture of the Sun on my photo
editor. Showing the negative of the image really makes viewing of sunspots a lot easier. Tell me
what you think.
Astronomers often use the trick of looking at the negative of an image instead of the
positive. It's particularly used in stellar astronomy. When displaying data to the public,
though, one almost always displays it as a positive. Otherwise, it would be too confusing to
those not used to looking at the data.
Personally, it hadn't occurred to me to try the trick with images of sunspots, but I can see
where that could be very helpful.
Go back to "Instrument" questions and answers.
Go back to "Dr. SOHO's FAQ."
SEND US YOUR COMMENTS
Go To
Other SOHO Web Pages
Author: Shane Stezelberger
Co-Author/Curator: Therese A. Kucera
[email protected]
Responsible Official: Art Poland([email protected])
Last modification: Monday, 08-Nov-1999 10:05:02 EST
http://sohowww.nascom.nasa.gov/explore/faq/instruments.html (22 of 22) [2000/01/11 11:57:59 AM]
Dr. SOHO's FAQ: Space Missions
Go back to "Dr SOHO's FAQ"
Go back to the "SOHO" main page.
●
●
Will a magnet work in space?
●
Can a spaceship go around the asteroid belt?
●
Where can I find information about solar sails?
●
Can you tell me about the spaceplane that NASA is working on?
●
What's the fastest speed at which a space vehicle of today can travel?
●
Will NASA be launching a mission to sample the newly-discovered lunar water?
●
Would it help to use a rotating spacecraft on a manned Mars mission?
●
Why has Venus been ignored in current and future planetary-exploration missions?
●
Who has flown the closest to the Sun?
SPARTAN-201
❍ What can you tell me about the Spartan satellite?
❍
Could the SPARTAN-201 satellite have been grappled at its center of rotation?
❍
Could SPARTAN-201 have been retrieved via a reel-and-cable system?
❍
Could SPARTAN have been yanked out of orbit by disturbing its rotation?
Will a magnet work in the same way in space as it does on the ground?
Yes! Magnets work quite well in space. Magnetism does not require gravity (as convection does)
or an atmosphere (as combustion does). The Earth itself has a good magnetic field. It makes
compass needles point north. In fact, satellites sometimes use magnets to "push against" the Earth's
magnetic field and thus straighten themselves out like a compass needle.
SPARTAN-201 is one of these satellites. The spacecraft was spinning out of control during its
1997 flight. It got going fast enough that its own magnetic system turned on automatically and
stopped the spin.
http://sohowww.nascom.nasa.gov/explore/faq/space_missions.html (1 of 9) [2000/01/11 11:58:03 AM]
Dr. SOHO's FAQ: Space Missions
The Hubble Space Telescope has some long, straight rods on its exterior, about where the thick
section meets the thin. These are its magnetic "torquers," which help maintain the telescope's
attitude control. They provide a counter-torque to Hubble's reaction wheels, but I am fuzzy on the
details.
SOHO (our satellite) is in a strange orbit, very far beyond the Earth's magnetic field. We cannot
use magnets to control our spacecraft. SOHO has large flywheels instead. They act like gyroscopes
inside the satellite. They force the bulk of the spacecraft around them to point at the Sun
constantly.
Go back to "Space Missions" questions and answers.
●
Can a spaceship go around the asteroid belt?
Yes and no.
The asteroid belt is spread out in a ring around the Sun, so a spaceship couldn't really "orbit
around" it. If you tried to orbit around the Sun faster than the asteroids do at that distance, you'd
burn up a good deal of fuel and end up coasting into a higher orbit.
The orbital mechanics can get tricky, you see.
However, there is no reason you couldn't simply send a spacecraft to the asteroid belt itself. Is this
what you were asking? In fact, this was done with the Galileo spacecraft -- which flew through the
asteroid belt anyway on its journey to Jupiter. NASA's NEAR probe is heading toward an asteroid,
though its target is not from the main asteroid belt:
http://sd-www.jhuapl.edu/NEAR/
(Home of the Near Earth Asteroid Rendezvous Mission)
Go back to "Space Missions" questions and answers.
I was looking for information on when we could expect to build and launch a solar sail. I am aware
of prototypes that have been produced, but I understand that there has yet to be a true solar sail
launched into space. I had heard that NASA was doing some work in this area and came across this
website. Do you have any information or contacts that would be helpful in finding out more about the
future of solar sails?
●
The principle behind solar sails has been known for a long time. If you have a reflective surface,
such as aluminum- coated mylar, then sunlight bouncing off of it will impart some momentum,
just like the wind imparts momentum to a sailboat. The trick is to build a large enough solar sail to
be useful.
Let's deal with some numbers. At the distance of the Earth from the Sun, the light pressure from
the Sun is approximately 0.0001 dynes/cm^2. In order to accelerate a spacecraft of 1000 kilograms
by 1 percent of the gravitational acceleration at the Earth's surface, then one would need a solar
sail of 100,000 square meters, or a square approximately 300 meters on a side.
http://sohowww.nascom.nasa.gov/explore/faq/space_missions.html (2 of 9) [2000/01/11 11:58:03 AM]
Dr. SOHO's FAQ: Space Missions
As one gets farther away from the Sun, the light pressure drops off as the inverse square of the
distance, and the size of the sail would have to be correspondingly bigger to get the same
acceleration. A practical sail size would most likely be on the order of a kilometer or more on a
side. So one can see that this is technically challenging, but not impossible.
In the SOHO project, our emphasis is on the physical study of the Sun, and not on the practical
uses of the solar flux. I've generally heard of solar sails mentioned in conjunction with possible
planetary missions, such as comet fly-bys. The planetary astronomy community, for example at
JPL, is probably a better place to inquire about future plans that NASA might have in regards to
developing solar sails:
http://www.jpl.nasa.gov
(The Jet Propulsion Laboratory)
Elsewhere, you might find these sites helpful:
http://www.ec-lille.fr/~u3p/index.html
(Solar Sails -- Home Page)
http://www.ugcs.caltech.edu/~diedrich/solarsails/
(Solar Sails)
Go back to "Space Missions" questions and answers.
Could you please send me some information on the experimental space plane that NASA is working
on? I would be grateful. Thank you.
●
NASA's next-generation launch vehicle may follow the X-33 program. X-33 itself is a testbed, not
a launcher:
http://rlv.msfc.nasa.gov
(Reusable Launch Vehicle)
You might also be referring to the X-38 emergency return vehicle. I have copied the entire press
release on its latest test, which is probably how you came to hear of it recently. I am also including
a web address for an ABC online news story on the vehicle that includes a photo of it. There are
also links to stories on the international space station in case that is what you are seeking
information on.
http://www.abcnews.com/sections/science/DailyNews/lifeboat0310.html
(X-38 Has Successful Test Flight)
Go back to "Space Missions" questions and answers.
What is the fastest a space vehicle of today can travel? What is the expected travel time to Mars at
the above speed? What is the fastest speed and travel time of vehicles of the near future?
●
Are you are asking about vehicles with humans in them? These are quite different from spacecraft
http://sohowww.nascom.nasa.gov/explore/faq/space_missions.html (3 of 9) [2000/01/11 11:58:03 AM]
Dr. SOHO's FAQ: Space Missions
without humans, as humans require a lot of heavy life support systems and the ability to come
back when they are done. The only space vehicle for humans we [NASA] have had for some time
is the space shuttle, which just goes in and out of Earth orbit. I guess you might count space station
Mir and its Soyuz ferries, too, although they don't actually go anywhere outside of low Earth orbit.
You also have to ask: how fast compared to what? The Earth is already going around the Sun at a
rate of almost one billion km/year (over 10,000 km/hr)!
It is projected that it will take about 6-9 months to send humans to Mars. Allowing some time
spent on Mars and the return voyage, the entire trip would take 2.4-3 years. If the route is
reasonably direct then the speed outward from the Sun would be = (Distance between Earth and
Mars at closest approach/time of the trip) = 78 million km divided by 6 months = almost 18,000
km/hour radially outward from the Sun (on average). There would also be a component of the
motion in the direction in which Mars and Earth orbit.
The current record-holders are still, I believe, the four far travelers: Pioneer 10, Pioneer 11,
Voyager 1, and Voyager 2. They are on their way out of the solar system, so they have exceeded
the Sun's own escape velocity. That velocity is about 18.47 kilometers per second (at Jupiter's
orbital radius) relative to the Sun.
For further information on Mars missions, you may want to look at the question and answer
service about Mars missions at:
http://quest.arc.nasa.gov/mars/ask/question.html
(Mars Team Online - Asking Questions)
In particular, they include some answers to previous questions about sending people to Mars at:
http://quest.arc.nasa.gov/mars/ask/humans-on-mars/
(Questions and Answers about Humans and Earth Life on Mars)
If you have a pdf viewer, you can look at this NASA document about a possible mission sending
humans to Mars:
http://www-sn.jsc.nasa.gov/marsref/
(Human Exploration of Mars: The Reference Mission)
Go back to "Space Missions" questions and answers.
I am writing to you because I would like to be a part of the flight to Mars. And I have an idea of
how to get there. I know that NASA already knows how to get the men and women of the mission to
Earth back from Mars (by the time the crew lands on Mars, a return shuttle will already be on Mars).
But I also know that NASA is having trouble with how the men and women of this mission will get to
Mars. I know NASA has many ideas (some from the movie: 2001 A Space Odyssey). Well, I would like
to add another idea to the batch, how about a UFO-like spacecraft? It will be launched from Earth
like a hand launching a frizbee. With this launching motion, the spacecraft will have artificial gravity
(so the problem with weakening muscles will not be a problem at all). Back to the reason I am writing
to you: could you send me as much information about Mars and space travel as possible?
●
http://sohowww.nascom.nasa.gov/explore/faq/space_missions.html (4 of 9) [2000/01/11 11:58:03 AM]
Dr. SOHO's FAQ: Space Missions
Here at SOHO, I'm hardly a Mars expert, but I'll try to answer your questions.
Unfortunately, getting to Mars is still a big part of the problem. We humans do not yet have much
practice building large, manned, interplanetary spacecraft to operate for several years at a time.
Even a minimal Mars spacecraft is going to be expensive to build. The political and financial
obstacles are probably the most daunting of all.
There are also issues about life-support; cosmonauts can live for more than a year on Mir, but they
have constant resupply from the ground.
Thirdly, the "human factors" of long-term spaceflight are a potential problem. Flying to Mars has
been compared to living on a submarine or at an Antarctic base. Living in cramped quarters for
months on end can put a strain on normal group dynamics. Look at Biosphere 2: the first 8 people
to live in it were all professionals -- and good friends from their long training program. Several
months of life Inside split them into two groups of four. They worked and ate separately, hardly
spoke to one another, and didn't correspond after they got out. We don't want a Mars crew to
psychologically self-destruct, so we will need to send the right team.
Concerning your idea for spinning the spacecraft, it certainly helps as a source of artificial gravity,
but it probably won't help the actual launch. Launching anything is always the hard part. The
rocket must overcome gravity, atmospheric resistance, etc. At the present time, launching is a
messy, dangerous, and expensive business. (That may change someday.) Once you get into orbit,
you're halfway to anywhere else in the solar system.
You are correct in saying that a return vehicle (or a fuel supply for the return trip) will probably be
sent to Mars first. Our ideas and plans about Mars missions have changed dramatically in the last
ten years. Robert Zubrin's book on this subject, The Case For Mars (1996), may interest you. Here
is a related web site:
http://marssociety.org
(Headquarters for the Mars Society) and especially:
http://www.nw.net/mars/marsdirect.html
(Mars Direct Home Page)
For example: Mars has a carbon dioxide atmosphere. Some chemical processes can produce rocket
fuel and oxidizer from this atmosphere. If we can send a small chemical processor and an empty
fuel tank to Mars instead of a fully loaded fuel tank, we can save weight. The less weight needed,
the cheaper a Mars mission can be. It costs something to launch every kilogram.
I hope this helps. Please write back if you need more information. If you have web access, the
Mars Pathfinder site will certainly interest you... but you've probably been there quite often
already! http://wwwmpf.jpl.nasa.gov
(JPL Mars Home Page)
I'm less familiar with this site, but SEDS (Students for the Exploration and Development of Space)
has planetary and spaceflight information:
http://www.seds.org/
http://sohowww.nascom.nasa.gov/explore/faq/space_missions.html (5 of 9) [2000/01/11 11:58:03 AM]
Dr. SOHO's FAQ: Space Missions
(SEDS Internet Headquarters)
Go back to "Space Missions" questions and answers.
I've noticed that after sending two probes to the surface of Venus and Mars, we have sent many
more probes back to Mars, but none to Venus! I know Venus has a "corrosive" atmosphere, but it
seems to me we should be able to get past that. Is there a reason we have abandoned this? Should we
let 150 parts per million of sulfur dioxide stop us from exploring Venus?
●
Personally, I think that all of the planets in the solar system are interesting and deserving of
attention. However, budgets are tight, and one does have to prioritize. I can't really speak for
NASA management, but I can guess some of the reasons that Mars appears to be getting more
attention than Venus right now. One reason is that a manned mission to Mars is a high-priority
goal in NASA's long term planning. Another reason is the discovery of hints of life on the Allen
Hills 84001 meteorite.
Cost is probably also a major factor when NASA chooses between missions to Mars and Venus.
Mars is probably a less demanding target in terms of building a spacecraft capable of landing on
the surface and surviving. We can hope that future technology developments might help in future
Venus missions.
There's an interesting discussion about the possibility of future Venus missions, and what they
might do, at:
http://192.101.147.17/publications/newsletters/lpib/lpib74/venus74.html
(Venus After Magellan: Where Do We Go From Here? - by Walter S. Kiefer)
I'm sure that NASA intends to go back to Venus again someday, when the right mission plan
comes along, and the budget allows it.
Again, let me emphasize that I can't speak for NASA management. When it comes to knowing
what's going on in those higher levels of government, I get my information the same way you do,
from hearing it on the news.
Remember also that Pluto has never been probed by any spacecraft, Hubble photos
notwithstanding. Mercury has been ignored since Mariner 10 in 1972, too - and there has been
occasional talk of a Mercury orbiter since then. (After all, a good chunk of that planet's surface has
never been mapped or photographed....)
Go back to "Space Missions" questions and answers.
●
What is the closest to the Sun that someone has been?
Hmmm... I think the answer would depend upon a number of factors, such as the position of Earth
along its orbit during many key space missions during the late 1960s.
The 1966 Gemini X crew (John Young and Michael Collins) set an altitude record of more than
800 miles (over 1280 km), which stood until the Apollo lunar flights a few years later. You could
http://sohowww.nascom.nasa.gov/explore/faq/space_missions.html (6 of 9) [2000/01/11 11:58:03 AM]
Dr. SOHO's FAQ: Space Missions
say that, during these orbits, they were 800 miles closer to the Sun than anyone on Earth. Of
course, relative to the Sun-Earth distance of 93 million miles this was not much closer than us on
the ground. There's never been a manned "mission to the Sun." We at SOHO telecommute to the
L1 point.
I should point out that the astronauts going to (and landing on) the Moon were actually the farthest
from the Sun. All the lunar missions were launched when the Moon was farther from the Sun than
the Earth was. This put the "near side" of the Moon in daylight. Having said that, I should also
point out that the astronauts' lunar trajectories may have carried them closer to the Sun than
Gemini X. I do not have the necessary data here to check that.
(If you include unmanned space probes, the present record-holder is the US/German Helios B. It
flew within 45 million km of the Sun in 1976.)
Go back to "Space Missions" questions and answers.
●
Where can I get info about the Spartan mission?
Information on the Spartan satellite can be found on the web:
http://umbra.nascom.nasa.gov/spartan/
(SPARTAN 201)
Images from the latest flight (November 1998) can be found at:
http://thalia.gsfc.nasa.gov/~gibson/SPARTAN/
(SPARTAN 201-5 Operations Page)
Spartan contains two instrument, the ultraviolet coronal spectrometer (UVCS) and the white-light
coronagraph. UVCS is a spectrometer that images the corona in the ultraviolet regions, recording
intensity and spectral information of emission lines such as Ly-alpha, O5+ and Si2+. From this
data, scientists working on Spartan can better understand the dynamics of the corona and solar
wind - the constant stream of ionized particles streaming out from the sun streaming into our solar
system. The interaction of the solar wind with Earth is complex, affecting weather patterns,
communications and, in high latitudes, the Aurora Borealis or northern lights.
Go back to "Space Missions" questions and answers.
Could the tumbling SPARTAN-201 satellite, in November 1997, have been captured at its point of
axial or center of gravity?
●
No. Not this time.
The problem was that SPARTAN was tumbling around a random axis. There was only one place
on SPARTAN for the shuttle's arm to grab, and SPARTAN was not spinning exactly at that point.
This arm-grapple pin on SPARTAN was out near the "rim" of the spin, rather than at the "hub."
The astronauts tried at first to grab the satellite with the arm using very careful timing, but it just
did not work.
Go back to "Space Missions" questions and answers.
http://sohowww.nascom.nasa.gov/explore/faq/space_missions.html (7 of 9) [2000/01/11 11:58:03 AM]
Dr. SOHO's FAQ: Space Missions
Was there any way of hooking a free spinning device to the center axial of SPARTAN and cabling
that device to the arm of the shuttle, then pulling it in like a fishing pole?
●
Probably, but using a cable doesn't solve the basic problem of attaching the spinning SPARTAN to
a non-spinning shuttle. NASA decided to perform this attachment using the astronauts' arms
instead of the shuttle's arm. It worked, but it took a little longer than first planned. Also, there was
probably nothing aboard Columbia which could be used as a cable. And SPARTAN wasn't going
to wait patiently for Columbia to return with such a cable system on a later mission!
NASA has had "mixed" (good and bad) results using cables and tethers in space. The Gemini XI
mission in 1966 attached a 30m cable to a co-orbiting rocket stage, then tried to spin with it. You
may remember the two shuttle flights of the Tethered Satellite System a few years ago; the tether
jammed during the first mission and broke on the second.
Cable systems need reels, just like a fishing pole -- and reels consist of moving parts which are
hard to lubricate in the vacuum of space. That's just one problem.
(However, look closely at the astronauts the next time you see a spacewalk. Their spacesuit tethers
coil up into little reels near the waist ring of the suit. That is a tether with moving parts, and it
seems to work just fine. It is very small, of course. I'm not a spacesuit expert, so I'm just
speculating here. Direct any detailed questions to the NASA Johnson Space Center web pages.)
Go back to "Space Missions" questions and answers.
●
Could SPARTAN have been yanked out if its orbit by disturbing its rotation?
No. At least, not very quickly. The satellite's rate of tumble had nothing at all to do with its orbit.
Neglecting air resistance, the SPARTAN will not "fall out of orbit" any faster because of its spin.
However, even at several hundred kilometers above the Earth, there is still a trace amount of
atmospheric density. There are a few air molecules bouncing around at that altitude. Each time a
satellite (ANY satellite in low orbit, including the shuttle itself) bumps into an air molecule, it
slows down. Eventually, it will lose so much speed that it falls out of orbit and burns up during
re-entry.
Most satellites die like that. Skylab re-entered after years of faint atmospheric friction. Eventually,
so would SPARTAN, if Columbia's crew hadn't grabbed it.
Go back to "Space Missions" questions and answers.
Now that NASA has found what appears to be a form of water on the Moon, is their a plan to send a
mission to take physical samples? If so, how soon will it be possible to accomplish the task?
●
The Lunar Prospector team has indeed announced their confirmation of ice found around the lunar
poles, specifically in deep craters where there is constant shadow. Presently, there is no planned
NASA mission to obtain samples. However, an excellent discussion of future possibilities and
implications for the ice can be found on the Lunar Prospector web pages, in particular:
http://lunar.arc.nasa.gov/science/results/lunarice/mine.html
http://sohowww.nascom.nasa.gov/explore/faq/space_missions.html (8 of 9) [2000/01/11 11:58:03 AM]
Dr. SOHO's FAQ: Space Missions
(Lunar Prospector at NASA ARC: Science)
and following links.
Exploration of the Moon will also have to be balanced with our planned explorations of Mars.
Although the Moon is significantly closer, Mars offers many advantages for long term exploration,
including ice and a thin atmosphere. You can read about the many possible Mars missions at:
http://cmex-www.arc.nasa.gov/MarsNews/Missions/Missions.html
(Mars Missions)
and at:
http://www.nw.net/mars/marsdirect.html
(The Mars Direct Home Page)
Lastly, you ask how soon would it be possible to accomplish this task. We certainly have the
technology to obtain lunar samples, and could probably do so within five years. I would say this is
more a policy and budgetary decision.
I know of at least two private corporations which are planning to send spacecraft to the Moon. One
is even planning to return some surface samples:
http://www.lunacorp.com
(LunaCorp)
http://www.appliedspace.com
(Applied Space Resources, Inc.)
Go back to "Space Missions" questions and answers.
Go back to "Dr. SOHO's FAQ."
SEND US YOUR COMMENTS
Go To
Other SOHO Web Pages
Author: Shane Stezelberger
Co-Author/Curator: Therese A. Kucera
[email protected]
Responsible Official: Art Poland([email protected])
Last modification: Tuesday, 13-Apr-1999 17:33:11 EDT
http://sohowww.nascom.nasa.gov/explore/faq/space_missions.html (9 of 9) [2000/01/11 11:58:03 AM]
Dr. SOHO's FAQ: Comets!
Go back to "Dr SOHO's FAQ"
Go back to the "SOHO" main page.
●
What is a comet's tail made of?
●
Can you tell me more about comets, and where can I find comet pictures?
●
How might a CME affect a comet?
●
How might a solar flare affect a comet?
●
Can a sungrazing comet cause a CME?
●
Do you have a list of sungrazing comets and asteroids?
●
I saw an unusual object near the Sun from an airliner recently. What was it?
To whom this may concern, I've been watching Comet Hale-Bopp. I went to an astronomy program a couple
of weeks ago, and the man running it said that if you condensed the entire 100-thousand-mile long tail of
Hale-Bopp to the density of water, it would not even fill a swimming pool. I didn't get the chance to ask him,
but thought you might be able to answer my question: if what he said is true, what is the tail made of?
●
Comet tails are made up gas and dust. These evaporate off the comet itself, which is made of ice and rock.
The ice is both regular water ice (H20) and other kinds, like carbon-dioxide ice (C02, also known as "dry
ice").
Comets often have two tails: A "plasma tail," which is made up of charged ions, and a "dust tail" which is
made up of dust. The lighter ions are "blown" directly away from the Sun by the solar wind, but the heavier
dust particles form a tail which points at an angle closer to the path of the comet's motion.
There are a number of places on the web where you can find out more about Hale-Bopp and other comets,
including:
http://www.halebopp.com/
(Comet Hale-Bopp)
http://pao.gsfc.nasa.gov/gsfc/comet/hale-bopp.htm
(Goddard's Hale-Bopp Comet Page)
http://encke.jpl.nasa.gov
(The Comet Observation Home Page)
http://sohowww.nascom.nasa.gov/explore/faq/comets.html (1 of 8) [2000/01/11 11:58:07 AM]
Dr. SOHO's FAQ: Comets!
Go back to "Comet" questions.
I am a 10 yr old doing my science project on Comets. There are a lot of really nice pictures on your internet
site. I would like to get your permission to print them out for use with my science project. Could your please tell
me who I need to contact? Thank You very much. I would also like any information you can send me about
comets and why they move in space.
●
First of all, go ahead and use any images you find on SOHO web pages! Such images are already
public-domain.
A comet is basically a small chunk of frozen gases and rock that is thought to be left over from the
formation of the planets. They are sometimes referred to as "dirty snowballs," and this is really a very good
description of what a comet is like!
Billions of years ago, our solar system was basically a great big cloud of gas and dust orbiting around the
Sun. Small particles within this cloud collided with each other, and sometimes joined together to form
bigger objects. Some of these objects grew to become the planets, while others didn't quite make it that far
and became comets. Gravity caused these smaller comets to be expelled from the inner part of the solar
system (near the Sun), and they gradually accumulated in a distant region called the Oort cloud (named
after Jan Oort, the Dutch astronomer who first predicted its existence).
These distant comets are much too small to be visible from Earth (a typical comet is less than 10 miles
across). Sometimes, though, a comet will make its way more or less by chance into the inner solar system,
where it gets "trapped" in a new orbit around the Sun. The heat from the Sun causes some of the frozen
gases in the comet to evaporate and these gases form a "tail" behind the comet as it moves around the Sun.
This is often when a comet becomes visible to observers on the Earth.
The tail can be hundreds of thousands of miles long, so it is much bigger than the original comet!
Sometimes you can see more than one tail -- one tail will be dust particles blowing off of the comet's
surface, and the other will be mostly ionized gases. The main part of the comet is called the "nucleus" (this
is the part that is only about 10 miles across). The bright comets that we sometimes see in the night sky are
those that come close to the Sun, and most of the light that we see from the comet is really reflected
sunlight.
Comets move in the sky because they are in orbit around the Sun, just like the planets. The time for a comet
to complete an orbit (called its "period") can be hundreds or even thousands of years, so that most people
don't see the same comet more than once in their lifetime.
If you want to find some more information, these websites might be useful to you:
http://sungrazer.nascom.nasa.gov/
(Doug B's Home Page. This page is maintained by one of our
SOHO team members, and has a lot of other good pictures
that you could use for your project.)
http://medicine.wustl.edu/~kronkg
(Comets and Meteor Showers)
http://encke.jpl.nasa.gov
(The Comet Observation Home Page. At NASA's Jet Propulsion Laboratory in California)
http://www.seds.org/nineplanets/nineplanets/comets.html
(Comets at The Nine Planets Web Site)
http://sohowww.nascom.nasa.gov/explore/faq/comets.html (2 of 8) [2000/01/11 11:58:07 AM]
Dr. SOHO's FAQ: Comets!
http://spaceart.com/solar/eng/comet.htm
(Comets at Views of the Solar System)
Go back to "Comet" questions.
I haven't found any commentary addressing the possibility of a solar wind from the recent flare (07 April
1997) disrupting the configuration of the gas and/or dust tails of Comet Hale-Bopp. Might there be a shock
wave which could alter the volume or direction of the outpourings from the comet's nucleus?
●
Although I'm not an expert on comets, I'll try to answer your question. If the coronal mass ejection (CME)
was directed towards the comet (I don't believe that this particular CME was), then it could indeed have an
effect on the configuration of the comet tail. The comet halo consists of gases which are emitted by the
comet's surface as it warms up as it approaches the Sun. The gas also carries away with it some dust. This
gas becomes electrically charged as electrons are knocked off the solar ultraviolet radiation, and is then
affected by the magnetic fields carried out by the solar wind.
The shock wave of the magnetic cloud associated with a CME does affect the structure of a comet's tail,
such as Hale Bopp's. One of the possible effects is known as a "disconnection" event, where a blob of
material appears to separate from the rest of the tail and be left behind. This occurs when the magnetic field
in the shock changes rapidly from one orientation to the opposite. One example where this was seen was in
the last passage of Comet Halley a few years ago. The actual brightness of the comet should not be
affected.
(I am unaware of any existing coordinated observations between solar-physics experiments and
comet-watching experiments. However, SOHO did observe Comet Hyakutake during its perihelion passage
in May 1996, though I do not think that that comet flew straight through any CMEs during that time.)
(There were probably no significant optical effects as a result of viewing the comet through the magnetic
cloud or CME... other than the aurorae, of course. CMEs are very tenuous stuff, and it takes a sensitive
telescope to even detect them at all.)
Go back to "Comet" questions.
I heard about the recent massive solar flare (on 07 April 1997). Was anyone seeing if it had any effect on
Comet Hale-Bopp? I would be very interested in hearing if the was any research done on this. Thank you for
your time.
●
Solar flares in themselves would not be expected to have any particular effect on comets. However, in this
case the solar flare was accompanied by a large ejection of mass from the Sun. This is called a "coronal
mass ejection" (CME). It is not uncommon, but the interest in this case was caused by the fact that the
CME was seen to be headed quite directly toward the Earth.
The resulting very large, but very diffuse, cloud of solar plasma (ionized gas) did in fact collide with Earth
on Thursday evening, 10 April 1997. There were numerous effects caused by the interaction of the
interplanetary cloud with the Earth's magnetosphere and upper atmosphere. The most visible of these was
an expansion of the auroral oval to much lower latitudes (New Hampshire, for example) than those where it
is normally seen.
The question, then, is whether this interplanetary cloud had hit the comet. Should such an event occur, it is
http://sohowww.nascom.nasa.gov/explore/faq/comets.html (3 of 8) [2000/01/11 11:58:07 AM]
Dr. SOHO's FAQ: Comets!
quite possible that the comet's tail could be disturbed, causing a wiggling, or in severe cases, a complete
disconnection of the tail. Such things have been observed in the past. In this case, however, the CME
appeared to have headed in the wrong direction (toward Earth and not toward Hale-Bopp), so we didn't
expect to see any "tail wagging" resulting from that particular CME of 7th April 1997.
At no time did LASCO or EIT observe Comet Hale-Bopp. It simply never got close enough to the Sun for
these instruments to detect it.
SWAN, our Lyman-alpha sky survey, DID observe Hale-Bopp, Hyakutake, and Comet Wirtanen. See:
http://www.aero.jussieu.fr/~jgoutail/
(The Solar Wind Anisotropies Instrument)
Go back to "Comet" questions.
Thank you for this service as I do have a question. While running the animation "LASCO C3" I noticed
that the sun-grazing comet from December 1996 appeared to have either impacted the Sun or completely
sublimated. There was a "cloud" that moved away from the Sun in the comet's probable orbit after perihelion,
but it was diffuse and did not appear to have a nucleus. Am I correct in my observations about the comet not
surviving the passage? If you can supply me with any further information, I would appreciate it.
●
You are perfectly correct -- the comet SOHO observed on 23-24 December 1996 was either completely
sublimated, or actually crashed into the Sun. If I understand correctly what you are referring to as a "cloud"
after the comet impact, you are referring to the large, bright activity in the northwest (upper right). That is a
coronal mass ejection (CME) of the sort that we see quite routinely, as the Sun spews out ionized plasma
that becomes the solar wind.
Such emissions of solar plasma are thought to result from a sudden release of stored up energy from a place
in the solar atmosphere that somehow becomes unstable, resulting in sudden (indeed, explosive) conversion
of energy from its stored form (probably magnetic) to kinetic energy (the observed outward rush).
It is not impossible that this particular CME could be related to a destabilization resulting from the impact
of the comet -- which would be described as a sudden introduction of foreign, cold material into a high
localized spot in the solar atmosphere. However, I can say most of us consider that scenario not very likely.
If you consider the probable size of the cometary nucleus (likely not more than a kilometer or so in
diameter, and probably much less) compared to the vastness of the solar sea into which it drops, the
energetics do not seem favorable. The CME, as it emerges from behind the instrumental occulting disk, on
the northwest side, has already expanded to a volume larger than the Sun itself (1.4 million km diameter!)
For serious analysis of a similar event (the first observation of a comet colliding with the Sun), you might
wish to check an article in the journal Science, 26th February 1982 (Volume 215). It should be available in
your library. Good luck!
Go back to "Comet" questions.
●
Is there a record being kept of the impacts of asteroids and comets on the Sun? If so, where can I access it?
The problems of observing impacting asteroids and observing sungrazing comets are actually slightly
different.
SOHO has discovered a number of small comets which have disappeared into the Sun. We might not say
http://sohowww.nascom.nasa.gov/explore/faq/comets.html (4 of 8) [2000/01/11 11:58:07 AM]
Dr. SOHO's FAQ: Comets!
they "impact," as we think they burned up in the Sun's outer atmosphere (the corona), rather than hitting its
surface (the photosphere).
All comets that come close enough to the Sun to burn up in this way are part of the Kreutz family of
comets. You can read more about them at:
http://sungrazer.nascom.nasa.gov/
(Doug Biesecker's Home Page)
This page is set up by Dr. Doug Biesecker. He discusses these sungrazing comets and lists the ones
observed by SOHO so far. (Doug himself has found the lion's share of these, and the rest of us at LASCO
are understandably jealous!)
Comets contain a lot of volatile ices which sublimate (evaporate without melting) to form their bright
comas and tails when they get close to the Sun. Asteroids are rocky and without ice, so they would be much
harder to see. We haven't observed any asteroids disappearing into the Sun. I don't know if others have, but
I expect such observations are rare -- perhaps nonexistent. (This assumes a firm distinction between
"comet" and "asteroid," and many solar system bodies straddle these definitions!)
While we at SOHO do not keep track of comets in general, our LASCO coronagraph is well-suited for the
discovery of these sungrazing comets. All known sungrazing comets (and there were several sighted
throughout history, prior to the space age) follow the same orbit, so astronomers think they were all
"calved" from the same parent comet.
Let me backtrack a bit: All the sungrazing comets discovered before the late 1970s were bright, naked-eye
objects (i.e. Comet Ikeya-Seki in 1965). Some were seen after their closest approach to the Sun, so they did
not hit the Sun on the way around. Since 1979, sungrazing comets have been discovered by satellite
coronagraphs. These comets are too faint and too close to the Sun to be detected by other means. However,
by its very nature, a coronagraph is utterly unable to see the Sun's surface. Coronagraphs use opaque disks
to block out the light of the Sun in order to see the faint solar atmosphere. This means that we can track
sungrazing comets on their way in towards the Sun, but LASCO cannot see them during their closest
approach.
Of the many sungrazing comets LASCO has seen so far, none have been observed after closest approach.
They might have hit the Sun, they might have completely evaporated, or they may have become too dim to
see. (A typical human eye can see down to magnitude 6 from the Earth's surface; LASCO's outermost
coronagraph can see down to at least mag 9.)
LASCO cannot see things smashing into the Sun. Other solar telescopes, whether on Earth or on satellites
like SOHO, cannot detect these small, faint objects so near to the Sun. As far as I know, nobody has any
images of such collisions.
What we can do, however, is plot the orbits of these comets. In theory, an orbit can be plotted from only
three images of the comet -- but the more, the better. Unfortunately (and here I am speculating a little), the
orbits that get plotted from these images have some degree of uncertainty. We're not quite sure whether or
not the comet strikes the Sun by looking at the orbit alone.
As for impending sungrazers: we don't know. Nobody knows what the population of the Kreutz orbit is.
And other, non-sungrazing comets do wander into our field of view sometimes. When this happens, we
quickly refer to:
http://encke.jpl.nasa.gov
(Comet Observation Home Page)
http://sohowww.nascom.nasa.gov/explore/faq/comets.html (5 of 8) [2000/01/11 11:58:07 AM]
Dr. SOHO's FAQ: Comets!
This is JPL's Comet Observation Home Page. It is highly useful for professionals and novices alike. We've
often compared comet coordinates here to the current solar coordinates to decide if any known comets
might be visible to LASCO.
More about sungrazing comets:
http://sungrazer.nascom.nasa.gov/ (as above)
(Doug Biesecker's Home Page)
http://galaxy.cau.edu/tsmith/sungrazer.html
(Sungrazer Comets, by Tony Smith)
Go back to "Comet" questions.
Today May 14th [1998], I was flying in a plane at more or less 2000 ft above sea, it was 20:40 GMT, the
plane was approaching the Maiquetia Airport in Venezuela, that is north from Caracas. The local time was
6:40 PM. It was a very clear day with a little haze and very few clouds, and was a beautiful sunset. The Sun
was about to touch the water, then I realize like a light stripe near the Sun and I figured out that it was an
annoying reflection in the plane's window. I moved my head looking for a different angle of view to avoid the
reflection, but I couldn't get rid of it. From my point of view the stripe was completely steady, vertical, and had
an intensity equal or greater than the Sun at that moment, the stripe was like 1/3 of the Sun's size and it was
right (north) from the Sun at a distance of the size of the Sun. It was also above the water at a distance about
twice the size of the sun, the width of the stripe was thin, like 1/10 of an inch. I looked on it to see if it was the
the sun reflecting on another plane, but because the size, intensity and steadiness was obvious that it couldn't
be a plane, and also because it was in a vertical position.
●
The plane begun to turn slowly for the final approach, and suddenly a little cloud came in front of the stripe
and it covered it, which showed me that it was not a reflection of the window, and as the plane changed
position, the stripe kept the same position relative to the Sun. Some passengers noticed the same stripe, and
everybody was equally puzzled by it. They were in different seats with different angles and windows.
I thought that it could be the launching of a rocket because that was more or less how it looked like. But I
check NASA´s site and there were no launches today, and also the position was not pointing to the US, it was
pointing to southern Mexico or Central America. Please help me out, I would like to know what I saw. I am 30
years old, I travel very frequently and I never have seen something like that. Thank you very much in advance.
I am not sure what you saw from your aircraft on 14 May. Could it have been a distant jet contrail?
One possibility my coworkers and I suggest is: Comet C/1998 J1, or "SOHO-49." In May 1998, SOHO
discovered a fairly bright comet orbiting near the Sun. Calculations indicated that, as the comet drifted
away from the Sun, it should become visible to Earthly observers.
Since then, many astronomers around the world have tried to find this comet. Many have succeeded, and
for many of them the comet was visible to the naked eye. See the Comet Observation Home Page for more
details:
http://encke.jpl.nasa.gov
(Comet Observation Home Page)
Also, see the SOHO/LASCO sungrazing comet page (although this comet was not a Kreutz sungrazer):
http://sungrazer.nascom.nasa.gov/
http://sohowww.nascom.nasa.gov/explore/faq/comets.html (6 of 8) [2000/01/11 11:58:07 AM]
Dr. SOHO's FAQ: Comets!
(Dr. Doug Biesecker's Science Pages)
We will forward your message to astronomers who might be able to verify if your object actually was this
comet. I have also included a recent report of observations of Comet 1998 J1. This report is full of technical
language, but one key parameter is "magnitude" or "m1." This is the brightness of the comet. Bright stars
are magnitude 0 and very faint stars are magnitude 6. Stars too faint to see without telescopes are given
higher numbers than 6.
I hope this helps. Please ask us if you have further questions, and we will send you new details as they
arrive. Remember, if it was the comet, the object would have appeared to move with the Sun toward the
horizon.
----- Begin Included Message ----Subj: IAUC 6906: C 1998 J1
Issued by CBAT: Sat, 16 May 1998 15:13:20 -0400
Circular No. 6906
Central Bureau for Astronomical Telegrams
INTERNATIONAL ASTRONOMICAL UNION
Mailstop 18, Smithsonian Astrophysical Observatory, Cambridge, MA 02138, U.S.A.
[email protected] or FAX 617-495-7231 (subscriptions)
URL http://cfa-www.harvard.edu/iau/cbat.html
COMET C/1998 J1 (SOHO)
More than 100 positions were reported (cf. IAUC 6894, MPEC 1998-J13, -J14 and -J21) of this comet as it
moved across the top of the field of the SOHO-LASCO C3 coronagraph between May 3.78 and 8.87 UT.
Attempts to record the comet from the ground were often inconclusive, mainly because of the difficulty of
predicting the comet's position from the rough SOHO data. Visual observations that appear to be positive
are: May 11.23 UT, m1 approx. 0.5, coma diameter D = 0'.5 (N. Biver, Kahe Point, Oahu, HI, 0.26-m
reflector); 11.23, m1 approx. 0.5, D = 1'.0 (H. Dahle, same location, 20 × 80 binoculars); 14.35, m1 approx.
2-3, tail about 15' long toward the east (P. Nation, Christies Beach, South Australia, 11 × 80 binoculars);
16.24, m1 = 3.5, D = 1' and rather condensed with faint, short, fan-shaped tail about 5' long in p.a. 110 deg
(Biver); 16.34, m1 approx. 2.5-3, tail > 0.5 deg long in p.a. 90-130 deg (C. E. Drescher, Warrill View,
Queensland, Australia, 7 × 50 binoculars). In addition, the comet was apparently imaged (focal length 500
mm, ST-6) by O. Farago (Stuttgart, Germany) on May 10.8 UT, when the close proximity to the Pleiades
allowed J. Jahn (Bodenteich, Germany) to perform astrometry (internal consistency about 2") from four
stars bright enough for detection in strong twilight at 4.5 deg altitude in the 1-deg field! Positions and a
compromise parabolic orbit and ephemeris follow:
1998 UT
May 10.80208
16.249
16.34
T = 1998 May
R.A. (2000) Decl.
3 45 03.6
+23 43 15
4 47.7
+12 53
4 48
+12 50
8.352 TT
q = 0.15015 AU
1998 TT
R. A. (2000) Decl.
Observer
Farago
Biver
Drescher
Peri. = 107.649
Node = 355.021
Incl. = 68.346
Delta
http://sohowww.nascom.nasa.gov/explore/faq/comets.html (7 of 8) [2000/01/11 11:58:07 AM]
r
2000.0
Elong. Phase
m1
Dr. SOHO's FAQ: Comets!
May
13
15
17
19
21
23
25
27
4
4
4
5
5
5
5
6
13.70
35.43
54.17
10.73
25.62
39.18
51.67
03.27
1998 May 16
+19
+15
+11
+ 7
+ 2
- 1
- 4
- 7
54.2
41.6
18.8
00.0
52.4
00.0
35.6
54.1
0.894
0.865
0.851
0.848
0.853
0.865
0.883
0.905
0.246
0.309
0.371
0.432
0.490
0.547
0.603
0.656
(C) Copyright 1998 CBAT
(6906)
----- End Included Message ----Go back to "Comet" questions.
Go back to "Dr. SOHO's FAQ."
SEND US YOUR COMMENTS
Go To
Other SOHO Web Pages
Author: Shane Stezelberger
Co-Author/Curator: Therese A. Kucera
[email protected]
Responsible Official: Art Poland([email protected])
Last modification: Tuesday, 13-Apr-1999 17:33:09 EDT
http://sohowww.nascom.nasa.gov/explore/faq/comets.html (8 of 8) [2000/01/11 11:58:07 AM]
13.1
16.7
20.8
24.9
28.9
32.7
36.3
39.5
111.6
109.5
104.8
99.3
93.8
88.6
83.7
79.2
Brian G. Marsden
1.7
2.6
3.3
4.0
4.6
5.1
5.5
6.0
Dr. SOHO's FAQ: General Astronomy Questions
Go back to "Dr SOHO's FAQ"
Go back to the "SOHO" main page.
●
What is the location of the Sun in the Milky Way?
●
Where is the center of the Universe?
●
What is the difference between a star and a planet?
●
How many stars are in the Universe?
●
How many other solar systems have been discovered?
●
At what distances could other stars support life?
Star Life cycles
❍ Where can I find information about stellar death?
❍
Can you please explain the life cycle of a star?
❍
Where can I find sources about stellar evolution?
❍
How much energy is left in our Sun?
❍
What exactly is a supernova?
❍
What exactly is a Type 2 supernova?
❍
What were the locations of some recent supernovae?
Inside the Solar System
❍ Did Venus undergo a greenhouse effect? When?
❍
How bright is daylight on Mars?
❍
What can you tell me about Jupiter's moon Europa, and the possibilities of life there?
❍
What is the order of the planets?
❍
In which direction do the planets revolve?
❍
How large is our solar system?
❍
Has a 10th planet been discovered?
❍
How did other planets get their names, and why doesn't the Earth have one?
http://sohowww.nascom.nasa.gov/explore/faq/astronomy.html (1 of 36) [2000/01/11 11:58:28 AM]
Dr. SOHO's FAQ: General Astronomy Questions
❍
How did you name the planets?
❍
When was Mars discovered?
❍
Is the Earth's perspective always equatorial?
❍
Which planet is the oldest?
❍
Which planets have magnetic fields? Which have atmospheres?
❍
How many Jupiters could fit inside the Sun?
❍
Could the Sun be made of dark matter?
❍
Can a planet's gravity affect other planets?
❍
How would things be different if, for example, Jupiter didn't exist?
❍
How is it that some asteroids are nickel-iron while others are rocky?
❍
Is Jupiter a "brown dwarf?"
❍
How long does it take for the Earth to revolve around the Sun?
❍
How fast does the Moon spin? Does it wobble? What about a lunar base?
❍
Where can I find pictures of Jupiter's moons?
❍
Could the Martian meteorite really be from one of Mars' moons?
❍
Could Mars' moons have been created by an exploding asteroid?
❍
I recently heard about the asteroid Icarus, which sometimes passes near the Earth. Can it hit the
Earth?
❍
What happened to that meteor that hit Greenland?
❍
Where can I find information about meteor showers?
❍
The local news recently said something about a meteor shower. Can you please elaborate?
Distances
❍ How many light years away is the Sun?
❍
How many Earth years are in a light year?
❍
How long does it take sunlight to reach the different planets of the solar system?
❍
Since light from other stars takes a long time to get to Earth, could we ever see our solar
system as it was in the past?
●
Is there a connection between Jupiter's rotation and the solar cycle?
●
Does Jupiter have any influence on the solar cycle? Both have periods just over 11 years.
●
Is there a relation between planetary alignments and the solar cycle?
●
What does "exposure" mean in photography?
●
Why do Lagrange points exist? Where are they?
●
Will there be a big planetary alignment at the end of this century [20th]?
●
How can I calculate the times of local sunrise and sunset?
http://sohowww.nascom.nasa.gov/explore/faq/astronomy.html (2 of 36) [2000/01/11 11:58:28 AM]
Dr. SOHO's FAQ: General Astronomy Questions
●
●
How do we calculate local noon for our location?
What is the location of the Sun in the Milky Way?
The Milky Way galaxy is a 'spiral galaxy', meaning that it is a fairly flat disk shape, with bright spiral
arms coming out of the center (much like the shape made by the water coming out of a garden
sprinkler). The Sun is located in one of these spiral arms, about two-thirds of the way out from the
center of the galaxy.
Go back to "Astronomy" questions and answers.
●
If the Big Bang really did take place, where is the center of the Universe?
According to the Big Bang theory, the Universe did not even exist before the explosion occurred.
And space itself did not exist until the Big Bang made it and carried it along outwards into the
expanding Universe. So, in a sense, the center of the Universe, or the "place" where the Big Bang
happened, is spread out throughout the Universe.
This is a fairly hard concept for us three-dimensional beings to really grasp. It is outside our everyday
experience and common sense. But an analogy might help: Imagine that there is a two-dimensional
creature confined to the surface of a slowly expanding balloon, which is its universe. (This is a very
special balloon -- its skin can stretch a very long way without breaking and it doesn't need a blow
hole.)
Now the 2-D creature cannot sense the direction up from the surface of the balloon or down into its
interior since these are outside its universe, and it can only move along the surface of the balloon. If
the creature travels extensively, it will find that it can go all the way around the balloon back to
where it started without finding an edge (or a center) of its universe. (Well, it can go back to its
starting place if it moves faster than its balloon universe expands.)
From your 3-D perspective, you might argue that the "center" of the balloon is a point within the
volume of air inside the balloon. But such an explanation would make no sense to the 2-D creature -that is not part of its universe, the direction to it is incomprehensible, and it isn't the kind of center
that the 2-D creature is looking for. (Besides, that kind of solution is not in the spirit of our analogy,
which requires us to concentrate on the surface of the balloon and pretend that nothing else exists.
Unlike our imaginary balloon, the analogy can't be stretched too far without breaking!) But if the
creature has studied mathematics or physics, it might come to understand that the original point from
which the balloon grew has become the whole balloon surface.
I hope this helps. (Most of us are neophytes when it comes to intuitive understanding of general
relativity.)
Go back to "Astronomy" questions.
●
Could you tell me please: what's the difference between a star and a planet?
Observationally: Stars remain fixed on the night sky, while planets wander around. The ancients
noticed that all the stars seem to rotate together around the Earth; they came up with the idea of a
"celestial sphere" to which all the stars were thought to be attached. The planets were (are) exceptions
to that rule: they move around through the constellations over the course of the year. The word
http://sohowww.nascom.nasa.gov/explore/faq/astronomy.html (3 of 36) [2000/01/11 11:58:28 AM]
Dr. SOHO's FAQ: General Astronomy Questions
"planet" means "wanderer" in Greek.
Physically: Planets are cool objects, sometimes with a hard crust, that orbit stars. Our Earth is a
planet. Mars is a planet with a hard crust, like Earth is. Jupiter and Saturn are planets that have no
hard surface - they are called "gas giants" because their atmospheres continue far down into the
depths of each planet. (Earth's atmosphere is only about 100 km thick, which is about 1.5% of the
Earth's radius). Most planets in our own solar system are "only" a few hundred million kilometers
from the Earth. Earth is a smallish but fairly typically-sized planet.
Stars are much much bigger than planets. The Sun, the closest star to us, is about a million times
larger than the Earth. Stars are so big that deep inside them, high pressures and temperatures cause
nuclear fusion to occur. (Here on Earth, nuclear fusion only normally occurs inside hydrogen
bombs!) The nuclear fusion heats stars, which then get so hot that they shine with their own light.
Most stars (the Sun is the only exception) are more than a thousand times farther away than the
planets are.
The Sun's other planets appear to wander around the night sky because we're all orbiting the sun at
different rates, and so they appear to move much as landmarks in a landscape appear to change
position when you walk or drive through the landscape. The other stars don't appear to move because
they're so far away that our orbital motion is insignificant. (Think of walking a couple of kilometers
while watching a distant mountain range: the distant mountains don't seem to change much.)
The Sun's planets appear bright in the night sky because they're lit up with sunlight, just like the Earth
is. Each planet has phases, like the Moon, because only the part of the planet that points toward the
Sun is lit up. (Normally, we can only see such phases on, I believe, Venus. It and Mercury are the
only worlds between us and the Sun, and Mercury is often hidden in the Sun's glare)
Other stars may have planets too. Some "gas giant" planets (like Saturn and Jupiter) have been
detected around other stars, but we don't yet know whether there are any other planets like Earth.
There are a bunch of interesting references about planets on the web. Try:
http://www.yahoo.com/Science/Astronomy/Solar_System/Planets/
(Yahoo! Planets)
http://www.seds.org/nineplanets/tnp/
(The Nine Planets, by Bill Arnett)
Go back to "Astronomy" questions.
●
How many stars are in the Universe?
Lots and lots, but we are not sure exactly how many. There are tens of billions of stars or more in a
galaxy and tens of billions of galaxies, so that's a few hundred thousand trillions (or a hundred billion
billions, in any case it would be written out as 100,000,000,000,000,000,000) at least.
Want to know how we calculate the number of galaxies? Check out the Hubble Deep Field:
http://oposite.stsci.edu/pubinfo/education/amazing-space/hdf/wholehdf.htm
(Estimating the Number of Objects in the Hubble Deep Field)
http://sohowww.nascom.nasa.gov/explore/faq/astronomy.html (4 of 36) [2000/01/11 11:58:28 AM]
Dr. SOHO's FAQ: General Astronomy Questions
Go back to "Astronomy" questions.
●
How many solar systems have been counted to date?
I don't know the current answer. There are approximately a dozen extrasolar planets that are
considered confirmed, but the count increases every day, as does the criticism of their validity.
Fortunately, a colleague pointed out a web site with some very recent findings in this field:
http://www.obspm.fr/encycl/encycl.html
(The Extrasolar Planet Encyclopedia: in English and French)
You might also be interested in:
http://starchild.gsfc.nasa.gov/docs/StarChild/questions/question10.html
(StarChild: Other Solar Systems)
http://www.ph.adfa.edu.au/e-mamajek/exo.html
(Exoplanets)
For more detailed information about our own solar system, try this site:
http://www.seds.org/nineplanets/tnp/
(The Nine Planets)
Go back to "Astronomy" questions.
The Sun, old Sol, is classified as a Class G2 yellow dwarf star, if I am not mistaken. What other stars,
within 10 parsecs, are also in the Sun's classification? What spectral types of stars, and what size
parameters, are capable necessary for protoplasmic life to evolve? For instance, could a Class K8 star
support Earth-like life, or a F1 at say Mars distance? I know this is speculative, but I am curious. Thank
you.
●
Hi!
I am forwarding you the following from a friend who taught a class on this subject. As you and he
say, this is all pretty speculative. We don't know for sure what is needed for life to form, we just
know about one case, the Earth. You can find a copy of a table of stars within 5 pc in the book
Exploration of the Universe by Abell, Morrison, & Wolff. I am sure you can find the information
other places as well, but I haven't yet turned up a 10 pc list.
------- Begin Forwarded Message ------Date: Thur, 6 Mar 1997
Subject: Re: stars classifications
Below is a table with the spectral classifications of stars and some rough properties for the middle
level of those stars (eg for B stars, the properties listed are for B5 stars).
Energy
Spec. Surface
Mass Output Lifetime No. in
http://sohowww.nascom.nasa.gov/explore/faq/astronomy.html (5 of 36) [2000/01/11 11:58:28 AM]
Dr. SOHO's FAQ: General Astronomy Questions
Class
O
B
A
F
G
K
M
Temp(K)
40,000
15,500
8,500
6,600
5,500
4,100
2,800
xSun
40
6.5
2.1
1.3
0.9
0.7
0.2
xSun
500,000
800
20
2.5
0.8
0.2
0.008
(years)
5e5
8e7
1e9
5e9
1e10
4e10
3e11
Galaxy
5.5e4
3.6e8
2.4e9
1.2e10
2.8e10
6.0e10
2.9e11
Life on Earth took about a billion years to get the chemistry organized to form the first crude cells
without nuclei, and another billion and a half years to form cells with nuclei and DNA. The lifetime
argument pretty much eliminates O and B stars from consideration, as they live too short a time for
basic life to start. A-type stars should also probably be eliminated because life would just get started
by the time the star dies. We will also eliminate M stars because they are too dim, and give off very
little ultraviolet light, which is necessary to spur the chemical reactions that cause and sustain life.
That leaves us with F, G, and K stars, of which there are 100 billion in the Milky Way Galaxy. There
are nine F, G, or K stars within 5 parsecs of the Sun. (Sorry, I don't have the numbers for 10 parsecs.)
One star (Alpha Centauri) is a G2 star, but it is part of a multiple system, which complicates matters
with such concerns as planet formation, stable planetary orbits, and illumination from the other stars.
In fact, multiple star systems are very common in the universe.
We think that the "zone of habitability" for our G2 Sun is from 1 to 2 AU (Earth and Mars). If we
scale this "zone" by energy output for other stars, we get 1.3 to 2.6 AU for F5 stars, 0.9 to 1.8 AU for
G5 stars, and 0.7 to 1.4 AU for K5 stars. The odds of F, G, or K stars having a planet within their
respective "zones" is a complete unknown at this point.
All of this is based on a lot of assumptions, mostly that life elsewhere will start in a similar manner as
we think it did on Earth. I hope this answer helps.
------- End Forwarded Message ------Regarding the entire subject of life in the Universe, the following web site is a wonderful place to
start:
http://www.astrobiology.com
(The Astrobiology Web)
Go back to "Astronomy" questions.
Sorry to bother you, but I'm a University student who is taking an astrology [sic] course. I'm working
on a small research paper about what happens to a star when it dies. Would it be possible for someone
there to send me in the right direction for a website that could offer information on this subject. I would
sincerely appreciate it.
●
Although it is also advisable to consult an astronomy textbook on such matters, you can easily find
such information over the net. Some possible sites are listed below for your convenience. More sites
http://sohowww.nascom.nasa.gov/explore/faq/astronomy.html (6 of 36) [2000/01/11 11:58:28 AM]
Dr. SOHO's FAQ: General Astronomy Questions
can be found by following various links at these sites:
http://zebu.uoregon.edu/text.html
(Astronomy HyperText Textbook)
http://imagine.gsfc.nasa.gov/
(Imagine the Universe! -- former High-Energy Astrophysics Learning Center)
http://heasarc.gsfc.nasa.gov/docs/www_info/webstars.html
(WebStars -- was HEASARC's Astronomy Resources)
Go back to "Astronomy" questions.
Could you explain the life cycle of a star in detail? I am in 9th grade and need a interview for my
project, and must have a statement that says I was interviewed on (the date). If you could do this, that
would be great. Thanks for your time.
●
I cannot explain the "big picture" about a star's life cycle very well, I'm afraid. This is an extremely
broad topic. Here is one web site that gives an overview:
http://starchild.gsfc.nasa.gov/docs/StarChild/universe_level2/stars.html
(Stars)
As for your interview, I will be glad to answer more specific questions for your project. As you
continue your research, I am sure you will have many questions which the library and the web do not
fully answer. That is where we at Dr. SOHO can help you.
In the meantime, please check your school and local libraries for astronomy books. Some of my
colleagues recommend Isaac Asimov's nonfiction for students in your situation. I strongly
recommend Carl Sagan's COSMOS, both the book and the television series. Get a large, illustrated
copy of the book and check out some of the middle chapters (in particular, I think, "The Lives of the
Stars"). Dr. Sagan's writing is very eloquent and well worth your time.
As I said, this topic is broad, with many interesting parts. You've probably already heard how a
spoonful of white-dwarf material would weigh thousands of tons. Here's another factoid about stellar
life cycles. Every element on the Periodic Table higher than Iron (Fe) was formed inside a supernova.
Every atom of gold, platinum, and uranium on Earth was once in the core of an exploding star.
(Essentially every atom of the Earth itself, and those of everyone living on it, was once part of a star,
too. Carl Sagan was fond of reminding us that "we are starstuff.")
Get to work on those sources, and write back to me with the interview questions which will certainly
develop in the process. Good luck!
Go back to "Astronomy" questions.
I'm a junior in high school, and I have a research project that requires a detailed report of the birth of
the Sun -- somewhat beginning with the Big Bang, but mainly focusing on the actual birth of our Sun to
its eventual death.... I've been to the library, but everything in the books is so scattered that a story line of
our Sun is very vague and ambiguous. I would really appreciate an address for a site that would have
●
http://sohowww.nascom.nasa.gov/explore/faq/astronomy.html (7 of 36) [2000/01/11 11:58:28 AM]
Dr. SOHO's FAQ: General Astronomy Questions
this specific information, if such a site exists. Sorry that this isn't just a question. Thank you.
I'm not sure if this is what you're looking for, but these are a few sites that I found when I typed in the
words "solar formation" into the Infoseek search engine (www.infoseek.com):
http://www.csulb.edu/~gordon/solarneb.html
(Formation of the Solar System)
http://www.arval.org.ve/section3_3.htm
(The Origin of the Solar System)
http://www.physics.gmu.edu/classinfo/astr103/CourseNotes/sls_form.htm
(ASTR 103: Solar System - Formation)
Some other keywords to search on might be "stellar formation" and "stellar evolution".
I wouldn't give up on your local library yet. One book you might try is The Physical Universe: An
Introduction to Astronomy by Frank H. Shu (1982). Any introductory astronomy textbook should
contain similar material. Marc Kutner's Astronomy: A Physical Perspective (1987) is another
accessible book. Although it is written at the college level with much calculus and chemistry, it (and
any similar textbook) may help answer some of your detailed questions.
Go back to "Astronomy" questions.
●
How much energy is left in the Sun, and when is it expected to burn out?
Thanks for asking Dr. SOHO! As you may know, stars are vast clumps of hydrogen gas. Stars shine
by fusing hydrogen into helium in their hot, dense cores. When a star of the Sun's size runs out of
usable hydrogen in its core, it will "puff out" its outer layers. This will create a bubble-like planetary
nebula. The Sun's interior will collapse into a tiny white dwarf star. This white dwarf will be very
dense -- a 0.1 inch cube of white dwarf material would weigh half a ton on Earth. White dwarfs
(dwarves?) glow with leftover heat, but they generate no new energy by themselves.
Stars much larger than the Sun can explode as a supernova. The remnants of a supernova collapse
beyond the white dwarf stage and into a neutron star, or even a black hole.
Now, how much time does the Sun have left? Astronomers have never actually watched a star go
through all the phases of its life cycle -- but they have made models and simulations of stars. They
can also observe many stars in various stages of their evolution. Based on stellar models, we know
that a star like the Sun (that is, with the Sun's brightness and mass) should shine for about nine or ten
billion years.
Astronomers think the Sun is about halfway through those nine billion years. We know this through
several sources: those solar models, the hydrogen- to-helium ratio of the Sun, and the geologic age of
the Earth and of Moon rocks. (We think the Earth and the other planets formed at about the same
time the Sun began to shine.)
So the Sun should burn for another 4 to 5 billion years. I am not sure how much energy it will
consume during that time. We could make a rough guess of this energy by estimating how much
mass is in the Sun's core. Solar fusion turns mass into energy via Einstein's famous E=mc2 equation.
http://sohowww.nascom.nasa.gov/explore/faq/astronomy.html (8 of 36) [2000/01/11 11:58:28 AM]
Dr. SOHO's FAQ: General Astronomy Questions
(Now, there are some complications: the Sun will not fuse all of its core hydrogen before it stops.
And it is fairly inefficient in the meantime, too!)
Go back to "Astronomy" questions.
●
What exactly is a supernova?
A supernova is what happens when a very massive star runs out of the fuel in its core. Stars run on
nuclear fusion, which converts lighter elements to heavier ones + energy. Eventually the core gets
full of iron, which soaks up energy rather than releasing it. The energy is released in a powerful
explosion called a supernova. You can read about this process at:
http://heasarc.gsfc.nasa.gov/docs/StarChild/universe_level1/stars.html
(StarChild: Stars)
http://heasarc.gsfc.nasa.gov/docs/StarChild/universe_level2/stars.html
(StarChild: Stars)
http://zebu.uoregon.edu/text.html
(Astronomy HyperText Textbook)
Go back to "Astronomy" questions.
I was wondering if you had specific information on Supernovae Type 2. I've read a lot of articles on
them, but some of them assume you understand everything. I don't know about everyone else but I don't.
We are doing research for astronomy class, and I need information that I can put into my report, and am
able to understand so I can present. Thanks.
●
Rest assured, our Sun will never become a supernova. It's just not big enough. Only the very massive
stars end up with this fate. It won't become a nova either - these are believed to only occur in binary
star systems.
The final fate of the Sun will be to end up as a white dwarf, with only a small, hot, collapsed core
remaining. The outer layers of the Sun will be ejected as a ring known as a planetary nebula. The
Earth may or may not be swallowed by the Sun during its "red giant" phase. This process is outlined
elsewhere in this FAQ.
A Type II supernova, on the other hand, is a much more catastrophic event. This occurs to a star with
many times more mass than the sun. In this kind of star, the core goes through a number of phases of
nuclear burning: hydrogen into helium, helium into carbon, etc., until finally the core is converted
into iron. At this point, no more energy can be generated in the core. Any nuclear reaction involving
iron doesn't release energy - it absorbs it. The iron core starts to collapse. The collapse causes it to
heat up. Nuclear reactions of iron start to occur which drain energy away from the core. This causes it
to shrink even further, resulting in a vicious cycle which leads to a complete collapse of the core. It's
estimated that once the core starts to shrink, the whole process takes only an hour. This core collapse
sends a shock wave through the rest of the star, igniting the outer layers of the star and creating a
sudden massive explosion. When the explosion clears away, all that's left is a core which has
collapsed to either a neutron star with the incredible density of an atomic nucleus, or a black hole
where light itself cannot escape.
http://sohowww.nascom.nasa.gov/explore/faq/astronomy.html (9 of 36) [2000/01/11 11:58:28 AM]
Dr. SOHO's FAQ: General Astronomy Questions
For more information, I suggest reading up on supernovae in your local library. There are a number
of astronomy texts that should be able to help you. You might also try the following links:
http://www.yahoo.com/Science/Astronomy/Astrophysics/Stellar_Phenomena/Novae_and_
Supernovae/
(Yahoo! Supernova Pages)
http://www.ast.cam.ac.uk/RGO/leaflets/supernovae/supernovae.html
(Royal Greenwich Observatory Information Leaflet No. 63: What is a Supernova?)
http://cssa.stanford.edu/~marcos/sne.html
(Supernova and Supernova Remnant Pages on the WWW)
http://www.chapman.edu/oca/benet/mrgalaxy.htm
(Mr. Galaxy's Supernovae)
Go back to "Astronomy" questions.
Could you tell me the location of around five past or present supernovae Type II explosions? What I
mean is, could you tell around which constellation they could be found in, and what their location in
celestial sphere is?
●
Here is a site which lists some recent supernovae:
http://www.ggw.org/freenet/a/asras/supernova.html
(Supernovae in NGC and IC galaxies)
You have to look at the entries carefully to distinguish the Type I supernovae from the Type IIs. Here are a
few examples of Type II supernovae:
Designation
1998bm
1998ar
1998Y
1998X
1998W
1998S
Galaxy
IC 2458
NGC 2916
NGC 2415
NGC 6754
NGC 3075
NGC 3877
Position
R.A. = 9h21m30s.49, Decl. = +64o14'25".4
R.A. = 09h34m58s.86, Decl = +21o42'57".5
R.A. = 7h36m56s.93, Decl. = +35o14'36".0
R.A. = 19h11m.4, Decl. = -50o39'
R.A. = 09h58m54s.83, Decl = +14o25'25".7
R.A. = 11h46m06s.18, Decl = +47o28'55".5
Right ascension is given in hours, minutes, seconds, and the declination is given in degrees, arc minutes,
and arc seconds. I'm told that 1998S is the most exciting of the recent supernovae.
Go back to "Astronomy" questions.
Please tell me if you think there has once been a "greenhouse effect" at Venus, which has run out of
control -- and if so, when did it happen?
●
Yes. Venus' surface temperature is about 482 degrees C. (Earth's is more like 27 deg. C, of course)
http://sohowww.nascom.nasa.gov/explore/faq/astronomy.html (10 of 36) [2000/01/11 11:58:28 AM]
Dr. SOHO's FAQ: General Astronomy Questions
According to current theory, a strong greenhouse effect maintains this high temperature.
A greenhouse effect occurs in a planet's atmosphere when there are lots of "greenhouse gases"
present. Gases like carbon dioxide and water vapor can prevent absorbed sunlight from reflecting
back out into space. They act like the windows of a greenhouse; light gets in, but cannot get out.
(Technically, much of the light does get re-radiated at longer wavelengths, after heating up the
surface.)
Earth's atmosphere is chiefly nitrogen and oxygen. Venus is almost all carbon dioxide at much higher
pressures than Earth. Venus is also closer to the Sun, so it gets about twice as much sunlight. These
two factors make Venus' greenhouse effect literally strong enough to melt lead.
(The controversy here on Earth concerns a possible artificial greenhouse effect. Some scientists think
fossil fuel use puts enough carbon dioxide into the atmosphere to raise Earth's surface temperature.
Other scientists aren't so sure. NASA is conducting a long research program called the Earth Science
Enterprise to get hard data on possible greenhouse changes here on Earth. Stay tuned!)
I can find less data about the second part of your question: when did this greenhouse effect start? We
had no space probes billions of years ago to watch the planets form, so we cannot know directly. One
book (Cattermole's Venus: The Geological Story) implies that the inner planets began acquiring their
present atmospheres about four billion years ago.
In addition to Peter Cattermole's book, I would recommend to you David Grinspoon's witty,
informative Venus Revealed (1997). Parts of this book are available on-line at:
http://sunra.colorado.edu/david/book.html
(Venus Revealed)
Additionally, try "Venus" under the Nine Planets link at SEDS:
http://www.seds.org/nineplanets/tnp/
(The Nine Planets: A Multimedia Tour of the Solar System)
Go back to "Astronomy" questions.
●
If I were standing on Mars, how bright would the day be?
The short answer is: Martian daylight would be dimmer than a typical Earth day, but you'd never
notice the difference!
The technical answer: Mars is roughly 1.5 times farther from the Sun than the Earth. (Orbits are not
circular, so this distance varies throughout the Martian year.) Anyway, sunlight - like most radiation fades with distance according to the inverse-square law, or 1/R2 where R is the distance.
Doing the math, we find that sunlight at Mars' distance should be about 57% dimmer than at Earth's
distance. This is roughly a factor-of-two difference.
However, human eyes are very good at handling changes in brightness. Our eyes can adapt to
factor-of-ten changes. Even stepping from a dark building into a bright, sunny yard, one's eyes adjust
almost completely in a few seconds. There was a discussion on the space newsgroups about this very
http://sohowww.nascom.nasa.gov/explore/faq/astronomy.html (11 of 36) [2000/01/11 11:58:29 AM]
Dr. SOHO's FAQ: General Astronomy Questions
topic some weeks ago; the consensus was that Mars daylight would appear "normal" to humans.
Remember the pictures from the Vikings and Mars Pathfinder? These images were properly exposed
for the Martian light level, and they look normal to us. One more thing: Mars' day is only about one
half-hour longer than Earth's; the two planets rotate at almost exactly the same speed.
Here's my favorite Mars Pathfinder picture, although it was taken very late in the day:
http://mars.jpl.nasa.gov/ops/sunset_rt.gif
(GIF image 226x222 pixels)
Interesting question! I hope this helps. Please contact Dr. SOHO again with any further solar
questions.
(Mars orbital parameters from "TRW Space Data," 4th Edition, 1992, TRW, Inc. Space and
Technology Group Communications.)
Go back to "Astronomy" questions.
I am currently in Year 10, in Australia. I would really like to know more about Jupiter's moon Europa
and the possibilities of it having primitive life. Thank-you for your time, as it is greatly appreciated.
●
It is exciting to think that three prerequisites for life are present on Europa. It has vast quantities of
water (possibly even some liquid water); energy inputs; and minerals in solution. Whether organic
life could emerge from this slurry is not known. (The liquid water - or lack of it - may be decisive.
Dr. Chris McKay, a NASA scientist who has spent years finding life forms in extreme Earth
environments, recently said this: at least on Earth, the search for life "...is the search for liquid
water.")
Based on what we know of life forms here on Earth, you do not need sunlight at all. All you need is
some source of energy and heat to drive the chemistry of life. On Europa, it is believed that the core
of the satellite is under lots of gravitational 'tidal' stress from Jupiter being so close. This causes the
interior of Europa to be rather warm. Also, any trapped radioisotopes would warm the interior a bit.
The best models of Europa show it to have a rocky core with a very thick layer of water in the form
of a ocean and a thick ice crust. NASA would have to send a spacecraft with a rocket 'penetrator' to
burn its way through 2-5 kilometers of solid ice before breaking into the subsurface ocean. It could
then do a chemistry study to see if it finds life signs, bacteria and algae... or whatever. Such a probe
may be launched early next century.
For more information about the possibilities for life elsewhere in the Universe, see the Astrobiology
Web:
http://www.astrobiology.com
(The Astrobiology Web)
In particular, see their excellent Europa resources:
http://www2.astrobiology.com/astro/europa/index.html
(Europa Revealed)
http://sohowww.nascom.nasa.gov/explore/faq/astronomy.html (12 of 36) [2000/01/11 11:58:29 AM]
Dr. SOHO's FAQ: General Astronomy Questions
Go back to "Astronomy" questions.
My question is fairly straightforward and simple... yet, it has been a while for me, so I am asking you
this: What is the order of the planets starting with the Sun outwards? The reason I pose this question is
that I have acquired an "orrery" clock, which shows the planetary movement of the first seven, along
with a ring listing the sign of the Zodiac, and I would like to align them (the planets on the clock), to
accurately reflect their position. So, in essence I need to know their position in relationship to the signs
of the Zodiac, so I can set them. Thanking you in advance for your response and am anxiously awaiting
it!
●
The order of the planets from the Sun AT THIS TIME (Late 1997) is Mercury, Venus, Earth, Mars,
Jupiter, Saturn, Uranus, Pluto, Neptune. Most of the time Pluto is the most distant planet from the
Sun, but it has a highly elliptical orbit and is closer than Neptune until some time in 1999.
If you are asking the location of the planets in the sky, your best bet would be to get a amateur
astronomy magazine like Sky & Telescope or Astronomy. The positions of the inner planets change
pretty quickly. There are also a number of web sites which might be helpful, including:
http://www.currentsky.com/current/
(Current Sky)
http://www.pa.msu.edu/abrams/diary.html
(Abrams Planetarium Skywatcher's Diary)
http://www.skypub.com/whatsup/saag.shtml
The first of these sites, in particular, has the angular position of all the planets listed for different
days.
Go back to "Astronomy" questions.
What is the direction of the planets' orbits around the Sun? Clockwise or counterclockwise? Thank
you.
●
Counter-clockwise!
In detail: if you could fly above the top of the Sun -- above its north pole -- and look down at the
solar system, you would see all the planets in their orbits. If these orbits were clock faces, the planets
would be orbiting counter-clockwise.
All nine known planets (and most known asteroids, I think) orbit "prograde," in the same counterclockwise direction around the Sun. The solar system follows the "right-hand rule:" stick your right
thumb straight up. Your thumb points straight up out of the Sun's north pole, and your fingers curl
around in the direction of the planetary orbits.
Astronomers think that this common revolution is left over from the spin direction of the giant
spinning gas cloud that probably formed our solar system approximately 4.5 billion years ago.
I was not sure of this answer immediately, so I checked one of LASCO's movies. Our C3 telescope
shows many stars in its background; they appear to move from left to right. (You can see this on our
web page.) This left-to-right motion confirms a counter- clockwise orbit.
http://sohowww.nascom.nasa.gov/explore/faq/astronomy.html (13 of 36) [2000/01/11 11:58:29 AM]
Dr. SOHO's FAQ: General Astronomy Questions
Put something -- a basketball -- in the middle of a room. Now walk around that basketball
counter-clockwise. You are now "orbiting" it in the same direction as the planets orbit the Sun. Look
toward the basketball, and you will see the walls of the room passing behind it from left to right.
Go back to "Astronomy" questions.
●
How large is our solar system (in light years)?
This depends on your definition. The farthest of the nine planets is usually Pluto. Right now, in 1998,
it is Neptune. By 1999 Pluto will be further from the Sun again.
(The orbits are ellipses, and they cross each other.) Pluto's orbit is roughly 39 times farther from the
Sun than the Earth. That is about 3.627 billion miles (5.8 billion km). That's only about 0.0006 light
years. Light travels six trillion miles in a year, you see.
However, the solar system contains more than just planets. Astronomers think that there is a "cloud"
of comets -- the Oort cloud -- far beyond the orbit of Pluto. This belt of tenuously-captured comets
may extend out to about a tenth of a light year, possibly more. (The Nine Planets web page, linked
below, says it may go out to a light year.) Comets are far too small to detect at such a distance. But
when Oort cloud comets get knocked into the inner solar system (through collisions or gravitational
perturbations), we can measure their orbits. That is how we know the cloud is there.
I can think of a third factor to define the size of the solar system: the heliopause. The heliopause is
the area of space where the Sun's solar wind hits the interstellar medium. There should be a shock
wave here, and the two Voyager spacecraft are listening for it. They will detect it as they cross it,
sometime in the next few decades. Stay tuned! (The heliopause should be much farther out than
Pluto, but much closer than the Oort cloud.)
Try this web site for more information: http://www.seds.org/nineplanets/tnp/
(The Nine Planets)
Go back to "Astronomy" questions.
Hi. Can you tell me if a new planet (number 10) has been discovered? If so, where in the solar system
is it, and what is its name?
●
We have not discovered a 10th planet. We are, however, starting to discover planets around other
stars.
Go back to "Astronomy" questions.
I am a writer for the science radio series Earth&Sky. A friend of mine at the University of Pittsburgh
suggested that you might be able to answer one of our listener's questions. Here it is:
How come the moons of other planets have names (Jupiter has Io, for example) while Earth's
doesn't? Somehow 'The Moon' doesn't seem to cut it since there ARE other moons. It seems very
Earth-centric which I would understand if the other planets were called 'earths' (as in the Earth
'Jupiter'). Do all languages refer to our moon in a generic way?
●
http://sohowww.nascom.nasa.gov/explore/faq/astronomy.html (14 of 36) [2000/01/11 11:58:29 AM]
Dr. SOHO's FAQ: General Astronomy Questions
First, and most generally, I would appreciate any insight you have into this question.
Second, do you know (or could you tell me where to find out) if "Earth," "Moon," and "Sun" are the
official names for those bodies? (I have often seen references to "Terra," "Luna," and "Sol," in line
with the Roman derivations of the other planets and satellites.)
Finally, does the following story seem plausible (assuming your answer to my first question doesn't make
it absurd):
Before Galileo discovered Jupiter's satellites, (1) "Moon" was the name (solely) of Earth's satellite, (2)
"Planet" meant any body that moved against the background of the stars, and (3) "Earth" designated
the Earth, but also meant something like "center of the solar system (or universe)." When Galileo
discovered Jupiter's satellites, (1) the term "moon" was extended to refer metaphorically to satellites in
general. After the Copernican view became established, (3) Earth was more naturally classified as a
planet (since it moved as planets do), and (2) "planet" came to mean a body (of a certain size) that
revolves around the Sun.
If this is not a question that relates well to your field, I wonder if you could refer me to a colleague who
might be a good source. Thanks very much for your time.
My understanding of the situation is pretty similar to yours: "Earth" originally referred to this special
place where we live and "moon" referred to a particular bright object in the sky. "Planets" referred to
other smaller "wanderers" in the sky. Once we understood that the Earth and Moon were analogous to
the planets and their satellites, we extended the names in both directions, referring to Earth as a planet
and the other planet's satellites as "moons". I don't know enough other languages to make a blanket
statement about them - I assume most of them have a special name for the Moon, but I have no idea
how many extend that word to the satellites of other planets.
I am not a special authority on this. Here at NASA people just use the words "Earth," "Moon," and
"Sun" (when speaking English, anyway). I don't know if these names are "official," though.
I think you might want to talk to someone at the International Astronomical Union. They are the ones
officially responsible for naming celestial bodies. See:
http://wwwflag.wr.usgs.gov/USGSFlag/Space/nomen/nomen.html
(Planetary Nomenclature Introduction)
You may also find this page interesting:
http://www.seds.org/nineplanets/tnp/days.html
(Appendix 5: "Planetary Linguistics," at The Nine Planets)
Go back to "Astronomy" questions.
●
How did you name the planets?
People have known about Mercury, Venus, Mars, Jupiter, and Saturn for a long time because we can
see them easily from Earth. Different cultures have given them different names. The Romans named
them after their gods, and we use those names. When the other three planets were discovered
(Uranus, Neptune, and Pluto), they were named after other Roman gods so that the names would fit
http://sohowww.nascom.nasa.gov/explore/faq/astronomy.html (15 of 36) [2000/01/11 11:58:29 AM]
Dr. SOHO's FAQ: General Astronomy Questions
with the names of the other planets.
Each planet's moons were usually named from Greek and Roman mythology, too. For example,
Phobos and Deimos were cohorts to Mars, the Roman god of war. One big exception to this pattern is
Uranus; all of Uranus' moons are named for characters in Shakespeare. Well, all except for the newly
discovered "S/1997 U 1" and "S/1997 U 2," of course. (Oh, and the one moon named for an
Alexander Pope character!)
A good source for the science of and stories behind the planets in our solar system, try The Nine
Planets:
http://www.seds.org/nineplanets/tnp/
(The Nine Planets)
In particular, see their page on "Planetary Linguistics":
http://www.seds.org/nineplanets/tnp/days.html
(Appendix 5: Planetary Linguistics)
(UPDATE, April 1998: Those two new moons of Uranus may soon have "real" names. The moons'
discoverers have proposed "Caliban" and "Sycorax" -- the native in Shakespeare's The Tempest and
his mother. Soon, the International Astronomical Union will vote to make these names official.)
Go back to "Astronomy" questions.
●
When was Mars discovered?
Before we go any further, I must encourage you to check an astronomy book in the library for Mars
facts. I also recommend two good web sites for planetary information:
http://www.seds.org/nineplanets/tnp/
(The Nine Planets -- by Bill Arnett)
http://www.tcsn.net/afiner/
(The Nine Planets -- for Kids!)
Mars was not discovered. It is visible to the naked eye, so it was quite familiar to our prehistoric
ancestors. Mercury, Venus, Mars, Jupiter, and Saturn are all quite visible to us. Check an astronomy
magazine or website to find out where to look. (Mercury is the hardest to see, because it stays so
close to the Sun in our sky.)
Have you ever looked at Mars in the sky? It really does have a distinct reddish-orange color. That's
how the equally orange- looking star Antares got its name: Anti-Ares, the Rival of Mars.
Mars' moons, Phobos and Deimos, were discovered quite recently. Asaph Hall, an astronomer at the
U.S. Naval Observatory in Washington, DC, found them in August of 1877.
Go back to "Astronomy" questions.
Given discrete polarization and axial rotation of the Sun, is the operation of celestial mechanics such
that the Earth's perspective is always equatorial rather than polar?
●
http://sohowww.nascom.nasa.gov/explore/faq/astronomy.html (16 of 36) [2000/01/11 11:58:29 AM]
Dr. SOHO's FAQ: General Astronomy Questions
The Sun, like the Earth, rotates on fixed axis relative to the stars. The Earth revolves about the Sun in
its orbit, and axis of this orbit is also fixed relative to the stars, aside from any small perturbations
from other planets such as Jupiter. The angle between these two axes is about 7.25 degrees. Thus, the
Earth is never more than 7.25 degrees away from the Sun's equator. The other planets in the solar
system also have orbits which lie close to the Sun's equator. It is believed that this is because all the
planets condensed out of a disk which revolved about the Sun in the early days of its formation. This
disk was left behind as the Sun contracted, and thus had the same rotation as the Sun itself.
Go back to "Astronomy" questions.
This question came up at a lunch discussion at work. What is the oldest planet? Has that been
determined, or were they basically all formed at the same time?
●
We currently believe that all the planets of the solar system formed at the same time, about 4.5 billion
years ago. If you have Web access, you can read more about the formation of the solar system at:
http://www.seds.org/nineplanets/tnp/origin.html
(The Origin of the Solar System)
People sometimes refer to comets as leftovers from the early times of the solar system. This is
because they believe that comets haven't changed much since their formation, while planets that are
closer to the Sun have lost some of the more volatile elements (such as hydrogen compounds) that
may have been present in their initial atmospheres. Also, the surfaces of some planets are older than
others. The Earth's surface changes a lot, because of geological processes and erosion due to air,
water, and life. Thus, its surface is "younger" than that of the Moon. That's one reason why we see so
many more craters on the Moon than on the Earth.
Go back to "Astronomy" questions.
I have another question for you about astronomy. Which planets have magnetic fields? Which have
atmospheres? Which have water? Can you tell me all the moons of Jupiter and the size of each?
●
You asked about atmospheres and magnetic fields. An EXCELLENT reference for this information is
The Nine Planets website:
http://www.seds.org/nineplanets/tnp/
(The Nine Planets, by Bill Arnett)
Think of it as an excellent, well-illustrated astronomy textbook which is both easy to read and kept
up-to-date with the latest discoveries. You will find information about planetary atmospheres here.
I ran through it to give you the summaries of known magnetic fields. The following quotes are direct.
What strikes me (as it will you, when you read about the atmospheres) is the tremendous variety of
our solar system:
Mercury: "...has a small magnetic field whose strength is about 1% of Earth's."
Venus: "...has no magnetic field, perhaps because of its slow rotation."
Earth: We know Earth has a magnetic field, because compasses work. Scientists are still somewhat
http://sohowww.nascom.nasa.gov/explore/faq/astronomy.html (17 of 36) [2000/01/11 11:58:30 AM]
Dr. SOHO's FAQ: General Astronomy Questions
unsure of what causes a planet's magnetic field. Earth's Moon has no overall magnetic field, but there
are ancient pockets of magnetism all over the crust.
Mars: After decades of trying, astronomers finally found parts of an ancient,weak Martian magnetic
field in 1997. The Mars Global Surveyor spacecraft made this discovery, using a magnetometer
provided by a scientist right here at Goddard Space Flight Center.
Jupiter: I know that Jupiter has a HUGE magnetic field. This field traps powerful radiation belts
around the planet, which are much larger versions of Earth's Van Allen Belts.
Saturn: "Like the other jovian [gas-giant] planets, Saturn has a significant magnetic field."
Uranus: has a magnetic field which "...is odd in that it is not centered on the center of the planet and
is tilted almost 60 degrees with respect to the axis of rotation. It is probably generated by motion at
relatively shallow depths within Uranus."
Neptune: yes, but "...like Uranus', oddly oriented and probably generated by motions of conductive
material (probably water) in its middle layers."
Pluto: Probably unknown. The Nine Planets does not yet mention a magnetic field. (I'm trying to
think of ways to detect a Plutonian magnetic field. If it were really strong, like Jupiter's field, we
could probably "hear" it in a radio telescope. If it were weak, like Mars', we would need to send a
sensitive magnetometer on a spacecraft. NASA is working on a "Pluto Express" spacecraft, but it
won't be ready for several more years. Until then, we won't know - and I'm not certain Pluto Express
will have a magnetometer.)
You also asked about water. As far as I know, liquid water has never been confirmed anywhere
beyond Earth. I can name several places where water ice is known or suspected: Europa, the Martian
poles, the lunar poles, even Mercury's poles(!).
Water is important, scientifically. As NASA scientist Christopher McKay recently said, the search for
life in weird places on Earth is the search for liquid water. A search for life on other planets will
probably also be a search for liquid water. This is part of why people have gotten so excited about
Europa: there might be liquid water beneath the surface.
Regarding your question about Jovian moons, I recommend The Nine Planets website. They can keep
track of such things far better than we at SOHO can!
Go back to "Astronomy" questions.
●
About how many Jupiters could fit inside the Sun?
The volume of a sphere is 4/3 × r3, where r is the radius of the sphere and r3 means (r × r × r). The
Sun has a radius of 700,000 kilometers and Jupiter has a radius of about 70,000 km. (See:
http://www.seds.org/nineplanets/nineplanets/nineplanets.html
(The Nine Planets)
or an astronomy text book.) That means that the volume of the Sun is about 1000 times that of Jupiter
(700,0003 divided by 70,0003 -- talk to your teacher about this if it isn't clear), so you could fit about
1000 Jupiters inside the Sun. That assumes that you could mush all the Jupiters in without any spaces
http://sohowww.nascom.nasa.gov/explore/faq/astronomy.html (18 of 36) [2000/01/11 11:58:30 AM]
Dr. SOHO's FAQ: General Astronomy Questions
between them.
Go back to "Astronomy" questions.
●
Is it possible that the Sun and/or black holes are made up of black matter?
I assume that by "black matter" you mean the "dark matter" to which astronomers sometimes refer.
The idea behind "dark matter" is a problem with two different ways of calculating the amount of stuff
in the Universe.
Astronomers can estimate the mass of a star just by looking carefully at its brightness and color
(spectrum). Similarly, they can estimate the mass of a whole GALAXY by using a statistical model
of the stars that make up that galaxy. In this way, they can determine just how massive each galaxy
that we can see really is.
There's another way of estimating mass of galaxies. Astronomers can watch the actual motions of
stars as they orbit around the cores of their home galaxies, by using the Doppler shift: the side of the
galaxy that happens to be spinning away from us looks redder than the side that is spinning toward
us. The more massive the galaxy is, the faster the stars orbit it.
The problem is that the mass (as determined from Doppler shifts) is about 10 times higher than the
mass (as determined from the brightness of stars) for most galaxies. So there's got to be a WHOLE
LOT of matter out there that isn't glowing -- stars must be a kind of special case. That extra matter is
called "dark matter". Black holes may or may not qualify -- while no light can escape from the black
hole itself, matter that is falling into the hole heats up and glows very brightly on the way in.
Our Sun wouldn't qualify as "dark matter," because, well, it's glowing. So if there are any alien
astronomers looking at our star, they will count its mass properly.
Go back to "Astronomy" questions.
Could other planets pull on other planets causing their crust to move? I recently heard that scientists
had found Neptune by the pull it made on another planet, and that this same method is used to find other
planets at distant stars.
●
The gravitational influence of a distant planet falls off like the square of the distance to that planet, so
that distant planets' effects on each other are very weak, indeed. Astrologers might sometimes tell
you that at your birth, the gravitational influences of distant planets (such as Jupiter) affect the course
of the rest of your life. In fact, the doctor that delivers you has more gravitational influence on you
than Jupiter does!
The reason that the doctor has such a strong gravitational pull on you (compared to Jupiter) is that,
while Jupiter is much much bigger than your doctor, it is also much, much farther away.
The tidal influence of a distant planet is caused by the variation in the distant planet's influence
versus position. If you know calculus, it is the derivative of the gravitational force. This tidal
influence falls off like the CUBE of the distance to the planet -- even faster than the gravitational
force itself.
For this reason, the tides caused by distant planets are usually very weak. Tides on the Earth are
http://sohowww.nascom.nasa.gov/explore/faq/astronomy.html (19 of 36) [2000/01/11 11:58:30 AM]
Dr. SOHO's FAQ: General Astronomy Questions
caused (roughly equally) by the Sun and the Moon. The Moon is much smaller than the Sun -- but it's
also much closer, and those two comparisons tend to cancel out.
Gravity and tides from distant planets are certainly detectable. You heard correctly: Neptune's
discovery was mathematically predicted from gravitational abnormalities in Uranus' orbit. But
remember that the tides from a distant planet are much, much less than the gravity from that planet,
because gravitational force falls off like only the square of the distance, while tidal force falls off like
the cube.
You're also quite right that planets' gravitational influences are used to find planets of distant stars. If
you grab a big, heavy rock and then spin around and around while holding the rock at arms' length,
you'll find that you don't spin around the center of your body like you would without the rock. You
always spin around your center of gravity, and holding the rock out moves the center of gravity away
from the center of your body.
Similarly, our Moon doesn't orbit around the very center of the Earth. Both the Moon AND the Earth
together orbit around their mutual center of gravity. Because the Moon doesn't have much mass
compared to the Earth (only about 1/250th of the Earth's mass), the CG is still inside the Earth itself
-- but it's offset toward the Moon. The Earth and Moon both orbit in the same amount of time, so that
the CG always stays deep inside the Earth on the line between the center of the Earth and the center
of the Moon.
Likewise, planets pull their stars' centers of gravity away from the center of the star. The star, too,
orbits around the center of gravity of its whole solar system.
But if a star is orbiting, it is moving -- which Doppler- shifts its light! So, if a star is slightly bluer for
a few months, then slightly redder, then slightly bluer again... then we know that it's orbiting about a
center of gravity with something else (like a planet) that's massive and not glowing on its own.
Go back to "Astronomy" questions.
We know that the "planets" in our solar system exist. But what I don't know is the significance of
them. For example: If Jupiter did not exist, would things be different? If Neptune didn't exist? You see
what I'm getting at? How do they affect us?
●
That's a really tough one. We'd have to ask ourselves whether the solar system as we know it would
exist without Jupiter. We think all the planets in orbit around the Sun were formed at about the same
time, out of the cloud of gas and dust that surrounded the newly-born Sun. This is well explained at:
http://www.seds.org/nineplanets/tnp/origin.html
(The Origin of the Solar System, at The Nine Planets website)
Jupiter is by far the largest planet in the solar system. It has as much mass as all the other planets
combined. As it was forming, it must have had a profound influence on the formation of the small
inner planets. Without Jupiter, maybe Earth would not have formed as it did.
If we supposed that Earth would have formed okay without Jupiter, we're still not out of the woods.
Jupiter's gravity disturbs icy bodies in the outer solar system, and sometimes they end up falling
down into the inner solar system, where we see them as comets. Sometimes these comets hit the
Earth. Without Jupiter, there might be fewer comets in the inner solar system. In one sense, this
http://sohowww.nascom.nasa.gov/explore/faq/astronomy.html (20 of 36) [2000/01/11 11:58:30 AM]
Dr. SOHO's FAQ: General Astronomy Questions
sounds good - nobody wants a comet falling on their head!
On the other hand, some scientists believe that Jupiter's presence prevents many comet impacts on
the Earth. I read recently that Jupiter theoretically gets hit by comets 1,000 times more often than the
Earth, thanks to its tremendous gravitational influence. We all remember Comet Shoemaker-Levy 9.
Without Jupiter, many more comets would careen around the inner solar system to potentially collide
with Earth. Unfortunately, I cannot find the reference for these speculations.
The other planets of our solar system are far less massive than Jupiter. Their gravitational influence is
less. Aside from Jupiter, would major differences in the solar system have had an effect on human
history? Not that I can think of. A different solar system could have affected our future, though: the
one we inhabit now is brimming with extraterrestrial resources (lunar oxygen, solar energy, asteroid
ores, etc.) But that is another story.
Go back to "Astronomy" questions.
I have read that asteroids are considered to be relics of the formation of the solar system. I have also
read that many asteroids are predominately iron-nickel bodies. It seems to me that these two facts are not
really compatible. I would think if asteroids were left over "stuff" from the formation of the solar system,
that there composition would be less uniform and include a wider variety of materials. There should not
be predominately iron-nickel asteroids. It seems to me that you would only get such bodies if you broke
up an already-formed planet. This would have allowed the heavier materials to separate into layers (like
the Earth and other rocky planets). Breaking up such a planet could easily result in iron-nickel asteroids
as well as "rocky" asteroids. Where is my thinking wrong?
●
Your thinking isn't so far wrong. The inner solar system asteroids are thought to have originally
consisted of fewer but larger bodies, possibly about the size of the present-day asteroid Ceres. Over
time these larger bodies collided with each other, and broke up into pieces. The pieces from the outer
mantles of these larger bodies make up the carbonaceous chondrite family of asteroids, and those
from the cores make up the iron-nickel family. One of the pieces of evidence that we have for this
theory is an analysis of the asteroid orbits. (There is a good deal of research still to be done about
asteroids. Space missions like NEAR and NEAP should help.)
Go back to "Astronomy" questions.
I am currently writing a science fiction story about the Sun, and I am seeking some information. Is is
true that Jupiter is considered a Brown Star?
●
According to the definition in the text book I am looking at (Exploration of the Universe by Abell,
Morrison, and Wolff), Jupiter is too small to be a brown dwarf. They say that a brown dwarf is "an
object intermediate in size between a planet and a star. The approximate mass range is from about
twice the mass of Jupiter up to the lower mass limit for self-sustaining nuclear reactions, which is
0.08 solar masses."
Here are some web sites which talk about brown dwarfs:
http://www.bahnhof.se/~davidgr/browndwf/bd_home.html
(Brown Dwarfs)
http://whyfiles.news.wisc.edu/017planet/contrary.html
http://sohowww.nascom.nasa.gov/explore/faq/astronomy.html (21 of 36) [2000/01/11 11:58:30 AM]
Dr. SOHO's FAQ: General Astronomy Questions
(I say "planet," and you say "brown dwarf")
Go back to "Astronomy" questions.
●
How many days, hours, and minutes does it take for the Earth to revolve around the Sun?
The sidereal year - the period of the Earth's orbit with respect to the stars - is 365d 6h 9m 10s
The tropical year - the time it takes from one vernal equinox (the start of Spring) to the next is 365d
5h 48m 46s.
These are different because of the precession of the Earth's axis - the Earth is slowly wobbling, which
affects the apparent position of the Sun against the stars slightly.
I got this information from the astronomy text book Exploration of the Universe by Abell, Morrison,
and Wolff.
Go back to "Astronomy" questions.
My friends and I have a few questions for you: What is the exact period of rotation of the Moon?
What is the exact period of revolution? As it turns, the Moon wobbles. Why? We know that there are
plans to build a colony or research station on the moon. Can you show us a sketch of it?
●
Let me answer your questions one at a time:
What is the exact period of rotation of the moon?
Approximately one month, or 27.322 days. (According to Zelik and Smith, Introductory Astronomy
and Astrophysics, 2nd edition)
What is the exact period of revolution?
Ah, this depends on how you define a revolution. The Moon takes about 27.322 days (the same as the
rotation) to orbit the Earth. Actually, it is orbiting the "barycenter" (center of mass) of the
Earth-Moon system. Think of us as a double-planet system.
However, it takes the Moon 29.5 days to get from full Moon to the next full Moon. This is the
rotation rate relative to the Sun. The Earth and Moon move about 1/12 of the distance around the sun
during each month.
The basic result is that the Moon rotates exactly once for every revolution. The Earth is at the center
of that revolution, so we only see one side of the Moon.
There is no "dark side of the Moon;" the lunar farside gets just as much sunlight as the nearside!
As it turns, the moon wobbles. Why?
I cannot find technical data about the physical wobble of the Moon's rotation, but there is an
apparent wobble. The Moon's rotation rate is constant. However, the Moon's orbit is not a perfect
circle. It is an ellipse (not an oval or an egg-shape), and the Moon's orbital speed varies along this
ellipse.
http://sohowww.nascom.nasa.gov/explore/faq/astronomy.html (22 of 36) [2000/01/11 11:58:30 AM]
Dr. SOHO's FAQ: General Astronomy Questions
At the high point of the orbit, the Moon is orbiting a little slower than it is rotating. At the low point,
it is orbiting (or revolving) a little faster. This way, we can "peek" at parts of the Moon's eastern and
western edges during each month.
There's also some north-and-south "peeking," because the Moon's orbit is inclined relative to the
Earth's. At times, we here on Earth are looking up at part of the Moon's south pole, for example. The
result of all these so-called "libration" effects means we can see about 59% of the Moon's surface
each month, not just one-half.
(I cannot draw a picture of the Moon's orbit here, but there is a good diagram in Chapter 4 of Zelik
and Smith's astronomy book. You will find it - or a similar astronomy book - in a good library.)
You can understand the rotation-and-revolution business with a demonstration. See, for example, the
"Bad Astronomy" page:
http://smart.net/~badastro/bad/moon_spin.html
(Astronomy Misconceptions: The Moon Does Not Spin)
We know that there are plans to build a colony or research station on the moon. Can you show us a
sketch of it?
The story of post-Apollo plans to return to the Moon is a long and complicated one. Many of these
proposals have come from outside NASA. I don't know many of the details. However, I can suggest
the following links:
http://www.seds.org/
(Students for the Exploration and Development of Space.
They have links and resources all over the place!)
http://www.ssi.org/
(Space Studies Institute.
They are working on space colony research projects, which includes future factories on the Moon.)
http://www.asi.org/
(The Artemis Society.
They want to build a private Moon base in the next decade. They have many pictures.)
http://www.millenial.org/
(First Millenial Foundation.
Also a non-NASA group, they are working to promote space colonization.)
Check your local libraries for a 1987 Ben Bova book, Welcome to Moonbase. It is well-illustrated by
Pat Rawlings, a renowned space artist.
Lunar bases are crucial to the space-colony concepts of the late Gerard O'Neill and others. Look for
books by O'Neill, T.A. Heppenheimer, and G. Harry Stine.
Go back to "Astronomy" questions.
I am helping my nephew with a project for school. He is in the 4th grade studying Jupiter. He is
looking for a picture of Jupiter and it's moons. We haven't had any luck finding one -- do you have one
or know where we can get one?
●
http://sohowww.nascom.nasa.gov/explore/faq/astronomy.html (23 of 36) [2000/01/11 11:58:30 AM]
Dr. SOHO's FAQ: General Astronomy Questions
I think I can get you off to a really good start. For one thing, there are some very nice images here at
Goddard at:
http://nssdc.gsfc.nasa.gov/photo_gallery/photogallery-jupiter.html
(National Space Science Data Center Photo Gallery: Jupiter)
And there are excellent images at:
http://spaceart.com/solar/eng/homepage.htm
(Views of the Solar System)
As always, visit The Nine Planets for all of your planetary science needs:
http://www.seds.org/nineplanets/tnp/
(The Nine Planets)
Happy viewing!
Go back to "Astronomy" questions.
●
What are the chances that the piece of Mars which came to Earth is really from one of Mars' moons?
The chances are almost nil, because a meteorite from Mars' moons would possess a chemical
composition different from that of a true "Martian meteorite."
There are about a dozen meteorites found here on Earth which have very similar chemical
compositions. These compositions are a little different from more typical meteorites. We think they
all came from the same place. Trapped gases in one of these meteorites matched the 1976 Viking
lander measurements of Mars' atmosphere. Therefore, we think that these dozen meteorites all came
from Mars. They were probably ejected by the impact of a giant meteoroid or comet.
Mars' moons, Phobos and Deimos, do not have any atmosphere, and they are thought to be captured
asteroids. Any meteorite originally from these moons would be chemically different from a Martian
meteorite.
One year ago, a NASA/industry research team announced that they had found possible evidence of
microscopic fossils in one of these Martian meteorites. I think we have all heard the details of this
story -- it is very interesting.
Much more recently, another scientist has found evidence for other microscopic life in a different
type of meteorite. This type of meteorite chemically matches certain kinds of asteroids. Phobos'
albedo (or reflectivity) matches certain kinds of asteroids -- specifically, the "C-type" ones. (We've
physically sampled neither Phobos nor any asteroid, but we can take remote spectra of each.)
Therefore, it might be possible that this second mysterious meteorite might be from Phobos, but
there is only one small Phobos and there are many, many C-type asteroids. The odds are against it.
Go back to "Astronomy" questions.
You stated that NASA has found many C-type asteroids that might be from Phobos. But you also
stated that there are many C-type asteroids and only one Phobos. Well, what are the chances that an
●
http://sohowww.nascom.nasa.gov/explore/faq/astronomy.html (24 of 36) [2000/01/11 11:58:30 AM]
Dr. SOHO's FAQ: General Astronomy Questions
asteroid exploded (for whatever reason) next to Mars, and one of the pieces got captured by Mars'
gravitation and became Phobos, while the other pieces went storming to Earth?
I did not imply that the C-type asteroids all came from Phobos, I merely said that one particular
meteorite may have originated on Phobos -- and that supposition is based solely upon my own
sketchy knowledge of asteroidal theory.
Please remember that "asteroid" and "meteorite" mean two different things. An asteroid is a rocky
body in outer space. A meteorite is any rock from space which lands on the Earth's surface.
Meteorites can come from many sources: comets (possibly), the asteroids themselves, or other planets
(like the famous Martian meteorites).
(There's even a class of meteorites, called "tektites," that are thought to be from Earth itself! When a
giant object slams into Earth's surface, it blows pieces of the crust high above the atmosphere. Many
of them reenter and land just like extraterrestrial meteorites. However, their chemistry identifies them
as terrestrial rocks. I hope you're less confused than I am at this point. I'm not an expert.)
Other than that, you are half-correct. Most current theories say that the asteroid belt (between the
orbits of Mars and Jupiter) was not formed by an exploding planet. It seems more likely that the
asteroids are remnants that never condensed into a planet in the first place. Perhaps Jupiter's massive
gravitational influence prevented any such formation.
If I remember correctly, some scientists do think that one or both of Mars' moons are captured
asteroids. (Some don't, though.) I recently read that both Phobos and Deimos appear to have C-type
chemical compositions. But that is the most I can tell you, with my own education and the sources at
hand. You will need to do further research on your own.
Some books that might help you are The New Solar System, by Beatty, Chaikin, et. al. and any book
by John S. Lewis. His latest books are Rain of Iron and Ice and Mining the Sky.
Go back to "Astronomy" questions.
I read some time ago about the Icarus Asteroid, which came some millions of km to the earth in 1949
and 1968. According to some scientists it may be coming dangerously close to Earth in 2006. Is there
information about this? Is there a homepage dedicated to Icarus?
●
Ah, Icarus! Icarus, or asteroid #1566, was discovered by Walter Baade at Palomar on 26 June 1949.
Icarus was the 13th "Apollo object" to be discovered; an Apollo object is simply an asteroid whose
orbit crosses that of the Earth.
If it crosses Earth's orbit, it may hit us sooner or later.
I looked Icarus up in two excellent books about asteroids, Islands in Space by Cole and Cox (sadly
out of print) and Rain of Iron and Ice by John S. Lewis. If I am reading my orbital mechanics
correctly, Icarus has an aphelion (high point) of 1.968 AU and a perihelion of 0.187 AU. One AU is
the distance between the Earth and the Sun. Icarus itself is approximately 1.4 kilometers in diameter.
Enough about the orbital mechanics, save for one thing: every 19 years, this asteroid swings by Earth.
That is how Baade found it in 1949. It encountered Earth again on 14 June 1968, at a safe distance of
4 million miles (almost 6.5 million km). The next encounter is in AD 2006, as you correctly indicate.
http://sohowww.nascom.nasa.gov/explore/faq/astronomy.html (25 of 36) [2000/01/11 11:58:30 AM]
Dr. SOHO's FAQ: General Astronomy Questions
(As far as I know, the 2006 encounter is not considered an impact threat.)
The 1968 encounter inspired a famous design project at MIT in 1967. The students were told: pretend
Icarus is on a collision course with Earth, and design a system to destroy/deflect it safely. The result
was "Project Icarus," which proposed the use of Saturn rockets and nuclear warheads to deflect
Icarus' orbit.
Here is an excerpt from the Preface to the Icarus report:
"The impact of Icarus would produce a crater only 10 or 15 miles in diameter. Its effects would
be felt world-wide, however. The energy involved is the equivalent of 500,000 megatons of
TNT -- two orders of magnitude above that involved in the largest recorded earthquake, and
four or five orders of magnitude more than Krakatoa... If the strike occurred in midocean,
tsunamis in the 100-foot category would cause world-wide damage. If the strike occurred on
land, the blast wave would level trees and buildings within a radius of several hundred miles,
and some 108 tons of soil and rock dust would be thrown into the stratosphere, where for
several decades it would act to reduce the solar radiation ordinarily received at earth's surface
and threaten the triggering of an ice age."
- MIT Report No. 13, "Project Icarus" Interdepartmental Student Project in Systems Engineering.
MIT Press, Cambridge, Massachusetts, 1968
This report directly inspired the 1979 film Meteor (widely classified as a bad movie, though it did
have Sean Connery), although the killer-asteroid idea was used in science fiction literature
throughout the 1970s. Hollywood will release two such killer-asteroid films in 1998. (Yes, one of
them will feature a comet, but that is the same basic principle.)
In the 1980s, killer-asteroid impacts became a respectable topic among scientists. The Alvarez team
theorized that an impact had killed the dinosaurs 65 million years ago. In 1994, Comet
Shoemaker-Levy 9 impacted Jupiter. The splash marks in Jupiter's atmosphere were bigger than the
Earth.
A comet like that one could swoop in on us with little warning, and astronomers keep finding new
Icarus-like bodies throughout the inner solar system. Should we worry? Yes. Icarus' orbit is
well-known, but there are many potential killer-asteroids to worry about.
Well, your question was rather timely, wasn't it? For much of yesterday, the Western news media
made it sound like we had only 30 years left to live! Now, it looks like Asteroid 1997XF11 will pass
us harmlessly in AD 2028.
Remember: the vast majority of asteroids in this solar system orbit in a belt between Mars and
Jupiter, or even further out. A few, however, do not. These "near Earth asteroids"or "near Earth
objects" pose a certain threat. Our home planet has been hit by asteroids many times before, and it
might be hit again someday.
It might. Or it might not; spaceflight has given our species the tools with which to defend ourselves.
(Comets can also hit us, just like Shoemaker-Levy 9 hit Jupiter in 1994. Comets can be even scarier,
because they can plummet into the solar system with only a few months' warning. Hyakutake was
discovered only a few months before closest approach, and that was a relatively large, bright comet.)
http://sohowww.nascom.nasa.gov/explore/faq/astronomy.html (26 of 36) [2000/01/11 11:58:30 AM]
Dr. SOHO's FAQ: General Astronomy Questions
You asked for information and websites. The following page is technical and policy-oriented, but
please look up the books in the bibliography:
http://k2.space.swri.edu/clark/ncar.html
("Impact Hazard: Predictability and Policy Issues," by Clark Chapman, a respected planetary
scientist)
Here are a few more links on the subject, from a list someone posted yesterday on the net:
http://k2.space.swri.edu/clark/ncar.html
(Today's Astronomy Picture of the Day -- Asteroids!)
http://www.llnl.gov/planetary/
(Planetary Defense Workshop: Proceedings)
http://impact.arc.nasa.gov/index.html
(Asteroid and Comet Impact Hazard)
http://www.au.af.mil/au/database/research/ay1995/acsc/95-225.htm
(Planetary Asteroid Defense Study)
Go back to "Astronomy" questions.
Do you have any information concerning the "asteroid" that recently hit Greenland? My teacher gave
this information about the Greenland incident, and I just want to confirm the info. Thank you.
●
Actually, the Greenland object was a meteor, not an asteroid. Asteroids are large chunks of rock (50 100's km in size) while meteors are small pieces of an asteroid or small pieces of a comet. If an
asteroid hit the Earth, it wold be potentially devastating for all of us.
This meteor (or "bolide") was seen by many people on 1997 December 9 as it lit up the sky upon
entry near southern Greenland. It is predicted to have impacted in the snow fields near 61d 25' North
and 44d 26'West and not far from the town of Qaqortoq. No physical evidence has been found so far,
but many expeditions are planned to this region in the spring after the area becomes passable.
It was pretty bright, however. A parking lot surveillance camera in Nuuk, 600 kilometers away seems
to have recorded the flash from this bolide - reflected off a car body. Weather satellite observations
were reported to have seen an impact cloud over the site, but this was later found not to be a related
event. The Norwegian Seismic Array recorded a 10 second event from about this area at the same
time, but this was observed several minutes after the impact which is the wrong time scale for a shock
wave to travel to the array from that region, and seismic stations in Greenland have not turned up any
seismic events from their own backyards! For more information, visit the Dutch Meteor Society
home page; also, the 1997 December 30 issue of EOS from the American Geophysical Union has an
article about it.
Go back to "Astronomy" questions.
●
Where can I find information on the meteor shower that is coming this November [1998]? I am
http://sohowww.nascom.nasa.gov/explore/faq/astronomy.html (27 of 36) [2000/01/11 11:58:30 AM]
Dr. SOHO's FAQ: General Astronomy Questions
interested in the likelihood of this happening, the likelihood of the satellites being hit, and if so, the
probable extent the damage?
Another reader asked:
I am mailing you in regards to the 1,000 or so meteors that are heading for Earth. I have heard they are
only the size of the width of a single hair. That is all I have heard about them. I'm not sure if you can
answer this inquiry or not, but I was wondering if you could email me some info about and perhaps some
pictures of the meteors.
Here are some Web sites pointing to news stories on this upcoming Leonid meteor shower
phenomena:
http://cnn.com/TECH/space/9804/27/leonid.meteor/index.html
(Meteor shower poses threat to Earth satellites)
http://www.abcnews.com/sections/science/DailyNews/satellites980427.html
(Huge Meteor Shower Expected in November)
And here is another site that may interest you: http://www-space.arc.nasa.gov/~leonid/index.html
(Leonid '98 Meteor Outburst Page)
In general, a "meteor" is an object which is burning up as it enters the Earths atmosphere.
"Meteoroids" are what we call them while they are in space - before they hit the atmosphere.
Meteroroids as small as the ones you mention are certainly not anything to worry about on the ground
(although they are quite pretty), and I doubt we have pictures of them in space. You can see a few
meteors on almost any clear, dark night of the year - though they do cluster in "meteor showers" at
the same times each year.
These annual meteor showers are usually caused when the Earth moves through the orbit of a comet.
The orbital path contains lots of small particles, and a meteor shower results. There are a number of
web sites which talk about these. Here are just a few of them (the first includes pictures of meteors
burning up in the Earth's atmosphere):
http://csep10.phys.utk.edu/astr161/lect/meteors/showers.html
(Meteors and Meteor Showers)
http://stardate.utexas.edu/resources/ssguide/meteors.html
(StarDate Online | Solar System Guide | Meteors)
http://www.lclark.edu/~wstone/skytour/meteors.html
(METEORS)
The Leonid meteor shower has been getting a lot of attention lately, because it peaks in intensity
every 33 years. The next peak will be November 1998 or November 1999. Space engineers are
worried about the possibility of damage to satellites and spacecraft from these tiny bits of debris.
Go back to "Astronomy" questions.
I live in Detroit, Michigan, and on our news on Thursday the 30th of April [1998] they said there were
something like 1,000 meteorites heading for Earth. They said that they were no bigger than the width of
a hair. They said the meteorites would enter the atmosphere at 2:00 a.m. over Asia for what would be a
splendid light show. They also said the meteorites would be traveling at speeds of over 20,000 mph. The
●
http://sohowww.nascom.nasa.gov/explore/faq/astronomy.html (28 of 36) [2000/01/11 11:58:30 AM]
Dr. SOHO's FAQ: General Astronomy Questions
news reported that these meteorites may collide with some of the satellites in orbit. This could cause
major problems with cellular phone companies and maybe cable companies. I was wondering if you have
foreseen any problems for yourselves or other companies that could possibly have satellites hit.
This sounds like a description of a regular meteor shower: the "Eta Aquarids," which are leftover bits
of Comet Halley. It is easier to see in the Southern Hemisphere. See the 1998 May 5 "StarDate"
program at:
http://stardate.utexas.edu/radio/calendars/s_current_cal.html
(StarDate Online Program Calendar)
or:
http://www.maa.mhn.de/Comet/Shower/eta_aquarids.html
(Eta Aquarids)
http://www.ticetboo.demon.co.uk/halley.htm
(Comet Halley's Debris)
These things are common. They can hit spacecraft, but usually they are not considered a major
hazard. We are worried about damage to spacecraft from the Leonid meteor showers in November
1998 and November 1999. These events apparently intensify every 33 years. Further discussion of
the Leonids is available elsewhere in this FAQ.
Go back to "Astronomy" questions.
●
Can you please tell me How many light years we are away from the Sun?
The answer to your question is:
0.00001598 light years from the Sun to the Earth.
That's about 1/63,000 th of a light year.
Light from the Sun only takes about 8.4 minutes to reach the Earth. Light travels at 300 thousand
kilometers per second -- that's fast enough to circle the Earth seven times in one second. The second
closest star to the Earth, Proxima Centauri, is just over 4 light years away from us. (We can't see the
Centauri system from here in the Northern Hemisphere, but I think you can!)
Scientists say that the Sun is an "average" star. In fact, if you were about 55 light years away from the
Sun, it would be too dim to see without a telescope. Space is big, and many of the stars we see at
night are relatively close to us.
(I say "relatively" because all are at least 4.5 light years away, and most stars are much further than
that. The stars in Orion's belt are about 1,200 light years away.)
From the Northern Hemisphere, I can only see one object that is outside of our Milky Way galaxy.
That is the Andromeda galaxy, which is 2 million light years away. (Or perhaps a little further. There
have been many refinements in the distance to the Andromeda galaxy just in the past ten years. The
"official" distance seems to change frequently!)
Go back to "Astronomy" questions.
http://sohowww.nascom.nasa.gov/explore/faq/astronomy.html (29 of 36) [2000/01/11 11:58:30 AM]
Dr. SOHO's FAQ: General Astronomy Questions
Now I am wondering if you could tell me how many light years are in an Earth year, and how many
Earth years in a light year.
●
The name "light year" is quite misleading. A light year doesn't measure time. It measures distance,
like a mile or a kilometer.
In my earlier answer, I described how very far apart most stars are. To measure these huge distances,
astronomers like to use a measurement much longer than a mile or a kilometer. So they use the light
year.
A light year is simply the distance light travels in one Earth year (365 days). Light always travels at
the same speed in space, and that speed is about 300,000 kilometers per second!
(We're pretty sure that nothing can travel faster than light, but that's another story.)
If you multiply the speed of light (300,000 km/sec) by the number of seconds in a year, you will get
one light year -- in kilometers.
As you can see, it's a big number. That is why astronomers prefer light years to kilometers when they
talk about large distances. It lets them write much smaller numbers!
To sum up, a lightyear is simply a measurement of distance. An Earth year is the amount of time the
Earth takes to orbit the Sun once: a little over 365 days.
P.S. When astronomers talk about distances smaller than the distance between stars, they probably
won't use light years. For example, to measure distances between planets, a light year becomes a little
too big. It's like measuring the height of a classroom ceiling in kilometers. To measure planetary
distances, they use ordinary kilometers... or else something called an "astronomical unit."
An astronomical unit (AU) is simply the distance between the Earth and the Sun: about 150 million
kilometers (or 0.00001598 lightyears!). For example, the distance between the Sun and Mars is 1.54
astronomical units.
(You'd better double-check that Mars number in a science book, though....)
Go back to "Astronomy" questions.
I would like to know how long it takes the light from the Sun to travel to each of the planets, and to the
asteroid belt.
●
Here is a table of how long it takes light to travel from the Sun to each of the planets. The speed of
light is constant in a vacuum (space) and is given below, as are the distances from the Sun to each of
the planets (and asteroid belt).
1 AU = 149 million kilometers = 92.5 million miles speed of light = 300,000 kilometers/second =
186,390 miles/second
Planet
Mercury
Venus
Earth
Distance from Sun (AU)
0.387000
0.723000
1.00000
Time for Light to travel (min.)
3.20350
5.98483
8.27778
http://sohowww.nascom.nasa.gov/explore/faq/astronomy.html (30 of 36) [2000/01/11 11:58:31 AM]
Dr. SOHO's FAQ: General Astronomy Questions
Mars
1.52400
12.6153
asteroids 3.00000 (an average value) 24.8333
Jupiter 5.20300
43.0693
Saturn 9.53900
78.9617
Uranus 19.1800
158.768
Neptune 30.0600
248.830
Pluto
39.5300
327.221
The orbits of the planets are not perfect circles; they are ellipses. The above times are averages.
Actual values will vary a bit throughout each planet's "year."
Go back to "Astronomy" questions.
My question is quite simple: Can we see our own solar system in the past? I have been told that light
from other 'stars' takes millions of years to reach Earth. Could one of these distant stars be 'our' own, in
the past? From what I understand, the universe is expanding, which causes the positions of the solar
system to change.
●
The light from our Sun travels outward from the solar system. There really isn't any practical way for
it to turn around and come back to us. But let us consider the various ways that it might happen.
The solar system is moving, so one might imagine the solar system passing some of its old light, and
we would see ourselves behind us. However, this isn't possible, because nothing made of matter can
travel as fast as light, and nothing can travel faster than light.
Light can be bent by gravity, and in fact we can see this effect in places. Light from far away galaxies
is sometimes bent by nearby galaxies in between us and the more distant galaxy. This effect is called
gravitational lensing. We've also sometimes seen light from stars in our own galaxy bent by other
stars. However, the amount of bending is small, nowhere near enough to turn light completely around
in the direction it came from. To do that, we'd have to have a series of stars, each bending the light in
turn. Since gravitational lensing is a rare phenomenon - the stars doing the bending have to be in just
the right position - this is highly unlikely.
One can imagine the light from our Sun bouncing off of something, and coming back at us that way.
This phenomenon is known as light echoes, and has been seen in supernovae, where the light of the
supernova explosion bounced off of gaseous clouds that were a few lightyears away from the star that
exploded. However, this echo is very diffuse, and doesn't look like a star. For that, you'd need
something that acted more like a polished mirror, and nothing like that is known in space.
You mention the expansion of the Universe. We know that the universe is not flat, but curved, and
the amount of curvature is related to the rate of expansion of the Universe. There are two possible
ways the universe could be curved. It could be open, with the Universe expanding forever, or closed,
with the Universe eventually reaching a maximum size and then start collapsing. Light follows the
curvature of the Universe. In a closed Universe, it's possible for the light to curve completely around
the Universe and come back to its starting point. However, only near the end of the Universe would
the light have had enough time to come back to its starting point. As somebody once said, once you
see your past self, you'd better duck.
So, the answer to your question is that there is no real way to see ourselves in the past. However, we
http://sohowww.nascom.nasa.gov/explore/faq/astronomy.html (31 of 36) [2000/01/11 11:58:31 AM]
Dr. SOHO's FAQ: General Astronomy Questions
can look at other stars which are similar to our own solar system, and at other galaxies which are like
our own, and see what they were like in the past. This helps tell us about our own past.
Go back to "Astronomy" questions.
Is the sunspot activity, which is at its low occurrence, related to Jupiter's orbit, which is traveling at its
fastest speed? We noticed that the perihelion date of Hale-Bopp was in the middle of the recent sunspot
activity. Is there any relation to that?
●
The short answers are: no, and no.
The solar cycle varies with an 11-year average period, although it can vary by a few years. Juptier's
"year" is 11.86 Earth years. They are close, but scientists do not believe there is a causal relation
between the two. The sunspot cycle is driven by magnetic field activity in the Sun's interior -- but
scientists aren't quite sure about the specifics of the process.
Jupiter has a large, strong magnetic field. If its magnetic field were a visible object, it would appear
in Earth's skies to be larger than the full Moon. But the Sun's magnetic field is much larger and
stronger still.
Comets, on the other hand, have no detectable magnetic field of their own. (We drew this conclusion
from spacecraft observations of Comet Halley back in 1986, which failed to find any magnetic field
as close as 4,000 km from the comet's nucleus. There was a colloquium here at Goddard on the
subject of comets just yesterday (25 April 1997), and a Goddard scientist reiterated this point.)
I say this because sunspots are known to be magnetic phenomena. They are, in fact, the places on the
photosphere where large magnetic field lines have protruded from the Sun's surface. Their cause lies
within the Sun, not without. It is unlikely that magnetically-neutral comets have much effect on
sunspots.
(When I say, "magnetically neutral," I may not be precisely correct. Comets and their tails can and do
become electrically charged by the solar wind, but you'd need to consult a comet expert for further
details.) Thanks for your inquiry!
(Postscript: Jupiter orbital parameter from "TRW Space Data," 4th edition,
TRW Science and Technology Group, 1992.)
Go back to "Astronomy" questions.
Does Jupiter have any influence on the solar cycle? I ask, because both Jupiter's orbit and the sunspot
cycle have approximately an 11-year period. Even if Jupiter does not cause the solar cycle, could it
control the cycle's intensity? Could Jupiter (or other planets) raise tides on the Sun in the same way the
Moon causes tides on the Earth?
●
This is an oft-raised question; scientists who have looked at it have found no causal relation between
the approximately 11-year solar cycle and Jupiter's 11.86-year orbital period. Those are "Earth"
years, of course.
Tides are a gravitational phenomenon. Sunspots are believed to be magnetic. Although Jupiter has a
huge and powerful magnetic field (if it were visible, it would look bigger than the full Moon in our
http://sohowww.nascom.nasa.gov/explore/faq/astronomy.html (32 of 36) [2000/01/11 11:58:31 AM]
Dr. SOHO's FAQ: General Astronomy Questions
sky), the Sun's magnetic field is bigger and stronger still.
Jupiter's gravity does make the Sun "wobble" back and forth as the big planet revolves around it.
Remember that Jupiter has 99% of the non-solar mass in our solar system, so none of the other
planets produce much of an effect. This type of wobbling is what allows astronomers to detect
possible large planets circling nearby stars.
The following page at Dr. Sten Odenwald's ASK THE ASTRONOMER site contains many
references to journal articles on this and similar subjects. Nature and Science should be widely
available in local libraries, but Solar Physics may be harder to find -- try a large university library.
Good luck, and keep us in mind for future questions!
http://image.gsfc.nasa.gov/poetry/astro/q923.html
(Do the planets affect the sunspot cycle?)
Go back to "Astronomy" questions.
Using the site at: http://www.fourmilab.ch/cgi-bin/uncgi/solar I found that the planets seem to be most
aligned at the periods of maximum solar activity. Has your team found this to be true?
●
The SOHO team has not been studying the relationship of the position of the planets to solar activity.
The solar cycle is believed to be driven by a magnetic dynamo process in the interior of the Sun.
There has been no scientific demonstration that the positions of planets can drive or be physically
related to the 11-year solar cycle.
Go back to "Astronomy" questions.
What is the meaning of exposure through a camera (e.g. 30 minutes, 1 hr, 45 min, 2 hr)? When you
are taking astronomy pictures, do you have to use a tracking motor on your telescope?
●
The exposure times are how long you must leave your camera shutter open. The stars are quite dim,
so you need to collect photons for much longer than for a regular daylight or flash photo.
In general, you do have to track astronomical objects if you want to get good photos of them.
Otherwise, you will get streaks as they move through the sky. This can create a nice effect as well,
though.
Go back to "Astronomy" questions.
●
Why do five (?) Lagrange points exist between the Earth and the Sun, and where are they lying?
Yes, There are 5 Lagrange points.
They are where low-mass objects can theoretically stay in gravitational equilibrium with two massive
ones orbiting around each other. The L1 point (where SOHO is) is in between the two bodies.
If the two large bodies (M1 & M2) have same mass the Lagrange points are arranged more or less
like:
http://sohowww.nascom.nasa.gov/explore/faq/astronomy.html (33 of 36) [2000/01/11 11:58:31 AM]
Dr. SOHO's FAQ: General Astronomy Questions
L4
L3
M1
(Sun)
L1
(SOHO)
M2
(Earth)
L2
L5
The L1-L3 points are shifted if the masses of the two large bodies are not equal. The L1, L2, and L3
points are points of "unstable equilibria" -- like putting a marble on top of a hill. The L4 and L5
points are points of "stable equilibria." Putting a spacecraft there is like putting a marble in a valley;
if it moves a bit, it will roll back to where it was. That is why you sometimes hear people talking
about putting space colonies at the Earth-Moon L5 point.
(In our solar system, the gravity from other planets is enough to disturb objects at the L4 and L5
points. There simply is no true "three body system" in a real solar system.)
For more information, go to:
http://www-spof.gsfc.nasa.gov/Education/wlagran.html
(Lagrangian Points)
Go back to "Astronomy" questions.
Recently, I had a chat with a person on the internet about planets, comets, asteroids, etc... and he
came up with the story that sometime between now and the end of the century, our planets (from Pluto to
the Sun) would be in line - making great changes to the Earth's gravitational pull. Doc, I don't know the
first thing about any of this but I've heard more than one person talking about the same thing. Is any of
it true? Or is it just pure baloney? Honestly I thing that it's quite intriguing, but I can't help but to
wonder about all this commotion towards the end of the century!
●
To answer your question about planetary alignments, below is a partial quote I got from a colleague
of mine who runs the "Ask the Astronomer," NASA's IMAGE Space Science Questions and Answers
site:
"There was a planetary alignment in December 1997 which was actually not very striking,
since the planets were spread out over a sizable stretch of the sky along the Ecliptic. In May,
2000 a better alignment from the earth will happen and extend about 40 degrees from the sun,
but split into a sunrise and sunset viewing opportunity as the planets are to the east and west of
the sun. Still more compact 'alignments' happen every few hundred years or so. The second
kind of 'alignment' is what you would see from a location near the sun with planets positioned
in their orbits within a few degrees of each other. Plotted with the sun at the center of
concentric orbit rings, the planets would form a line directed roughly towards the sun. To get
such an alignment where the planets are spread within a cone about 90 degrees in width is
http://sohowww.nascom.nasa.gov/explore/faq/astronomy.html (34 of 36) [2000/01/11 11:58:31 AM]
Dr. SOHO's FAQ: General Astronomy Questions
again very common..once a century or so in a Grand Conjunction. To get within 10 degrees,
the timing has to be even more precise and this 'coincidence' or synchrony takes thousands of
years. Even tighter alignments take millions of years, and even billions of years. I seem to
recall that, given the different orbit inclinations and periods, to get the planets to cluster within
a diameter equal to the moon as seen from the sun...a VERY tight line, takes several billion
times the age of the universe itself!!!"
In terms of its gravitational effect, it basically has none. All of the planets combined have only about
0.1% of the mass of the Sun (mostly in Jupiter). Gravitational effects are caused by mass and
distance, so even Jupiter (which you would need over 1000 Jupiters to equal the Sun) since it is 5
times farther than the Sun has almost no effect. The distance part of the equation is why the Moon
(relatively close) can cause tides. In terms of some real numbers:
The net tidal gravitational pull would be about 1/70 of what the Moon and the Sun produce and
would cause ocean water tides to be a millimeter or so taller averaged over the entire earth! If the
Moon and the Sun tides have little effect upon you, don't expect miracles from the other planets!!
In closing, no doomsday worries here!
Go back to "Astronomy" questions.
Hello! I am looking for a program to calculate or tables that show lunar rise and set times, and phase.
I am in Atlanta, Georgia. I am interested in this information for the next 50 years if possible. Can you
help?
●
Computing accurate lunar positions and phases is not easy, because there are a lot of small
perturbations that need to be taken into account. There are a number of books that give algorithms
that work to varying degrees of precision. Some even include programs that you can use directly. Try
these books (or any other book you can find that deals with astronomical computing or astrometry):
Astronomy With Your Personal Computer, by Peter Duffett-Smith, published by Cambridge
University Press.
Astronomical Algorithms, by Jean Meeus, published by Willmann-Bell.
Astronomy on the Personal Computer, by Oliver Montenbruck and Thomas Pfleger, published by
Springer-Verlag.
Accurate tables are published on an annual basis in the Astronomical Almanac, by the US Naval
Observatory in Washington, DC. These tables would not be of any use for predictive purposes
beyond the current year. However, The Observatory also has a web site with information on lunar
rise and set times:
http://aa.usno.navy.mil/AA/data/
(The US Naval Observatory)
Go back to "Astronomy" questions.
●
We live in Mississauga, Ontario, Canada. Can you help us calculate local noon at our site?
http://sohowww.nascom.nasa.gov/explore/faq/astronomy.html (35 of 36) [2000/01/11 11:58:31 AM]
Dr. SOHO's FAQ: General Astronomy Questions
Here's how to calculate local noon at your site. First of all, you'll need to know your longitude. Let's
take for example, the NASA Goddard Space Flight Center. It's located in Greenbelt, MD, which is
just outside of Washington, D.C. I look this up in the atlas, and find that the longitude is 76 degrees,
53 arcminutes west, or 76.88 degrees. This can be converted into a time by dividing by 15 degrees for
every hour. Thus:
76.88 degrees / (15 degrees/hour) = 5.125 hours = 5h 8m
(I rounded off to the nearest minute for simplicity.) The next step is to look up the time of meridian
passage in the Astronomical Almanac. For example, for Feb 11 this is given as 12:14. Since we're
west of Greenwich, we add the longitude factor calculated above and get:
12:14 + 5:08 = 17:22
Next, there's a small correction related to somewhat different definitions of time. The above time is
given in "Terrestrial Dynamic Time" (TDT). To convert this into normal time, add 64 seconds during
1998. This correction factor is given in section K as TDT-UT. To the nearest minute, this would
make the time of local noon as 17:23.
This is, of course, the time expressed in "Greenwich Mean Time" (GMT), which astronomers prefer
to refer to as "Universal Time" (UT or UTC). To convert that to local time, you need to find out how
many time zones you are from Greenwich time. For example, Eastern Standard Time (EST) is 5
hours different from GMT, so local noon on Feb 11 at Goddard would be at:
17:23 - 5 = 12:23
During the summer, the difference would only be 4 hours, because of daylight savings time.
Go back to "Astronomy" questions.
Go back to "Dr. SOHO's FAQ."
SEND US YOUR COMMENTS
Go To
Other SOHO Web Pages
Author: Shane Stezelberger
Co-Author/Curator: Therese A. Kucera
[email protected]
Responsible Official: Art Poland([email protected])
Last modification: Tuesday, 13-Jul-1999 11:31:21 EDT
http://sohowww.nascom.nasa.gov/explore/faq/astronomy.html (36 of 36) [2000/01/11 11:58:31 AM]
Dr. SOHO's FAQ: Miscellaneous Inquiries
Go back to "Dr SOHO's FAQ"
Go back to the "SOHO" main page.
●
What are those flying saucer-shaped objects in the LASCO images?
●
How will society benefit from the SOHO mission?
●
Can you give me a short primer on solar neutrinos, particularly as the subject relates to Arthur C.
Clarke's The Songs of Distant Earth?
●
Is AD 2000 a Double Leap Year?
●
Do you have any suggestions for developing a fictional character to live on the Sun?
●
Why are there no stars in pictures taken from the Moon?
●
Can you suggest Sun-related books for elementary through high-school grades?
●
If the Sun is emitting heat, why do regions of extreme cold exist within the atmospheric envelope?
●
If interstellar space is a near-vacuum, how does the Sun's supposed heat reach us?
●
Is anyone at SOHO an amateur radio operator?
●
I thought fusion reactions are always stronger than fission reactions -- yet I recently read a book
which contradicts this. Please explain.
●
What is the strength of UV-A and UV-B radiation at mid-day in mid-July?
●
Are there any space pictures of the Sun taken from distances much greater than the Earth?
●
At what distance does the Sun look like "just another star?" What formula do you use for
brightness vs. distance?
●
Where can I find ephemerides for the sun and Comet Hale-Bopp during March and April 1997?
●
When our Sun dies, will another star replace it?
●
Where should we go before the Sun dies and consumes us?
●
Do you wear safety goggles to observe the Sun?
●
Why is the Sun yellow if it is red in space?
●
Is there such a thing as an alien?
http://sohowww.nascom.nasa.gov/explore/faq/misc.html (1 of 23) [2000/01/11 11:58:45 AM]
Dr. SOHO's FAQ: Miscellaneous Inquiries
●
●
●
What are SOHO's most prominent discoveries so far?
Resultats de SOHO en Francais?
Science Careers and Scientists
❍ How did you become a scientist and learn cool stuff?
❍
What classes and degrees do I need to pursue for a professional career in astronomy?
❍
Where did you go to school? What did you study?
❍
Do you have partners in your work?
❍
What is it like to be a scientist? Do you enjoy your job?
❍
Do you get a lunch break?
❍
How do scientists feel about religion?
●
How can I put a latitude/longitude grid on my solar images?
●
Does light really bend?
●
Does light actually have vitamin D in it?
What are those flying saucer-shaped objects in the LASCO images?
Thanks for the question. The "funny-looking spheroid" is a typical response of the SOHO LASCO
coronagraph CCD detector to an object (planet or bright star) of small angular extent but so bright
that "bleeding" occurs along pixel rows.
CCD stands for charge-coupled detector, and refers to a silicon chip, usually a centimeter or two
across, divided into a grid of cells, each of which acts like a small photomultiplier in that an
incoming photon knocks loose one or more electrons. The electrons are "read out" by row (fast
direction) and column (slow direction), the current converted to a digital signal, and each cell or
picture element ("pixel") thus assigned a digital value proportional to the the number of incoming
photons in that pixel (the brightness of the part of the image falling on that pixel). This is the same
kind of detector as is used in a hand-held video camera, though until recently, the analog-to-digital
conversion was left out in consumer devices.
If you point a video camera at a very bright source (say, the Sun), the image "blooms" or brightens
all over --- there are so many electrons produced in the pixels corresponding to the bright source
that they spill over into adjacent rows and column, perhaps over the entire detector. Better CCD's
will "bleed" only along the fast readout direction (a single row), and perhaps a few adjacent rows.
The LASCO and EIT CCD cameras include "anti-bleed" electronics which limit the pixel bleeding
around bright sources to less than the full row (and usually no adjacent rows). In the case of a
marginally too-bright object, the pixel bleeding will be only a few pixels in either direction along
the fast readout direction. Thus, the "flying saucer" images.
A few of the LASCO images that have appeared on the "extraterrestrial" Web sites show much
larger and brighter, but still saucer-like features. These images are in fact obtained with the
instrument door closed, but with an incorrectly long exposure. The big "saucers" result from
http://sohowww.nascom.nasa.gov/explore/faq/misc.html (2 of 23) [2000/01/11 11:58:45 AM]
Dr. SOHO's FAQ: Miscellaneous Inquiries
massive pixel bleeding along every row of the detector containing part of the image of the "opal,"
or small diffusing lens, in the instrument door, that is used for obtaining calibration data.
If your correspondents still prefer to believe that the pixel-bled images of planets or bright stars are
something else, ask them why the extended part of the "saucers" (i.e., the pixel bleeding) always
occurs in the same direction relative to the image --- even when the spacecraft is rolled relative to
its normal orientation relative to the Sun.
I don't know if I'd blame the interest in the pixel-bled planetary images in LASCO data as hysteria,
or as a combination of healthy curiosity with an unfortunate lack of critical thought. If it's the
latter, I would tend to blame the lack of critical thought on our educational system. A healthy
skepticism is one that is applied not only to the statements of authority figures, but also to the
statements of self-appointed experts, I would hope.
Go back to "Miscellaneous" questions and answers.
●
I am fascinated by all of the information on SOHO. I am hoping to create a presentation for my
class to educate them on this satellite. I am curious as to the specific ways society will be able to
apply SOHO's findings. I understand that it tracks magnetism, but in what way will society be
able to use this information to it's benefit?
The fascination with SOHO is a fascination with the Sun, "Our Star," which SOHO was
designed to study. Because the Sun is the only star close enough to have real and dramatic
effects on our life here on Earth, we certainly expect and hope that improving our
observations and our understanding of this beautiful, awesome object will in the course of
time bring about beneficial applications. While it's never possible to tell where the quest for
knowledge will lead, at this time the area where we have the greatest expectation of useful
fallout is in the "space weather" arena.
"Space Weather" may sound abstruse, but it's a concept that is growing in importance as
mankind pushes further and further against the limits within which we live. When a farmer
had only an acre or two to worry about, a look out the window was a good enough weather
forecast for the day's plowing. When he has thousands of acres to plow, seed, and fertilize,
he may find it necessary to plan on a much broader scale in order to avoid disaster; thus, we
need weather satellites and global forecasting systems for tropospheric weather. Similarly,
when communication and electrical grids connected only local communities, the worst
threat might have been a lighting strike on a local utility pole. But today, our electrical
power grids span entire continents, and our communication lines reach across hemispheres,
linked by synchronous-altitude satellites. It's not too early to be thinking about the effects on
these extended systems, of vast clouds of atomic particles and magnetic fields thrown out by
the Sun on an almost daily basis.
It is important, sir, to impress on your students the value of seeking knowledge for it's own
sake. Don't let them think that knowledge is good only if it fattens someone's 'bottom line.'
We study in order to learn the truth, to discover what the world about us really is, and how it
works. We don't always know where the quest will lead us, but if we don't seek wisdom, we
can be sure it will lead us nowhere.
http://sohowww.nascom.nasa.gov/explore/faq/misc.html (3 of 23) [2000/01/11 11:58:45 AM]
Dr. SOHO's FAQ: Miscellaneous Inquiries
Go back to "Miscellaneous" questions and answers.
●
I do not know if you are an aficionado of science fiction, but I am. I have read a book, and for
the life of me both the title and the author escape ("SONGS OF DISTANT EARTH?" I think...)
Anyway, the premise is that Old Sol goes nova NOT 5 billion years hence but within the next
10,000 years, due to a much lower-than-expected or predicted by theory neutrino output. The
book didn't have as much science as I would have liked, but it was a good read. Could you
please give me a short primer on solar dynamics, the role of neutrinos, the theoretical lifespan
of the sun and what is likely to happen at the end of the Sun's life -- and most importantly, how
we know.
(Some of us here may take issue with any perceived lack of science content in THE SONGS
OF DISTANT EARTH, by Arthur C. Clarke, which is indeed the plot you describe. I
enjoyed it, too! But let us answer your questions.)
You are asking some pretty big questions here. You might want to consult a good
college-level introductory astro text. The Exploration of the Universe by Abell, Morrison, &
Wolff is good, Also, Shu's The Physical Universe might be worth looking at. It has a more
mathematical approach.
Even with a non-graphics browser, the following page on solar neutrinos would be
interesting: http://www.maths.qmw.ac.uk/~lms/research/neutrino.html
(The Solar Neutrino Problem)
Essentially, we are not detecting as many neutrinos as we would expect based on current
theory. One possible explanation for this anomaly is that our models of the solar interior are
not quite right. It is possible that, with a change in these models, our predictions concerning
the Sun's lifetime would change, although I doubt it would go from 5 billion to 10,000
years. If I recall correctly, Clarke himself is purposely vague when describing the Sun's
death mechanisms in that novel.
As you say, the current theory predicts that the Sun will last another 5 billion years (we
think it has been around about that long already. At that point it is expected that it will have
used up all the hydrogen being used as nuclear fuel in its core. This would cause the core to
contract. The contraction, in turn, would lead to heating and nuclear reactions in layers
outside the core and the expansion of the star into a "red giant." The Sun's outer layers
would expand past Mercury's orbit and maybe further.
Eventually, the energy due to the collapsing core would run out. The Sun would shrink
down to a small body about the size of the Earth called a "white dwarf," which would
continue cooling for tens of billions of years. Remember that this white dwarf has most of
the heat -- but little of the surface area -- of a much bigger star. Other, larger stars might
finish up in more exotic end-states such as neutron stars or black holes. These theories are
based on a number of things. Scientists have used physics to develop models of the interior
of the Sun which model and predict the physical parameters of the Sun and other starts
pretty well. Also, astronomers have observed the other stars in the galaxy, and the theories
also explain the existence and distributions of different type of stars.
http://sohowww.nascom.nasa.gov/explore/faq/misc.html (4 of 23) [2000/01/11 11:58:45 AM]
Dr. SOHO's FAQ: Miscellaneous Inquiries
Of course, the solar neutrino problem is one area where most current models of the Sun
don't work so well. The Sun seems to be producing fewer neutrinos than is predicted by the
models. One possible solution would be that the Sun is actually a little cooler deep inside
than we've thought. SOHO instruments which probe the Sun's interior via helioseismology
may be able to help answer some questions about the Sun's interior. For more information
on helioseismology, check out:
http://helios.tuc.noao.edu/helio.html
(Global Oscillation Network Group)
http://soi.stanford.edu
(The Solar Oscillations Investigation)
Go back to "Miscellaneous" questions and answers.
●
Greetings from the other side of the planet. I am a South African citizen, resident in the United
Arab Emirates. Could you answer the following question: Is the year 2000 a Double Leap Year
or not? I would appreciate an answer plus a solid explanation, like for instance -- the Earth
travels around the Sun in 365.251 days.
The year 2000 is a leap year in the normal sense, i.e. the 29th of February exists in that year.
There is no such thing as a "double" leap year.
The Gregorian calendar in use today assumes that the Earth goes around the Sun every
365.2425 days. The formula for adding leap days at the end of February is as follows: 1.
Years divisible by 4 are a leap year, unless overridden by rule 2.
2. Years which are divisible by 100 are not a leap year, unless overridden by rule 3.
3. Years which are divisible by 400 are a leap year.
This calendar was decreed by Pope Gregory XIII in a papal bull in February 1582. However,
many countries did not adopt it until centuries later. Before the Gregorian calendar, the
Julian calendar was used. This calendar, decreed by Julius Caesar, assumed a year of 365.25
days. By 1582, the error had accumulated to 10 days.
The actual length of the year is fairly close to 365.2425. According to the Astronomical
Almanac, a more accurate value is 365.24219. Thus, the Gregorian calendar should get out
of step by one day every 3226 years, and by the year 33840, we'll be in the same situation
Pope Gregory found himself in.
In the SOHO project, we try to be careful about how we handle time, to the point where we
not only worry about leap years, but also about leap seconds, which are added to the year at
irregular intervals to adjust for the varying rotation rate of the Earth's crust. More
information about time at this level of accuracy can be obtained from:
http://tycho.usno.navy.mil/time.html
(US Naval Observatory Time Service Department)
http://hpiers.obspm.fr
http://sohowww.nascom.nasa.gov/explore/faq/misc.html (5 of 23) [2000/01/11 11:58:45 AM]
Dr. SOHO's FAQ: Miscellaneous Inquiries
(INTERNATIONAL EARTH ROTATION SERVICE)
Go back to "Miscellaneous" questions and answers.
●
I am doing a science project about the Sun, and I am wondering if you have any suggestions or
ideas that I could use to create a fictional character that could adapt to living on the Sun.
Hmmm, an interesting question. As you probably know, the Sun's surface is VERY hot -around 5500 degrees C at the photosphere (the layer you see in visible light). Below AND
above that, it mostly just gets hotter. At those temperatures, atoms are disassociated and
ionized, so it would be hard to hold anything we normally think of as life together. Sunspots
are a little cooler -- there have actually been spectroscopic measurements of water vapor
(steam) in them, but NOT liquid water. That means molecules can stay intact in some
locations. NASA is thinking of sending a spacecraft "to" the Sun, but they don't expect to be
able to get it much closer than about 4 times the Sun's radius.
The other things to think about would be the high gravity and lack of any solid surface
(which is related to the temperature, of course).
I am trying to think of science-fictional examples. I know David Brin put a life form on the
Sun in his novel Sundiver, but I can't remember much about it. Hal Clement's first story,
"Proof," describes a spacefaring race of "neutronium" creatures who evolved in the Sun.
(First published in 1942, it was recently reprinted in The Ascent of Wonder.) A colleague
here says that Arthur C. Clarke has a widely-reprinted story from the late 1950s entitled
"Out of the Sun." Evidently, it is about a sentient bubble of solar plasma. Authors who have
bestowed anthropomorphic qualities -- more metaphysical than physical -- upon the stars
themselves include Benford & Eklund ("If the Stars are Gods"), Madeleine L'Engle (A
Wrinkle in Time), and Olaf Stapledon (Star Maker, et cetera).
Robert Forward's Dragon's Egg (and its sequel, Starquake,) envisions a sentient race
evolving on the surface of a neutron star. Our Sun is, however, quite different from a
neutron star.
Anyway, I guess it depends on how "realistic" you want to be. For any sort of life, you'd
want coherence -- not just a bunch of hot ions bouncing around. Maybe you could contain,
structure, and protect a life form with a magnetic field -- like how scientists try to contain
atomic fusion particles in laboratories, only in reverse. The Sun has lots of magnetic field
structures with which you can work:
http://sohowww.nascom.nasa.gov/explore/poster.html
(New Views of the Sun)
So far as we know, there are no living creatures on the Sun. If there were, they would have
to be very different from any sort of living being we know on Earth. With the temperature at
the "surface" of the Sun at about 10,000 degrees Fahrenheit, there is almost no oxygen, and
no liquid water. I put the word "surface" in quotes because the Sun does not have a solid
http://sohowww.nascom.nasa.gov/explore/faq/misc.html (6 of 23) [2000/01/11 11:58:46 AM]
Dr. SOHO's FAQ: Miscellaneous Inquiries
surface like the Earth; instead it is a very low-density gas, mostly hydrogen. So these
hypothetical creatures would need a very tough skin to survive the high temperatures, would
need to be able to breathe hydrogen, survive without water, and move around on a gaseous
surface.
For a look at the latest scientific activity concerning possible life elsewhere in the Universe,
we suggest the Astrobiology Web. I doubt you will find much mention of Sun-dwelling
creatures here, though:
http://www.astrobiology.com
(The Astrobiology Web)
I hope this is of help. Good luck. If you come up with something neat, we'd like to hear
about it!
Go back to "Miscellaneous" questions and answers.
●
I have noticed over many years that any photographs taken of the sky from the Moon fail to
show any stars. One would think that far more stars would be visible from the Moon than from
the Earth, with its atmosphere and load of particulate matter. I have conjectured that it is a
photographic effect; namely when the Earth is in the photographic field, the camera's aperture
has to be closed down so as not to be "blinded" by the brilliance of the Earth glow. Is this
explanation approximately correct? If not, what is?
You are right -- the stars as seen from the Moon ought to be very bright, but the sky
certainly does look black in those photos, even the ones without the Earth. I think you are
basically correct in that the dynamic range is just not wide enough to have the Earth or the
Moon's sunlit surface in the same shot with the stars. How exactly they optimized the
images to show what they are interested in, I can't say. It would depend on exactly which
space probe or Apollo mission the image is from. (Indeed, all of the six Apollo landings
occurred on the Lunar nearside. Therefore, the sunlit Earth was always visible in the sky to
the astronauts and their cameras.)
I don't observe the Moon, but we have similar problems in our observations of the Sun.
When viewing especially bright objects we do a number of things, including using a small
aperture (as you suggest), decreasing the exposure time, using a detector with low
sensitivity, or just arranging the color scale of the resulting image so that the features in
which we are interested are emphasized. All these techniques could lead to images without
relatively dim, "uninteresting" objects.
I looked around in some of the Clementine mission images and found a few that show stars
and planets, but no spectacular star-scapes. (Yes, they are not exactly "from" the Moon -they're from lunar orbit, to be precise -- but they are nice anyway):
http://www.nrl.navy.mil/clementine/clem_collect/limb.html
(A Clementine Collection. The caption mentions the Earth saturating the sensor)
http://www.nrl.navy.mil/clementine/clem_collect/venus.html
(A Clementine collection: Venus)
http://sohowww.nascom.nasa.gov/explore/faq/misc.html (7 of 23) [2000/01/11 11:58:46 AM]
Dr. SOHO's FAQ: Miscellaneous Inquiries
http://www.nrl.navy.mil/clementine/clem_collect/planets.html
(A Clementine Collection: The Moon, Sun, and Planets)
Clementine was small space probe that the US Defense Department launched in 1994 to test
new sensors. The inexpensive project was the first American lunar mission since Apollo,
took more than one million images of the Moon, and was widely credited as the first modern
demonstration of the "smaller faster cheaper" method of space exploration.
One final note: it is true that astronomy should be easier on the airless Moon than on the
Earth. Stars seen from the Moon will not "twinkle." A Naval Research Laboratory
experiment during the Apollo 16 mission used this advantage; this ultraviolet camera
became the first human observatory on another planetary body. A mockup of this
experiment is on display in the Smithsonian's Air and Space Museum, and its Principal
Investigator (Dr. George Carruthers) is still active at NRL as of this writing (mid-1998) One
of his astronaut "observers" on Apollo 16, John Young, is still working at NASA Johnson
Space Center in Houston.
Go back to "Miscellaneous" questions and answers.
●
Can you please give me a list of books, all related to the Sun of course, that would be useful to
an upper elementary, middle school, or high school student?
1. Sun, Earth and Sky, by Kenneth Lang. We frequently use this book around here.
Comprehensive and well-illustrated, it would be appropriate for high school students. It is
quite recent; it was published just before SOHO launched, and it has information and
pictures from the Yohkoh mission.
2. Exploration of the Universe, by Abell, Morrison, and Wolf. Many general astronomy
texts probably have good material. I have a copy of Abell, Morrison, and Wolf; they have a
good chapter on the Sun, again at a high school or introductory college level.
3. Catching the Light: The Entwined History of Light and Mind, by Arthur Zajonc (Oxford
University Press, 1993. ISBN 0 -19-509575-8. Not specifically about the Sun, but "...it's a
delightful book which traces how humans have endeavored to understand the phenomenon
of light. Blends myth, science, literature, art, religion, and physics into one unified
exploration of the nature of light. Easy to read, it might captivate a few high-schoolers."
4. For young elementary students, one colleague of ours likes Isaac Asimov's astronomy
series (20+ books; one is about the Sun).
5. Sun Up, Sun Down, by Gail Gibbons. I recently saw this book and looked through it
quickly. Aimed at first-graders, it has things about the seasons as well as basic stuff about
the Sun (such as: the Sun is a star, and it is the center of our solar system).
6. Someone here has a copy of the Audubon Society's The Sun and the Moon pocket guide.
This has essentially no flow. However, each little blurb seems accurate, and the pictures are
quite good.
http://sohowww.nascom.nasa.gov/explore/faq/misc.html (8 of 23) [2000/01/11 11:58:46 AM]
Dr. SOHO's FAQ: Miscellaneous Inquiries
7. Guide to the Sun, by K. J. H. Phillips (Cambridge University Press, 1996). This book is a
useful resource for specific information about solar science. It contains extensive
information on the Sun, particularly current knowledge about the physics and astronomy of
the Sun. It is written at the level of popular science magazines. Mathematics has been kept
to a minimum.
8. Exploring the Cosmos, by L. Berman and J. C. Evans (HarperCollins College Press,
1987), is another valuable source of information on the Sun and astronomy in general. This
is a well organized, well written, and easy to understand introductory text.
From the Solar Flare Theory page, General Interest:
Guide to the Sun, Kenneth J. H. Phillips. (Cambridge University Press, 1992) Secrets of the
Sun, Ronald Giovanelli. (Cambridge University Press, 1984) Sun, Earth and Sky, Kenneth
R. Lang. (Springer-Verlag, 1995) Interested in observing the Sun for a project or as a
hobby? These books will help:
Observing the Sun, Peter O. Taylor (Cambridge University Press, 1991) Peter Taylor
manages a Forum on CompuServe (GO SUNSPOT) Solar Astronomy Handbook, R. Beck,
H. Hilbrecht, K. Reinsch, and P. Volker, (Willmann-Bell Inc., Richmond VA, 1995)
Go back to "Miscellaneous" questions and answers.
●
I have been informed that you are one of the most eminent researchers on the subject of the
Sun. Many believe that current beliefs and science are upside-down and polar opposites of
reality. I put forth some questions here, and if you can answer them, great: (1) If there is an
outpouring of heat from the Sun, as is currently believed, then why do regions of extreme cold
exist within the atmospheric envelope? (2) If interstellar space is a near vacuum, with a thermal
temperature of minus 273.15 degrees Celsius or 0 Kelvin, then how does the Sun's supposed
heat reach us? I propose that the Sun uses cold centripetal processes and NOT expansive heat
processes. Are you interested? I can prove what I say.
I am not certain I understand your opening remark. It is certainly true that "conventional
wisdom" in any given field of science is not necessarily correct. However, when we succeed
in acting as scientists, in spite of our biases, we hold our scientific "beliefs" as tentative, and
try to stay open-minded to the possibility that new information may contradict what we
think we know. We try to base those "beliefs" on reasonable interpretations of the most
reliable data available, i.e., measurements and results that can be reproduced. And when a
theory is developed to explain existing data, it must have testable consequences, including
predictions of information that is not yet known.
Scientists tend to be somewhat conservative about holding on to ideas which seem to have
worked well in the past, and also skeptical about new ideas which seem to run counter to our
hard-won physical "intuition," but that is how we keep each other honest. We cross-check
each other's measurements, numerical codes, and analytic derivations. We argue about
competing interpretations and constantly seek new ways to test theories. If someone
introducing a new idea wants to be taken seriously, the burden of proof is on him or her to
demonstrate that the new idea is consistent with the existing data, and to show why past
http://sohowww.nascom.nasa.gov/explore/faq/misc.html (9 of 23) [2000/01/11 11:58:46 AM]
Dr. SOHO's FAQ: Miscellaneous Inquiries
interpretations are incorrect or incomplete.
With respect to your specific questions:
(1) If there is an outpouring of heat from the Sun, as is currently believed, then why do
regions of extreme cold exist within the atmospheric envelope?
I am unaware of any evidence that there are regions of extreme cold (i.e., much less than a
few thousand degrees K) in the solar atmosphere within a few solar radii of the solar
surface. Of course, in some sense the solar atmosphere extends throughout the solar system.
I'll get to the material further away in a bit.
There is strong evidence of several kinds that the bulk of the material in the solar corona
(the outer layer of the Sun), within distances of a few times the solar radius, has a
temperature near a million degrees K or more. This evidence consists primarily of in-situ
particle detections or spectroscopic measurements of highly ionized trace elements whose
atoms have many electrons stripped away. This hot ionized gas or plasma produces spectral
emission line radiation in visible light, ultraviolet, extreme ultraviolet, and soft X-ray
wavelengths, which is characteristic of the specific ions (atoms with electrons removed)
present in the plasma at the given temperatures. There is a lot of redundant information from
ions from different elements. Further, the thermal (kinetic) motions of the ions along the line
of sight broaden the spectral emission lines -- emission from ions moving toward us are
blue-shifted while emission from ions moving away is red-shifted, so that the line spreads
out in wavelength. The resulting thermal widths of the lines imply kinetic temperatures
(temperatures corresponding to the random thermal motions) that are comparable to the
ionization temperatures (the temperatures corresponding to the ionization states or number
of electrons stripped away) of the respective ions. In-situ particle detectors which measure
the ionization states and the velocities of the particles confirm the picture of
high-temperature plasma.
As the plasma expands outward from the Sun, into the solar wind, the plasma cools
somewhat from the expansion, so that the kinetic temperatures drop down to only 10,000 to
500,000 degrees K in the vicinity of the Earth. At this distance from the Sun, the density of
the ions drops to about one particle per cubic centimeter, so that there is a low probability
that the particles will collide and redistribute their energy (so the plasma is called
"collisionless"). Some would argue that the whole concept of temperature doesn't have much
meaning for a plasma with almost no collisions, but it is still possible to measure (with
in-situ particle detectors) the spread in the velocities of the particles (which is usually taken
as a measure of the "kinetic temperature" - the temperature associated with the particle
motions) and to count the number of ions of a given element in each ionization state, and so
determine the "frozen in" temperature from when the plasma was still "collisional" (i.e.,
when particles had a high probability of colliding and redistributing their energy) back when
they had higher density, closer to the Sun. The measured, "frozen in" temperatures agree
with the million-or-so-degree temperatures measured by spectrometers which measure
emission lines from plasma when it is closer to the Sun.
Besides particles from the solar wind plasma, there are also some particles from interstellar
http://sohowww.nascom.nasa.gov/explore/faq/misc.html (10 of 23) [2000/01/11 11:58:46 AM]
Dr. SOHO's FAQ: Miscellaneous Inquiries
space in the near-Earth environment, but their density is about a factor of 10 lower than the
solar wind particles. So the probability for the interstellar particles to collide with the solar
particles is much lower even than the probability for the solar particles to collide with each
other; thus the effective temperature of the solar wind particles doesn't change, although the
plasma is very tenuous.
(2) If interstellar space is a near vacuum, with a thermal temperature of minus 273.15
degrees C or 0 Kelvin, then how does the Sun's supposed heat reach us?
The solar wind and energetic particle streams can buffet the Earth's magnetosphere, and
cause geomagnetic disturbances, communication breakdown, and power grid failure, as well
as aurorae. These can be quite dramatic transfers of energy -- but most of the energy from
the Sun reaching the Earth is from sunlight. It is more a question of the Sun's energy, rather
than the Sun's heat, reaching us. Light travels just fine through a near-vacuum.
Also, I wonder if you are confusing interstellar space -- the space between the stars -- with
interplanetary space -- the space within the solar system? Interplanetary space is continually
bathed -- and sometimes buffeted -- by the solar wind. At Earth, the solar wind is a much
more important factor than interstellar gas, but the total energy transfer to the Earth from the
solar wind plasma is nearly negligible compared to the total energy transfer from sunlight.
Go back to "Miscellaneous" questions and answers.
●
Is anyone on this address also an Amateur Radio operator? Is anyone a member of the Goddard
Amateur Radio Club?
There may be some ham operators here, but I don't know of anyone in particular. You can
write the Goddard Amateur Radio Club directly:
http://garc.gsfc.nasa.gov/www/garc-home-page.html
(Goddard Amateur Radio Club Home Page)
The contact seems to be Jim Blackwell at [email protected]).
Go back to "Miscellaneous" questions and answers.
●
Since childhood, I have always been convinced that the energy released from a nuclear fusion
reaction was many magnitudes greater than that of a corresponding nuclear fission reaction.
recently I came across a contrary description while reading a book titled In The Shadow of a
Star. I forgot the author's name but it was a history of the pursuit of neutrinos and neutrino
detection. The author described a fission reaction as producing 10X the energy of a fusion
reaction. Please comment. I will return to the library if further details on author, etc. are
necessary.
It depends on what you mean by "corresponding nuclear fission reaction." For a particular
reaction formula (e.g. 1 hydrogen + 1 deuterium -> 1 helium + 1 neutron + 16.6 MeV of
energy) fission reaction are about 10 times more energetic than fusion reactions. However,
the particles that participate in fusion are much less massive than those of fission reactions.
Thus, if the reactions are described in terms of energy per unit mass, fusion reactions
http://sohowww.nascom.nasa.gov/explore/faq/misc.html (11 of 23) [2000/01/11 11:58:46 AM]
Dr. SOHO's FAQ: Miscellaneous Inquiries
produce a few times more energy than fission reactions. (Reference: Chp. 11 of Modern
Physics by Tippler, 1st ed.)
Go back to "Miscellaneous" questions and answers.
●
I'm doing some personal research on the effects of alpha/beta radiation as it hits Earth at
different times of the year. For example, what is the strength of UV-A's and UV-B's in mid-July
at mid-day? What is the unit of measure of each, and what type of instrument is used to measure
them? Any help will be greatly appreciated or for additional source(s) would be a help.
Your question has more to do with meteorology than it does with solar physics. There's very
little variation in the solar irradiance at those wavelengths. The changes you see at the
ground are due almost entirely to atmospheric effects. Thus, I'm not sure I'm able to answer
your questions to your satisfaction. However, I'll try.
1. What is the strength of UV-A's and UV-B's in mid-July at mid-day?
I think that matters a lot on where you are. It'll be different in the Caribbean than in Maine,
for example. Variations in the ozone layer also matter. The ozone hole over the south pole
lets through more ultraviolet radiation than other parts of the atmosphere.
For example, here is a Web site I found which gives predicted amounts of UV-B light for
Australia:
http://www.bom.gov.au/info/about_uvb.shtml
(Forecasts for Sun Safety at the Bureau of Meteorology)
I suggest searching other meteorological sites for similar information.
2. What is the unit of measure of each?
Here is a good web site that explains this:
http://www.intl-light.com/handbook/ch01.html
(Light Measurement Handbook: What is Light?)
UV-A is defined to be the total amount of light between 340-400 nanometers. This is the
ultraviolet light which is just shortward of the visible light spectrum, and in fact overlaps the
visible spectrum slightly. UV-B is somewhat shorter than that, from 290-340 nanometers). It
appears that the meteorology and radiometry groups use slightly different definitions for
these bands.
A nanometer is one billionth of a meter, or 10-9 meters. You may also encounter an older
unit of measure, called the Angstrom. An Angstrom is 1/10th of a nanometer, or 10-10
meters. Nanometers are considered to be a more standard way of expressing wavelengths in
this spectral regime, but many astronomers still use Angstroms.
The unit of measure in these two wavelength bands would be in Watts per square meters
(W/m2), or in Watts per square centimeters (W/cm2). There also appears to be a UV index
http://sohowww.nascom.nasa.gov/explore/faq/misc.html (12 of 23) [2000/01/11 11:58:46 AM]
Dr. SOHO's FAQ: Miscellaneous Inquiries
system for the forecasted amount of UV-B radiation, but I haven't been able to find out how
that relates to W/m2.
3. What type of instrument is used to measure them?
A photometer, calibrated in those wavelength ranges. For example, check out:
http://www.intl-light.com/products/photothe.html
(PRODUCTS: Phototherapy Measurement Systems)
There was one thing in your mail message that confused me. You talked about alpha/beta
radiation. To me, that's something very different, and not at all related to, UV-A and UV-B.
Alpha radiation refers to the flux of alpha particles, which consist of two protons and
neutrons bound together. Beta radiation consists of electrons.
Go back to "Miscellaneous" questions and answers.
●
Are there any space mission photos with perspectives looking back at the Sun from distances
greater than Earth, i.e. from the vicinity of Mars, Saturn, Jupiter, Neptune, or Pluto?
One candidate is my very favorite Mars Pathfinder image, because there is something
special about this sunset:
http://wwwmpf.jpl.nasa.gov/ops/sunset_rt.gif
(A Martian Sunset)
Mars is only about 50% further out from the Sun than the Earth, so the Sun still largely
"looks like" the Sun.
In the early 1990s, one of the two Voyager spacecraft took several images of the inner solar
system from a distance of many billions of kilometers. The late Carl Sagan published some
of them in the book Pale Blue Dot. The title comes from the appearance of the planet Earth
at such a distance. Here's what we look like from 6.4 billion km away:
http://newproducts.jpl.nasa.gov/calendar/vgr_fam.html
(The Voyager Family Portrait)
http://www.seds.org/billa/psc/pbd.html
(Reflections on a Mote of Dust -- The Pale Blue Dot)
As you can see, the Sun is 40 times smaller in diameter when viewed from such a distance.
Remember that most spacecraft -- the Hubble Space Telescope, for example -- try very hard
to keep their instruments pointed away from the Sun. Staring at the Sun can be as damaging
to a spacecraft sensor as it can be to your own eyes. Mars Pathfinder was one exception, as
is SOHO.
Go back to "Miscellaneous" questions and answers.
●
When or at what distance does the Sun start to take on the appearance of "just another star?"
Is there a simple math formula that can be used to calculate relative size-brightness of the Sun
http://sohowww.nascom.nasa.gov/explore/faq/misc.html (13 of 23) [2000/01/11 11:58:47 AM]
Dr. SOHO's FAQ: Miscellaneous Inquiries
with respect to distance?
Both the brightness of the Sun and the area it covers in the sky go down as the square of the
distance away. What would it take for the Sun to seem to be "just another star?" I suppose if
you couldn't resolve it as a disk, that would be star-like. Offhand, however, I don't know
how small something has to appear before that is that case. On Earth it is probably
controlled by the atmosphere, but in space we get a clearer view.
Another way to do it would be to calculate how far away the Sun would have to be before it
was only as bright as the brightest Star, Sirius. From Earth the Sun is about 10 billion times
brighter (1010) than Sirius. Because the brightness goes down as the square of the distance,
the Sun would have to be about 100 thousand (105) times further away than it is to be as
bright (or as dim) as Sirius. This is about 1.6 light years. Sirius is actually much farther
away (8.8 light years) -- it is intrinsically much brighter than the Sun. Pluto is only 0.0006
light years from the Sun, so the Sun should still be quite bright from there.
The formula I am using is just a proportion:
Luminosity1 / Luminosity2 = (distance2/distance1)2
I looked up the relative brightnesses of the Sun and Sirius in a astronomy textbook.
Go back to "Miscellaneous" questions and answers.
●
I'm working on a paper about Hale Bopp. I'm just calculating the positions and orbit data. To
be able to do this, I need the ephemeris (X, Y, Z) of the Sun between March and April 1997.
Please help me finding some links to this item!
Well, the most interesting co-ordinate system for your problem is the Sun-centered
co-ordinate system. So you probably don't need an ephemeris for the Sun at all! You need
orbital data about the Earth.
If you want to be quick and dirty about it, you can assume the Earth is in a 1 A.U. circular
orbit in the plane of the Ecliptic. Remember that the Earth's position is fixed at the Vernal
Equinox: that's when the Sun crosses (northbound) the intersection between the celestial
equator and the Ecliptic.
Check the "Comets" section of Dr. SOHO's FAQ for some useful web sites. I also took a
quick look at the net with Alta-Vista and found the following:
ftp://navigator.jpl.nasa.gov/ephem/export
(JPL Export)
which is the Jet Propulsion Laboratory's Export site. You might try checking there.
Go back to "Miscellaneous" questions and answers.
●
I was wondering, when our Sun dies, which will be in 6 billion years, will another Sun or star
replace it?
http://sohowww.nascom.nasa.gov/explore/faq/misc.html (14 of 23) [2000/01/11 11:58:47 AM]
Dr. SOHO's FAQ: Miscellaneous Inquiries
Essentially, no.
When the Sun reaches the end of its normal existence in, as you say, about 6 billion years
(most think it's 5 billion), it will go through a number of stages of evolution. Our
understanding of the future of the Sun is based on observations of many other stars of
similar type at various stages of their lives.
The first thing to happen will be the collapse of the central core when the hydrogen fuel for
nuclear fusion reactions runs out. This core collapse will provide a "bounce" that will cause
the outer layers of the Sun's atmosphere to expand outwards and become cooler. The outer
layers will then envelope the inner planets, including Earth -- killing off any remaining
forms of life.
The Sun at this stage will be what is known as a "red giant", which is just what it sounds
like: a big star which is red in color. (It gets red because the outer atmosphere is cooler than
it is now. Red is a "cooler" color than yellow). The core will then start to burn helium, and
there is enough helium present to sustain the red giant phase for a long time (probably
millions of years). When the helium runs out, the core will collapse further and even heavier
elements may begin to burn. Eventually, the outer layers will be blown off completely, and
all that will remain of the Sun will be its core, which will gradually cool to form what we
call a "white dwarf."
If the Sun were 3 or 4 times heavier than it actually is, there is a good chance that it would
end its life as a black hole rather than a white dwarf.
The white dwarf will cool slowly over tens of billions of years. No new star will
automatically replace the Sun. However, the hydrogen envelope ejected after the Sun's red
giant phase may eventually help form other stars elsewhere in the galaxy.
Go back to "Miscellaneous" questions and answers.
●
Where should we go before the Sun dies and consumes us?
That shouldn't happen for another 5 billion years or so. That is a long time. There may not
even be any humans around by then.
If there are, they will need to leave. Earth probably will not survive the Sun's red giant
phase, as you indicate. Although the moons of the outer planets may survive, they will be
hard for us to inhabit without solar energy. Any existing "O'Neill style" space colonies won't
survive, and they require solar power in the first place. The eventual white dwarf will not be
energetic enough to sustain nearby civilizations.
In the absence of a Sun, this solar system will have few options for life. Interstellar travel
may be a way out, but it will be very difficult. Flying to nearby star systems won't be like
"Star Trek." The energy requirements for any interstellar vessel are huge. Scientists, even
the fair minded ones who read science fiction and who think aliens could exist, tend to
discourage hopes for real interstellar travel.
http://sohowww.nascom.nasa.gov/explore/faq/misc.html (15 of 23) [2000/01/11 11:58:47 AM]
Dr. SOHO's FAQ: Miscellaneous Inquiries
The other stars are very, very far away. However, traveling out to them represents human
civilization's only chance to outlive the Sun.
Go back to "Miscellaneous" questions and answers.
●
Do you wear safety goggles to observe the Sun?
I mostly observe the Sun with telescopes on the spacecraft, so I don't need to look directly at
it. Astronomers using telescopes on the ground observe with special filters that block out
much of the Sun's light. (Even then, special goggles are not uncommon at telescopes like the
McMath at Kitt Peak, New Mexico.):
http://www.nso.noao.edu/nsokp/mp.html
(The NSO McMath-Pierce Facility at Kitt Peak)
There are kinds of "safety glasses" you can use to look at the Sun briefly, but unless you are
very sure you have the right kind, it is better to project a light from the Sun on a wall or
piece of paper and look at that. One way to do that is with a "pin hole camera." I'll try to find
some good instructions and send them to you.
Above all, NEVER look at the Sun with your naked eyes -- not even for a quick glance.
Obey this rule and your retinas will thank you.
Go back to "Miscellaneous" questions and answers.
●
Why is the Sun yellow if it is red in space?
Note: we have a slightly different take on this answer at:
http://sohowww.nascom.nasa.gov/explore/faq/basics.html#COLOR
(Basic Solar Questions: What color is the Sun?)
It is difficult to define just what color the Sun is. Some books define sunlight as "white
light," meaning that it has every color in it. Remember that sunlight can be split into a
rainbow spectrum by a prism or a raindrop. Still, the Sun does appear "yellow" in our sky -but DO NOT LOOK AT IT to confirm this! Astronomers used to call the Sun a "yellow"
star because of its temperature. (Now they call it a "G2V star," which is a far more
official-sounding designation.)
I've never seen the Sun from space -- of course, I've never even been in outer space at all!
To a human eye, sunlight would probably look the same from space as it does from the
Earth's surface. To a scientific camera (like the cameras on SOHO), the Sun can look totally
different from space. The Sun emits all kinds of wavelengths and radiation which cannot
penetrate the Earth's atmosphere. We can see things from space that we cannot see from the
ground. That is why we launched SOHO into space.
SOHO's pictures -- the green and orange ones, for example -- are usually taken in these
special wavelengths. We cannot normally "see" ultraviolet light, so these pictures have no
http://sohowww.nascom.nasa.gov/explore/faq/misc.html (16 of 23) [2000/01/11 11:58:47 AM]
Dr. SOHO's FAQ: Miscellaneous Inquiries
real "color." When we put them on a computer, we give these pictures "false" colors. The
Sun isn't green or orange, but these pictures are.
Scientists use false color for many reasons, but one important reason is simply that it looks
nice. False colors also help separate images of different wavelengths. Otherwise, all of our
ultraviolet pictures would be grey. Different wavelengths would be hard to tell apart.
Go back to "Miscellaneous" questions and answers.
●
Is there such a thing as an alien?
Nobody knows. Despite all the talk of UFOs, we have no evidence of visiting aliens or of
crashed alien spaceships. We have never detected a definite radio message from aliens, but
we're not finished listening yet! Nobody knows -- yet.
There are a billion stars in our galaxy alone, but we don't know how many of them have
planets. We don't know how many of those planets have life. With all those stars out there,
many scientists think that there might be some kind of aliens somewhere. But without
evidence of any kind, one way or the other, they cannot say for sure. We have not yet
bumped into any of them. In short, nobody really knows.
But some are trying to find out. "Alien" could include the possible micro-fossils from the
famous Martian meteorite. One place to keep up with all the latest news about life elsewhere
in the Universe is the Astrobiology Web:
http://www.astrobiology.com
(The Astrobiology Web)
Go back to "Miscellaneous" questions and answers.
●
I am researching the possibility of writing a feature about SOHO for a magazine. It would
feature the most prominent scientific discoveries that SOHO has afforded us. What do you think
they should be?
In response to your request for SOHO's most prominent discoveries, I would say that the
magnetic carpet coronal heating result ranks very highly, followed by the subsurface jet
stream result. More info on these results can be found at:
http://soi.stanford.edu/press/ssu8-97/
(SOI/MDI SSU8-97 Press Release)
Also check SOHO's CDS home pages for information about X-ray bright points, another
new SOHO find. LASCO and EIT have been contributing to our understanding of "space
weather," and you can find out more space weather information at the ISTP web site:
http://www-istp.gsfc.nasa.gov/
(International Solar-Terrestrial Physics)
http://sohowww.nascom.nasa.gov/explore/faq/misc.html (17 of 23) [2000/01/11 11:58:47 AM]
Dr. SOHO's FAQ: Miscellaneous Inquiries
Go back to "Miscellaneous" questions and answers.
●
Astronome amateur francais, je recherche un site presentant les differents resultats de SOHO
en langue francaise. Si un tel site existe, je vous serai tres reconnaissant de bien vouloir me le
faire savoir. Dans l'attente de votre reponse, je vous souhaite une bonne continuation dans vos
recherches et continuez a nous faire REVER.
Bonjour,
As a Matra engineer I work on the SOHO operations. Since I'm French, I got your e-mail
because nobody else was able to understand it. So I strongly recommend to write your future
requests in English.
To answer your questions about SOHO results described in French:
http://www.medoc-ias.u-psud.fr/
(medoc fournit des donnees sur SOHO)
A part cela voir aussi a l'IAS (www.isa.fr) les infos sur SUMER. Apres j'ai bien peur que
tout soit en Anglais, notamment les publications scientifiques:
http://sohowww.nascom.nasa.gov/publications/CDROM1/papers/index.html
(The First Results from SOHO)
Alors bon courage.
Go back to "Miscellaneous" questions and answers.
●
How did you become a scientist and learn cool stuff?
I started getting interested in science and astronomy when I was very young. I joined an
amateur astronomy club in high school, and I took lots of science and math classes along
with my other ones. In college, I majored in physics. After college, I went on to graduate
school. Then I got a job here at NASA.
Go back to "Miscellaneous" questions and answers.
●
Do you have partners you work with to find out these answers?
Yes. People often work on scientific projects together. Most reports are co-written by
several scientists. In solar astronomy, we collaborate and share data with observatories all
over the world. When it is nighttime in North America, the Sun is visible to astronomers in
Australia.
Running a spacecraft like SOHO takes a lot of people; that is part of what makes it fun.
Scientists from a dozen countries cooperate with the engineers to plan each day's
observations.
Go back to "Miscellaneous" questions and answers.
http://sohowww.nascom.nasa.gov/explore/faq/misc.html (18 of 23) [2000/01/11 11:58:47 AM]
Dr. SOHO's FAQ: Miscellaneous Inquiries
●
Could someone help me? I need to find out what classes/degree(s) I need to get to pursue
astronomy professionally. I have a math degree and a good background in chemistry. I would
appreciate any suggestions.
Your question is one we like to hear, because it means you might consider a career in one of
the oldest, and at the same time, newest and most exciting branches of science. Bear in
mind, however, that astrophysical careers are difficult to begin and even harder to maintain.
From your note, it actually sounds as though you have already acquired a good background.
Math and chemistry are the places to start. The next logical steps would be toward courses
in physics and, of course, astronomy. If there is a Department of Astronomy at your college,
go talk to them, and see what they offer. Many colleges and universities, even large ones, do
not have a separate Astronomy department; often it is combined in some way with the
Physics Department. If your school does not, then talk to the folks in Physics. Try to find
someone in the Physics Dept. that has an interest in astronomy or related areas, such as
spectroscopy or atomic physics, or especially plasma physics.
Think about whether your talents are such as to lead you to lean toward theoretical work or
experimental. But to begin, the introductory courses in physics and astronomy are the place
for you.
Go back to "Miscellaneous" questions and answers.
●
Where did you go to school, and what did you study in order to be a scientist?
(A reply from a SOHO scientist): I went to grade school and high school in Evanston,
Illinois, where I grew up. I took lots of college prepatory classes, including math and
science. For college I went to a place called Carleton College in Minnesota, where I majored
in physics. Then I went to graduate school in astrophysics at the University of Colorado.
(A reply from a SOHO engineer): I attended grade school and high school in Pennsylvania,
also taking college-preparatory classes. I went to Virginia Polytechnic Institute, which I
chose because of its engineering reputation, and I majored in aerospace engineering. Upon
my graduation, I applied for and was offered this instrument operations position at the
SOHO project.
If you think you want to follow either of us on a similar career path, please start preparing
now. Take as many math and science courses as you can, but do not neglect things like
English, either (communications skills are vital). Try to take some calculus classes before
you finish high school, for example. Apply to the best college you can, and work as hard as
you can once you get there. Remember that a scientific or technical education can open
many opportunities for you. Good luck!
Go back to "Miscellaneous" questions and answers.
●
What is it like to be a scientist? Do you enjoy your job?
http://sohowww.nascom.nasa.gov/explore/faq/misc.html (19 of 23) [2000/01/11 11:58:47 AM]
Dr. SOHO's FAQ: Miscellaneous Inquiries
It is usually pretty fun. I especially like my current job because I get to do all sorts of
interesting things. I use the instruments on the spacecraft and work with the data when it
comes back. I am trying to answer questions about how the Sun works. Then I have to tell
people about what I have been doing so that they understand. Sometimes these things are
hard and I get frustrated, but mostly it is very interesting and challenging too.
I like to study the Sun for a number of reasons. I am interested in astronomy, and the Sun is
the closest star. It makes for a very nice target; studying the Sun is one way to find out
things about stars in general. I also like studying something that is related to us directly. The
Sun supplies the energy that keeps life on Earth going! Changes on the Sun can affect us.
This is true now more than ever, because explosions on the Sun can affect spacecraft,
powerlines, and astronauts.
Go back to "Miscellaneous" questions and answers.
●
Do you get a lunch break?
Yes.
Go back to "Miscellaneous" questions and answers.
●
My husband is always saying, "They" are finding so much now... pretty soon they will blow
religion out of the water. He makes it seem as though anyone in science/astronomy, are all
Satan worshipers! What is the status on the "religion" issue? Mostly antireligion, proreligion,
neutral??
I don't have the numbers in front of me, but I've seen surveys which show that scientists are
just as likely to say that they believe in God as a non-scientist. Evidently, a training in
science is no hindrance to religion. Science can tell us many things about how the universe
was formed, but it doesn't address the questions of why the universe exists.
That said, a scientist's view of religion does conflict with some people's religious beliefs. A
scientist, like a police detective, looks at the evidence. This leads us to conclude that the
Earth is billions of years old, and the Universe as a whole is even older, which is in conflict
with some people's belief that everything was formed in the year 4004 B.C. There's nothing
new in this - scientists have known that the Earth is old for at least a century. But this is not
in conflict with those who believe that the formation of the stars and planets is part of a
grand design of an omniscient God. Most of the established religions, including most
Christian churches, would agree.
Go back to "Miscellaneous" questions and answers.
●
I am looking for helpful code to enable placing the heliographic longitude and latitude on my
solar images. Can you help me?
Most of our software is written in a language known as IDL (Interactive Data Language)
from Research Systems, Inc. If you also use IDL, then you can retrieve our software by one
of two methods:
http://sohowww.nascom.nasa.gov/explore/faq/misc.html (20 of 23) [2000/01/11 11:58:47 AM]
Dr. SOHO's FAQ: Miscellaneous Inquiries
1. You can install the Solar Software (SSW) Library by following the instructions on the
page:
http://sohowww.nascom.nasa.gov/software.html
(SOHO software)
This is a software library incorporating analysis routines for most of the major solar satellite
missions. The generic section has many general-purpose routines for solar data analysis. If
you want to get involved in the analysis of data from any of these missions, I strongly
encourage you to install this library.
2. You can retrieve individual routines by going through the SOHO anonymous ftp server
at:
ftp://sohoftp.nascom.nasa.gov/solarsoft/
(Directory of /solarsoft)
In particular, you may wish to visit:
ftp://sohoftp.nascom.nasa.gov/solarsoft/gen/idl/solar/
(Directory of /solarsoft/gen/idl/solar)
One routine in that directory which may be of interest is plot_helio.pro.
The IMAGE_TOOL package in the directory:
ftp://sohoftp.nascom.nasa.gov/solarsoft/gen/idl/image_tool/index.html
(Directory of /solarsoft/gen/idl/image_tool)
is also very good for displaying solar images from FITS files, and it gives one the option of
over-plotting heliographic longitude and latitude. It's oriented toward extracting images
from a particular SOHO database, but it does give one the option of pulling in images from
personal directories.
Even if you don't use IDL, you may wish to look at the routine plot_helio.pro and possibly
convert it to FORTRAN, C, or whatever language you do use.
Go back to "Miscellaneous" questions and answers.
●
Light bends? Really, could light be a 'new' matter? What we see are the fragments breaking
apart... and when the fragments become so dispersed they lose their momentum and turn back.
Okay... take a seed. If you leave the seed in the ground and water it without light, you would be
lucky to get mold! But... add light and you have a tree. Is there some exchange of matter taking
place here?
Yes, light rays do bend in the presence of gravitational fields. This was first predicted by
Albert Einstein as part of his General Theory of Relativity.
In quantum mechanics, light consists of particle-like entities called photons. So yes, in some
sense light is a kind of matter, although not in the normal meaning of the term. These
photons have no mass. However, Einstein showed that the apparent mass of an object
http://sohowww.nascom.nasa.gov/explore/faq/misc.html (21 of 23) [2000/01/11 11:58:47 AM]
Dr. SOHO's FAQ: Miscellaneous Inquiries
increases as it approaches the speed of light. Since photons are never standing still, but
always move at the speed of light, even though they're massless, they act like they have a
mass. Thus, they're attracted by a gravitational field, and this causes the light rays which go
closely past a large object to be slightly bent.
I'm afraid I don't understand your point about "fragments breaking apart". Photons cannot
break apart into fragments. They can interact with something else and be scattered,
reflected, or absorbed. For example, the light coming from a lamp can scatter off a chair,
and that's how you see the chair. The chair also absorbs some of the light, and that warms it
up, especially if it's dark colored. A mirror reflects light.
When plants absorb light, what they get is energy, not matter. Light is a pure form of
energy. When light is absorbed by a plant, that energy is converted into chemical energy
which the plant can then use. Animals, on the other hand, get their energy from the food
they eat. Ultimately, that energy came from a plant absorbing light.
Go back to "Miscellaneous" questions and answers.
●
Well... I do like our discussion. I try to keep my observations 'down to earth.' I remember the
News telling people that the Sun was good at some point, because it's light has vitamin 'D' in it.
I see breaking up of light in fragments, and the fragments are what we see. But... because light
is giving this essence away, it loses concentration and falls!! And that might be the reason you
see it bending into gravitational fields that are more powerful. But... it withheld its light or
concentration up to that point.
It's not strictly correct to say that light has vitamin D in it, but one does get vitamin D from
exposure to sunlight. The energy in the sunlight causes chemical reactions in our skin, and
this chemical reaction makes vitamin D.
I can only say again that a photon cannot break up into fragments, unless it bumps into
something. What you may be thinking of is that light can be broken up into its component
colors. Pass white light through a prism, and you'll see different colors come out the other
end. All those colors existed in the light to begin with. Each photon has its own particular
color. The white light going into the prism has photons in it of all colors. Each photon, as it
passes through the prism, is sent in a different direction depending on its color, and on the
other end you see the photons sorted by colors to make a rainbow.
Think of a photon as being like a baseball. Both fall within a gravitational field. A baseball
doesn't fall because it breaks up into fragments. It falls because it has weight. The same is
true for a photon, although only in a relativistic sense can one say that a photon has weight
(mass). It's really as simple as that.
Go back to "Miscellaneous" questions and answers.
Go back to "Dr. SOHO's FAQ."
SEND US YOUR COMMENTS
http://sohowww.nascom.nasa.gov/explore/faq/misc.html (22 of 23) [2000/01/11 11:58:47 AM]
Dr. SOHO's FAQ: Miscellaneous Inquiries
Go To
Other SOHO Web Pages
Author: Shane Stezelberger
Co-Author/Curator: Therese A. Kucera
[email protected]
Responsible Official: Art Poland([email protected])
Last modification: Tuesday, 13-Apr-1999 17:33:10 EDT
http://sohowww.nascom.nasa.gov/explore/faq/misc.html (23 of 23) [2000/01/11 11:58:47 AM]